Vous êtes sur la page 1sur 207

!

1. EL TEST DE LA D-XILOSA ALTERADO INDICA:

1. 1. DEFECTO DE LA DIGESTIÓN DE LOS CARBOHIDRATOS.


SÍNDROME DE MALABSORCIÓN POR DEFECTO DE LA PARED DEL
2. 2.
INTESTINO.
3. 3. DÉFICIT CONGÉNITO DE DISACARIDASAS.
4. 4. OBSTRUCCIÓN BILIAR.
Gráfico de respuestas
Comentario

La D-xilosa se absorbe en el yeyuno por difusión pasiva, sin necesidad de que intervengan las
enzimas pancreáticas o las sales biliares. Por ello, está alterada en procesos que afectan a la
pared del tubo digestivo (respuesta 2 correcta), sin verse influida por alteraciones pancreáticas ni
biliares.

El test de la D-xilosa consiste en administrar este azúcar en ayunas, y luego se mide su presencia
en orina, cuando pasan unas horas. Si la encontramos en orina, quiere decir que se ha absorbido a
nivel intestinal.(R/2)
!

2. Un marinero de 37 años acude a consulta por presentar dos úlceras de


aproximadamente l cm de diámetro en el orificio del prepucio. Son dolorosas, de bordes
irregulares y el fondo de las úlceras está cubierto de exudado necrótico. En la exploración
física aparece una adenopatía inguinal izquierda inflamatoria de carácter doloroso y
sensible al contacto. El resto de la exploración es normal. El paciente reconoce haber
mantenido contactos sexuales con prostitutas cada vez que su barco fondeaba en un
puerto, siendo la última vez hace una semana. ¿Cuál sería su diagnóstico?

1. 1. Chancro blando.
2. 2. Linfogranuloma venéreo.
3. 3. Sífilis primaria.
4. 4. Herpes genital.
Gráfico de respuestas
Comentario

Dentro de las ETS, es importante que sepa realizar el diagnóstico diferencial entre ellas. Veamos
opción por opción:

Chancro blando: Produce lesiones múltiples, dolorosas y con adenopatías unilaterales.

Sífilis primaria: El chancro suele ser único, no doloroso, y con adenopatías bilaterales.

Linfogranuloma venéreo: No produce un chancro como tal, sino una lesión papulosa, o incluso
nada en absoluto… Pero después aparecen adenopatías unilaterales de gran tamaño, que acaban
fistulizando.

Herpes genital: Lesiones ulcerosas múltiples, con adenopatías bilaterales y molestias dolorosas.

!
!
!

En este caso, la presencia de una adenopatía unilateral, así como el dolor y el período de
incubación breve (una semana) habla a favor de un chancro blando.(R/1)
!

3. Mujer de 45 años que tras intervención quirúrgica de masa ovárica izquierda presenta
dolor en flanco izquierdo, náuseas, vómitos, ileo paralítico e hidronefrosis izquierda. ¿La
prueba que usted pediría para valorar la lesión que probablemente tenga la paciente,
sería?

1. 1. USG abdominal.
2. 2. USG transvaginal.
3. 3. Arteriografía.
4. 4. Urografía.
Gráfico de respuestas
Comentario

Es una pregunta de dificultad media que debe ser tenida en cuenta. Se trata de un posible ligadura
de uréter que ocasiona hidronefrosis con fiebre, dolor en flanco, náuseas, vómitos e íleo. Se podría
estudiar mediante USG, pero el mejor método diagnóstico en estos casos es la urografía ya sea
intravenosa o retrógrada. En cuanto al tratamiento, la primera mediad debe se la derivación
urinaria.(R/4)

4. Excluyendo la válvula aórtica bicúspide, ¿cuál es la cardiopatía congénita más


frecuente?:

1. 1. Conducto arterioso persistente.


2. 2. Comunicación interventricular.
3. 3. Coartación aórtica.
4. 4. Tetralogía de Fallot.
Gráfico de respuestas
Comentario

Es importante que domine bien su clasificación así como los aspectos clínicos de las principales
cardiopatías congénitas. Recuerde que la cardiopatía congénita más frecuente es la válvula aórtica
bicúspide, seguida de la CIV. Ahora bien, si le preguntan por la cardiopatía congénita cianótica
más frecuente debe contestar la transposición de los grandes vasos, en el RN y la tetralogía de
Fallot a partir del primer año de vida.(R/2)

5. Una de las siguientes NO es causa habitual de hemoptisis:

1. 1. Bronquiectasias.
2. 2. Tuberculosis.
3. 3. Estenosis mitral.
4. 4. Enfisema pulmonar.
Gráfico de respuestas
Comentario
El conocimiento de la semiología en medicina es muy importante porque es necesario dominar
determinados síntomas para así conseguir seleccionar la respuesta correcta en un caso clínico, ya
sea porque lo conocemos o simplemente descartando el resto de las opciones. La hemoptisis se

!
!
!

define como la presencia de sangre en el esputo, normalmente procedente de vías aéreas


subglóticas, y siempre es motivo de preocupación tanto para el enfermo como para el médico. Esta
sangre puede proceder de las vías respiratorias, del parénquima pulmonar o de los vasos
sanguíneos. Las enfermedades del árbol respiratorio pueden ser inflamatorias (bronquitis, que es
la causa más frecuente de hemoptisis, bronquiectasias, fibrosis quística,...) o neoplásicas
(carcinoma broncógeno o carcinoide). Las enfermedades del parénquima pueden ser localizadas
(neumonía, absceso, tuberculosis o Aspergillus) o difusas (sd de Goodspasture y hemosiderosis
pulmonar idiopática). Las enfermedades vasculares que pueden cursar con hemoptisis son el TEP,
las malformaciones y todas aquellas situaciones que aumenten la presión en el árbol pulmonar
(p.e. estenosis mitral). De esta forma vemos que el enfisema no suele producir hemoptisis, por lo
que la seleccionamos como opción correcta.(R/4)

6. De los siguientes, el factor más relacionado con el crecimiento intrauterino retardado


tipo simétrico es:

1. 1. Infección congénita.
2. 2. Hipertensión.
3. 3. Diabetes con vasculopatía.
4. 4. Edad de la mujer por encima de los 35 años.
Gráfico de respuestas
Comentario

Los fetos pequeños para la edad gestacional son aquellos que se encuentran debajo del percentil
10. Se dividen en:

PEQUEÑO PARA EDAD GESTACIONAL CONSTITUCIONAL: no hay alteraciones estructurales,


son normales pero en rango de crecimiento bajo.

PEQUEÑO PARA LA EDAD GESTACIONAL ANOMALO (CIR SIMETRICO): Presentan anomalias


genéticas estructurales o secundarias a una infección TORCH. Son "armónicos". Todos los
parámetros son proporcionados pero retrasados respecto a la edad gestacional.

CIR II O ASIMÉTRICO: Secundarios a una funcion placentaria alterada. Doppler alterado "les falta
comida". Son disarmónicos con BDP y LF acorde con la edad pero DA retardado.(R/1)

7. Lactante de 2 meses y medio, que presenta un cuadro de catarro de vías altas de 2 días
de evolución, acude hoy al servicio de urgencias por rechazo de las tomas y dificultad
respiratoria. En la inspección tiene tiraje subcostal e intercostal moderado, con ruidos
inspiratorios de vías altas; se auscultan roncus y sibilancias diseminadas en ambos
hemitórax; el resto de la exploración es normal. Tiene una saturación de O2 del 90%.
¿Qué patología sospecha?

1. 1. Bronquiolitis aguda.
2. 2. Bronquiolitis obliterante.
3. 3. Cardiopatía congénita descompensada.
4. 4. Asma bronquial.

!
!
!

Gráfico de respuestas
Comentario

Pregunta muy fácil y que no debe fallar, por lo típico y lo frecuente del cuadro clínico. Aunque la
definición de bronquiolitis habla de lactantes menores de dos años con su primer episodio de
dificultad respiratoria, se da más en los menores de 6 meses.

Es importante tener en cuenta los síntomas previos para su diagnóstico, porque en los lactantes
más mayores, sobre todo con antecedentes familiares u otros factores de riesgo para
hiperreactividad bronquial, como la dermatitis atópica, las alergias alimentarias, o el tabaquismo
pasivo, la bronquiolitis puede confundirse con el asma. Las crisis de broncoespasmo, sin embargo,
suelen tener una aparición brusca; en la auscultación, se escuchan fundamentalmente sibilancias
(sin ruidos de secreciones), y lo más importante, las crisis se repiten con frecuencia.(R/1)
!

8. En una postmenopáusica sana, el tratamiento sustitutivo adecuado es:

Antidopaminérgicos tipo veralipride para el síndrome climatérico y asociar


1. 1.
alendronato.
2. 2. Fitoestrógenos.
3. 3. Estrógenos y progestágenos.
4. 4. Asociación de fitoestrógenos con bifosfonatos y calcio con vitamina D3.
Gráfico de respuestas
Comentario

Pregunta sobre el tratamiento de la menopausia que puede resultar algo complicada si no tenemos
los conceptos claros. No hay que confundirse por la variedad de respuestas, ya que en el
enunciado tenemos la clave: postmenopáusica sana. En las opciones 1 y 4 se incluyen
tratamientos para la osteoporosis (raloxifeno y bifosfonatos como el alendronato), con lo que ya no
se trata de mujeres sanas. Los fitoestrógenos son sustancias de origen vegetal con acción
estrogénica débil, con lo que será efectivas en un grupo reducido de pacientes que presenten
síndrome climatérico débil. Es obvio pensar que el tratamiento sustitutivo adecuado es la
combinación de estrógenos y progestágeno, ya que mejora los síntomas del síndrome climatérico y
es útil en la prevención de la pérdida de masa ósea; así, la opción correcta es la nº4.(R/3)
!

9. Señale cuál de los siguientes síntomas NO aparecería en una fractura-luxación del D12
con sección medular, de muy poco tiempo de evolución:

1. 1. Hiperreflexia.
2. 2. Atonía muscular.
3. 3. Pérdida completa de la sensibilidad por debajo de nivel lesionado.
4. 4. Abolición de reflejos superficiales.
Gráfico de respuestas
Comentario

Pregunta difícil y de un tema poco preguntado como es el manejo de los traumatismos medulares.
La sección medular de poco tiempo de evolución se corresponde con el denominado choque
medular, que consiste en la pérdida transitoria de todos los reflejos espinales con parálisis fláccida
por debajo del nivel de lesión (opción 1 incorrecta). Recuerde que el choque medular precede en

!
!
!

un tiempo variable a la lesión medular establecida, caracterizada por signos de 1ª motoneurona por
debajo de la lesión (espasticidad e hiperreflexia).(R/1)

10. Hombre de 62 años que consulta por molestias faríngeas de 3 meses de evolución. A
la exploración se aprecia tumoración ulcero-vegetante de cara laríngea de epiglotis que
infiltra bandas ventriculares. Cuerdas vocales libres con movilidad conservada.
Adenopatías bilaterales múltiples, pequeñas, móviles y no dolorosas. El diagnóstico
anatomopatológico es carcinoma epidermoide moderadamente diferenciado. ¿Cuál de
las siguientes opciones terapéuticas considera más CORRECTA?:

1. 1. Laringuectomía horizontal supraglótica y vaciamiento radical bilateral.


2. 2. Hemilaringuectomía vertical y vaciamiento funcional bilateral.
3. 3. Laringuectomía horizontal supraglótica y vaciamiento funcional bilateral.
4. 4. Hemilaringuectomía vertical y vaciamiento radical bilateral.
Gráfico de respuestas
Comentario
Respecto al tratamiento de los tumores supraglóticos, la cirugía es uno de los tratamientos más
utilizados. El tumor descrito es un tumor supraglótico bajo (supraglotico infrahioideo) que respeta el
plano glotico a nivel de comisura anterior, cuerda vocal y aritenoides y además no paraliza la
cuerda vocal, con lo cual la cirugía a realizar será una laringuectomia horizontal supraglotica. El
estado del cuello es el factor que empeora más el pronostico en estos tumores, ya que las
metástasis regionales cervicales son frecuentes y hace necesario un vaciamiento cervical bilateral
en este paciente que es N2, pero también estaría indicado aunque fuese N0. El vaciamiento si sólo
es un N2 y no afecta a paquete vascular cervical suele ser suficiente con que sea funcional.(R/3)

11. El CO2 es el gas ideal para la realización de un neumoperitoneo en una laparoscopia,


ya que para que un gas sea utilizado en laparoscopia debe ser:

1. 1. Medianamente soluble, irritante e incomburente.


2. 2. Insoluble, comburente e incoloro.
3. 3. Soluble, incoloro e incomburente.
4. 4. Incoloro, insoluble y volátil.
Gráfico de respuestas
Comentario

Se han empleado para el neumoperitoneo diferentes tipos de gases, desde aire, nitrógeno, argón,
helio, CO2 y óxido nitroso. Cada uno de ellos reviste características distintas, pero en términos
generales sólo los gases solubles (CO2 y N2O) tienen aplicaciones en la práctica de la
laparoscopia. A diferencia del N2O, el CO2 es incombustible, evitando el riesgo de chispazos al
usar el electrobisturí.(R/3)

12. Una mujer de 85 años consulta por cansancio y debilidad especialmente por las
mañanas. A veces se encuentra inestable al caminar y tiene que sentarse para recuperar
el equilibrio. En dos ocasiones ha tenido que sentarse para no caer pero niega síntomas
de mareo. Tiene hipertensión arterial, incontinencia urinaria y artrosis. Su tratamiento es
hidroclorotiazida (25 mg/d), oxibutinina (10 mg/d), lisinopril (10 mg/d), calcio (1500 mg/d)
y paracetamol (3 g/d). A la exploración destaca una tensión arterial de 115/70 mmHg,
pulso 80 lpm. Sus movimientos son lentos. Tiene un temblor moderado en las manos.
Puede levantarse de la silla lentamente pero sin necesidad de apoyarse en los brazos.
Camina levemente inclinada hacia delante con poco balanceo de los brazos. Gira

!
!
!

lentamente pero sin perder el equilibrio. No es capaz de mantenerse sobre un solo pie.
¿Cuál de las siguientes posibles actuaciones realizaría en primer lugar?

1. 1. Valorar la agudeza visual.


2. 2. Estudio con mesa basculante.
3. 3. Medir la tensión arterial tumbada y levantada.
4. 4. Intento terapéutico con L-dopa.
Gráfico de respuestas
Comentario

Pregunta difícil.

El caso clínico nos describe a una mujer de 85 años con clínica probablemente debida a una
enfermedad de Parkinson (cansancio, debilidad, inestabilidad con la marcha, bradicinesia, temblor
distal, escaso balanceo con la marcha…). Sin embargo, la paciente consulta por cansancio y
debilidad que aparecen “especialmente por las mañanas”, y además “ha tenido que sentarse para
recuperar el equilibrio y para no caer” negando síntomas de mareo.

Estos datos nos hacen sospechar un cuadro de hipotensión postural en el contexto de una posible
enfermedad de Parkinson. Además, la presencia de tratamiento antihipertensivo, especialmente los
diuréticos, hablan a favor de esta sospecha. También toma un anticolinérgico, que puede estar
contribuyendo (oxibutinina). La clave de esta pregunta, viene dada por el enunciado final: nos pide
la actuación en primer lugar.

Las respuestas 1 y 2 son claramente falsas. La disminución de la agudeza visual no produce este
cuadro, no existen datos de focalidad neurológica que nos hagan sospechar que una RM aportará
información al cuadro, y la mesa basculante es un test realizado para el diagnóstico diferencial del
síncope vasovagal con riesgo de hipotensión grave y asistolia que no debe realizarse en ancianos.

La respuesta 4 es también falsa, aunque lleva a confusión. Es cierto que al tratarse de una anciana
con clínica parkinsoniana el intento con L-dopa es una buena actitud a llevar a cabo. Sin embargo,
en la pregunta se nos pide la primera actuación, y en el contexto de una posible hipotensión
postural, es prioritario descartar esta y tratarla, disminuyendo por ejemplo la dosis de diurético.
Además, el tratamiento antiparkinsoniano con L-dopa puede empeorar o causar hipotensión
ortostática, haciéndose menos recomendable en este cuadro.

Por tanto, la respuesta correcta es la 3, medir la tensión arterial tumbada y levantada, siendo este
el test más fácil y rápido para confirmar el ortostatismo.(R/3)

13. A 4-year-old boy with a history of sickle cell anemia and splenic infarcts becomes to
the emergency department presenting with fever and malaise. Laboratory results shows
anemia, reticulocytosis and leukocytosis. What would be expected in a peripheral blood
smear?

1. 1. Howell Jolly Bodies.


2. 2. Heinz bodies.
3. 3. Dacryocites.
4. 4. Hipersegmented PMN.
Gráfico de respuestas
Comentario
!
!
!

Howell-Jolly bodies. Howell-Jolly bodies are histopathological findings of basophilic nuclear


remnants (clusters of DNA) in circulating red blood cells.They usually indicate spleen damage. In
patients affected by sickle cell anemia hyperviscosity causes spleen infarcts so its function gets
markedly reduced.(R/1)

14. ¿Cuál de los siguientes NO es un factor asociado a candidiasis?

1. 1. Hemorragia intermenstrual.
2. 2. Diabetes mellitus.
3. 3. Corticoterapia.
4. 4. Antibióticos.
Gráfico de respuestas
Comentario

Si se fija la respuesta 2 o 3 puden causar inmunosupresión celular, por lo que en estos pacientes
se incrementa el riesgo. El uso de antibióticos aumenta las probabilidades de candidiasis por
destruir la flora bacteriana habitual. La respuesta que debe elegir es la 1.(R/1)

15. Con la clasificación de choque hipovolémico, por ATLS, señale verdadero o falso: ( 1
) Pérdidas de sangre Clase I, son equivalentes a la donación sanguínea. ( 2 ) En Clase II,
hay marcado compromiso como taquicardia, taquipnea e hipotensión arterial. ( 3)
Pérdidas Clase III, pueden ser manejadas solo con solución salina. ( 4 ) Pérdidas Clase
IV, se pierde aproximadamente 40% de la volemia y ponen en riesgo la vida del paciente.

1. 1. FFVV.
2. 2. VFFV.
3. 3. FVFV.
4. 4. VFVF.
Gráfico de respuestas
Comentario

La opción correcta es la 2 ya que:

-Clase I: V. Las pérdidas son equivalentes a la donación al ser menores de 750ml.

-Clase II: F. No hay marcado compromiso cardiovascular ya que la TA es normal y la taquicardia no es muy marcada.

-Clase III: F. Se deben administrar cristaloides y concentrados de hamatiesen este nivel.

-Clase IV: V. Las perdidas son del 40% del volumen o mayores con riesgo vital(R/2)

!
!
!

16. Médico traumatólogo de 40 años que en una serología rutinaria se le detecta niveles
de antiHBs de 30 UI/I. Refiere vacunación completa según pauta estándar frente hepatitis
B hace 4 años. Sería recomendable:

1. 1. Reiniciar pauta vacunal (0-1-6 meses).


2. 2. Reiniciar pauta vacunal (0-1-2-12 meses).
3. 3. No revacunar.
Darle inmunoglobulina anti-hepatitis B y reiniciar pauta vacunal (0-1-2-12
4. 4.
meses).
Gráfico de respuestas
Comentario

El paciente presenta niveles protectores de AcHBs (>10UI/l) por lo que no requiere más
inmunización frente a la hepatitis B.(R/3)

17. A 43-year-old woman with a


history of multinodular goiter comes to the clinic for a follow-up examination after a
gammagraphy has been performed, which is shown in the image. Laboratory studies
show: TG and TPO antibodies slightly elevated, TSH level 0.001 microUi/ml (normal range:
0.35-4.95), free T4 3.569 ng/dl (normal 0.700-1.480) and free T3 > 30 pg/ml (normal 1.71 -
4.53). Which of the following is the most likely diagnosis?

1. 1. Graves disease.
Functioning metastatic follicular carcinoma of the thyroid gland. Thyroglobulin
2. 2.
levels should be measured.
3. 3. Toxic multinodular goiter.
4. 4. Toxic thyroid adenoma.
Gráfico de respuestas
Comentario

Para acertar esta pregunta, dependemos bastante de la imagen gammagráfica adjunta. En


cualquier caso, no resulta difícil darse cuenta de que se trata de un adenoma tóxico (tiroides
prácticamente abolido, con una enorme hipercaptación en una zona redondeada). Este diagnóstico
encaja perfectamente con el perfil hormonal que nos describen (elevación de las hormonas
tiroideas con TSH suprimida, es decir, el de un hipertiroidismo). Si se tratara de un bocio
multinodular tóxico, la captación se vería en forma de múltiples nódulos, como su nombre indica.

!
!
!

Tampoco podría tratarse de un estruma ovari, donde tendríamos una gammagrafía hipocaptante
(es un tumor ovárico que secreta hormonas tiroideas y por ello la TSH estaría suprimida). Es difícil
pensar en una metástasis de un cáncer folicular, ya que estamos a nivel del tiroides, donde en todo
caso sería un tumor primario… Y, aun así, sería muy difícil pensar en un cáncer, por tratarse de un
nódulo “caliente”. Por último, la enfermedad de Graves-Basedow produciría una hipercaptación
tiroidea, pero difusa.(R/4)

18. En un paciente que presenta una eritrodermia, los siguientes signos clínicos harían
sospechar una etiología psoriásica EXCEPTO uno, señálelo:

1. 1. Límite neto de las lesiones.


2. 2. Exfoliación seca de todo el tegumento, incluido el cuero cabelludo.
3. 3. Estado general conservado.
4. 4. Prurito marcado en las localizaciones que la psoriasis afecta típicamente.
Gráfico de respuestas
Comentario

Se trata de un pregunta complicada, que exige un conocimiento bastante concreto de las


características de un tipo de psoriasis no muy preguntada en el ENARM.

La eritrodermia psoriásica no es uno de los apartados más preguntados.

Debe recordar al menos: - definición de eritrodermia: enrojecimiento generalizado, con


descamación que no representa una entidad en sí, si no que refleja una presentación clínica de
diferentes entidades. - En este caso nos preguntan por la causa psoríasica de la misma. La
psoriasis representa en 20% del total de las causas de eritrodermia. Suele precederse de lesiones
típicas de psoriasis en placas y suele ser también el resultado del cese de tratamientos como el
metotrexate o corticoides sistémicos. - Clínicamente puede orientar a su diagnóstico: o cambios
unguelaes o descamación plateada en placas delimitadas en cara, rodillas y codos no de forma
generalizada.(R/4)

19. ¿Cuál de las siguientes situaciones NO predispone al Kernicterus en el recién


nacido?:

1. 1. Ingesta materna de sulfamidas.


2. 2. Sepsis neonatal.
3. 3. Hipoalbuminemia neonatal.
4. 4. Alcalosis metabólica.
Gráfico de respuestas
Comentario
El kernicterus es la consecuencia del depósito de bilirrubina indirecta sobre los ganglios basales
del SNC. Entre sus causas se necesita una situación de hiperbilirrubinemia muy importante (debida
a hemólisis, sepsis, hemorragia interna, TORCH) y factores adicionales (como prematuridad,
acidosis metabólica, hipoxia, enfermedad de base, ingesta materna de sulfamidas,
hipoalbuminemia,...)(R/4)

20. De las siguientes características, una de ellas NO es característica de la intolerancia


a la lactosa:

!
!
!

1. 1. Lactato fecal aumentado.


2. 2. Heces acuosas y con olor ácido.
3. 3. Deposiciones explosivas.
4. 4. Cuerpos reductores (Clinitest) negativos en heces.
Gráfico de respuestas
Comentario

La intolerancia transitoria a la lactosa es un cuadro autolimitado en el tiempo que aparece tras una
gastroenteritis invasiva , debido a la destrucción de los enterocitos.

La incapacidad de digerir la lactosa hace que este azúcar permanezca en heces. Al ser la lactosa
un azúcar reductor, se acidifican las heces, apareciendo diarrea explosiva y eritema anal por
irritación, aumentando el lactato fecal y apareciendo en el Clinitest presencia mayor del 0.5% de
cuerpos reductores, es decir, test POSITIVO, por eso la respuesta 4 es la única falsa.(R/4)

21. Señale en qué supuesto hay que estudiar siempre la causa de la ictericia:

1. 1. La ictericia que aparece a las 48 horas de vida.


2. 2. La bilirrubina alcanza una cifra superior a 10 mg/dL.
3. 3. La bilirrubina directa supera el 20% de la total en cualquier momento.
4. 4. En el prematuro alcanza un valor superior a 12 mg/dL al 7º día de vida.
Gráfico de respuestas
Comentario

Esta pregunta puede parecer tediosa y compleja, pero no lo es tanto si tiene las nociones claras.
Ha de saber que las hiperbilirrubinemias pueden ser de dos tipos en función de la cifra de
bilirrubina directa: colestáticas (si BID excede el 20%) y no colestáticas (si BID menor del 20%). La
opción 3 designa una ictericia colestática, tributaria por tanto de estudio. El resto de las opciones
forma parte del espectro de la ictericia fisiológica.(R/3)

22. La edad más frecuente de convulsión febril es:

1. 1. 1 mes - 4 años.
2. 2. 6 años - 10 años.
3. 3. 3 meses - 5 años.
4. 4. 7 dias - 5 años.
Gráfico de respuestas
Comentario

Es simplemente un dato memorístico, o la sabe o no. La respuesta correcta es la número 3: 3


meses a 5 años.(R/3)

23. Un RN perderá calor por radiación si:

1. 1. Esta expuesto al aire.


2. 2. No se le seca correctamente.
3. 3. Sus ropas están muy frías.
4. 4. Se le desnuda en un ambiente frio.
Comentario
La reanimación neonatal es un tema que debe dominar.

!
!
!

Pregunta sencilla. La respuesta correcta es la 4. Repase la siguiente lista.

Evaporación: a través de piel y pulmones.

Conducción: hacia sólidos que están en contacto.

Radiación: hacia sólidos que no hacen contacto.

Convección: diferencia de temperatura con el aire circundante.(R/4)

24. Marque V ó F, respecto a la meningoencefalitis bacteriana: ( ) La glucosa aumenta en


el LCR, ( ) Las proteínas aumentan en el LCR, ( ) Los gérmenes ingresan al encéfalo por
los plexos coroideos, ( ) Para disminuir el edema cerebral usamos los corticoides, ( ) La
ceguera como complicación es muy frecuente.

1. 1. F V V F F.
2. 2. F V F V V.
3. 3. V V V V F.
4. 4. F F F F F.
Gráfico de respuestas
Comentario

La meningitis bacteriana se caracteriza por:

- Disminución de la glucosa en el LCR.

- Proteinas aumentadas en el LCR.

- Los gérmenes entran por los plexos coroideos.

- No se usan corticoides para reducir el edema cerebral

- No es frecuente la ceguera como complicación sino la sordera.(R/1)

25. Un paciente alcohólico, vagabundo, acude a Urgencias por síndrome febril de larga
evolución y deterioro del estado general. A la exploración el paciente está sucio y
descuidado y presenta un soplo sistólico en foco aórtico. Se realiza un ecocardiograma
que demuestra la presencia de una vegetación en la válvula aórtica, a las dos semanas
los hemocultivos continúan negativos. Este paciente presenta una endocarditis por:

1. 1. Bartonella quintana.
2. 2. Streptococcus bovis.
3. 3. C. albicans.
4. 4. S. aureus.
Gráfico de respuestas
Comentario

!
!
!

La endocarditis infecciosa (EI) es un tema muy importante en el examen. Lo más preguntado es la


etiología, clínica, tratamiento y profilaxis.

Nos describen una EI en la que los hemocultivos son negativos, y de las 4 respuestas la más
probable es la Bartonella quintana (RC- 1), que se ha descrito con mayor frecuencia en alcohólicos
indigentes, cono el paciente de este caso. El cuadro se conoce como “fiebre de las trincheras”.

Otros microorganismos que producen endocarditis infecciosa con hemocultivos negativos son los
bacilos del grupo HACEK, que precisan medios de cultivo enriquecidos e incubación durante un
tiempo prolongado.

Coxiella y Brucella son también gérmenes productores de EI con hemocultivos negativos, suelen
afectar a la válvula aórtica y, con frecuencia, precisan cirugía para su tratamiento.(R/1)

26. Niño de 3 años, previamente sano, en el cual al examen físico presenta fiebre, cefalea,
signos meningeos y fotofobia, cual sería la etiología más probable:

1. 1. Glaucoma agudo.
2. 2. Neuritis óptica.
3. 3. Meningitis.
4. 4. Rabia.
Gráfico de respuestas
Comentario

No puede equivocarse en esta pregunta. La respuesta correcta es la 3 meningitis por presentar


fiebre, cefalea, fotofobia y signos meningeos.(R/3)

27. ¿Cuál de las siguientes opciones caracteriza la fase aguda de las dermatitis
eccematosas?:

1. 1. Eritema, edema y vesículas.


2. 2. Formación de costras y descamación.
3. 3. Aparición de estrías de Wickham.
Pápulas y placas eritematosas bien delimitadas y cubiertas por varias capas de
4. 4.
escamas finas y nacaradas.
Gráfico de respuestas
Comentario

El eccema es una reacción cutánea inflamatoria que se caracteriza histológicamente por


espongiosis, acantosis más o menos marcada y un infiltrado linfohistiocitario perivascular
superficial. La espongiosis es el edema intercelular de la epidermis que conduce a la ruptura de las
uniones intercelulares con la formación de vesículas. Los eccemas pueden dividirse, desde el
punto de vista evolutivo, en tres fases. En la fase aguda predomina la espongiosis y la formación
de vesículas. Clínicamente esto se traduce en la presencia de eritema, edema, vesículas, ampollas
y exudación. En la fase subaguda la espongiosis y las vesículas disminuyen mientras aumenta la
acantosis. Además se forma un estrato córneo paraqueratósico. En la clínica se observan lesiones
costrosas y descamación. Finalmente en la fase crónica se encuentra hiperqueratosis con áreas de

!
!
!

paraqueratosis y acantosis que clínicamente se manifiesta como grietas, fisuras y liquenificación.


En el liquen plano se forma un reticulado blanquecino en la superficie de las pápulas que se
denomina "estrías de Wickham". La opción 4 corresponde a las lesiones de la psoriasis.(R/1)

28. Una mujer de 55 años acude a consulta por presentar rectorragia con heces de
aspecto mucoide. En el examen endoscópico se encuentra una tumoración plana de
superficie vellosa, de 4 x 3 cm, que se reseca endoscópicamente aparentemente de forma
completa. ¿Cuál es la actuación médica prioritaria en la paciente?

Hacer un estadiaje tumoral inmediato, sobre todo con el fin de descartar


1. 1.
metástasis hepáticas precoces.
Realizar como tratamiento inmediato resección intestinal de la zona que incluya
2. 2. el lecho tumoral, precedida de quimioterapia neoadyuvante y radioterapia
preoperatoria.
Realizar un estudio histológico exhaustivo de la pieza con el fin de descartar áreas
3. 3.
de carcinoma intramucoso o infiltrante.
Realizar un estudio clínico, radiológico y genético urgente, con el fin de descartar
4. 4.
poliposis múltiple familiar y cáncer colorrectal hereditario.
Gráfico de respuestas
Comentario

Estamos ante una mujer de edad media con rectorragia y el hallazgo de una tumoración que se
reseca. Ante todo, la actitud prioritaria será filiar el tipo de lesión con el que nos encontramos, ya
que de ello dependerá nuestra actitud posterior (opción 3 correcta). Lo primordial a descartar es la
existencia de focos de malignidad, que no puede presuponer de antemano (opciones 1 y 2
incorrectas). Por último, en general las poliposis familiares no debutan en este tipo de edades y
hay otros datos clínicos añadidos para ponernos tras la pista (opción 4 incorrecta).(R/3)
!

29. En una mujer embarazada de 18 semanas, sin antecedentes médicos de interés, con
correcto seguimiento de la gestación y sin patología de interés durante las primeras 18
semanas, con niveles de beta-HCG dentro de la normalidad, PAPP-A dentro de la
normalidad, glucoproteína b-1 específica de la gestación en valores de normalidad,
detectamos una alfafetoproteína sérica elevada. ¿Cuál es la malformación congénita que
más probablemente detectemos en el feto?

1. 1. Atresia esofágica.
2. 2. Onfalocele.
3. 3. Mielomeningocele.
4. 4. Gastrosquisis.
Gráfico de respuestas
Comentario

La elevación de la alfafetoproteína se relaciona estrechamente con las alteraciones del tubo neural,
como el mielomeningocele (respuesta 3 correcta). El resto de las opciones también producirían
elevación de esta proteína, pero si nos preguntan la malformación congénita más probable,
debemos marcar las de carácter neurológico.(R/3)

!
!
!

30. Sobre la atención inmediata del RN, marque la CORRECTA:

1. 1. El score APGAR determina la conducta a seguir en el RN.


2. 2. La aspiración traqueal siempre es mandataria y obligada.
3. 3. La dosis de vitamina K es: 0.1 mg intramuscular.
4. 4. El APGAR expresa la buena adaptación vital del RN a la vida extrauterina.
Gráfico de respuestas
Comentario

Respuesta 1 falsa. El Apgar no determina la conducta a seguir en el RN.

Respuesta 2 falsa. No siempre habrá que realizar aspiración traqueal. Tiene indicaciones precisas.

Respuesta 3 falsa. La dosis de vitamina K es de 0.5 a 1 mg IM.(R/4)

31. ¿Cuál de los siguientes NO es factor de riesgo de osteoporosis?:

1. 1. Obesidad.
2. 2. Multiparidad.
3. 3. Tabaco.
4. 4. Raza blanca.
Gráfico de respuestas
Comentario

Pregunta muy fácil sobre la osteoporosis en el climaterio y sus factores de riesgo. Se puede
responder de forma directa, señalando como opción correcta la 1, ya que las mujeres con
obesidad, al tener mayor panículo adiposo, poseen mayor cantidad de aromatasa, que es la
enzima encargada de la conversión periférica de los andrógenos en estrona (estrógeno principal en
el climaterio). Por ello, la obesidad puede considerarse incluso como un factor de protección.

La raza blanca está más predispuesta a padecer osteoporosis (opción 4), al igual que las
fumadoras (el tabaco produce hipoestronismo, opción 3), y las multíparas (opción 2) relacionado
con el estado estrogénico.(R/1)

32. ¿Cuál es la vía de diseminación más frecuente del cáncer de ovario?:

1. 1. Implantación directa en cavidad abdominal.


2. 2. Vía linfática.
3. 3. Vía hematógena.
4. 4. Transporte a través del tracto genital.
Gráfico de respuestas
Comentario

La vía de diseminación más frecuente del cáncer de ovario es por contigüidad; en este caso,
implantación directa en la cavidad abdominal.(R/1)

33. Un recién nacido a término nacido por cesárea presenta dificultad respiratoria,
taquipnea e imagen de "redistribución" de los vasos pulmonares, diafragmas aplanados
y líquido en las cisuras. No hay hipoxemia, hipercapnia, ni acidosis; ¿cuál es el proceso
más probable?

!
!
!

1. 1. Enfermedad de la membrana hialina.


2. 2. Neumomediastino.
3. 3. Taquipnea transitoria del recién nacido.
4. 4. Síndrome de Wilson-Mikity.
Gráfico de respuestas
Comentario

La taquipnea transitoria es la modalidad de dificultad respiratoria típica del RN término o casi


término. Para su génesis, se requiere además un desencadenante obstétrico (parto por cesárea o
vaginal rápido), que hace que el tórax del RN no sufra compresión en el canal del parto, con lo que
el líquido pulmonar no sería expulsado. Precisamente, ese líquido pulmonar en la radiología y la
evolución favorable dan la clave para llegar al diagnóstico. Respuesta 3 correcta.(R/3)

34. A 5-year-old badly behaved child is brought to the pediatrician's office by his mother,
because he has very poor school performance. She has noticed that often the boy stares
blankly at the wall for a few minutes. He has a normal physical and neurological
development. What test should be performed next?

1. 1. EEG study.
2. 2. Holter ECG study.
3. 3. Serological blood tests.
4. 4. Lumbar puncture.
Gráfico de respuestas
Comentario
EEG study. The reported symptoms are compatible with absence seizures. Absence seizures are
defined by a brief loss and return of consciousness, generally not followed by a period of lethargy.
In this case, consciousness gaps are responsible for poor school performance. Valproic acid is an
effective treatment. A complete medical history and physical examination is the most important
diagnostic tool, but EEG is also useful.(R/1)

35. Ante una imagen mamográfica de microcalcificaciones agrupadas y moldeadas, sin


tumoración mamaria palpable, ¿cuál es el diagnóstico más probable?:

1. 1. Tumor filodes.
2. 2. Fibroadenoma.
3. 3. Carcinoma intraductal.
4. 4. Carcinoma intralobulillar infiltrante.
Gráfico de respuestas
Comentario

Pregunta sumamente importante para el ENARM. Lo más importante de esta pregunta es que los
signos mamográficos descritos le hagan sospechar un cáncer de mama. La presencia
demicrocalcificaciones agrupadas nos obliga a descartar este diagnóstico, por lo que habrá que
obtener una muestra histológica.

Observe que en el enunciado dicen: “sin tumoración mamaria palpable”. Si hubiese un nódulo
claramente definido, la forma de obtener la muestra podría ser mediante una PAAF… Pero si no se
palpa nódulo alguno, realizar una PAAF sería como puncionar a ciegas. Por ello, en un caso así,
habría que emplear una biopsia guiada con arpón.(R/3)

!
!
!

36. Mujer de 32 años de edad con AP de trastorno bipolar en tratamiento crónico con litio.
Ingresa en psiquiatría con síntomas de fase maniaca, aunque refiere buen cumplimiento
terapéutico. Durante el ingreso se cursa interconsulta a nefrología por poliuria de 7-8
litros al día con polidipsia y nicturia. Exámenes de laboratorio sin alteraciones.
Osmolaridad en orina 140 mosm/l. La sospecha diagnóstica y la actitud más adecuadas
serán:

Diabetes insípida por litio. No precisa pruebas diagnósticas de confirmación; se


1. 1.
debe suspender cuanto antes tratamiento con litio.
2. 2. Potomanía. No precisa otras pruebas diagnósticas ni actitud terapeútica alguna.
Probable tubulopatía por ingesta crónica de litio. Está indicado realizar un test de
3. 3.
deshidratación.
Tubulopatía proximal por litio. Suspender tratamiento con litio tras descartar
4. 4.
potomanía con un test de deshidratación.
Gráfico de respuestas
Comentario
Es importante recordar que la ingesta crónica de litio se encuentra entre las causas más frecuentes
de diabetes insípida nefrogénica adquirida. En este caso concreto y dado que la paciente presenta
descompensación de su trastorno psiquiátrico de base, estaría indicado hacer el diagnóstico
diferencial con la potomanía, haciendo un test de deshidratación. (R/3)

37. A 2-day-old infant has a markedly increased cranial perimeter (30 cm). He weighs 2400
gr and thorough physical examination reveals: multiple bilateral cervical and inguinal
lymphadenopathies, a bulging fontanel, hepatosplenomegaly and opacities in both eyes.
Cardiac auscultation shows a systolic murmur in the second left intercostal space. His
mother had a febrile episode during her first trimester of pregnancy but she did not seek
medical advice. What is the most likely diagnosis?

1. 1. Perinatal CMV infection.


2. 2. Congenital rubella.
3. 3. Birth-acquired herpes.
4. 4. Congenital Toxoplasmosis.
Gráfico de respuestas
Comentario

Ante un neonato que presenta antecedente de retraso del crecimiento intrauterino, hiperplasia
retículo-endotelial (hepatoesplenomegalia, adenopatías), anemia y trombopenia, debemos
considerar una posible infección connatal. Este niño presenta patología ocular (leucocoria,
probablemente por cataratas), cardiopatía (soplo) y microcefalia. De los tres datos que integran la
tríada de Gregg (cardiopatía, cataratas y sordera neurosensorial), nos han mencionado dos, y el
resto del cuadro es compatible, por lo que el diagnóstico más probable corresponde a una rubéola
congénita.(R/2)

38. En el desarrollo de venas varicosas en la extremidad inferior el factor MENOS


importante es:

1. 1. Antecedentes familiares positivos.


2. 2. Arteriosclerosis obliterante.
3. 3. Edad.
4. 4. Sexo.
Gráfico de respuestas
!
!
!

Comentario

Pregunta de dificultad moderada.

Estas venas pueden ser primarios o secundarias.

Las primarias resultan de un desarrollo defectuoso de las paredes venosas con debilidad congénita
de las mismas.

Las varices secundarias se producen por insuficiencia venosa profunda y por incompetencia de las
venas perforantes.

La etiología del proceso varicoso no está clara, aunque se conocen una serie de factores que
predisponen y favorecen la aparición de las mismas. El factor más importante es la postura.
Aproximadamente la mitad de los pacientes con varices tienen antecedentes familiares. Otros
factores son aquellos que dificultan el flujo del retorno venoso como el embarazo, la obesidad o las
masas tumorales. Las varices aumentan con la edad y son mucho más frecuente en mujeres.(R/2)

39. Un niño acude a consulta de control al mes de edad y tiene las siguientes medidas:
talla 52 cm, peso 4 Kg y perímetro cefálico 37 cm, correspondiendo dichas medidas al
percentil 40. Si el niño recibe lactancia materna exclusiva y es un niño sano, usted
esperaría en el siguiente control mensual:

1. 1. Que solo la talla persista en el percentil 40.


2. 2. Que el peso suba al percentil 90.
3. 3. Que todas sus medidas continúen en el percentil 40.
4. 4. Ninguna de las anteriores.
Gráfico de respuestas
Comentario

Es un paciente que se encuentra dentro de la normalidad, por lo que en el control mensual


siguiente continuará en dicho percentil.(R/3)

40. Una paciente de 30 años, con un cuadro clínico de sangrado menstrual irregular,
dismenorrea, dispareunia y una esterilidad de 3 años de evolución, presenta exámenes
de laboratorio hormonales normales y un ultrasonido transvaginal que informa de un
útero normal y múltiples formaciones quísticas ováricas bilaterales de 4 cm con signos
de sospecha. Teniendo en cuenta el diagnóstico más probable, ¿cuál de las siguientes
es de elección para obtener el diagnóstico de sospecha?:

1. 1. Ultrasonido abdominal.
2. 2. TC abdominopélvica.
3. 3. Laparoscopia.
4. 4. Culdocentesis.
Gráfico de respuestas
Comentario

Ante un posible caso de endometriosis, dada la edad de la paciente y la sintomatología, en cuyo


estudio, no tenemos las cosas del todo claras, con unos datos hormonales que no están alterados,

!
!
!

lo mejor que podemos plantear es la laparoscopía exploradora, para realizar un diagnóstico de


certeza y podremos tomar biopsias para un posterior estudio anatomopatológico. (R/3)

41. Paciente con antecedente de alcoholismo que ingresa en urgencias por melenas de 5
días de evolución. En la endoscopía se observan várices esofágicas y gástricas grado III
con signos de hemostasia reciente. En un ultrasonido abdominal se aprecia una masa en
cabeza de páncreas compatible con pancreatitis crónica que se confirma en el TAC. Un
estudio de las presiones portales demuestra que existe hipertensión portal no
dependiente del hígado. Para evitar nuevos episodios de sangrado digestivo,
recomendaría:

1. 1. Propranolol.
2. 2. TIPS.
3. 3. Resección pancreática.
4. 4. Derivación portosistémica quirúrgica.
Gráfico de respuestas
Comentario

Nos encontramos ante el caso clínico de un paciente con antecedentes de alcoholismo, sin
diagnostico previo de hepatopatía o pancreatitis crónica, que debuta con un episodio de
hemorragia digestiva alta y en cuyo estudio descubrimos la existencia de várices esofágicas y
gástricas. Lo habitual sería atribuir el origen de éstas a una hepatopatía evolucionada con
hipertensión portal. Sin embargo, el estudio posterior demuestra que ésta no depende del hígado y,
por otra parte, existe una masa en cabeza de páncreas que provoca hipertensión portal
(probablemente por compresión a nivel de la porta). Es decir, tenemos una causa orgánica que
justifica el origen de la hipertensión portal, y que por otro lado no podemos descartar que sea una
neoplasia. Así pues, nos decidiremos por una técnica quirúrgica resectiva.(R/3)

42. Una de las siguientes entidades se asocia a fibrosis retroperitoneal, mediastínica y


colangitis esclerosante:

1. 1. Tiroiditis de Riedel.
2. 2. Tiroiditis de Hashimoto.
3. 3. Carcinoma papilar de tiroides.
4. 4. Carcinoma medular de tiroides.
Gráfico de respuestas
Comentario
Las tiroiditis son procesos en los que existe una inflamación de la glándula. En muchas de ellas el
patrón funcional es de una primera fase de hipertiroidismo por liberación de hormonas tiroideas
preformadas al torrente sanguíneo y una segunda fase de hipotiroidismo que puede ser
autolimitada. La tiroiditis fibrosante o de Riedel consiste en una fibrosis del cuello que puede
afectar al mediastino y al retroperitoneo de forma concomitante, en la que la función tiroidea suele
ser normal, existiendo hipotiroidismo hasta en el 25% de los casos, pero el hipertiroidismo no está
descrito en esta entidad.(R/1)

43. Una niña de 1 año de edad presenta fiebre alta, vómitos y llanto al miccionar, por lo
que se sospecha infección de vías urinarias. En este caso la decisión inicial más
adecuada es:

1. 1. Solicitar examen completo de orina tomado con bolsa colectora.

!
!
!

2. 2. Pedir urocultivo tomado mediante cateterismo vesical o punción suprapúbica.


3. 3. Indicar ultrasonido de vías urinarias.
4. 4. Iniciar tratamiento antibiótico.
Gráfico de respuestas
Comentario

Ante la sospecha alta de infección de orina, como en este caso, se puede obviar el sedimento de
orina con muestra procedente de bolsa y hacer directamente el sondaje, ya que si el sedimento por
bolsa sale alterado, lo que es muy probable, la tendremos que sondar igualmente, y así ganamos
tiempo ante una niña con fiebre alta y vómitos, por lo tanto con bastante malestar.(R/2)

44. Entre las contraindicaciones ABSOLUTAS del tratamiento fibrinolítico en el contexto


del infarto agudo de miocardio, NO estaría incluido:

1. 1. Antecedentes de hemorragia intracraneal.


2. 2. Hipertensión arterial.
3. 3. Ictus isquémico durante los tres últimos meses.
4. 4. Infarto agudo de miocardio debido a disección aórtica.
Gráfico de respuestas
Comentario
Contraindicaciones absolutas de tratamiento fibrinolítico: 1. Hemorragia activa (excluida la
menstruación). 2. Cualquier hemorragia intracraneal previa. 3. Ictus isquémico en los últimos 3
meses. 4. Lesión cerebrovascular estructural conocida o neoplasia intracraneal. 5. Sospecha de
disección aórtica. La hipertensión arterial es una contraindicación relativa, porque podemos
intentar controlarla previamente al tratamiento de reperfusión.(R/2)

45. ¿Cuál es el agente causal más frecuente de bronquiolitis en los lactantes?:

1. 1. Virus sincitial respiratorio.


2. 2. Virus de la gripe.
3. 3. Streptococcus penumoniae.
4. 4. Adenovirus.
Gráfico de respuestas
Comentario

Esta pregunta sobre la etiología de la bronquiolitis es muy sencilla e importante.

Recuerde que el virus respiratorio sincitial es el agente causal principal , aunque otros virus como
el virus de la parainfluenziae, adenovirus, influenza, rinovirus y, de forma infrecuente, M.
pneumoniae, se han asociado a esa enfermedad.(R/1)

46. Embarazada de 37 semanas a la que se le practica monitorización fetal no estresante


con el siguiente resultado: frecuencia cardíaca fetal 135 lpm, ondulatoria normal,
movimientos fetales escasos, sin ascensos en la frecuencia cardíaca y sin
deceleraciones. ¿Qué actitud tomaría?

1. 1. Inducción del parto.


2. 2. Amnioscopía.
3. 3. Amniocentesis tardía.
4. 4. Prueba de Pose.

!
!
!

Gráfico de respuestas
Comentario

Para valorar el bienestar fetal en el tercer trimestre (37 semanas de gestación tenemos varias
opciones, pero en este caso la más acertada será realizar un RCTG estresante o prueba de Pose.

•! No estaría indicado una inducción de parto ni una cesárea urgente, dado que todavía no
queda claro si existe sufrimiento fetal.
•! No estaría indicado realizar una amnioscopía dado que existe riesgo de rotura de
membranas y en principio no tenemos intención de inducir el parto.
•! La amniocentesis tardía se suele realizar en la semana 32 para medir la madurez pulmonar
fetal, medir la bilirrubina en sospecha de isoinmunización Rh, evacuadora en caso de
polihidramnios o amnioinfusión en oligoamnios o tratamiento de la transfusión feto-fetal de
gemelares. Este tipo de amniocentesis no se usa por tanto para el diagnóstico
prenatal.(R/4)

47. Mujer de 70 años que desde hace tres meses presenta vómitos y dolor abdominal que
no cede con la comida. Se acompañaba de pérdida de peso y anorexia. A la EF: palidez
mucocutánea y ligero dolor a la palpación epigástrica. En los exámenes de laboratorio se
detecta Hb 8.9 g/L, VCM 70, amilasa 45 u/L. El test del aliento para H. pylori fue positivo.
En el ultrasonido abdominal se observaron adenopatias en territorio gástrico. Para esta
paciente, ¿cual de las siguientes pautas recomendaría en primer lugar?:

1. 1. Gatroduodenoscopia.
2. 2. Pantoprazol 8 semanas y posterior evaluación.
3. 3. PAAF de adenopatias dirigido por TAC.
4. 4. Anticuerpos anti H. pylori.
Gráfico de respuestas
Comentario

En este caso, con un síndrome general evidente, dolor en epigastrio y anemia microcítica, la
primera sospecha diagnóstica debe ser una tumoración gástrica. Por tanto, es necesaria una
prueba de imagen (gastroduodenoscopia) y, posteriormente, una confirmación histológica, con
biopsia de la lesión. Tanto por la edad de la paciente, como por la positividad del test del aliento
para Helicobacter pylori, lo más probable es que se trata de un adenocarcinoma gástrico de tipo
intestinal.(R/1)

48. Femenino de 60 años de edad, que presenta sangrado genital escaso por orificio
cervical, con ecografía que indica hiperplasia endometrial. ¿Cuál será la conducta más
apropiada?

1. 1. Reevaluación ecográfica.
2. 2. Histeroscopia - biopsia.
3. 3. Laparotomía.
4. 4. Histerectomía abdominal.
Gráfico de respuestas
Comentario

!
!
!

La hiperplasia endometrial es un trastorno que consiste en la proliferación del endometrio por


acción de los estrógenos sin el efecto compensador de la progesterona. Es una enfermedad que
se da fundamentalmente en mujeres con ciclos anovulatorios.

El diagnóstico es anatomopatológico. Ante la sospecha de patología endometria por la presencia


de clínica (metrorragia perimenopáusica o posmenopáusica) o por hallazgos ecográficos
(engrosamiento endometrial mayor de 5 mm en mujeres posmenopáusicas o superior a 12 mm en
mujeres premenopáusicas), es obligatorio obtener matieral para estudio anatomopatológico.

El método diagnóstico de elección es la histeroscopia-biopsia, ya que ofrece la ventaja de


visualizar la cavidad a la hora de realizar la biopsia. Otros métodos también útiles son el legrado
fraccionado o las cánulas flexibles tipo Cornier o Pipelle.

Entre las opciones disponibles, la más adecuada es la opción 2, siendo la 3 y la 4 demasiado


invasivas sin haber obtenido un diagnóstico previo.(R/2)

49. En cuanto al manejo del recién nacido prematuro, señale lo INCORRECTO:

El uso de bicarbonato en prematuros extremos condiciona la presentación de


1. 1.
hemorragia intracerebral.
Los recién nacidos prematuros requieren menores volúmenes de líquidos para
2. 2.
infusión intravenosa que los recién nacidos a termino.
3. 3. Una natremia de 150 mEq/L sugiere disminuir la infusión de líquidos.
4. 4. Todos los recién nacidos de bajo peso son prematuros.
Gráfico de respuestas
Comentario

No todos los RN con bajo peso son prematuros

Se ha asociado a la hemorragia intraventricular con el uso de bicarbonato, por lo que la opción 1


no la debe de seleccionar. Por obvias raones en una hipernatremia el tratamiento es diinuir la
infusión de suero fisiológico e iniciar glucosado o al 0.45%.(R/4)

50. Paciente de 34 años con antecedentes personales de cólicos biliares que se encuentra
en el 7º mes de gestación y que acude a urgencias por presentar dolor abdominal
localizado en cuadrante superior derecho de aproximadamente 24-36 h de evolución, tras
el cual ha presentado dos vómitos, y que actualmente presenta fiebre de 38ºC. Tras ser
valorada por Ginecología sin apreciarse patología ginecológica es remitida a su servicio
de urgencias. ¿Qué actitud diagnóstica seguiría?:

1. 1. Exploración física. Exámenes de sangre y Rx de abdomen.


2. 2. Exploración física. Exámenes de sangre y orina y Rx de abdomen.
3. 3. Exploración física. Exámenes de sangre y orina y ultrasonido abdominal.
4. 4. Exploración física y Exámenes de sangre y orina.
Gráfico de respuestas
Comentario

!
!
!

Pregunta deducible, que debe contestar porque aunque dude con el diagnóstico hay varias
opciones que puede descartar y por tanto: hay que arriesgarse!. Para contestar bien esta pregunta
debe recordar que la urgencia quirúrgica más frecuente en embarazadas es la apendicitis aguda.
No le debe despistar la localización del dolor, ya que a medida que progresa la gestación el
apéndice se localiza en una posición más alta y más lateral, lo cual puede dificultar el diagnóstico.
La apendicitis es un diagnóstico fundamentalmente clínico y se apoya en exámenes de sangre (en
la que lo más frecuente es encontrar una leucocitosis con desviación izquierda). En caso de una
mujer embarazada, por ser el diagnóstico dudoso suele estar indicado la realización de una técnica
de imagen como es un ultrasonido que es inocuo tanto para la madre como para el feto. La Rx de
abdomen y el TAC NO estan indicada en embarazadas.(R/3)

51. Cuál de los siguientes enunciados, NO es criterio para considerar que un neonato
presenta ictericia fisiológica:

1. 1. Incremento diario de bilirrubina > 5 mg/dl.


El nivel máximo de bilirrubina en el RNT se da entre el 2-3 día de vida y en el
2. 2.
RNPT entre el 4-6 día de vida.
3. 3. La bilirrubina indirecta no debe llegar en ningún momento a ser > 15 mg/dl.
4. 4. Una de las causas es el aumento de la circulación enteropática de bilirrubina.
Gráfico de respuestas
Comentario

Las características de la ictericia no fisiológica son las siguientes :

Inicio en las primeras 24 horas de vida.

Duración superior a 10-15 días.

Bilirrubina total mayor de 12 mg/dl en RNT o mayor de 15 mg/dl en RNPT.

Incremento de la bilirrubin a superior a 5 mg/dl/24 horas.

Bilirrubina directa mayor a 1 mg/dl o superior al 20% de la bilirrubina total.(R/1)

52. Referente a la crioglobulinemia mixta esencial. Todo es cierto, EXCEPTO:

1. 1. Se relaciona con crioglobulinas IgG e IgM monoclonal.


2. 2. Puede ocasionar GN rápidamente progresiva.
3. 3. Los niveles de complemento son normales.
4. 4. La erradicación del proceso infeccioso es de utilidad en el tratamiento.
Gráfico de respuestas
Comentario

Esta pregunta es de dificultad media. La opción 1 es correcta y no debe dudarla. La clínica renal
aparece en un 50% de los casos en forma de hematuria y/o proteinuria, aunque también puede
ocasionar una GN rápidamente progresiva (opción 2). La opción 4 es correcta porque se ha
demostrado que el tratamiento de la hepatitis puede ayudar al tratamiento de la vasculitis. Por
último la opción 3 es INCORRECTA porque precisamente los niveles de complemento suelen estar
descendidos por la existencia de inmunocomplejos con IgG e IgM que lo activan.(R/3)

!
!
!

53. A 71-year-old woman presents to her physician with a 4-month history of asthenia and
arthralgia and a 2-month history of nasal congestion and a 2-week history of cough. She
is especially worried now because she has been coughing up blood-tinged sputum for
the last 4 days. These symptoms are not accompanied by fever, nausea or vomiting. She
has hypertension and migraine. Over the past 4 weeks, she has been taking over-the-
counter NSAIDs for her arthralgia. Her pulse is 90/min, breathing frequency are 12/min
and blood pressure is 140/83 mm Hg. On physical examination, the patient appears
fatigued, with diffuse tenderness in her joints which are not warm or erythematous. A
clear nasal discharge is also noted, with no other significant findings. Lab tests show:
Hematocrit 32%, Mean corpuscular volume 82 μm3, Leukocyte count 15,100/mm3,
Segmented neutrophils 71%, Eosinophils 2%, Lymphocytes 15%, Monocytes 12%,
Platelet count 325,000/mm3, Serum Urea nitrogen 32 mg/dL, Creatinine 3.4 mg/dL,
Antinuclear antibodies 1:256, Rheumatoid factor negative, Antineutrophil cytoplasmic
antibodies positive. Urinalysis shows: Blood 3+, Protein 3+, RBC 15–19/hpf, WBC 1–5/hpf,
RBC casts rare. Which of the following is the most likely causative mechanism of this
patient's renal failure?

1. 1. Tuberculosis.
2. 2. Interstitial nephritis.
3. 3. Hypertension.
4. 4. Vasculitis.
Gráfico de respuestas
Comentario
Vasculitis. The combination of pulmonary (blood-tinged sputum), upper airway, arthralgia and
kidney failure in a patient with anti-neutrophil cytoplasm antibodies (ANCAs) is suggestive of
VASCULITIS, particulary GRANULOMATOSIS WITH POLYANGIITIS a.k.a WEGENER'S DISEASE
(if the antibodies react against proteinase= c-ANCA) or CHURG STRAUSS (if the antibodies are p-
ANCA). Remember that the presence of such antibodies is not always detected in patients with
these vasculitides.(R/4)

54. Estamos estudiando a una paciente de 45 años, con antecedente personal de vitíligo,
que en un control de laboratorio de rutina ha presentado cifras de GOT de 75 U/ml y GPT
de 102 U/mL; inmunoglobulina G 1,800mg/dL; serología para hepatitis víricas tipo B y C
negativas. La paciente no refiere hábitos tóxicos. Se realiza estudio de anticuerpos que
nos pone sobre la pista de una hepatitis autoinmune, que se confirma mediante biopsia.
En relación con la hepatitis autoinmune, indique cuál de las siguientes le parece
CORRECTA:

1. 1. Es una enfermedad benigna con escasa morbilidad.


La hepatitis tipo I, además de presentarse con títulos de anticuerpos antinucleares
2. 2.
elevados, presenta serología positiva para el virus C.
3. 3. Los autoanticuerpos anti-LKM-2 se presentan en el tipo II.
Los mecanismos inmunitarios humorales parecen los responsables de las
4. 4.
manifestaciones extrahepáticas de la enfermedad.
Gráfico de respuestas
Comentario

La hepatitis autoinmune tipo 1 cursa con ANA y anti-músculo liso positivos. En la tipo 2, aparecen
anticuerpos anti- LKM 1 y anti-citosol hepático 1. No obstante, aunque no conozca estos detalles,
la respuesta 4 resulta bastante lógica. Si se trata de una enfermedad autoinmune en la que se

!
!
!

implican autoanticuerpos, resulta lógico pensar que las manifestaciones a distancia tendrán alguna
relación con este mecanismo.(R/4)

55. Which of the following statements is CORRECT in relation to iron metabolism?

1. 1. Iron absorption occurs in the ileum.


2. 2. Most of the serum iron is bound to ferritin.
3. 3. Food intake worsens absorption of oral iron.
4. 4. Approximately 20% of the ingested iron is absorbed.
Gráfico de respuestas
Comentario

La absorción del hierro se realiza en el duodeno y los tramos proximales del yeyuno, y se ve
favorecida por el medio ácido. De modo que, con la digestión de los alimentos que aumenta el pH
o con fármacos antiulcerosos, la absorción está disminuida. Además, la proporción de hierro
procedente de la dieta que se absorbe es de un 10%.(R/3)

56. ¿Cuál de los siguientes esteroides tópicos es el de mayor potencia?:

1. 1. 17 alfa hidrocortisona.
2. 2. Propionato de clobetasol.
3. 3. Butirato-propionato de fluometasona.
4. 4. Valerato de Betametasona.
Gráfico de respuestas
Comentario
Pregunta compleja, de estudio en profundidad de Segunda Vuelta. Los Esteroides Tópicos, en
Dermatología, se dividen en tres grupos según su potencia. Muy potentes: tratamientos en palmas
y plantas y dermatosis crónicas, hiperqueratósicas. El más potente de todos es el Propionato de
clobetasol (respuesta 3 correcta) y otros menos potentes, también incluidos en este grupo, como el
Dipropionato de beclometasona. Moderadamente potentes: los más utilizados. Por ejemplo,
Butirato- propionato de fluometasona, Valerato de betametasona, Acetónido de flucinolona. Poco
potentes: utilizados en niños y zonas de piel fina, cara, pliegues. El prototipo es la
Hidrocortisona.(R/2)

57. After slight pain in his right eye for two days, an otherwise asymptomatic 28-year-old
man comes to the emergency department. He bought steroids eye drops at the pharmacy
because his eye itched, using it as required during the previous week. After staining with
fluorescein, examination reveals a branch-like lesion in lower central cornea. Which of
the following is the most appropriate treatment for this patient’s eye lesion?

1. 1. Maintain steroid eye drops treatment until symptoms cease.


2. 2. Add pilocarpine eye drops to the treatment with steroids.
3. 3. Start acyclovir eye drops therapy.
Suspend treatment with steroids. Start with acyclovir and pilocarpine eye drops
4. 4.
therapy.
Gráfico de respuestas
Comentario

!
!
!

La presencia de una imagen fluoropositiva de forma ramificada es muy sugestiva de que el


paciente presente una queratitis herpética. Esta posibilidad se ve muy reforzada por el hecho de
que el paciente se haya aplicado un corticoide tópico durante los días previos. Los corticoides por
su efecto inmunosupresor pueden favorecer la reactivación del virus herpes, y por ello están
contraindicados en esta paciente. También está contraindicada la pilocarpina porque al inducir un
espasmo del músculo esfínter de la pupila, acentúa el dolor. (R/3)

58. En una TAC abdominal realizada a una paciente de 42 años para estudio de un
traumatismo se detecta una imagen de masa renal derecha de 2.5 cm de diámetro máximo
y aspecto graso. Refiere encontrarse asintomática. El resto de la TAC es estrictamente
normal. Señale la FALSA:

Puede aparecer de forma esporádica, lo cual es cuatro veces más probable en las
1. 1.
mujeres.
Surge en la esclerosis tuberosa, en cuyo caso es múltiple, bilateral, de mayor
2. 2.
tamaño y con probabilidad de causar hemorragias espontáneas.
3. 3. La biopsia es fundamental en el diagnóstico.
Las principales complicaciones del AML renal son la hemorragia retroperitoneal
4. 4.
y la hemorragia en el sistema colector urinario.
Gráfico de respuestas
Comentario
El angiomiolipoma (AML) es un tumor mesenquimatoso benigno. Representa en torno al 1 % de los
tumores extirpados quirúrgicamente. La ecografía, TC y RM confirman a menudo el diagnóstico
debido a la presencia de tejido adiposo. La biopsia rara vez resulta útil. El resto de las opciones
otorgan datos fundamentales para el estudio.(R/3)

59. Señale la respuesta FALSA respecto a la fisiología renal:

Aclaramiento de una sustancia es el volumen de plasma que se limpia de una


1. 1.
sustancia determinada al pasar por el riñón en la unidad de tiempo.
2. 2. Es normal una proteinuria de hasta 150 mg/día.
3. 3. La aldosterona actúa fundamentalmente en el túbulo proximal.
4. 4. Una FENa menor de 1 indica fracaso renal funcional.
Gráfico de respuestas
Comentario

Esta pregunta entraña una cierta complejidad, pero debe dominar todos estos conceptos para
defenderse con soltura ante las preguntas de nefrología. Analicemos cada una de las opciones:

1. La definición de aclaramiento de una sustancia corresponde a lo que dice el enunciado.

2. La proteinuria tiene esos límites antes de ser patológica: 150 mg/día.

3. Intenta aprender la localización de los principales transportadores de iones en la nefrona, tanto


para entender el funcionamiento de ciertos diuréticos como la fisiopatología de algunas
enfermedades, las tubulares hereditarias y otras. Si es necesario, haga un dibujo de una nefrona y
píntelos. Es en el túbulo distal donde actúan tanto la ADH como la aldosterona, regulando la
volemia y el agua eliminada. La aldosterona estimula un transportador que reabsorbe Na+ y
elimina K+ e H+, ése es el transportador que está alterado en la enfermedad de Liddle.

!
!
!

4. La excreción fraccional de sodio (aclaramiento de sodio dividido entre el de creatinina ) menor


de uno indica que el fracaso renal es funcional, en esos casos el organismo apenas aclara sodio,
intenta reabsober todo el sodio posible para así aumentar la volemia.(R/3)

60. ¿Qué actitud adoptaría ante un paciente ingresado por presentar un infarto de
miocardio y que presenta una taquicardia ventricular sostenida, aunque sin deterioro
hemodinámico?:

1. 1. Amiodarona.
2. 2. Masaje cardíaco.
3. 3. Bretilio.
4. 4. Verapamil i.v.
Gráfico de respuestas
Comentario

Se trata de una arritmia ventricular primaria. Si produjera deterioro hemodinámico, al tratarse de


una situación de riesgo vital se debería proceder a una desfibrilación eléctrica. Como el estado
hemodinámico es bueno, se prefiere intentar primero el tratamiento farmacológico con amiodarona.
En caso de que no fuese eficaz, se procedería a la cardioversión eléctrica.(R/1)

61. Un paciente portador de una prótesis mitral metálica va a someterse a una resección
prostática. ¿Cuál de las siguientes le parece la profilaxis más adecuada?:*

1. 1. Ampicilina + gentamicina.
2. 2. Ampicilina.
3. 3. Amoxicilina + gentamicina.
4. 4. Ampicilina + metronidazol.
Gráfico de respuestas
Comentario

En esta pregunta es importante saber que, entre los pacientes que precisan profilaxis de la
endocarditis bacteriana ante determinados procesos, se incluyen los portadores de cardiopatías
adquiridas, y dentro de ellos, están los que llevan prótesis intracardiacas como la prótesis mitral de
este caso clínico. En segundo lugar hay que recordar que la profilaxis en procedimientos
genitourinarios es con ampicilina 2g i.v. y gentamicina 1.5mg/kg de peso i.v o i.m. 30 min antes del
procedimiento, seguidos de ampicilina 1g, 6 horas después. Para los procesos odontológicos
recuerda que la profilaxis se realiza con ampicilina.

Aunque recientemente se han publicado las guías americanas, en las que ya no se recomienda
profilaxis antimicrobiana en procedimientos gastrointestinales, ni genitourinarios.(R/1)

62. La enterotoxina colérica es responsable del cuadro diarreico característico de esta


enfermedad. ¿Cuál es su mecanismo de acción?:

1. 1. Efecto citotóxico directo sobre el enterocito.


2. 2. Provocando la activación continua de la adenilato ciclasa.
3. 3. Por activación del GMP cíclico intracelular.
Provocando la liberación de citoquinas por las células del sistema inmune
4. 4.
intestinal.
Gráfico de respuestas
Comentario

!
!
!

Pregunta directa sobre el mecanismo fisiopatológico de producción de diarrea del Vibrio cholerae.
Vibrio cholerae, agente causal del cólera, es una bacteria productora de enterotoxinas, las cuales
actúan directamente sobre la superficie de los enterocitos, sin destrucción de la mucosa, alterando
el intercambio iónico y favoreciendo el paso de agua libre hacia la luz intestinal (a través de la
activación continua de la adenilato ciclasa) por eso genera tanta deshidratación para ocasionar una
diarrea acuosa sin productos patológicos y sin leucocitos al microscopio.(R/2).

63. Ante un paciente con demencia de reciente comienzo y una serología treponémica
positiva en sangre, ¿qué prueba complementaria considera esencial para establecer el
diagnóstico de neurosífilis?

1. 1. EEG.
2. 2. RM craneal.
3. 3. TC craneal con contraste.
4. 4. Análisis del LCR.
Gráfico de respuestas
Comentario

El VDRL, que es una prueba serológica no treponémica, se utiliza para diagnosticar la neurosífilis.
Lógicamente, el estudio se realizará en LCR. Este examen consiste en buscar determinados
anticuerpos, llamados reaginas, en el líquido cefalorraquídeo. Cuando se trata de una sífilis
primaria o secundaria, esta prueba sería negativa. Sin embargo, en una neurosífilis, el resultado
sería positivo.

Recuerde que las pruebas serológicas para la sífilis se dividen en dos grupos:

- Reagínicas: VDRL y RPR, menos sensibles y específicas. Pueden negativizarse con la


curación de la enfermedad, por lo que tienen gran valor para el seguimiento.

- Treponémicas, como el FTA- Abs. Su sensibilidad y especificidad son mayores. Sin


embargo, una vez que son positivas, ya nunca negativizan. Por ello, no tienen valor para el
seguimiento de un paciente, ni para diagnosticar una reinfección.(R/4)

64. La operación de Whipple consiste en la exéresis de:

Parte distal del estómago, duodeno, cabeza de páncreas, vesícula, vía biliar distal
1. 1.
y yeyuno proximal.
2. 2. Vesícula, vía biliar distal y cabeza de páncreas.
3. 3. Vesícula, vía biliar distal y parte distal gástrica.
4. 4. Vesícula, vía biliar proximal y distal y cabeza de páncreas.
Gráfico de respuestas
Comentario
Se trata de una pregunta específica sobre técnica quirúrgica y en concreto sobre la intervención de
Whipple o duodenopancreatectomía cefálica, que es la técnica quirúrgica de elección con intención
curativa indicada en el cáncer de cabeza de páncreas. En esta técnica se resecan el antro gástrico,
todo el marco duodenal y unos cms del yeyuno proximal en bloque junto con la cabeza del
páncreas que se secciona a nivel del llamado cuello pancreático sobre la porta y que incluye la vía
biliar principal distal, es decir el colédoco distal. Como gesto añadido se debe resecar la vesícula
biliar, lo cual es mandatorio siempre que se practica una intervención en la vía biliar principal. Así

!
!
!

pues debemos reconstruir haciendo tres anastomosis a saber: 1) anastomosis entre el conducto de
Wirsung y el tubo digestivo 2) anastomosis entre la vía biliar y el tubo digestivo y 3) reconstrucción
del tránsito digestivo mediante anastomosis entre el estómago y el tubo digestivo. En general todas
estas anastomosis se realizan sobre un mismo asa de yeyuno.(R/1)

65. Una paciente, diagnosticada de infección crónica por VHB en fase replicativa y en
tratamiento con interferón alfa-2b, consulta porque, a las ocho semanas de iniciado el
tratamiento, está más cansada y un estudio de laboratorio demuestra que hay un ligero
aumento de la bilirrubina y de las transaminasas. ¿Qué consideraciones se haría?

1.! ! 1.! Probablemente!tiene!una!hepatitis!de!otra!causa.!


2.! ! 2.! Se!debe!añadir!esteroides!al!tratamiento.!
3.! ! 3.! Probablemente!está!haciendo!la!seroconversión!anti<HBe.!
4.! ! 4.! Probablemente!el!paciente!no!se!está!poniendo!el!interferón.!
Gráfico de respuestas
Comentario

En la hepatitis crónica B en fase replicativa, el objetivo del tratamiento con IFN alfa no es la
negativización del virus (esto sucede en muy pocos casos), sino el paso de fase replicativa a no
replicativa. Cuando esto ocurre, se debe a que el IFN alfa ha conseguido potenciar la respuesta
inmune del paciente frente al virus, siendo ésta suficiente para pasar conseguir el objetivo
propuesto. Esto es lo que se llama seroconversión anti-HBe, y clínicamente suele acompañarse de
un aumento transitorio de la bilirrubina, transaminasas y ligero cansancio. Es como una hepatitis
aguda, pero más leve, traduciendo la potenciación de la respuesta inmunológica frente al VHB, con
la consiguiente inflamación transitoria a nivel hepático.(R/3)

66. Paciente de 34 años con esterilidad


primaria de 3 años de evolución sometida a tratamiento inductor de la ovulación para
ciclo de Fertilización In Vitro a la que se realizó 24 horas antes la punción folicular bajo
control ultrasonográfico sin incidencias. Acude a Urgencias por presentar dolor pélvico
severo, postración, mareo y aumento del perímetro abdominal. El USG ginecológico es
el que se muestra en la imagen. Señale la afirmación CORRECTA acerca del cuadro que
presenta la paciente:

Se trata de una estimulación ovárica normal: reposo, analgesia, control diuresis y


1. 1.
transferencia embrionaria en 48 horas.

!
!
!

Se trata de un síndrome de hiperestimulación ovárica: reposo, analgesia y


2. 2.
transferencia embrionaria en 48 horas.
Se trata de un síndrome de hiperestimulación ovárica: reposo, analgesia, control
3. 3.
de diuresis y cancelación transferencia embrionaria.
Se trata de un síndrome de hiperestimulación ovárica: laparotomía exploradora y
4. 4.
punción folicular.
Gráfico de respuestas
Comentario

El síndrome de hiperestimulación ovárica es una posible complicación de la fecundación in vitro.


Se trata de una respuesta excesiva del ovario a la inducción de la ovulación, que desencadena una
extravasación de líquido. Las pacientes con síndrome de ovario poliquístico tienen más riesgo de
hiperestimulación. Suele ser un cuadro clínico que se desencadena tras la administración de hCG
y se resuelve en 1- 2 semanas, cursando con distensión abdominal, grandes ovarios llenos de
folículos, dolor abdominal, vómitos, diarrea y ascitis. Puede llegar a ser grave y provocar fracaso
renal, tromboembolismos, SDRA y poner en riesgo la vida de la paciente.

En los casos leves-moderados, el tratamiento consiste en reposo, hidratación y control de las


constantes vitales. En los casos graves puede llegar a requerir una laparotomía urgente, en la que
intentaremos ser lo más conservadores posible. Se debe evitar la gestación cancelando las
inseminaciones o criopreservando los embriones en el caso de pacientes sometidas a fecundación
in vitro.(R/3)

67. En el estudio de esterilidad de la paciente del caso clínico anterior indique cuál de las
siguientes pruebas NO se considera fundamental:

1. 1. Determinación de FSH, LH y estradiol en fase folicular.


2. 2. Ultrasonido ginecológico transvaginal.
3. 3. Histerosalpingografía.
4. 4. Biopsia de endometrio en fase lútea.
Gráfico de respuestas
Comentario

En todo estudio de esterilidad, se deben realizar las pruebas siguientes:

- Anamnesis, exploración y estudio de sangre (BH, bioquímica, orina, Rh y serologías frente a


rubéola, toxoplasma, sífilis, VHB, VHC y VIH).

- USG transvaginal, que nos informa sobre posibles alteraciones morfológicas del útero, anexos,
patología endometrial, endometriosis, ovarios poliquísticos, etc.

- Valoración de la ovulación, midiendo FSH, LH y estradiol en fase proliferativa (3-5 días del ciclo).
Entre el día 20-22 (fase lútea), se medirían PRL y progesterona.

- Seminograma y recuperación de espermatozoides móviles.

- Histerosalpingografía.

Existen otras pruebas que se realizan de forma puntual, cuando existen determinadas sospechas.
Concretamente, la biopsia de endometrio se utiliza cada vez menos y en este caso no existen
razones para realizarla, al menos no de entrada.(R/4)

!
!
!

68. Señale la afirmación ERRÓNEA en relación con el cáncer de vulva:

1. 1. Son factores de riesgo el tabaquismo y la infección por VPH.


2. 2. La localización más frecuente es en labios mayores.
3. 3. El síntoma principal es la hemorragia.
4. 4. La edad de aparición más frecuente es alrededor de los 70 años.
Gráfico de respuestas
Comentario

El síntoma principal del cáncer de vulva NO es la hemorragia, sino el prurito vulvar que tiene una
evolución larga. Generalmente ante cuadros de prurito vulvar de larga evolución, generalmente en
los labios mayores, en mujeres mayores, debemos sospechar cáncer de vulva.(R/3)

69. En pacientes con episodio de metrorragia disfuncional severa, la conducta


terapéutica inmediata es:

1. 1. Histerectomía.
2. 2. Legrado uterino.
3. 3. Administración de oxitócicos por la vía IV.
4. 4. Transfusión sanguínea total fresca.
Gráfico de respuestas
Comentario

La clave en esta prgunta es una metrorragia disfunciónal SEVERA, la conducta a seguir sería el
legrado uterino, respuesta 2, ya que de entrada realizar histerectomía es un tratamiento invasivo
que podríamos utilizarlo en caso que no funcione el legrado.(R2)

70. If a patient with vasculitis presents with saddle nose deformity, which of the following
options is the most likely diagnosis?

1. 1. Polyarteritis nodosa.
2. 2. Henoch-Schönlein purpura.
3. 3. Wegener's granulomatosis.
4. 4. Allergic granulomatosis.
Gráfico de respuestas
Comentario

La deformidad en silla de montar dentro de las vasculitis es propia de la enfermedad de Wegener,


que produce una destrucción del tabique nasal.

Con esta enfermedad debe realizarse diagnóstico diferencial con otras enfermedades que
producen destrucción de la línea media, como el granuloma de la línea media (proceso que forma
parte del grupo de las enfermedades inmunoproliferativas angiocéntricas), neoplasias de las vías
respiratorias superiores y linfomas extranodales, infecciones por hongos, micobacterias o
actinomicosis y la policondritis recidivante.

La biopsia y los cultivos demuestran el diagnóstico, aunque en algunos casos no se observa


vasculitis, y, en ese caso, el resto de la clínica y analítica debe orientarnos al diagnóstico. En el
resto de vasculitis que dan como opciones posibles no se produce destrucción del tabique ni
afectación típica de la vía respiratoria superior(R/3)

!
!
!

71. A 4-month-boy is brought to his pediatrician due to chronic constipation since birth.
His medical history included meconium plug syndrome at birth, which resolved with
enemas. He has been breastfeeding exclusively. Physical examination reveals that his
height and weight are

1. 1. Pathological rectal biopsy.


2. 2. No stool on rectal examination.
3. 3. Relaxation of the internal anal sphincter.
4. 4. Barium retention in the colon for more than 24 hours after its ingestion.
Gráfico de respuestas
Comentario

Estamos ante una enfermedad de Hirschprung o megacolón agangliónico. El diagnostico se realiza


mediante:

- Radiología simple: distensión importante de asas con ausencia de aire a nivel rectal.

- Enema ópaco: se aprecia cambio brusco en el diámetro del colon entre el segmento (estenosado)
y la porción sana (distensión). En el segmento agangliónico aparecen contracciones en dientes de
sierra. En el colon proximal dilatado, los pliegues transversales son paralelos. Existe también un
retraso en la eliminación del contraste.

-Manometría anorrectal: ausencia de relajación del esfinter anal interno ante un aumento de
presión a ese nivel (lo normal es la relajación del mismo).

- Biopsia: proporciona el diagnostico definitivo. En el segmento afectado se observa una ausencia


de células ganglionares (ausencia de plexo de Meissner y Auerbach) con aumento de la
acetilcolinesterasa y de las terminaciones nerviosas. En la zona previa a la zona dañada, existe
hipertrofia muscular.(R/3)

72. Un niño de 5 años presenta fiebre alta, dificultad respiratoria con estridor, disfagia y
babeo. En la exploración destacan las alteraciones del nivel de conciencia, con fases de
irritabilidad y somnolencia. ¿Qué diagnóstico clínico establecería?:

1. 1. Laringomalacia.
2. 2. Epiglotitis aguda.
3. 3. Laringitis aguda espasmódica.
4. 4. Traqueítis.
Gráfico de respuestas
Comentario

Esta pregunta es sencilla e importante. Presenta un caso clínico típico de epiglotitis. Las
alteraciones en la conciencia es signo de gravedad. Recuerde que es una urgencia en cuyo
manejo lo principal es asegurar la vía área.

Para pensar en laringomalacia nos tendrían que haber hablado de una lactante con estridor desde
el nacimiento que empeora en decúbito.

La clínica de laringitis se caracteriza por la triada estridor inspiratorio, tos metálica y dificultad
respiratoria. al contrario que la LAV.

!
!
!

Ante una laringitis que no mejora y con estridor en ambos tiempos respiratorios sospecharemos
traqueitis.(R/2)

73. El concepto de restricción en el crecimiento intrauterino implica:

1. 1. Recién nacido de peso inferior a 2,500 g.


Recién nacido de bajo peso para su edad gestacional independientemente de la
2. 2.
semana de gestación (debajo percentil 10).
3. 3. RN antes de la 37 semana de gestación.
4. 4. Ninguna respuesta es correcta.
Gráfico de respuestas
Comentario

Pregunta sobre el CIR en la que debemos tener claro el concepto. El CIR basa su diagnóstico en
las exploraciones ecográficas. Es una disminución patológica del ritmo de crecimiento fetal, cuyo
resultado será un feto que no alcanza su potencial de crecimiento (peso por debajo del percentil 10
para su edad gestacional).(R/2)

74. En el manejo del paciente asintomático infectado por el VIH se aconseja la realización
de serología para los siguientes gérmenes, EXCEPTO:

1. 1. Citomegalovirus.
2. 2. Toxoplasma.
3. 3. Criptococo.
4. 4. Virus C de hepatitis.
Gráfico de respuestas
Comentario

Puede plantearse la necesidad de realizar serología frente a CMV, ya que muchas personas son
positivas, con independencia de su infección por VIH… Pero, desde luego, lo que no puede pedir
es una serología que no es útil, como sucede con la criptocócica. Aunque esta prueba existe,
carece de utilidad debido a que la sensibilidad es muy escasa.(R/3)

75. Un paciente intervenido hace unos años por una úlcera duodenal acude a consulta
por la aparición de dolor abdominal. Ante la sospecha de una nueva úlcera se le practica
una endoscopía que muestra úlceras en bulbo duodenal y antro pilórico. La gastrina
sérica del paciente está elevada, y se eleva tras la administración de secretina. ¿Cuál
sería el tratamiento óptimo de este paciente teniendo en cuenta su sospecha
diagnóstica?

1. 1. Antagonistas H2 en dosis altas.


2. 2. Omeprazol.
3. 3. Resección completa del tumor.
4. 4. Gastrectomía total.
Gráfico de respuestas
Comentario

La primera sospecha diagnóstica ante un cuadro clínico con úlceras múltiples e hipergastrinemia
debe ser el síndrome de Zollinger- Ellison (gastrinoma), que además se confirma con la elevación
de la gastrina tras la estimulación con secretina. En relación al tratamiento de elección es siempre

!
!
!

quirúrgico, con extirpación del tumor, si se puede (ya que tiene una gran tendencia a la
diseminación).(R/3)

76. Femenino de 63 años que hace 8 semanas se sometió a un by-pass aortocoronario de


3 vasos. Vuelve a ingresar por fiebre de 38ºC y dolor pleurítico. En la base del pulmón
izquierdo se ausculta una disminución de los sonidos respiratorios. Se realiza una Rx en
decúbito lateral, mostrando un pequeño derrame pleural. Se extrae un líquido
serohemático con pH 7.40, LDH 380 U/l (sangre 90), proteínas 3.8 g/dl (sangre 6.3). El
examen citológico es negativo y la gammagrafía de perfusión es normal. ¿Qué
tratamiento o actitud recomendaría?

1. 1. Hacer angiografía pulmonar.


2. 2. Iniciar tratamiento con prednisona.
3. 3. Iniciar tratamiento con indometacina.
4. 4. Realizar un TC de tórax.
Gráfico de respuestas
Comentario

La presencia de dolor pleurítico, con derrame pleural, fiebre y el antecedente de un by- pass
aortocoronario reciente, debe sugerirnos un síndrome post- pericardiotomía. Después de una
intervención quirúrgica que manipule serosas (pleura y/o pericardio), es posible que éstas se
inflamen de forma reactiva, lo que produciría una pleuritis y/o pericarditis durante las siguientes
semanas. Esta entidad cursa con un derrame pleural tipo exudado, como en esta paciente. Como
tratamiento, basta el uso de un AINE, como la indometacina, y esperar a que se resuelva de forma
espontánea, como sucede en la mayoría de los casos.(R/3)

77. A 7-year-old Somalian child is brought to the pediatrician's office for a routine visit.
He is doing well in school and he is able to read and write. Physical examination shows
an adequate development and growth. He has familiar history of sickle cell disease (both
parents and a sister). Hemoglobin electrophoresis shows: Hemoglobin F 0%, Hemoglobin
S 35%, Hemoglobin A 65%. Which would be the most likely complication associated to
his condition?

1. 1. Hematuria
2. 2. Osteonecrosis
3. 3. Spleen infarction
4. 4. Recurrent pneumonia
Gráfico de respuestas
Comentario
Hematuria. Kidney involvement in sickle cell anemia, which is called sickle cell nephropathy,
presents with renal hyperperfusion, glomerular hypertrophy, and glomerular hyperfiltration. Many
patients eventually develop a glomerulopathy leading to glomerular proteinuria. Loss of kidney
function is secondary to cortical infacts. A high percentage of patients suffer painless hematuria,
probably due to medullary infarcts. The rest of the possible options are less common.(R/1)

78. Cuál de las siguientes aseveraciones NO es correcta respecto a la toxicología de la


heroína:

1.! ! 1.! Se!trata!de!una!sustancia!con!un!gran!poder!adictivo.!


El!síndrome!de!abstinencia!aparece!entre!las!8!y!10!horas!después!de!la!última!
2.! ! 2.!
administración.!

!
!
!

3.! ! 3.! La!clonidina!se!utiliza!en!el!tratamiento!del!síndrome!de!abstinencia.!


4.! ! 4.! El!tratamiento!de!elección!ante!un!caso!de!intoxicación!aguda!es!la!naltrexona.!
Gráfico de respuestas
Comentario

La heroína, es una sustancia muy adictiva sobre todo por vía IV > inhalatoria > oral ( opción 1
correcta).

El síndrome de abstinencia depende de la dosis, el tiempo de consumo, y la vida media.


Concretamente con la heroína aparece a partir de las 8 horas (opción 2 correcta) de la última
dosis, con la metadona, al tener vida media más larga, la abstinencia es más larga pero más leve y
por eso se utiliza como sustitutivo para la deshabituación. El opiode que más pronto induce
abstinencia la meperidina. Para evitar los síntomas de la abstinencia se una clonidina y así
disminuye la hiperactividad adrenérgica con agonistas alfa- 2.

Lo que es falso es que la naltrexona esté indicada para el tratamiento de la intoxicación aguda. Lo
que se usa es la Naloxona! (R/4)

79. Una de las siguientes NO es complicación frecuente del abruptio:

1. 1. Insuficiencia renal.
2. 2. Choque.
3. 3. Síndrome de Sheehan.
4. 4. Pielonefritis.
Gráfico de respuestas
Comentario

Como complicaciones del abruptio, se han descrito todas las respuestas, excepto la pielonefritis
(opción 4). Las más frecuentes son la CID y la insuficiencia renal. La CID también se presenta en
abortos diferidos y la patogenia es similar en ambos casos: liberación y paso de tromboplastinas
fetales a circulación materna. El tratamiento consiste en una adecuada reposición de factores de
coagulación y fibrinógeno, al mismo tiempo procedemos a la evacuación inmediata del útero,
mediante la realización de una cesárea. El choque y el síndrome de Sheehan son complicaciones
menos frecuentes.(R/4)

80. Niño de 4 años con fiebre alta de 6 días de


evolución acompañada de irritabilidad. La madre refiere además haberle notado las
manos y los pies hinchados y dolorosos. A la exploración llama la atención hiperemia

!
!
!

conjuntival intensa sin exudado, un exantema de predominio en tronco y labios


enrojecidos y resecos. Es ingresado con sospecha de enfermedad infecciosa y tratado
con antibióticos tras la toma de cultivos. Al 10º día de evolución, la fiebre no ha
desaparecido y comienza a presentar alteraciones en manos y pies como los que se
muestran en la imagen. En relación al cuadro clínico citado, señale la opción FALSA:

La imagen muestra la descamación en dedo de guante característica de la


1. 1.
enfermedad.
Para confirmar el diagnóstico es necesario realizar biopsia de algún ganglio
2. 2.
linfático accesible.
3. 3. La base patogénica de la enfermedad es una vasculitis.
La etiología de la enfermedad no está clara pero se basa en la teoría del
4. 4.
superantígeno.
Gráfico de respuestas
Comentario

La imagen, en esta pregunta, es sólo un dato más, y nos muestra lo que ya nos podríamos haber
imaginado por el contexto de la pregunta: descamación plantar. Se trata de una enfermedad de
Kawasaki, en cuya patogenia parece estar implicado un superantígeno que produciría una
sobreactivación policlonal de los linfocitos T, que producirían citoquinas responsables de las
manifestaciones de la enfermedad.
Recuerden que, en esta patología, el problema fundamental es que a veces se producen
dilataciones aneurismáticas en las coronarias, por lo que estos niños estarían predispuestos a
infartos de miocardio a edades muy precoces. El diagnóstico de la enfermedad de Kawasaki se
realiza en base a criterios clínicos y no precisaría confirmación histológica (R/2).

81. El niño de la pregunta anterior fue ingresado y tratado con gammaglobulina


intravenosa además de aspirina. Sobre el manejo de este paciente, señale la opción
FALSA:

Si la fiebre no cede con gammaglobulina el fármaco de segunda elección son los


1. 1.
corticoides.
La gammaglobulina es eficaz en la prevención de la aparición de aneurismas
2. 2.
coronarios en la mayoría de los casos.
La aspirina a dosis antiagregantes es importante en la prevención de las
3. 3.
trombosis.
La descamación en dedo de guante es característica de la fase aguda de la
4. 4. enfermedad y por tanto muy útil para el diagnóstico precoz y el inicio del
tratamiento.
Gráfico de respuestas
Comentario

La descamación es habitual cuando la enfermedad ya lleva cierta evolución, por lo que no tendrá
aplicación alguna para llegar a un diagnóstico precoz (respuesta 4 falsa). Tal como explicábamos
en el comentario anterior, el problema principal que puede complicar este cuadro son los
aneurismas coronarios, que aparecen con menor frecuencia cuando se utiliza la gammaglobulina
intravenosa, como se comenta en la opción 2.(R/4)

82. El fenómeno de Koebner se da en:

!
!
!

1. 1. Pénfigo.
2. 2. Psoriasis.
3. 3. Púrpura trombocitopénica.
4. 4. Vasculitis por hipersensibilidad.
Gráfico de respuestas
Comentario

Pregunta básica sobre un signo muy útil en el diagnóstico dermatológico, el fenómeno de Koebner.
Consiste en la aparición de determinadas dermatosis en las zonas de presión o traumátismo. Es
típico de las enfermedades eritemato-escamosas como psoriasis (opción 3), liquen plano, pitiriasis
rubra pilaris y Darier. Infecciosas con el molluscum o las verrugas y de otras como vitíligo,
xantomas, vasculítis y síndrome de Sweet. La consecuencia práctica es que el paciente debe
conocer que el fenómeno puede actuar como factor precipitante, por lo que debe evitar en la
medida de lo posible las lesiones cutáneas.(R/2)

83. El tratamiento más recomendable de la diverticulosis asintomática es:

1. 1. Laxantes salinos.
2. 2. Anticolinérgicos.
3. 3. Resección quirúrgica del segmento afecto.
4. 4. Dieta rica en residuos.
Gráfico de respuestas
Comentario

Cuando se habla de diverticulosis, el término se refiere a la presencia de divertículos, sin


complicación acompañante. La diverticulosis aumenta en frecuencia con la edad, y suele ser
asintomática. Por tanto, es evidente que, de inicio, no se indica ningún tratamiento quirúrgico. En
situación estable, sin clínica, no se precisa el uso de fármacos de ningún tipo. En cambio, sí parece
aconsejable una dieta rica en fibra y residuos, que “arrastre” el contenido del colon, lo que dificulta
el estancamiento de restos fecales intradiverticulares, lo que incrementa el riesgo de
diverticulitis.(R/4)

84. A patient with a past history of HIV infection comes to the emergency department
complaining of dyspnea and pallor. Current medication includes antiretroviral therapy
with AZT + 3TC + Efavirenz, for 14 months. Last determinations of CD4 and viral load were
350/mm3 and <200 copies / ml, respectively. Current laboratory studies show:
hemoglobin 7.8 g / dl, MCV 68 fl. The most likely diagnosis is:

1. 1. Aplastic crisis (parvovirus B19 infection).


2. 2. Disseminated Leishmaniasis.
3. 3. Disseminated infection by Mycobacterium avium-intracellulare.
4. 4. Chronic gastrointestinal bleeding.
Gráfico de respuestas
Comentario

Pregunta sencilla. Tenemos que percatarnos que se trata de un paciente con diagnóstico de VIH
controlado, con unos CD4 de 350 y una carga viral <100 copias, por lo que al encontrarnos con
una anemia microcítica, lo primero que tendriamos que descartar es una hemorragia
gastrointestinal crónica.(R/4)

!
!
!

85. A 79-year-old male is brought to the ER due to sudden onset intense abdominal pain,
with vomiting. His past medical history is positive for hypertension, diabetes, atrial
fibrillation and two ischemic strokes. His BP is 150/98 mm Hg, pulse is 120/min,
temperature is 38.4ºC and respirations are 20/min. Physical examination reveals clear
lungs to auscultation, severe abdominal pain to palpation and absent bowel sounds, with
rebound. Rectal examination shows heme-positive stools. Lab tests show: WBC
18,700/mm3, hemoglobin 13 g/dl, creatinine 1 mg/dl, troponin I < 0.017 (normal) INR 1.5.
What is the most likely diagnosis?

1. 1. Acute pancreatitis
2. 2. Acute diverticulitis
3. 3. Myocardial infarction
4. 4. Bowel infarction
Gráfico de respuestas
Comentario
Bowel infarction. Acute mesenteric ischemia (AMI) is a syndrome caused by inadequate blood flow
through the mesenteric vessels, resulting in ischemia and eventual gangrene of the bowel wall.
Although relatively rare, it is a potentially life-threatening condition. Broadly, AMI may be classified
as either arterial or venous. AMI as arterial disease may be subdivided into nonocclusive
mesenteric ischemia (NOMI) and occlusive mesenteric arterial ischemia (OMAI); OMAI may be
further subdivided into acute mesenteric arterial embolism (AMAE) and acute mesenteric arterial
thrombosis (AMAT). In this case, the patient presents wth occlusive mesenteric arterial ischemia,
most likely due to embolism. Of all the types of AMI, acute mesenteric arterial embolism typically
has the most abrupt and painful presentation as a consequence of the rapid onset of occlusion and
the inability to form additional collateral circulation. It has been described as abdominal apoplexy
and is sometimes referred to as a “bowel attack.” Often, vomiting and diarrhea (gut emptying) are
observed. Patients are usually found to have some cause of emboli. Because most emboli are of
cardiac origin, patients often have atrial fibrillation or a recent myocardial infarction (MI) with mural
thrombus. Early in the course of AMI, in the absence of peritonitis, physical signs are few and
nonspecific. Peritoneal signs develop late, when infarction with necrosis or perforation occurs.
Fever, hypotension, tachycardia, tachypnea, and altered mental status are observed.(R/4)

86. En relación al eczema marginado de hebra señale la respuesta INCORRECTA:

1. 1. Clínicamente puede aparecer un borde inflamatorio, incluso con pústulas.


2. 2. Son características unas pequeñas lesiones satélites.
3. 3. Ceden bien al tratamiento con imidazoles tópicos.
4. 4. Puede transmitirse por contacto sexual.
Gráfico de respuestas
Comentario

El eczema marginado de Hebra se trata de una tiña localizada a nivel inguinal, que se produce por
hongos dermatofíticos, concretamente por el E. flocosum y T. rubrum (especies también
relacionadas con la tiña pedis). El contagio se produce por contacto directo o a través de toallas,
ropas...Entre los síntomas que se producen destaca el prurito. Las lesiones comienzan por una o
varias manchas eritematosas, ligeramente descamativas, que progresan de forma excéntrica y dan
lugar a una placa más grande, con borde de crecimiento activo eritematoescamoso y en ocasiones
con pequeñas vesículas o pústulas produciendo aclaramiento central. Puede aparecer alguna
lesión satélite pequeña, pero no es característico.(R/2)

87. Todos los siguientes datos apoyan el diagnóstico de apraxia constructiva, EXCEPTO:

!
!
!

El paciente no es capaz de copiar en un papel un dibujo modelo cuando se le pide


1. 1.
verbalmente.
2. 2. El paciente comprende lo que se le está pidiendo.
3. 3. El paciente es arquitecto de profesión.
4. 4. El paciente presenta una hipoestesia hemicorporal derecha.
Gráfico de respuestas
Comentario

Esta pregunta sirve para repasar el tema de las apraxias. Debe recordar que la apraxia es la
incapacidad para llevar a cabo actos motores ante una orden verbal o imitación en un paciente con
una adecuada comprensión y sin déficits motores o sensitivos primarios. La apraxia constructiva,
más concretamente, es la incapacidad para dibujar o construir figuras o formas bi o
tridiminesionales. Se explora pidiendo al paciente que copie en un papel varios dibujos
modelo.(R/4)

88. A 2-year-old child is brought to the emergency department by her worried mother
presenting with fever and crying while urinating. The girl had previously been healthy and
had a normal development. The mother has also observed some blood in the urine
collected. Rectal temperature is 37.8ºC. Vital signs are: 100/70 mmHg (blood pressure)
and 99 bpm. Abdominal palpation is painful in suprapubic and right costovertebral areas.
Urine test results are: Gravity 1.004, pH 7, None proteins, Moderate blood, Negative
glucose, Negative ketones, Leukocyte esterase positive, positive nitrates, White blood
cells 45 +/hpf, Red blood cells 20-30/hpf, Casts none, Crystals none. Once antibiotherapy
is completed with good course, what is the best following step in the management of this
patient?

1. 1. Chronic prophylactic antibiotherapy


2. 2. Bladder and renal ultrasonography and voiding cystourethrogram.
3. 3. Renal and bladder ultrasound
4. 4. Expectation with observation
Gráfico de respuestas
Comentario
Bladder and renal ultrasonography and voiding cystourethrogram. Urinary tract infections are
common in children, especially girls and boys under one year of age. If infections appear at later
ages, it should raise the suspicion of renal abnormalities or urinary tract defects. Ultrasound is the
diagnostic test of choice in the acute phase because it can help to diagnose abscesses in the renal
parenchyma or obstruction in the urinary tract. It is an inexpensive, noninvasive test with no
radiation exposure.(R/2)

89. ¿Cuál de los siguientes antihipertensivos NO disminuye los niveles de renina?:

1. 1. Metildopa.
2. 2. Reserpina.
3. 3. Propranolol.
4. 4. Diazóxido.
Gráfico de respuestas
Comentario

Es importante que domines el tratamiento farmacológico de la HTA porque es muy preguntado en el ENARM
pero no se preocupe si ha fallado esta pregunta porque es difícil. Para acertarla, debe tener clara la fisiología
del sistema renina- angiotensina- aldosterona. Tenga en cuenta que la renina se libera en respuesta a varios

!
!
!

estímulos, entre los cuales se encuentran la activación del SNS y de las células yuxtaglomerulares. Fíje que
en las dos primeras opciones aparecen fármacos que bloquean este sistema, disminuyendo así la secreción
de renina. La metildopa y la clonidina son fármacos simpaticolíticos de acción central y la reserpina es un
depletor de catecolaminas. El propranolol es un betabloqueante que bloquea los receptores beta- 1 de las
células yuxtaglomerulares, inhibiendo así la liberación de renina. El diazóxido, sin embargo, es un
vasodilatador de uso intravenoso que disminuye la tensión arterial por otro mecanismo distinto.(R/4)

90. ¿Cuál de las siguientes manifestaciones NO es característica de una colestasis


prolongada?:

1. 1. Prurito.
2. 2. Osteopenia.
3. 3. Degeneración neuromuscular.
4. 4. Estreñimiento.
Gráfico de respuestas
Comentario
En el síndrome colestático, parte de la sintomatología se debe al déficit de vitaminas liposolubles.
Así, puede surgir osteopenia u osteomalacia por alteraciones en la absorción proteica y de la
vitamina D; y alteraciones neuromusculares o ataxia si hay déficit de vitamina E, situación de baja
frecuencia y que se observa básicamente en niños. Por otra parte, hay síntomas derivados de la
retención de bilis como ictericia, hipercolesterolemia, prurito y en ocasiones puede encontrarse
diarrea tipo esteatorrea por falta de absorción de las grasas al no aportarse sales biliares. No se ha
descrito estreñimiento.(R/4)

91. En la hemocromatosis idiopática primaria NO es característico encontrar:

1. 1. Disminución de la ferritina sérica.


2. 2. Elevación de la sideremia.
3. 3. Hiperglucemia.
4. 4. Saturación de la transferrina mayor del 70%.
Gráfico de respuestas
Comentario

Recuerda que un hallazgo característico de la hemocromatosis primaria es la elevación de la


ferritina, por aumento de los depósitos de hierro.(R/1)

92. La escarlatina es producida por:

1. 1. Adenovirus.
2. 2. Rotavirus.
3. 3. Neumococo.
4. 4. Toxina eritrogenica del estreptococo B hemolítico.
Gráfico de respuestas
Comentario

La escarlatina está ocasionada por S. pyogenes productor de toxinas eritrógenas. Afecta


principalmente a escolares entre 5-15 años y es infrecuente en menores de 3 años.

!
!
!

El tratamiento de elección es penicilina oral durante 10 dias.(R/4)

93. ¿Cuál de los siguientes NO es un factor protector para las siguientes patologías que
se mencionan a continuación?:

1. 1. Multiparidad en el cáncer de endometrio.


2. 2. Nuligesta en cáncer de cérvix.
3. 3. Anovulatorios en cáncer de ovario.
4. 4. Anovulatorios en la enfermedad inflamatoria pélvica.
Gráfico de respuestas
Comentario

El tema importante sobre de los factores de riesgo y protección en ginecología es importante,


sobre todo de los tumores.

El principal factor de riesgo para el cáncer de ovario es la ovulación incesante por lo que la
anovulación (respuesta 3) es un factor protector.

Los anticonceptivos orales además de proteger de el cancer de ovario, lo hacen del


adenocarcinoma de endometrio. En cuanto al cérvix, aumentan la displasia pero NO el cancer. Y le
pude ayudar recordar que disminuyen la EIP pero aumentan la vulvovaginitis.

El principal factor de riesgo en el cáncer de endometrio es la estimulación estrogénica sin


oposición de progesterona. Son factores de protección: menarquia tardia, multiparidad,
menopausia precoz...

Las nuligestas tienen mas riesgo de cáncer de endometrio y de ovario pero no necesariamente de
cérvix ya que el principal el principal factor de riego para cervix es el numero y edad de inicio de las
relaciones sexuales. El que sea nuligesta no implica que no tenga relaciones sexuales, por lo que
no puede ser considerado como factor de protección.(R/2)
on verdaderas.
Gráfico de respuestas
Comentario

El neuroblastoma es el tumor sólido extracraneal más frecuente en la infancia. La edad media de


los pacientes en el momento del diagnóstico son los dos años (el 90 % de los casos se producen
en menores de cinco años).

La localización más frecuente es el abdomen 70%. Dentro del abdomen, un 50 % se econtrarán en


suprarrenales. Otras localizaciones: tórax, mediastino, posterior y nasofaringe.

Se asocia a síndrome alcohólico fetal, hidantoínas, neurofibromatosis tipo 1, nesidioblastosis,


Hirschprung.

Clínica deriva de la localización, puede presentar algunos síndromes complejos como: síndrome de
opsoclono-mioclono, diarrea secretora intensa, hipertensión arterial y hematoma lineal en el
párpado.

Los estadios según International Neuroblastoma Staging System (INSS) son los siguientes:

!
!
!

I: localizado en el órgano de origen .

II: situado más allá de la estructura de origen, sin sobrepasar la línea media y sin afectación
ganglionar ipsilateral (HA) o con ella (IIB).

III: más allá de la línea media con/sin afectación ganglionar.

IV: muestra metástasis al hueso, a la médula ósea, al hígado, a la piel , a los ganglios linfáticos
distantes o en otras localizaciones .

IVs: tumor estadio I o II en un menor de un año con metástasis en el hígado, la piel o la médula
ósea.(R/4)

94. Un militar de la brigada paracaidista, de 25 años de edad, sufre un traumatismo


durante un entrenamiento. Como consecuencia de éste, sufre una fractura vertebral en
C6, así como una fractura de Colles en antebrazo izquierdo. Se recupera y, semanas
después, le cuenta que persiste cierto dolor, limitación de la movilidad y hormigueo en
los tres primeros dedos de la mano izquierda, que empeora claramente durante la noche.
Teniendo en cuenta el diagnóstico más probable, señale cuál de las siguientes causas
NO predispone al mismo:

1. 1. Desviaciones óseas postraumáticas.


2. 2. Amiloidosis.
3. 3. Artritis reumatoide.
4. 4. Hipertiroidismo.
Gráfico de respuestas
Comentario

Nos están describiendo una secuela de la fractura de Colles como es un síndrome del túnel del
carpo. Éste puede ser debido a muchas patologías más, todas las descritas en la pregunta excepto
el hipertiroidismo (sin embargo, sí se asocia al hipotiroidismo).(R/4)

95. Respecto al síndrome hepatopulmonar, ¿cuál de las siguientes afirmaciones es


FALSA?

1. 1. Es un cuadro clínico de carácter básicamente funcional.


2. 2. Existe un desequilibrio de las relaciones ventilación/perfusión pulmonares.
Es el causante de la hipoxemia arterial que presentan algunos pacientes con
3. 3.
hepatopatía crónica.
4. 4. Es irreversible incluso después del trasplante hepático.
Gráfico de respuestas
Comentario

El síndrome hepatopulmonar es un cuadro que se produce en pacientes con hepatopatia crónica


(igual que el hepatorrenal), y cursa con hipoxemia arterial debida en parte a la menor reactividad
vascular pulmonar a estímulos vasoconstrictores pulmonares (como la hipoxia). No obstante, al ser
una patología funcional, es reversible si se realiza un trasplante hepático con éxito.(R/4)

96. Los pliegues urogenitales dan lugar a:

!
!
!

1. 1. Clítoris.
2. 2. 2/3 inferiores de la vagina.
3. 3. Labios menores.
4. 4. Vestíbulo.
Gráfico de respuestas
Comentario

Pregunta muy difícil de Embriología.

Genitales externos: Durante la quinta semana de vida embrionaria, se forman pliegues tisulares a
cada lado de la cloaca que se unen por delante en la linea media para formar el tubérculo genital.
Con la división de la cloaca por el tabique urorrectal y la formación consecuente del perineo, estos
pliegues cloacales se conocen por delante como pliegues urogenitales y por detrás como pliegues
anales.

El tubérculo genital empieza a aumentar de tamaño, pero en el embrión del sexo femenino, su
crecimiento va haciendose mas lento de manera gradual hasta que se convierte en el clítoris, y los
pliegues urogenitales forman los labios menores.

En el varon, el tubérculo genital sigue creciendo para formar el pene, y se cree que los pliegues
urogenitales se fusionan para encerrar a la uretra del pene. Lateralmente a los pliegues
urogenitales, se desarrolla otro par de eminencias, que se conocen en la etapa indiferente como
eminencias labioscrotales.

En ausencia de andrógenos se quedan prácticamente sin fusionar y se convierten en labios


mayores.

El seno urogenital definitivo da origen al vestíbulo vaginal, en el cual se abren uretra, vagina y
glandulas vestibulares mayores. (R/3)

97. Señale la afirmación ERRÓNEA sobre el oncogén C-erb B2 (Her-2-neu):

1. 1. También se llama receptor del factor de crecimiento epidérmico humano 2.


Las pacientes con cánceres que lo sobreexpresan pueden ser tratadas con
2. 2.
trastuzumab.
3. 3. Los cánceres de mama en los que se sobreexpresa tienen mejor pronóstico.
Se ha relacionado con resistencia al tratamiento antihormonal, pero puede hacer
4. 4.
que sea más sensible a ciertos quimioterápicos.
Gráfico de respuestas
Comentario

Esta pregunta puede confundirnos si no dominamos este concepto. Los cánceres de mama que
sobreexpresan el Her- 2- neu (C- erb B2) generalmente son más agresivos y suelen tener
receptores hormonales negativos. Por tanto NO tienen mejor pronóstico como se afirma en la
opción 3. Otra cosa es que tienen un tratamiento específico con Trastuzumab (Herceptin®) y es lo
que nos hace pensar, erróneamente, que tienen mejor pronóstico.(R/3)

98. Femenino con diagnóstico reciente de adenocarcinoma de recto que se encuentra a


10 cm del margen anal. ¿Cómo se realiza el estudio de extensión?

1. 1. TC abdominal.

!
!
!

2. 2. Resonancia magnética.
3. 3. Rx simple de abdomen.
4. 4. TC torácico.
Gráfico de respuestas
Comentario

Ante un carcioma colorrectal, debemos hacer un estudio de extensión para valorar la N y la M del
TNM. La prueba de imagen de elección para conseguir éste objetivo es el TC. Respuesta 1
correcta.(R/1)

99. A 40-year-old man comes to the ER complaining of a 4-day history of abdominal pain
and bloody diarrhea. He reports 6 similar episodes in the past year. Endoscopic
evaluation shows extensive disease from terminal ileum to the rectum, with ulcerations
and pseudopopyps. About the disease that you suspect, which of the following is
pathognomonic?

1. 1. Discontinuous bowel involvement.


2. 2. Involvement of terminal ileum.
3. 3. Noncaseating granulomas on biopsy.
4. 4. Crypt abscesses on biopsy.
Gráfico de respuestas
Comentario
Crypt abscesses on biopsy. Our main suspition is ulcerative colitis, which typically presents cryps
abscesses. Noncaseating granulomas and cobblestone appearance in endoscopy are charateristic
of Crohn's disease(R/4)

100. Recién nacido de 3 días de vida


cuyos padres traen a urgencias por presentar vómitos biliares desde el nacimiento junto
con evacuaciones escasas, acólicas, sin productos patológicos. Está siendo alimentado

!
!
!

con lactancia materna exclusiva, presenta buena succión y realiza bien las tomas. Los
padres refieren además tinte ictérico desde las 48 horas de vida. No presenta fiebre ni
otra sintomatología. A su llegada a urgencias presenta los siguientes signos vitales: FC.
130 l.p.m. FR. 37 r.p.m, Tª 36.8 ºC, TA. 61/30 mmHg, saturación de O2 96 %. En la
exploración física del paciente se aprecia un adecuado estado de hidratación, con
ictericia cutánea hasta miembros inferiores, con auscultación cardiopulmonar normal y
un abdomen blando y depresible en el que no se palpan masas ni visceromegalias. En la
exploración neurológica se aprecia una fontanela anterior normotensa, se encuentra
reactiva, con buen tono muscular y reflejos arcaicos presentes y simétricos. Se realizan
examenes de laboratorio en la que se detecta hiperbilirrubinemia a expensas de
bilirrubina indirecta. Asimismo, se lleva a cabo un ultrasonido abdominal y
posteriormente un tránsito gastroduodenal, que puede ver en la imagen. Ante la historia
clínica, la exploración física y las pruebas complementarias, ¿cuál es el diagnóstico más
probable?

1. 1. Atresia duodenal.
2. 2. Atresia esofágica.
3. 3. Atresia pilórica.
4. 4. Atresia anal.
Gráfico de respuestas
Comentario

El paciente de esta pregunta presenta una obstrucción intestinal. La presencia de vómitos de


características biliares descarta una estenosis proximal al duodeno, ya que la vesícula biliar
desemboca en la primera porción del duodeno, por ello podemos descartar las respuestas 2, 3 y 4.

La estenosis hipertrófica de píloro no ocurre en los primeros días de vida, sino entre las 3 y las 6
semanas de vida. Se caracteriza por vómitos progresivos, alimenticios, nunca biliares.

La atresia pilórica presenta el mismo cuadro pero desde el nacimiento. Recuerde también que la
atresia esofágica más frecuente es la tipo tres (aTRESia esofágica)

La atresia anal se manifiesta por ausencia de evacuaciones y se detectaría en la exploración física.

El diagnóstico en este caso, es una atresia duodenal por los vómitos biliares, por el inicio neonatal
y por la presencia de dos burbujas en el tránsito (también visible en radiografía simple), que
corresponden una al estómago y otra al duodeno, sin que pase el contraste distalmente a la
estenosis.(R/1)

101. El paciente de la pregunta anterior fue sometido a intervención quirúrgica con una
evolución favorable. Respecto al cuadro clínico que presenta, ¿cuál de las siguientes
afirmaciones es CORRECTA?

1. 1. Se asocia a otras enfermedades, como el síndrome de Down.


2. 2. En la radiografía de abdomen se observa el signo de la burbuja.
3. 3. El tratamiento quirúrgico de elección es la pilorotomía extramucosa de Ramsted.
Aunque puede presentarse en el período neonatal, es típico que debute en la
4. 4.
tercera semana de vida.
Gráfico de respuestas
Comentario
!
!
!

El paciente presenta una atresia duodenal, cuyo principal diagnóstico diferencial es la obstrucción
a nivel del píloro.

La atresia duodenal debuta en los primeros días de vida, ya que el niño nace con una estenosis a
ese nivel, a diferencia de lo que ocurre en la estenosis hipertrófica de píloro en la que esa
obstrucción es progresiva, por lo que se manifiesta hasta la 3ª y la 6ª semanas de vida.

En la radiografía de la atresia duodenal observaremos 2 burbujas, una gástrica y otra duodenal,


mientras que en la obstrucción a nivel del píloro sólo veremos una burbuja, la correspondiente al
estómago.

La estenosis hipertrófica de píloro se asocia a la toma de macrólidos durante la lactancia y el


embarazo; sin embargo, la atresia duodenal se relaciona con el síndrome de Down (recuerde las 3
D: atresia Duodenal, síndrome de Down y Doble burbuja).

La pilorotomía extramucosa de Ramsted es el tratamiento de elección para la estenosis pilórica, no


para la atresia duodenal.(R/1)

102. Todo lo siguiente puede aparecer como consecuencia de la resección ileal,


EXCEPTO:

1. 1. Déficit de hierro.
2. 2. Esteatorrea.
3. 3. Aumento de la frecuencia de cálculos de colesterol.
4. 4. Diarrea por pérdida de la función de la válvula ileocecal.
Gráfico de respuestas
Comentario

La resección ileal no asocia déficit de hierro, pues este elemento se absorbe fundamentalmente en
regiones proximales del tracto gastrointestinal, sobre todo en el duodeno y el yeyuno proximal.
Recuerda que la absorción es mayor en forma hem, luego en forma ferrosa y como peor se
absorbe es en forma férrica.(R/1)

103. En relación al ciclo genital femenino es FALSO que:

1. 1. En la primera fase del ciclo, la FSH produce un aumento del recuento folicular.
2. 2. La LH tiene un solo pico, previo a la ovulación.
3. 3. El cuerpo lúteo produce progesterona y estrógenos.
4. 4. Al final de la fase lútea se produce un descenso de FSH previo a la menstruación.
Gráfico de respuestas
Comentario
Pregunta que versa sobre los conocimientos básicos del ciclo genital femenino. La GnRH
sintetizada en el hipotálamo estimula la secreción de FSH y LH por la hipófisis. La FSH tiene un
pico en primera fase (folicular) para estimular una cohorte folicular, y al final de la fase lútea
también se eleva sus niveles para estimular un nuevo grupo de folículos.(R/4)

104. ¿Cuál de los procesos cutáneos siguientes es MENOS frecuente en infectados por
el virus de la inmunodeficiencia humana?:

!
!
!

1. 1. Foliculitis eosinofílica.
2. 2. Exantema agudo retroviral.
3. 3. Reacción cutánea a medicamentos.
4. 4. Xerodermia.
Gráfico de respuestas
Comentario
Pregunta de alta dificultad, pues para contestarla, es preciso memorizar toda una lista de
afectaciones dermatológicas relacionadas con la infección por el VIH.(R/1)

105. Uno de los síntomas siguientes NO es característico de la anorexia nerviosa:

1. 1. Angustia.
2. 2. Perturbación de la imagen corporal.
3. 3. Interés por vestir ropas amplias.
4. 4. Amenorrea.
Gráfico de respuestas
Comentario

Es una pregunta muy sencilla pero hace dudar sobre las características clínicas básicas dela
anorexia nerviosa. Todas las opciones excepto la 3 se observan en la anorexia. Más que interés en
vestir ropas amplias, se esconden en ella para ocultar su delgadez. Las anoréxicas están además
en alerta y generalmente tristes, y apartadas aunque dependen de la fase. Generalmente son
hiperactivas aunque al final se convierten en hipo. Tienen cambios bruscos del estado de ánimo,
con rigidez de pensamiento, controladoras, manipuladoras, son desconfiadas, luchan por la
percepción, con personalidad obsesivo- compulsiva. No se olvide de repasar los criterios
diagnósticos.(R/3)

106. ¿Cuál de las siguientes aseveraciones en la fibrosis pulmonar idiopática es FALSA?:

1. 1. Fisiopatológicamente no se diferencia del conjunto de las fibrosis pulmonares.


Existen enfermos que evolucionan muy rápidamente (Hammam-Rich), pero la
2. 2.
mayoría tienen una evolución más lenta.
3. 3. En el lavado bronquialveolar, el hallazgo más común es la neutrofilia.
4. 4. Los corticoides son efectivos tanto en la fase de alveolitis como en la de fibrosis.
Gráfico de respuestas
Comentario
El tratamiento de la fibrosis pulmonar son los corticoides cuyo efecto es disminuir o suprimir la
alveolitis puesto que la fibrosis es irreversible. Cuando no hay respuesta se pueden anadir
inmunosupresores como la ciclofosfamida o azatioprina. Sólo hay respuesta en el 20% de los
casos. El trasplante de pulmón debe considerarse en la fases finales de la enfermedad.(R/4)

107. Todas las afirmaciones sobre la enterocolitis necrosante (ECN) son ciertas,
EXCEPTO:

Su incidencia es del 1 al 5 por 100 de los ingresos de recién nacidos en unidades


1. 1.
de cuidados intensivos.
2. 2. La situación de hipoxia no favorece este proceso.
3. 3. El estrés perinatal es un factor predisponente.
4. 4. El diagnóstico depende de mantener una sospecha constante sobre este proceso.
Gráfico de respuestas

!
!
!

Comentario

La enterocolitis necrotizante afecta fundamentalmente a íleon distal y colon proximal. Suele


relacionarse con situaciones de hipoxia y bajo gasto (respuesta 2 correcta). Clínicamente, debe
sospecharla ante la presencia de distensión abdominal y evacuaciones sanguinolentas. Existen
varios signos radiológicos, pero el más sugestivo es laneumatosis intestinal, que es diagnóstico de
esta enfermedad. El tratamiento consiste en dieta absoluta, sonda nasogástrica e hidratación
intravenosa. Debe administrase además antibioterapia, cubriendo gérmenes anaerobios y
gramnegativos. En caso de perforación intestinal o refractariedad a antibióticos, se plantearía la
cirugía.(R/2)

108. Señale la FALSA respecto a los tumores benignos del pulmón:

Los hamartomas, lo más frecuente es que sean asintomáticos y de localización


1. 1. periférica, y a pesar de su benignidad, se suelen resecar para asegurar su
benignidad.
La histología más frecuente de los tumores benignos del pulmón es adenoma
2. 2.
bronquial.
Lo más habitual en el tumor carcinoide del pulmón es que se diagnostique por el
3. 3. síndrome carcinoide, que además puede producirse sin necesidad de que existan
metástasis hepáticas.
4. 4. El tratamiento de elección de todos los adenomas es la resección quirúrgica.
Gráfico de respuestas
Comentario

La respuesta incorrecta es la 3, pues el tumor carcinoide pulmonar se diagnostica con mayor


frecuencia por una tos persistente, debido a su crecimiento endobronquial. La aparición de
síndrome carcinoide se relaciona con tumores de cierto tamaño, con lo que aparece después.
Recuerde que el tumor carcinoide constituye la histología más frecuente de los tumores benignos
del pulmón y su localización es central, ya que crece en el interior de los bronquios y deriva de las
células del sistema APUD (neuroendocrinas).(R/3)

109. De las siguientes enfermedades, ¿cuál es la que con mayor frecuencia produce
ulceraciones múltiples en las primeras porciones (duodeno-yeyuno) del intestino
delgado?

1. 1. Antinflamatorios no esteroideos (AINE).


2. 2. Enfermedad de Crohn.
3. 3. Tuberculosis intestinal.
4. 4. Infección por Yersinia.
Gráfico de respuestas
Comentario

La localización más corriente de la tuberculosis en el ámbito digestivo es el íleon terminal. En


consecuencia, sería raro pensar en las primeras porciones del intestino delgado, aparte de que la
tuberculosis gastrointestinal no es una enfermedad frecuente (respuesta 3 falsa).

La enfermedad de Crohn afecta preferentemente a las regiones terminales del intestino delgado
(íleon terminal). Recuerda que, en el diagnóstico diferencial de la enfermedad de Crohn, está la
ileítis por Yersinia, puesto que afecta a la misma zona (opciones 2 y 4 falsas).(R/1)

!
!
!

110. ¿Qué paciente está sometida a un mayor riesgo de cáncer ovárico de estirpe
epitelial?

1. 1. Tomadora de anovulatorios durante 10 años.


2. 2. Síndrome de ovario poliquístico.
3. 3. Disgenesia gonadal.
4. 4. Tratamiento con clomifeno o gonadotropinas.
Gráfico de respuestas
Comentario

El 90% de las tumoraciones malignas de ovario son de estirpe epitelial. Los factores de riesgo
tienen como punto en común el traumatismo constante en el recubrimiento epitelial del ovario, con
la posibilidad de que la reparación no sea adecuada y aparezcan células atípicas. Ejemplo claro de
ello: un ovario sometido a sucesivas inducciones de la ovulación, con numerosos folículos
reclutados desarrollándose y rompiéndose en cada ciclo (respuesta 4). Serán factores protectores,
por tanto, el consumo de anticonceptivos hormonales (anovulatorios) y el SOP (dónde con gran
frecuencia se producen ciclos anovulatorios). Aunque el síndrome de Morris predispone a un
cáncer de ovario, no sería de estirpe epitelial, sino germinal (el disgerminoma) y algunas
disgenesias gonadales también (el síndrome de Swyer se asocia con la aparición de
gonadoblastomas).(R/4)

111. Mujer de 59 años que acude a realizarse la mastografía del programa de screening
de cáncer de mama. Se observa un nódulo de 2 cm de diámetro, con bordes espiculados
y con microcalcificaciones agrupadas en su interior. La anatomía patológica de la pieza
quirúrgica confirma que se trata de un cáncer ductal infiltrante, con 3 de los 11 ganglios
extirpados positivos para metástasis. Indique cómo completaría el tratamiento:

1. 1. Radioterapia y poliquimioterapia.
2. 2. Radioterapia y hormonoterapia.
3. 3. Completar la mastectomía y añadir quimioterapia.
4. 4. Completar la mastectomía y añadir quimioterapia y hormonoterapia.
Gráfico de respuestas
Comentario

Está claro que esta mujer precisará radioterapia, como siempre que realizamos una resección
conservadora. Por otra parte, también es necesario administrar poliquimioterapia, ya que existen
ganglios linfáticos con invasión tumoral. Sin embargo, no disponemos de datos sobre la presencia
o ausencia de receptores hormonales en el tumor, por lo que no podemos saber con estos datos si
es tributaria o no de recibir hormonoterapia.(R/1)

112. Si al examinar a una paciente con cáncer de cervix usted encuentra una “pelvis
congelada” y el ultrasonido le informan hidronefrosis izquierda, el estadio clínico
correcto sería:

1. 1. IIb.
2. 2. IIIb.
3. 3. IVa.
4. 4. IIIa.
Gráfico de respuestas
Comentario

!
!
!

Concepto poco preguntado en el ENARM, eventualmente preguntan estadiajes pero no es lo más


común te aconsejo que los repases una semana antes cuando mucho antes. Estadio IIIb de la
FIGO: tumor que invade la pared pélvica, que causa hidronefrosis o riñón no funcional. (R/2)

113. En la sala de partos se observa que un lactante tiene movimientos respiratorios, pero
que el aire no entra en los pulmones si la boca está cerrada. El diagnóstico más probable:

1. 1. Atresia de coanas.
2. 2. Hernia diafragmática.
3. 3. Hipoplasia pulmonar.
4. 4. Cardiopatía congénita.
Gráfico de respuestas
Comentario

Esta pregunta es bastante intuitiva. Están describiendo una apnea obstructiva (en las apneas
centrales, como la inducida por la narcosis, no hay excursiones torácicas). La única posibilidad de
apnea obstructiva descrita en la pregunta es la atresia de coanas. Vemos que el niño tiene
compromiso respiratorio cuando no puede utilizar la boca para respirar. En la hipoplasia pulmonar
por hernia diafragmática, el aire no entra en ningún caso, tenga la boca abierta o cerrada el
pequeño.(R/1)

114. La displasia broncopulmonar es el resultado de :

1. 1. Daño inflamatorio pulmonar en un feto en desarrollo.


2. 2. Falla en el desarrollo de las arteriolas pulmonares en el RN.
3. 3. Falla en el desarrollo bronquial del RN.
El uso de oxígeno y presión positiva durante el síndrome de dificultad
4. 4.
respiratoria-RN.
Gráfico de respuestas
Comentario

Pregunta sencilla sobre DBP. La respuesta correcta es la 4, ya que es causada por el uso de
oxígeno y presión positiva. Repase la siguiente tabla.

!
!
!

115. Los anticuerpos anti-Ro no se asocian a:

1. 1. Lupus cutaneo subagudo.


2. 2. Haplotipo DR-3.
3. 3. Lupus neonatal.
4. 4. Wolf Parkinson White en lactantes.
Gráfico de respuestas
Comentario

En el lupus eritematoso sistémico (LES) es característica la relación de diversas manifestaciones


clínicas con la presencia de algunos anticuerpos. Un ejemplo es lo que ocurre con los anticuerpos
anti-Ro que aparecen en un 30% de los pacientes con LES, también denominados SS-A. Se ha
visto que se relacionan con el lupus cutáneo subagudo, LES del anciano y lupus neonatal. En esta
última situación el paso de anticuerpos de la madre al feto durante la gestación puede producir en
el recien nacido lesiones cutáneas o bloqueo cardiaco congénito, pero no un WPW como indica la
respuesta número 4. La aparición de este anticuerpo es más habitual en los sujetos con HLA- DR3.
Además de en el LES es muy haitual la presencia de anticuerpos anti- Ro en el síndrome de
Sjögren (60%). (R/4)

116. Masculino de 45 años acude a su consulta por presentar hematuria. En la exploración


clínica se aprecia linfedema en miembros inferiores. El tacto rectal muestra próstata con
superficies irregulares laterales. Es cierto que:

1. 1. Tendrá un PSA elevado con baja concentración de fosfatasa ácida.


2. 2. Tendrá un PSA bajo con elevada fosfatasa ácida.
3. 3. Tendrá elevado el PSA y elevada fosfatasa ácida.
4. 4. Tendrá disminuida la enolasa neuroespecífica.
Gráfico de respuestas
Comentario

El caso que nos plantean corresponde, con bastante probabilidad, a un cáncer de próstata, ya que
el tacto rectal nos habla de una glándula de superficie irregular. Por ello, lo más probable es que el
PSA esté aumentado, como suele suceder en el cáncer de próstata. En cuanto a la fosfatasa ácida
prostática, recuerde que, cuando está elevada, suele deberse a afectación extraprostática. La
presencia de linfedema en miembros inferiores es sugerente de afectación de los ganglios
linfáticos regionales, por lo que cabría esperar el aumento de este marcador.(R/3)

117. El limite recomendado de 15 mmHg para el neumoperitoneo, ¿tiene relación con que
parámetro?

1. 1. Distensibilidad de la pared abdominal.


2. 2. Compresibilidad de la pared arterial.
3. 3. Presión venosa.
4. 4. Presión de perfusión tisular.
Gráfico de respuestas
Comentario
!
!
!

Por encima de 15 mmHg el compromiso será de la circulación venosa (respuesta 3), de ahí que se
ponga ese límite para el neumoperitoneo.(R/3)

118. Respecto a la patología tumoral más frecuente en niñas, marcar lo CORRECTO:

1. 1. Ováricas.
2. 2. Vulvares.
3. 3. Cervicales.
4. 4. Trompas.
Gráfico de respuestas
Comentario

Los tumores de vulva, cérvix o trompas son típicos de mujeres adultas.

Los tumores de ovario, en cambio, pueden aparecer en todas las edades. En concreto, los tumores
germinales son muy frecuentes en niñas y adolescentes.(R/1)

119. Una mujer de 42 años de edad consulta en Urgencias de un hospital por presentar
desde hace 2 semanas, deterioro de su estado general, fiebre vespertina de bajo grado,
cifras elevadas de tensión arterial y edemas maleolares. Entre sus antecedentes
destacaba un episodio de artritis simétrica en ambos carpos dos años antes. Además
refería aparición ocasional de erupción cutánea en sus veranos en la playa. En los
exámenes de laboratorio realizados en Urgencias destacaba hemoglobina 10.2 g/dl,
creatinina 3.8 mg/dl, urea 75 mg/dl, y presencia de hematíes y cilindros en el sedimento
urinario. ¿Cuál sería la actitud más CORRECTA?

Iniciar tratamiento diurético y con antagonistas de los receptores de la enzima


1. 1.
conversora de la angiotensina y mandarla al domicilio para revisarla en consulta.
Iniciar tratamiento con esteroides e imnunosupresores aunque no disponga de
2. 2.
biopsia renal.
3. 3. Incluirla en protocolo de diálisis.
4. 4. Iniciar tratamiento antibiótico e hidratación intensa.
Gráfico de respuestas
Comentario

Ante la combinación de artritis, nefropatía y fotosensibilidad, deberíamos plantearnos un posible


lupus eritematoso sistémico. Nos plantean cuál sería la mejor actitud con respecto a sus problemas
renales. Vamos a enumerarlos para verlos con mayor claridad.

•! Creatinina y urea elevadas, es decir, existe un grado importante de insuficiencia renal, y


además de reciente aparición, por la historia que nos plantean (evolución de dos
semanas).
•! Hipertensión, edemas y alteraciones del sedimento (cilindros, hematuria), sugestivo de
síndrome nefrítico.

Existen varias formas de nefropatía lúpica, que pueden clasificarse con una biopsia renal y un
estudio histológico posterior. Sin embargo, estamos ante un caso grave, por lo que no podemos
hacer la biopsia y esperar a que nos llegue el resultado, porque corremos el riesgo de que alcance
un importante grado de deterioro. De hecho, ante la gravedad del caso que nos plantean, sería
bastante probable que estuviésemos ante una forma proliferativa difusa, que es la de mayor
gravedad. Aunque esto no se sabrá hasta realizarse el estudio histológico, debemos comenzar el

!
!
!

tratamiento cuanto antes, con esteroides e inmunosupresores, antes de que la función renal
empeore todavía más. La confirmación histológica podría plantearse en un segundo tiempo. Lo que
no podemos diferir es el tratamiento, si ha alcanzado un deterioro tan importante en sólo dos
semanas.(R/2)

120. La indicación más frecuente de trasplante hepático en la infancia es:

1. 1. Cirrosis post-hepatitis A.
2. 2. Atresia biliar.
3. 3. Fallo hepático fulminante.
4. 4. Enfermedades metabólicas.
Gráfico de respuestas
Comentario
Debes conocer ciertas cosas sobre el transplante hepático, por lo menos indicaciones,
contraindicaciones y principales complicaciones. En adultos, en general, todas las causas de
hepatopatía terminal son candidatas a transplante. Entre ellas la cirrosis etílica en abstinencia, la
viral, la cirrosis biliar primaria, cirrosis biliar secundaria, colangitis esclerosante primaria, hepatitis
fulminante, hemocromatosis, enfermedad de Caroli y hepatocarcinoma. El colangiocarcinoma no es
indicación de transplante hepático por sus malos resultados. En los niños la atresia de vías biliares
es la indicación más frecuente; otras indicaciones son los trastornos genéticos o heredados del
metabolismo, fibrosis hepática congénita...(R/2)

121. A 55-year-old man with a past medical history significant for smoking, alcoholism
and a recently diagnosed inoperable pancreatic adenocarcinoma comes to the
emergency department with concerns regarding the appearance of various tender bumps
on his legs. Some of this bumps are ulcerated. A biopsy is obtained. Which of the
following is the most likely result?

1. 1. Septal panniculitis without vasculitis.


2. 2. Septal panniculitis with vasculitis.
3. 3. Lobular panniculitis without vasculitis.
4. 4. Lobular panniculitis with vasculitis.
Gráfico de respuestas
Comentario

La lesión elemental que caracteriza a las paniculitis es el nódulo. Clínicamente, las paniculitis
pueden ser muy difíciles de distinguir entre sí, precisándose biopsia en muchos casos para el
diagnóstico, así como una buena historia clínica.

Las enfermedades pancreáticas pueden producir paniculitis, puesto que la liberación de enzimas
pancreáticas al torrente sanguíneo puede producir la digestión de la grasa del tejido celular
subcutáneo. Dado que se trata de una esteatonecrosis por las lipasas pancreáticas, lógicamente
será una paniculitis LOBULILLAR, porque la grasa está en los lobulillos. Histológicamente, no se
encontrará vasculitis (a diferencia del eritema indurado de Bazin, que es lobulillar CON
vasculitis).(R/3)

122. En un periodo de 3 meses, un hombre de 42 años ha presentado cuatro episodios


de desconexión de su entorno, asociados con postura distónica de la mano derecha y
movimientos de masticación, de un minuto de duración, quedando a continuación
confuso y con dificultad para la expresión verbal durante 10 minutos. Después se

!
!
!

recupera con normalidad pero no recuerda lo que le ha ocurrido.¿Cuál es la actitud más


correcta?

1. 1. Iniciar tratamiento con clobazam oral y realizar un electroencefalograma (EEG).


Iniciar tratamiento con carbamacepina y realizar una resonancia magnética (RM)
2. 2.
y un EEG.
Ingresar en la unidad de cuidados intensivos e iniciar tratamiento con fenitoína
3. 3.
ultravenosa.
4. 4. Recomendar observación por su familia y volver a revisión en tres meses.
Gráfico de respuestas
Comentario

El cuadro que nos describen corresponde a una crisis parcial. El siguiente paso sería distinguir el
tipo. La diferencia entre una crisis parcial simple y una compleja es el deterioro del nivel de
conciencia, que sólo se produce en las últimas, mientras que en las simples está conservado. Por
otra parte, dado que el paciente ha padecido ya cuatro episodios semejantes, la necesidad de
tratamiento está clara, aparte del EEG y las pruebas de imagen (RMN), de modo que la opción
correcta es la 2.

El tratamiento de elección, en este caso, sería la carbamacepina. Es interesante que conozca


algunos detalles sobre este fármaco, ya que se ha preguntado en múltiples ocasiones en el
examen ENARM es el tratamiento de elección de la neuralgia del trigémino.

•! Como antiepiléptico, es muy útil para las crisis parciales.


•! A nivel hepático, es inductor del metabolismo de otros fármacos, y también del suyo propio
(autoinductor).
•! Puede producir neutropenia, así pues es recomendable solicitar una BH en las revisiones
de estos pacientes.(R/2)

123. A 5-month-old infant is brought to the doctor's because of scaly asymptomatic


lesions accompanied by erythema. These lesions are predominantly located on his scalp
area and around his nose, eyebrows and eyelids. His father says that the spots improve
after shampoo application. What is the most likely diagnosis?

1. 1. Atopic dermatitis
2. 2. Psoriasis
3. 3. Seborrheic dermatitis
4. 4. Impetigo
Gráfico de respuestas
Comentario
Seborrheic dermatitis. Seborrheic dermatitis is an inflammatory skin disorder affecting the scalp,
face, and torso. Typically, seborrheic dermatitis presents with scaly, flaky, itchy and red skin. It
particularly affects the sebaceous-gland-rich areas of skin. When it affects the head is called cradle
cap. Psoriasis lesions are scaly white plates sometimes with an erythematous base. Atopic
dermatitis occurs in areas of extension, cheeks and hairless areas.(R/3)

124. Niño de 11 meses de edad, ingresa por urgencias con movimientos tónico clónico
generalizados, asociado a estornudos, rinorrea y tos seca desde hace un día, sin

!
!
!

alteración previa de su estado de conciencia y sin antecedentes de episodios previos


similares, con Tº axilar de 39.3ºC.; sin otros signos en el examen físico. Su hipótesis de
diagnóstico nº 1 sería

1.! ! 1.! Meningoencefalitis!bacteriana.!


2.! ! 2.! Convulsión!febril.!
3.! ! 3.! Hemorragia!subaracnoidea.!
4.! ! 4.! Hipoxemia.!
Gráfico de respuestas
Comentario

En esta pregunta tiene que fijarse en varias cosas: 1 la edad, 2 fiebre y 3 le piden que de su
hipótesis número 1, por lo que la respuesta correcta es la número 2 convulsiones febriles.

Las crisis febriles son un proceso típico de la edad infantil entre los 3 meses y los 5 años de edad,
que se relaciona más frecuentemente con el aumento de temperatura, lo que da lugar a una crisis
el primer día de un proceso febril, independientemente del origen del mismo.

Las crisis febriles simples son generalizadas, duran menos de 15 minutos, presentan buena
recuperación posterior y los hallazgos en el periodo intercrítico son normales o negativos. Con
frecuencia existen antecedentes familiares de crisis febriles o de epilepsia; son recurrentes en un
tercio de los casos, aunque sólo el 10% de los pacientes sufre más de dos episodios, de forma
más probable si las crisis se producen en el primer año de vida; no se relacionan con un mayor
riesgo de presentar epilepsia.

Las crisis febriles complejas son las que tienen signos focales, una duración superior a 15 minutos,
o se repiten en el curso del mismo episodio febril; se relacionan con un 2-5% de incremento del
riesgo de sufrir epilepsia con posterioridad.

Las crisis febriles pueden tratarse con diazepam vía recta o IV, aunque dado que ceden
espontáneamente, el manejo más adecuado es el control de la temperatura, preferiblemente con
paracetamol. En pacientes con crisis febriles típicas recurrentes puede administrarse diazepam oral
o rectal en situaciones de ascenso térmico. No está indicado el tratamiento continuado con
anticomiciales como profilaxis de crisis febriles.(R/2)

125. Niña de 4 meses de vida. Antecedentes de retraso en la evacuación de meconio y


estreñimiento que acude con distensión abdominal y vómitos biliosos de 24 h de
evolución. La radiografía simple de abdomen muestra distensión de asas intestinales y
ausencia de aire distal. Indique cuál debe de ser el manejo más adecuado:

Lo más probable es que el paciente presente una Enfermedad de Hirschprung que


1. 1. requerirá colostomía urgente con extirpación del segmento agangliónico dilatado
en el mismo acto.
En la medida de lo posible se recomienda un tratamiento conservador inicial con
enema de suero y estimulaciones rectales hasta la realización de manometría,
2. 2.
enema opaco y biopsia rectal que faciliten el diagnóstico y proporcionen datos
sobre la extensión del intestino afecto.
La toma de biopsias intestinales durante la realización de la ileostomía no debe
3. 3.
realizarse pues convierte una cirugía limpia en contaminada.

!
!
!

Si no existen antecedentes familiares de enfermedad de Hirschprung, lo más


4. 4. probable es que tenga un válvulo intestinal y su tratamiento es conservador con
sonda nasogástrica y antibioterapia intravenosa.
Gráfico de respuestas
Comentario

La enfermedad de Hirschprung o aganglionosis colónica es todo un clásico en las preguntas de


Pediatría.

Antecedentes típicos son estreñimiento desde el período neonatal (tapón meconial) y a lo largo del
primer año de vida (esta paciente tiene 4 meses), debido a la peristalsis inadecuada del segmento
agangliónico. Otros signos típicos serían la distensión abdominal (megacolon) debido a la retención
fecal y ausencia de gas distal, donde generalmente asienta el tramo agangliónico. Entre las
complicaciones probables nos encontramos la de la pregunta: obstrucción.

Inicialmente se toman medidas de desobstrucción, hasta la realización de pruebas más específicas


que confirmen la sospecha diagnóstica (enema, manometría y biopsia).(R/2)

126. ¿En cuál de los siguientes procesos NO está indicada la esplenectomía?:

1. 1. Esferocitosis hereditaria.
2. 2. Tricoleucemia.
Mielofibrosis idiopática con esplenomegalia gigante y molesta, y citopenias
3. 3.
debidas a hiperesplenismo.
4. 4. Drepanocitosis.
Gráfico de respuestas
Comentario

Utillizando algunos conceptos básicos sobre estas enfermedades podemos contestar esta
pregunta sin mayores problemas. La esplenectomía estará indicada en aquellas enfermedades
cuyo origen sea el bazo (opción 2) o como método paliativo para disminuir la destrucción de
células sanguíneas (esferocitosis hereditaria, púrpura trombocitopénica que no respoonde a
glucocortocoides, mielofibrosis con citopenia por hiperesplenismo). También debes recordar que
en la drepanocitosis existe una tendencia a microinfartos de repetición en múltiples órganos, entre
ellos el bazo. Esto condiciona una "autoesplenectomía" con hipoesplenismo, por lo que podrás
deducir que la esplenectomía quirúrgica aportará pocos beneficios a estos pacientes. (R/4)

127. A 12-year-old obese boy comes to his pediatrician reporting a 4-month history of
mechanic pain in his right thigh and knee. Physical examination reveals a 1-centimeter
discrepancy in the length of his legs and external rotation of his right leg. Which of the
following is the most likely diagnosis?

1. 1. Juvenile Idiopathic Arthritis.


2. 2. Slipped Capital Femoral Epiphysis.
3. 3. Stress fracture of the femoral neck.
4. 4. Perthes' disease.
Gráfico de respuestas
Comentario

Pregunta difícil de ortopedia, al tratarse de una afectación unilateral podríamos descartar una AJI.
Por otro lado una fractura por estrés no sería lo más frecuente en este período de edad.

!
!
!

Epifisiólisis femoral proximal

Concepto y epidemiología. Deslizamiento entre el cuello femoral y la epífisis femoral proximal,


quedando esta última posterior e inferior. Es la causa más frecuente de dolor, claudicación de la
marcha y limitación de la movilida d de la cadera del adolescente. Es 2-3 veces más frecuente en
pacientes afroamericanos y en el sexo masculino (en el qu e se produce entre los 11 y los 16 años)
que en el femenino (10-14 años). Se asocia a obesidad (el 70 % están por encima del percentil 95),
hiperactividad, retroversión femoral, alteraciones endocrinas qu e alteran la fisis (hipogonadismo,
hipotiroidismo, déficit de GH , hipopituituarismo), raquitismo renal y síndrome de Down .
Clásicamente, se pensaba qu e la incidencia de bilateralidad era del 25-30% , pero actualmente se
reconocen cifras mayores (40-60% ) , especialmente cuando se asocia a alguna
endocrinopatía.(R/2)

128. Se sospecha en EPI crónica, cuando una paciente presenta:

1. 1. Metrorragias.
2. 2. Infertilidad.
3. 3. Fiebre persistente.
4. 4. Signos de peritonitis.
Gráfico de respuestas
Comentario

Los datos característicos de la EPI crónica son dolor, dispareunia, dismenorrea, menorragia
(sangrado relacionado con la menstruación) e infertilidad.(R/2)

129. En cuál de las siguientes entidades esta especialmente indicada la plasmaféresis:

1. 1. Goodpasture.
2. 2. Wegener.
3. 3. Panarteritis nodosa.
4. 4. Schonlein-Henoch.
Gráfico de respuestas
Comentario

Esta pregunta requiere saber la patogenia de las enfermedades mencionadas en cada opción. Las
opciones 2, 3 y 4 son síndromes vasculíticos que afectan al riñón secundariamente a la inflamación
de sus vasos en cada caso. Sin embargo, debe recordar que la enfermedad de Goodpasture se
debe a autoanticuerpos antimembrana basal glomerular dirigidos expresamente contra el colágeno
tipo IV de la membrana basal teniendo como clínica característica la asociación de una hemorragia
alveolar difusa y GN rápidamente progresiva. La plasmaféresis es un tratamiento útil de esta
enfermedad puesto que sirve para lavar la sangre de estos autoanticuerpos, lo cual evita el daño
de dicha membrana.(R/1)

130. En uno de los siguientes trastornos está alterada la capacidad de dilución de la orina:

1. 1. Intoxicación por litio.


2. 2. Hipoalbuminemia intensa.
3. 3. Diabetes insípida.
4. 4. Fase poliúrica de la insuficiencia renal aguda.
Gráfico de respuestas
Comentario

!
!
!

En esta pregunta lo importante es que reconozca qué transtornos determinan una alteración en la
capacidad de concentración de orina (isostenuria) y por qué.

Tanto la hipercalcemia como la intoxicación por litio son causas conocidas de nefritis intersticial
crónica, entidad que ocasiona defectos en la acidificación de la orina y en la concentración urinaria.
En la diabetes insípida (tanto la central como la nefrogénica) se ve anulada la actividad de la ADH -
porque no existe o porque no funciona- con lo cual el organismo no puede retener agua en la zona
del túbulo colector dando así lugar a una orina muy diluída. En la fase poliúrica de la IRA los
túbulos se encuentran alterados, no reabsorben bien ni iones, ni agua de forma que la orina no se
puede concentrar. (R/2)

131. La glomerulonefritis proliferativa mesangial se asocia con MENOS frecuencia a:

1. 1. Púrpura de Henoch-Schönlein.
2. 2. LES.
3. 3. Crioglobulinemia.
4. 4. Aterosclerosis difusa.
Gráfico de respuestas
Comentario

Esta pregunta presenta cierta dificultad, pues es necesario conocer concretamente la anatomía
patológica de ciertas GN secundarias.

Si analizamos las opciones una a una:

La opción 1, la púrpura de Henoch- Schölein es considerada por muchos autores como una forma
clínica de la GN mesangial IgA, en la que además de las manifestaciones renales, se suman las
articulares, digestivas y cutáneas.

El LES (opción 2) produce daño renal que es clasificado por la OMS en distintos grupos, uno de
ellos es la GN mesangial lúpica, que a diferencia de las dos entidades anteriores, muestra depósito
de IgG, IgM y complemento además de IgA.

La opción 3, crioglobulinemia, es también una causa de formación de IC circulantes que se


depositan en el mesangio dando una GN mesangial o mesangiocapilar. No olvidar que se asocia a
hipocomplementemia y característicamente a pseudotrombos de globulina PAS+.

Finalmente, la opción que se asocia con menos frecuencia a GN proliferativa mesangial es la


aterosclerosis difusa (opción 4), puesto que en esta situación el daño histológico se centraría en
los vasos (arterias y arteriolas), y no justificaría por sí mismo una proliferación celular en el
mesangio.(R/4)

132. ¿A qué edad recomienda la Academia Americana de Pediatría que se detecte la


hipoacusia?

1. 1. Al nacer.
2. 2. 3 meses.
3. 3. 4-5 años.
4. 4. 12 años.
Gráfico de respuestas
Comentario
!
!
!

Pregunta difícil por ser memorísitca, la respuesta correcta es la 2, 3 meses.(R/2)

133. Respecto al tratamiento del cáncer de endometrio señale la respuesta FALSA:

1. 1. El tratamiento fundamental es quirúrgico.


La radioterapia tiene un papel importante en aquellos casos en los que no es
2. 2.
posible el tratamiento quirúrgico.
La hormonoterapia con altas dosis de estrógenos se utiliza en aquellos casos de
3. 3.
enfermedad avanzada, metastásica o recidivas.
4. 4. La quimioterapia es poco eficaz en esta patología.
Gráfico de respuestas
Comentario

Respecto al tratamiento del cáncer de endometrio todas son correctas excepto la hormonoterapia,
que sí tiene un papel en casos de enfermedad avanzada, metastásica o recidivas pero que se
realiza con gestágenos a altas dosis y no estrógenos.(R/3)

134. Which of the following is not a radiographic finding suggestive of emphysema?

1. 1. Increased retrosternal airspace.


2. 2. Increased cardiothoracic index.
3. 3. Increased retrocardiac airspace.
4. 4. Bronchovascular markings do not reach the peripheral lung fields.
Gráfico de respuestas
Comentario

Recuerde que en el enfisema, los pulmones están excesivamente llenos de aire. Por tanto, todas
las respuestas serán ciertas, excepto la 2. El índice cardiotorácico se calcula como diámetro
cardíaco/ancho del tórax en la placa PA de tórax en inspiración. Al aumentar el ancho del tórax en
el enfisema (el denominador), el índice cardiotorácico estará disminuido.(R/2)

135. El tratamiento a largo plazo de la poliposis nasosinusal se basa fundamentalmente


en:

1. 1. Antihistamínicos tópicos nasales.


2. 2. Furosemida tópica.
3. 3. Corticoides tópicos nasales.
4. 4. Cirugía radical.
Gráfico de respuestas
Comentario
La poliposis nasosinusal a día de hoy es una enfermedad crónica que no se cura pero que se
controla aceptablemente con una combinación de tratamientos, siendo fundamentales los
corticoides tópicos nasales. Los pacientes con síndrome ASA son los más difíciles de controlar y
pueden requerir varias cirugías.(R/3)

136. Lactante de 6 meses, acude a Urgencias por tos intensa en accesos que acaban en
gallo inspiratorio, congestión facial, lagrimeo, salivación y vómitos intensos que impiden
su alimentación en la última semana. Señale la respuesta FALSA:

!
!
!

En la biometría hemática lo más frecuente es encontrar leucopenia con


1. 1.
neutrofilia.
2. 2. La complicación más frecuente es la neumonía por sobreinfección bacteriana.
3. 3. El tratamiento adecuado sería eritromicina durante 14 días.
4. 4. El agente responsable es un cocobacilo gramnegativo.
Gráfico de respuestas
Comentario

Pregunta de dificultad media en la que debemos reconocer en el enunciado el dato clave de la tos
ferina, que son los accesos de tos que terminan en "gallo" inspiratorio.

Sabiendo la enfermedad que es, ya podemos contestar que la correcta es la 1, puesto que es dato
básico para conocer de esta enfermedad que se presenta como leucocitosis con linfocitosis
absoluta debido a que esta bacteria libera un factor estimulante del crecimiento de los linfocitos.

La determinación de IgG antifactor estimulante es la prueba más sensible y específica para su


diagnóstico.(R/1)

137. Secundípara de 30 años a término. Su embarazo ha transcurrido con normalidad. Se


ha puesto de parto de forma espontánea con evolución normal hasta que se rompe la
bolsa con una dilatación de 4 cm. A partir de entonces comienza con hemorragia de
sangre roja en moderada cantidad y aparecen signos de sufrimiento fetal agudo. El
estado general de la mujer es bueno y la dinámica uterina es normal. Este cuadro
corresponde a:

1. 1. Placenta previa central.


2. 2. Abruptio placentae.
3. 3. Rotura uterina.
4. 4. Rotura de vasa previa.
Gráfico de respuestas
Comentario

Debe dominar este tema al 100% para el ENARM. En esta pregunta te están describiendo un caso
típico de rotura de vasa previa, cuadro de comienzo brusco que coincide con la amniorrexis de
expulsión de líquido amniótico teñido de sangre con buen estado general materno, y sufrimiento
fetal con elevada mortalidad. El tono uterino acostumbra a ser normal y no existe dolor. Recuerda
su asociación con inserción velamentosa de cordón.

No olvide que dentro de las hemorragias del tercer trimestre no es de las causas más comunes. La
sospecha diagnóstica es la constatación de vasos que laten en la bolsa amniótica, y el tratamiento
se basa en la cesárea urgente dados los signos de sufrimiento fetal.(R/4)

138. Indique lo CORRECTO en relación a la sarcoidosis:

1. 1. La piel está afectada en muy pocos casos.


2. 2. La afectación cutánea aparece en un 35% de casos aproximadamente.
3. 3. Las adenopatía hiliares no suelen ser simétricas.
4. 4. Las adenopatías paratraqueales aparecen en pocos casos.
Gráfico de respuestas
Comentario

!
!
!

La piel está afecta en el 35% de los casos de sarcoidosis. La lesiones más frecuentes son el
eritema nodoso, las placas, las erupciones maculopapulosas, los nódulos subcutáneos y el lupus
pernio. La afectación neurológica ocurre en el 5% de los casos y lo más frecuente es la afectación
del VII par con parálisis facial unilateral. En la radiografía lo característico es el aumento de los
ganglios hiliares bilaterales y también es frecuente el aumento de las adenopatías paratraqueales,
generalmente derechas.(R/2)

139. En cuanto a la inversión uterina, NO es cierto:

Suele relacionarse con el mecanismo de tracción en el momento del


1. 1.
alumbramiento.
2. 2. Al estar aumentado el tono uterino, es poco frecuente el riesgo de hemorragia.
3. 3. Debe intentar reducirse de forma manual.
4. 4. La histerectomía obstétrica puede ser la última alternativa terapéutica.
Gráfico de respuestas
Comentario
La inversión uterina es el prolapso del fondo uterino a través del cervix. Está producida por
mecanismos de tracción durante el alumbramiento. El riesgo de sangrado es muy alto, aunque a
veces la reposición a su forma natural es dificultosa por el aumento del tono uterino o por la
formación de un anillo cervical que dificulta su reposición. El tratamiento es intentar una reposición
manual, aunque a veces, en caso de no conseguirlo o por atonía uterina o metrorragia postparto
hay que proceder a una histerectomía obstétrica como última alternativa.(R/2)

140. El tratamiento de elección en pacientes con hemorroides trombosadas y dolorosas


consiste en:

1. 1. Escisión de las hemorroides.


2. 2. Incisión de las hemorroides y evacuación de coágulos sanguíneos.
3. 3. Ligadura de las hemorroides con bandas de caucho.
4. 4. Inyección de una solución esclerosante.
Gráfico de respuestas
Comentario
Las hemorroides internas se producen por dilatación del plexo venoso hemorroidal interno formado
por venas rectales superior y media. Se clasifican en IV grados según la intensidad del prolapso. El
grado I permanece en recto y su tratamiento es conservador. El grado II prolapsa a través del ano
cuando el paciente puja, reduciéndose espontáneamente y se trata con ligadura de vaina de
caucho o bien esclerosis. El grado II prolapsa cuando el paciente puja pero es necesaria la
restitución manual y el tratamiento es con ligadura de vaina de caucho. El grado IV es el prolapso
persistente cuyo tratamiento es la hemorroidectomía.(R/1)

141. ¿Cuál es el tratamiento más efectivo en el espasmo infantil o síndrome de West?

1. 1. Valproato.
2. 2. Difenilhidantoína.
3. 3. Carbamazepina.
4. 4. ACTH.
Gráfico de respuestas
Comentario

!
!
!

El síndrome de West aparece en el primer año de vida, más frecuentemente entre el 4° y 7° mes, y
predomin a en varones (1.5:1). Puede ser de origen criptogénico o sintomático (más frecuente); de
hecho, cualquier daño cerebral importante que pueda generar epilepsia a esta edad,
probablemente lo hará en la forma de este síndrome, con lo que el listado de posibles causas es
muy extenso. La tríada que define el síndrome consta de :

Espasmos infantiles

Detención del desarrollo psicomotor.

Hipsarritmia intercrítica.

El tratamiento se basa en: ACTH , corticoides, clonazepam , vigabatrina. (R/4)

142. Señale la afirmación CORRECTA en referencia a los tumores de glándulas salivales:

Los tumores que asientan en la glándula parótida tienen mayor probabilidad de


1. 1.
ser malignos que los que asientan en las glándulas salivales menores.
El tumor de Warthin es un cistoadenocarcinoma papilífero que afecta
2. 2.
preferentemente la glándula submaxilar con preponderancia en el sexo femenino.
3. 3. El adenoma pleomorfo es el tumor benigno más frecuente en la glándula parótida.
El adenoma pleomorfo, a diferencia del tumor de Warthin, sufre una
4. 4.
hipercaptación característica del tecnecio 99.
Gráfico de respuestas
Comentario

Los tumores de las glándulas salivales no son un tema primordial, pero esta pregunta viene bien
para recordar algunos conceptos fundamentales:

•! A menor tamaño de la glándula, mayor probabilidad de que los tumores que asienten en
ella sean malignos. Por ello, al ser la parótida la más grande, lo más frecuente sería que
fuesen benignos.
•! La parálisis facial es típica de los tumores malignos (infiltración del nervio).
•! El tumor de Warthin recibe también el nombre de CISTOADENOLINFOMA (no
cistoadenocarcinoma). A pesar de que su nombre es horrible (termina en LINFOMA), no es
un verdadero linfoma, de hecho es benigno. Se llama CISTOADENOLINFOMA porque
forma quistes (CISTO), estructuras glandulares (ADENO) y presenta un infiltrado
linfocitario (LINFOMA). Pero NO es maligno.
•! El adenoma pleomorfo, o tumor mixto, es, como dice la respuesta 3, el más frecuente de la
parótida. Se llama tumor mixto porque no sólo genera estructuras glandulares derivadas de
la glándula en sí, sino también cartílago.
•! Tanto el adenoma pleomorfo como el tumor de Warthin aparecen como calientes en la
gammagrafía, pero el que presenta una imagen de mayor captación no es el pleomorfo
sino en el de Warthin, al contrario de lo que indica la respuesta 4.(R/3)

!
!
!

143. Una mujer de 63 años acude muy preocupada por escapes de orina a lo largo del día.
Al interrogatorio refiere que presenta escapes importantes con la tos, la risa y los
estornudos, pero que también los presenta a veces porque no le da tiempo de ir al baño.
En este caso:

Estamos ante una incontinencia urinaria de esfuerzo y deberá ser tratada con
1. 1.
anticolinérgicos.
Estamos ante una incontinencia de urgencia y la paciente deberá ser tratada con
2. 2. ejercicios de suelo pélvico, y si estos no funcionan, se le colocará una malla
suburetral.
Estamos ante una probable incontinencia mixta que se beneficiaría de la
3. 3. combinación de los tratamientos independientes para ambos tipos de
incontinencia.
4. 4. Bajo ningún concepto podrían usarse anticolinérgicos en esta paciente.
Gráfico de respuestas
Comentario

Existe un evidente componente de incontinencia urinaria de esfuerzo que podríamos tratar con
ejercicios de suelo pélvico. Además presenta lo que parece una incontinencia urinaria de urgencia
probablemente candidata a tratamiento con anticolinérgicos pero en este caso estamos ante una
incontinencia urinaria mixta que requerirá tratamiento independiente de sus dos componentes para
un buen resultado.(R/3)

144. ¿Qué anticuerpo se identifica mediante IFI sobre el hemoflagelado Crithidia luciliae?

1. 1. Anti-DNA nativo.
2. 2. Anti-DNA monocatenario.
3. 3. Anti-Sm.
4. 4. Anti-Scl-70.
Gráfico de respuestas
Comentario

No se preocupe si has fallado esta pregunta, su dificultad es evidente y es altamente improbable


que aparezca en el ENARM.

Las técnicas más específicas para detectar anticuerpos anti-DNA nativo o bicatenario son el
radioinmunoanálisis (RIA) y la inmunofluorescencia indirecta sobre Crithidia luciliae. Se trata de un
protozoo cuyo DNA es circular y en forma de hélice, contenido en el interior de una mitocondria
gigante. (R/1)

!
!
!

145. En la revisión de un paciente de 43


años realizada habitualmente por litiasis de repetición que ha precisado litotricia en
numerosas ocasiones se observa la imagen. Ante ella:

1. 1. Habría que programar nuevamente una sesión de litotricia.


2. 2. Habría que realizar una biopsia de manera preferente.
Los síntomas de esta patología son frecuentemente hematuria+dolor intenso de
3. 3.
larga evolución.
4. 4. Habría que continuar un programa de seguimientos habituales.
Gráfico de respuestas
Comentario

Una imagen ecográfica muy sugestiva de quiste renal simple. Recuerda que esta entidad tiene tres
características muy típicas y que, por sí solas, bastan para realizar el diagnóstico ecográficamente,
sin necesidad de confirmación posterior con TC.

- Contornos lisos bien definidos.

- Anecogénico.

- Signo del refuerzo posterior.

La actitud correcta ante un quiste renal simple no es la TC ni la biopsia renal, sino sencillamente la
observación periódica por su problema renal de base (litiasis renal).(R/4)

146. Con la determinación por encima de lo normal de hidrógeno espirado tras la ingesta
de 50 g de lactosa, podemos decir que el problema del enfermo consiste en:

1. 1. Un trastorno difuso de la mucosa intestinal.


2. 2. Déficit de lactasa primario o secundario.
3. 3. Una atrofia vellositaria.
4. 4. Infección por Helicobacter pylori.

!
!
!

Gráfico de respuestas
Comentario

Se trata de una pregunta directa. El test del aliento con lactosa marcada con H2 será patológica
cuando en el intestino haya déficit de lactasa (ya sea primario o secundario, tras diarrea severa,
por ejemplo). Esto impide que la lactosa pueda degradarse en azúcares simples (glucosa y
galactosa) para absorberse. Es un test muy sencillo, no invasivo y con una gran rentabilidad para
el diagnóstico de déficit de lactasa.(R/2)

147. La enfermedad de Paget de la mama tiene las siguientes características, EXCEPTO:

1. 1. Representa el 2.5% de todos los cánceres de mama.


2. 2. Pronóstico dependiente de la presencia o no de tumor subyacente.
3. 3. Infección crónica de pezón o areola.
4. 4. Lesión rojiza, eczematosa y areola.
Gráfico de respuestas
Comentario

Pregunta sobre la enfermedad de Paget de la mama que puede ser fácil contestando por
eliminación. La enfermedad de Paget debe considerarse como la extensión de un tipo de cáncer de
mama ductal (ya sea in situ o invasor) a la epidermis suprayacente del pezón y la areola, cursando
como lesión eccematosa; fíjese que en este eccema no influye ningún tipo de infección sino que es
por infiltración tumoral, por lo que debemos elegir la opción 3. Es poco frecuente representando el
1-3% de todos los cánceres de mama (opción 1).(R/3)

148. Paciente de 55 años, fumador y bebedor, que


refiere disfonía desde hace aproximadamente 2 semanas. Acude a su consulta. ¿Cuál es
la actitud adecuada respecto a este paciente?

1. 1. Debe ser valorado por un otorrinolaringólogo.


2 semanas es poco tiempo para ser considerado un dato de alarma. Según las
2. 2. guías de consenso internacionales debe esperarse como mínimo 1 mes para
considerarlo un dato de alarma e indicar exploraciones complementarias.
El tratamiento indicado, al no existir datos de alarma, es hidratación adecuada,
3. 3.
corticoterapia, y reposo vocal.

!
!
!

Ante el primer dato de disfonía en un paciente fumador y bebedor está indicada la


4. 4.
realización de un TC de cuello.
Gráfico de respuestas
Comentario

Pregunta sencilla de un tema muy importante para el examen.

Los tumores de laringe se dividen en tumores supraglóticos, glóticos y subglóticos. Los tumores
glóticos dan síntomas de manera precoz, habitualmente disfonía.

Los tumores supraglóticos y subglóticos dan síntomas de manera tardía, pudiendo dar disfonía en
fases avanzadas.

En pacientes de riesgo (fumadores y bebedores), una disfonía de 15 días debe ser valorada por un
otorrinolaringólogo, quien realizará una adecuada exploración física, a ser posible mediante
nasofibroscopia.

Si existieran dudas, o si se quisiera valorar la extensión de la lesión tumoral, se indicaría un TC con


contraste, pero no antes.(R/1)

149. La afectación de las articulaciones interfalángicas distales de las manos, con


deformidad, con nódulos de Heberden y con dolor es característico de:

1. 1. Reumatismo poliarticular crónico.


2. 2. Poliartritis crónica primitiva.
3. 3. Gota.
4. 4. Artrosis.
Gráfico de respuestas
Comentario

La artrosis es fácil en el ENARM, y debe conocer algunas formas clásicas: los nódulos de
Heberden por sí mismos piden a gritos respuesta 4, que es la correcta. La afectación de
interfalángicas distales es bien típica y frecuente, especialmente en mujeres, en los que existe un
patrón de herencia dominante. La artrosis genera dolor y deformidades articulares, aunque a veces
en esta localización tan sólo supone un problema estético (engrosamiento articular dorsal). Los
reumatismos poliarticulares suelen afectar a otras articulaciones, como las metacarpofalángicas,
muñecas, hombros, rodillas? y tienen un gran componente inflamatorio sinovial; la gota tiene
predilección por la primera metatarsofalángica, la clásica podagra, con gran inflamación
también.(R/4)

150. Gestante de 16 semanas, que en el urocultivo de control, presenta bacteriuria


asintomática por E. coli. Curso del embarazo hasta la fecha dentro de la normalidad, sin
patologia asociada, refiere alergia a la penicilina. Según el antibiograma, el E. coli es
sensible a la ampicilina, fosfomicina, gentamicina y ciprofloxacino. ¿Cual de las
siguientes afirmaciones seria INCORRECTA?:

1. 1. No debe tratarse, ya que la paciente está asintomática.


2. 2. El antibiótico más utilizado durante el embarazo es la ampicilina.

!
!
!

3. 3. En este caso estaria indicado el tratamiento con fosfomicina.


4. 4. La gentamicina esta incluida en la categoria C de toxicidad fetal.
Gráfico de respuestas
Comentario
La bacteriuria asintomática durante el embarazo siempre debe tratarse, para evitar complicaciones
infecciosas del tracto urinario, por la dilatación ureteral propia del embarazo. Tanto la Ampicilina
como la Fosfomicina pertenecen a la categoria B de toxicidad fetal: no se han demostrado riesgos
para el feto en estudios en animales, aunque no existen estudios controlados en embarazadas. No
demostrado riesgo para los humanos. La Gentamicina y el Ciprofloxacino pertenecen a la categoria
C de toxicidad fetal: Carencia de estudios en animales y en humanos, en determinadas ocasiones
pueden administrarse, si la relación riesgo/beneficio lo aconseja.(R/1)

151. Acude a consulta un paciente miope con la


siguiente exploración del fondo de ojo derecho e izquierdo (ver imagen). Señale cuál de
la siguientes opciones es VERDADERA:

1. 1. Tiene más riesgo de desarrollar un glaucoma de ángulo cerrado.


2. 2. Cuando se operan de catarata se les coloca una lente de menor graduación.
Las fotopsias son síntomas comunes de los miopes que no sugieren ninguna
3. 3.
patología.
4. 4. Utilizan lentes biconvexas.
Gráfico de respuestas
Comentario

!
!
!

El fondo de ojo que nos presentan corresponde a un paciente con miopía magna y algunos signos
de neovascularización. En cualquier caso, no era necesario que fuera capaz de interpretarlo.
Bastaba la palabra “miopía”, que sale en el propio enunciado.

R1: Los miopes tienen mayor riesgo de glaucoma de ángulo abierto. El de ángulo cerrado se
relaciona con los hipermétropes.

R3: Las fotopsias deberían hacernos pensar en ciertas complicaciones de la miopía. Puede
tratarse de algo relativamente poco importante, como el desprendimiento del vítreo posterior, pero
también podrían aparecer en un desgarro retiniano.

R4: Las lentes que necesita el miope son bicóncavas (divergentes).(R/2)

152. A heart-transplanted 6 year-old-boy who is actually receiving Cyclosporine,


Azathioprine and low-dose steroid therapy is brought to the physician by his mother
because she is concerned about side-effects on the growth of her child. Which of the
following is the normal growth rate in this child?

1. 1. 2 cm per year.
2. 2. 3 cm per year.
3. 3. 4 cm per year.
4. 4. 7 cm per year.
Gráfico de respuestas
Comentario
Como muchas veces ocurre en las preguntas sobre cifras o porcentajes, la respuesta correcta está,
una vez más, en el valor intermedio. Se habla de talla baja cuando se encuentra por debajo del
percentil 3 para su edad, o por debajo de - 2 DS. A la hora de evaluar la talla de un niño, hay que
tener en cuenta la correlación con la talla de los padres, y la velocidad de crecimiento, de forma
que está indicado un estudio de talla baja en los niños con velocidad de crecimiento por debajo del
percentil 25 o menor de 4 cm / año, de forma mantenida.(R/3)

153. En el hipotiroidismo secundario o hipofisario NO existe:

1. 1. Hipercolesterolemia.
2. 2. Aumento de CPK.
3. 3. TSH baja o en el límite bajo de la normalidad.
4. 4. A veces se acompaña de otros déficits de hormonas hipofisarias.
Gráfico de respuestas
Comentario

Existen desde el punto de vista clínico varias diferencias entre el hipotiroidismo central o
secundario y el hipotiroidismo primario. Debe recordar que en el hipotiroidismo existe una
alteración del metabolismo lipídico a nivel hepático que provoca la presencia de
hipercolesterolemia. En los pacientes con hipotiroidismo central no se encuentra esta alteración.
Los pacientes con hipotiroidismo central suelen llevar un menor tiempo de enfermedad que los
pacientes de etiología primaria, ya que en la mayoría de los casos se presentan con otros déficits
concomitantes que hacen que se diagnostiquen precozmente y sean tratados antes de que

!
!
!

desarrollen alteraciones del metabolismo lipídico. Otra característica que diferencia estas dos
entidades es la presencia de bocio en algunos casos de hipotirodismo primario que está siempre
ausente en los de etiología central, ya que este aumento de la glándula tiroidea es secundario a la
elevación de las cifras de TSH. A nivel de laboratorio, el hipotiroidismo primario se caracteriza por
unos niveles de T4 y T3 bajos y elevados de TSH, mientras que el de etiología central, presenta
unos niveles de hormonas periféricas bajos y una TSH disminuida o inadecuadamente normal.(R/1)

154. El tratamiento de elección en los síndromes trombóticos microangiopáticos consiste


en:

1. 1. Glucocorticoides.
2. 2. Esplenectomía.
3. 3. Antiagregantes plaquetarios.
4. 4. Plasmaféresis.
Gráfico de respuestas
Comentario

Recuerde que el tratamiento de elección de los síndromes trombóticos microangiopáticos (SHU y


PTT) es la plasmaféresis. Otros tratamientos que pueden ser útiles son: esplenectomía, esteroides,
antiagregantes, citostáticos y rituximab.(R/4)

155. ¿Cuál de las siguientes asociaciones es FALSA?:

1. 1. Timoma - ginecomastia.
2. 2. Timoma - síndrome de Cushing.
3. 3. Linfoma - hipercalcemia.
4. 4. Tumor germinal - hipoglucemia.
Gráfico de respuestas
Comentario

Pregunta complicada que puede solucionar descartando las opciones claramente verdaderas y que
debe dominar, aunque este tema no es demasiado importante para el ENARM. Lo más importante,
es la tendencia del teratoma (y otros tumores de células germinales) a calcificarse; además, este
tumor se ha asociado a ginecomastia, hipoglucemia y tirotoxicosis. También es muy característica
las asociación del linfoma con la hipercalcemia, mediada por la producción de vitamina D por parte
del tumor. En cuanto al timoma (la masa más frecuente del mediastino anterior) es importante
recordar su asociación con la aplasia pura de células rojas y con la miastenia gravis. Menos
importantes son la agammaglobulinemia y el Cushing.(R/1)

156. ¿Qué arterias irrigan la capa funcional del útero (endometrio)?, marque lo
CORRECTO.

1. 1. Arcuatas.
2. 2. Radiales.
3. 3. Espirales.
4. 4. Rectas.
Gráfico de respuestas
Comentario

!
!
!

La capa funcional del endometrio es la que se desprende en cada menstruación. Sus arterias
reciben el nombre de arterias espirales, debido a su forma, como se puede ver en el dibujo.

Las arterias arcuatas y radiales son ramas principales de las arterias uterinas, que se encuentran
en el espesor del miometrio. De ellas nacen las arterias rectas, que irrigan la capa basal del
endometrio (la que no se desprende), y de estas últimas, las arterias espirales, que irrigan la capa
funcional, y que por tanto mueren y se regeneran con cada ciclo menstrual.(R/3)

157. A 37-year-old woman comes to your office complaining of a breast mass. Her medical
history is irrelevant, except for her grandmother, who was diagnosed with breast cancer
at age 54. Breast examination shows a slighty retracted left nipple, and a fixed mass is
palpated in the lower outer quadrant of the left breast. Mastography shows a 2x2 cm
spiculated mass with coarse calcifications, and ultrasonogram shows a hypoechoic
mass. Multiple core biopsy samples show foamy macrophages and fat globules. Which
of the following is the most appropriate next step in management?

1. 1. Routine follow-up and no intervention.


2. 2. MRI of the breast.
3. 3. Genetic studies.
4. 4. Radiation therapy of the left breast.
Gráfico de respuestas
Comentario

Pregunta sencilla sobre el estudio diagnóstico del cáncer de mama. Si tenemos una paciente con
lesiones sospechosas, que en distintas biopsias no se diagnostica de neoplasia, lo adecuado sería
continuar con un seguimiento, sin requerir otro tipo de tratamiento. NO requiere estudios genéticos
adicionales, ni mucho menos iniciar tratamiento con radiación.(R/1)

158. Anciana de 78 años con insuficiencia cardíaca sistólica compensada. Respecto a su


patología señala la CORRECTA:

!
!
!

1. 1. Los betabloqueantes están contraindicados.


2. 2. El carvedilol debe empezarse a dosis plenas.
3. 3. Los IECA no deben unirse a la digoxina.
4. 4. Es beneficioso unir la espironolactona al tratamiento con diuréticos más IECA.
Gráfico de respuestas
Comentario

Una pregunta de dificultad media-baja sobre el tratamiento de la insuficiencia cardíaca. La


respuesta correcta es la 4, puesto que estamos hablando de dos fármacos que prolongan la
supervivencia: IECAs y espironolactona, que pueden combinarse sin problemas. Recuerde que,
tanto uno como otro, pueden elevar el potasio sérico. Sin embargo, podemos contrarrestar este
efecto con diuréticos del asa o con tiazidas.

El resto de las opciones son incorrectas. Los betabloqueantes no estarían contraindicados, al


tratarse de una IC compensada. Por supuesto, no se comienza con dosis plenas, sino más bien
bajas, subiendo progresivamente. Por otra parte, no hay razones para no combinar IECAs con
digoxina, ni la amiodarona estaría contraindicada en esta paciente, en caso de necesitarla.(R/4)

159. La madre de un lactante menor requiere información sobre como calentar la leche
materna que se extrae y guarda en el refrigerador. Por lo que usted indica lo siguiente.

1. 1. Calentar el recipiente en agua hirviendo.


2. 2. Hervir la leche.
3. 3. Sumergir el recipiente en agua tibia.
4. 4. Ninguna de las anteriores.
Gráfico de respuestas
Comentario

La respuesta correcta es la 3, o el llamado baño María, para calentar la leche materna y


posteriormente refrigerarla.(R/3)

160. Sobre el mecanismo de transmisión, para evitar la infección por el VHB, es útil lo
siguiente EXCEPTO:

1. 1. Es necesario aislar los pacientes HBsAg positivo.


2. 2. Todo el personal sanitario debe vacunarse.
3. 3. La ebullición a 100 ºC diez minutos consigue la esterilización.
4. 4. Es importante como mecanismo de transmisión el mecanismo sexual.
Gráfico de respuestas
Comentario

Teniendo en cuenta la vía de transmisión del VHB (hematógena y sexual), los pacientes portadores
del virus (HBsAg positivos) no precisan aislamiento, por lo que la respuesta correcta es la 1. Los
que sí lo precisan son los pacientes con hepatitis A, ya que la transmisión del VHA es
orofecal.(R/1)

161. En el tratamiento empírico inicial de la meningitis en el periodo neonatal se emplean


dos antibióticos asociados (generalmente las cefalosporinas de tercera generación
asociadas a la ampicilina) para cubrir los gérmenes más frecuentes en este período.
Señale, de ellos, el que suele ser resistente a las primeras y sensible a la segunda:

!
!
!

1. 1. Neumococo.
2. 2. Haemophilus influenzae.
3. 3. Listeria monocytogenes.
4. 4. Pseudomonas aeruginosa.
Gráfico de respuestas
Comentario

Pregunta muy importante que no debe de olvidar para el ENARM. La listeriosis durante
elembarazopuede ser asintomática o causar una enfermedad febril, que puede confundirse con
una gripe o síndrome gripal. Habitualmente, el diagnóstico de certeza sólo es posible mediante el
hemocultivo, donde crecerá un bacilo aerobio, móvil y grampositivo. La infección, ya sea oculta o
manifiesta, puede estimular el trabajo de parto. La listeriemia materna produce una infección fetal,
que provoca característicamente lesiones granulomatosas diseminadas con microabscesos. La
eliminación de meconio es frecuente cuando existe infección fetal.

El neonato es particularmente susceptible a la infección por Listeria (recuerde que los ancianos
también lo son). La tasa de mortalidad de la listeriosis neonatal es cercana al 50%. En estos casos,
es usual la sepsis neonatal precoz, mientras que la listeriosis de inicio tardío se manifiesta como
una meningitis a las 3-4 semanas de edad. Estas infecciones son muy parecidas a las causadas
por estreptococo beta-hemolítico del grupo B. Hay evidencia de que el tratamiento antimicrobiano
materno puede ser eficaz en caso de infección fetal. Como tratamiento, se recomienda la
combinación de ampicilina y gentamicina.(R/3)

162. ¿Cuál es el tratamiento antibiótico ambulatorio de elección en la enfermedad


inflamatoria pélvica leve/moderada?:

1. 1. Clindamicina y gentamicina.
2. 2. Metronidazol.
3. 3. Amoxicilina-clavulánico y doxicilina.
4. 4. Ceftriaxona y doxiciclina.
Gráfico de respuestas
Comentario

Los agentes infecciosos más frecuentes, y los que deben quedar cubiertos, en la EIP son
Chlamydia trachomatis y Neisseria gonorrhoeae. En la enfermedad moderada/leve, el tratamiento
es, normalmente, ambulatorio con ceftriaxona (monodosis i.m. para cubrir gonococo) y doxiciclina
(14 días v.o. para la chlamydia). Si a las 48 horas no ha respondido o en el primer momento se
considera grave, el tratamiento es hospitalario con doxiciclina + ceftriaxona e.v. completándose el
tratamiento con doxiciclia oral 14 días.(R/4)

163. Son datos distintivos en las vasculitis el tamaño del vaso que afectan ó bien por ser
una patología primaria o secundaria a enfermedad sistémica subyacente. Señale la
respuesta INCORRECTA:

1. 1. Son vasculitis de vaso grande la arteritis de la temporal y la arteritis de Takayasu.


La afectación renal en la vasculitis pauciinmune se caracteriza por ausencia de
2. 2. depósitos inmunitarios en la inmunofluorescencia, generalmente en el suero
existen ANCAs positivos.
El síndrome de Kawasaki es una vasculitis infantil que afecta a los vasos
3. 3.
pequeños.
El síndrome de Churg-Strauss afecta a vasos de calibre pequeño produciendo
4. 4.
inflamación de vías respiratorias con infiltrado eosinófilo.

!
!
!

Gráfico de respuestas
Comentario

Recuerde que la PAM puede tener positividad para los pANCAS (característicos del Churg-
Strauss) y para los cANCAS (característicos del Wegener).

Sobre el Kawasaki, recuerde sus criterios diagnósticos con el abecedario: A (adenopatías), B


(boca- - > fisuras labiales...), C (conjuntivitis bilateral no exudativa), D (descamación distal dedos),
E (exantema) y F (fiebre, imprescindible). Recuerde que recientemente se ha descubierto que su
agente etiológico es el Coronavirus New Heaven.(R/3)

164. Los siguientes hongos son responsables de micosis internas, EXCEPTO uno:

1. 1. Histoplasma capsulatum.
2. 2. Sporotrix shenckii.
3. 3. Malassezia furfur.
4. 4. Aspergillus fumigatus.
Gráfico de respuestas
Comentario

Esta pregunta es de dificultad media- alta. Los hongos últimamente se están poniendo de moda, en
relación con el paciente VIH u otros tipos de inmunodepresión.

Blastomyces,HistoplasmayAspergillusson hongos causantes de micosis invasivas, tal como


explicamos en el Manual. La duda razonable está entre las opciones 2 y 3. La esporotricosis es
una infección cutánea que produce nódulos a lo largo del trayecto de los vasos linfáticos,
normalmente en relación con laceraciones cutáneas (vegetal con espinas, como un rosal). Sin
embargo, en pacientes inmunodeprimidos, puede ir más allá de la piel y producir cuadros graves y
atípicos, como la esporotricosis ósea, o también formas diseminadas. Sin embargo, M. furfurestá
implicada en la pitiriasis versicolor, que es una infección únicamente superficial, por lo que no
cabría esperar que se desarrollasen formas invasivas.(R/3)

!
!
!

165. Primogénito masculino que a los 21 días de


vida comienza con vómitos en proyectil en todas las tomas, asociados a estado de
irritabilidad y avidez por las tomas. Realiza la siguiente radiografía (ver imagen). Ante los
hallazgos descritos usted solicitaría:

1. 1. Exámenes de laboratorio porque sospecho gastroenteritis aguda.


2. 2. USG abdominal porque sospecho reflujo gastroesofágico.
3. 3. USG abdominal porque sospecho invaginación.
4. 4. USG abdominal porque sospecho estenosis hipertrófica de píloro.
Gráfico de respuestas
Comentario

Esta pregunta es un buen ejemplo de que, normalmente, debemos dar prioridad a los datos del
enunciado con respecto a los que vemos en las imágenes, que no dejan de ser pruebas
complementarias.

La estenosis hipertrófica del píloro suelen preguntarla en el ENARM exactamente como aparece en
este enunciado: recién nacido de unos 20 días con vómitos en proyectil, que sin embargo tiene
hambre (avidez por las tomas). La imagen que nos ofrecen es sólo un dato más, donde
encontramos una franca dilatación gástrica (observa la gran burbuja, densidad aire, en cavidad
abdominal),(R/4)

166. Con respecto al cuadro anterior, señale la FALSA:

1. 1. El diagnóstico de elección se basa en hallazgos clínicos y ecográficos.


2. 2. Es más frecuente en mujeres.
3. 3. Se asocia a estreñimiento.
4. 4. Precisa corrección quirúrgica.
Gráfico de respuestas
Comentario

!
!
!

La estenosis hipertrófica del píloro es una entidad relativamente frecuente (3 por cada 1000 recién
nacidos vivos), más frecuente en hombres (respuesta 2 falsa), sobre todo de raza blanca y con
mayor frecuencia si son los primogénitos de la familia. La incidencia parece ser mayor en los
grupos sanguíneos B y O. Entre otras asociaciones, se ha relacionado con la administración
intravenosa de prostaglandinas a la madre, y con el uso de eritromicina en el neonato.(R/2)

167. Mujer de 55 años que consulta por prurito y aparición de lesión eritematosa y
descamativa a nivel de pezón derecho que se ha ido extendiendo hacia la areola. En la
exploración se objetiva lesión eczematosa de aproximadamente 3.5 cm que abarca
pezón-areola derechos. No se palpan nódulos subyacentes ni se observa secreción por
pezón (ni espontánea ni a la expresión). Mama izquierda sin alteraciones en unidad
areola-pezón. No se palpan nódulos. Ambas axilas negativas. Señale la CORRECTA:

1. 1. Podemos asegurar que la patología está limitada a la piel.


2. 2. Es una localización poco frecuente de dermatitis atópica.
En la histología de la lesión podemos encontrar células grandes típicas de
3. 3.
citoplasma amplio y claro.
4. 4. Sólo sospecharemos malignidad si se asocia una lesión subyacente palpable.
Gráfico de respuestas
Comentario

Pregunta de dificultad media sobre la enfermedad de Paget de la mama.

La enfermedad de Paget debe considerarse como la extensión de un tipo de cáncer de mama


ductal a la epidermis suprayacente, afectando pezón y la areola. Es poco frecuente, representando
el 1-3% de todos los cánceres de mama. Clínicamente, consiste en una lesión eczematosa,
asociando también prurito. Por ello, el eccema es su principal diagnóstico diferencial. Una de las
características por las que puede distinguirse del eccema es por ser unilateral (el eccema suele
afectar a ambas mamas). Otro rasgo diferencial son los límites, que suelen ser difusos en el
eccema y mejor definidos en el Paget mamario.

Recuerden que, histológicamente, son características las células de Paget. Son grandes,
redondeadas, PAS- positivas, con núcleos de gran tamaño y sin puentes intercelulares que las
unan.(R/3)

168. El infarto mesentérico agudo se debe con mayor frecuencia a:

1. 1. Oclusión de la arteria mesentérica superior.


2. 2. Trombosis de la vena porta.
3. 3. Oclusión de la arteria mesentérica inferior.
4. 4. Oclusión del tronco celíaco.
Gráfico de respuestas
Comentario
La isquemia mesentérica aguda es un proceso obstructivo brusco de la arteria mesentérica
superior debido con más frecuencia por una embolia en pacientes con fibrilación auricular u otro
proceso embólico. Con menos frecuencia se debe a una trombosis in situ de una placa de
arteriosclerosis o a bajo gasto.(R/1)

169. ¿Cuál de los siguientes síntomas es más sugestivo de las depresiones


melancólicas?:

!
!
!

1. 1. Apetencia por los dulces.


2. 2. Despertar precoz.
3. 3. Mejoría matutina.
4. 4. Reactividad del humor.
Gráfico de respuestas
Comentario
La apetencia por dulces es típica de la depresión atípica. El despertar precoz es típico de la
depresión endógena o "melancólica". La mejoría matutina es típica de las neurosis y de las
depresiones del polo neurótico. La reactividad del humor es propia de la neurosis y de la depresión
del polo neurótico, y la hiperreactividad del humor es típica de la depresión atípica. Las ideas de
desesperanza son comunes a todas las depresiones.(R/2)

170. Hombre de 59 años que ingresa por síndrome miccional y fiebre. Entre sus
antecedentes destacan insuficiencia renal crónica, trasplante renal hace dos años, en
tratamiento con prednisona (10 mg/día), ciclosporina (100 mg/día), azatioprina (80
mg/día). Unos 10 días antes del ingreso comienza con disuria y polaquiuria, en el exámen
general de orina se observaban 10 leucocitos/campo y 3-4 eritrocitos/campo y en el
cultivo de orina crecieron 10^4 colonias de flora mixta. Su médico de cabecera indicó
tratamiento con ciprofloxacino, sin mejoría en los síntomas y con aparición de fiebre,
motivo por el que ingresa. En la exploración se observa: Tª 38° C, aspecto cushingoide,
auscultación cardiopulmonar sin hallazgos, percusión renal no dolorosa y abdomen no
doloroso. Entre los datos de laboratorio destacan: Hb 12.1 g/dl, leucocitos 7,500
(neutrófilos 96%, linfocitos 4%), plaquetas 200,000, creatinina 2.3 mg/dl, ultrasonido renal
sin signos de uropatía obstructiva ni de lesiones en el parénquima renal y cultivo de orina
negativo. En esta situación, ¿qué le parece más apropiado?

1. 1. Realizar tacto rectal.


2. 2. Iniciar tratamiento con aciclovir IV.
3. 3. Iniciar tratamiento con doxiciclina IV.
4. 4. Realizar TAC renal.
Gráfico de respuestas
Comentario

Una pregunta difícil.

Clínicamente, se trata de una infección urinaria con fiebre, y no parece ser una pielonefritis
(ultrasonido renal normal y puñopercusión negativa). Recuerde que, dentro de las IVUs, las
prostatitis también pueden producir fiebre, por lo que habría que orientar la exploración hacia este
diagnóstico (opción 1 correcta).

Por otra parte, aunque no sea el motivo principal de la pregunta, vamos a analizarla más
profundamente. El paciente tiene como antecedente un trasplante renal, por lo que está en
tratamiento con distintos inmunosupresores. Otra de las claves es la falta de respuesta al
tratamiento habitual de las prostatitis bacterianas, el ciprofloxacino… Ante la inmunodepresión y la
no respuesta al antibiótico, habría que considerar otras etiologías menos habituales. Dado que el
criptococo es un hongo que se elimina a través de la orina, en pacientes inmunodeprimidos puede
llegar al parénquima prostático, produciendo una prostatitis.(R/1)

171. Pueden ser causa de hipotiroidismo congénito transitorio, EXCEPTO:

!
!
!

1. 1. Exceso de yodo.
2. 2. Idiopático.
3. 3. Tiroides ectópica.
4. 4. Déficit de yodo.
Gráfico de respuestas
Comentario

Pregunta sencilla que puede resolver sin conocimientos extensos de hipotiroidismo congénito
transitorio.

El exceso de yodo puede causar un bloqueo en la síntesis de hormonas tiroideas llamado efecto
Wolff-Chaikov. El idiopático no lo selecciona, por que de cada enfermedad habrá una causa
idiopática. El déficit de yodo por obvias razones causa hipotiroidismo, por lo que la única opción
que queda y es la respuesta correcta es la número 3... Ya que un tiroides ectópico producirá
hormonas tiroideas.(R/3)

172. El proceso de exclusión alélica es aquél por el que (señale la respuesta CORRECTA):

Proceso por el que sólo se reorganiza y expresa una de las dos copias de DNA de
1. 1.
las cadenas pesadas y ligeras.
Proceso por el que se excluye la expresión de los genes de los segmentos V del
2. 2.
complejo VDJ.
3. 3. Proceso por el que se excluye la expresión de los genes de las cadenas ligeras.
4. 4. Proceso por el que se excluyen los alelos implicados en el reordenamiento VDJ.
Gráfico de respuestas
Comentario

La exclusión alélica hace referencia a la situación por la cual una célula B sólo expresa los genes
de las cadenas pesadas y ligeras de uno de los alelos de los cromosomas homólogos (el materno
o el paterno), y el otro jamás será expresado por esa célula. (R/1)

173. Paciente de 25 años, gestante de 37


semanas, que acude a Urgencias por cefalea intensa. El embarazo ha cursado con
normalidad hasta ese momento. Tensión arterial 180/120 mmHg mantenida. En el USG se
objetiva un CIR tipo II con oligoamnios. El doppler es el que se muestra en la imagen. El
registro cardiotocográfico es normal en ese momento. La conducta a seguir es:

!
!
!

Ingreso y administración de hidralacina i.v. y sulfato de magnesio. Si se consigue


1. 1. control de la tensión, se continuará con tratamiento vía oral con alfametildopa
hasta inicio espontáneo del trabajo de parto.
Ingreso, reposo en decúbito lateral izquierdo y administración de diuréticos para
2. 2.
el control de la tensión arterial.
3. 3. Ingreso para finalizar la gestación y tratamiento antihipertensivo i.v.
Ingreso y tratamiento con nitroprusiato. Si no se consigue control de la tensión
4. 4.
arterial se realizará cesárea.
Gráfico de respuestas
Comentario

Tema de vital importancia para el ENARM. Por mucho que haya estudiado, identificar lo que
aparece en la imagen es realmente muy difícil, salvo para un ginecólogo o un residente avanzado
de la especialidad. Sin embargo, podría haber acertado la pregunta, que es de lo que se trata en
realidad.

Lo que le están contando es, claramente, un cuadro de preeclampsia con mala evolución: cefalea
intensa e importante grado de hipertensión. Si además nos dicen que existe un CIR tipo II, debe
considerar que estará relacionado con un cierto grado de isquemia placentaria, como ocurre con
frecuencia en este tipo de CIR. Sea como sea, tenemos una preeclampsia de mala evolución, por
lo que es necesario el ingreso (se descarta la 3). El tratamiento de la preeclampsia consiste en
poner fin a la gestación, que en este caso es perfectamente posible, ya que el feto tiene 37
semanas y por lo tanto habrá alcanzado la madurez pulmonar. Lo que muestra la imagen es un
Doppler con un flujo umbilical invertido. Insistimos en que el objetivo de esta pregunta no es que
aprenda a reconocer este dato gráficamente, sino que tenga recursos para defenderse, aunque no
comprenda nada de la imagen.(R/3)

174. En los exámenes de laboratrio que se realiza a la paciente existe proteinuria, anemia,
trombopenia, elevación de transaminasas y elevación de la creatinina, con pruebas de
coagulación normales. Señale la afirmación FALSA respecto al cuadro que presenta la
paciente:

El tratamiento de elección en esta paciente es finalizar la gestación ya que se trata


1. 1.
de una pre-eclampsia grave.
2. 2. La paciente presenta una preeclampsia complicada con un síndrome HELLP.
3. 3. Estará indicado iniciar profilaxis de la eclampsia con sulfato de magnesio.
La paciente presenta una preeclampsia grave que obliga a terminar la gestación y,
4. 4.
dado que no ha iniciado el trabajo de parto, se realizará cesárea.
Gráfico de respuestas
Comentario

El tratamiento ideal de la preeclampsia es terminar la gestación, idealmente por vía vaginal


(respuesta 4 falsa). Ya sabemos que la realidad clínica es otra, y que muchas preeclampsias
acaban en cesárea, pero en el examen preguntan lo que dice la teoría.(R/4)

175. Señale cuál de las siguientes afirmaciones respecto a la rubéola es FALSA:

1. 1. El signo más característico del período prodrómico son las adenopatías.


2. 2. El exantema suele aclararse al tercer día con mínima descamación.
La artritis afecta sobre todo a mujeres y niños preadolescentes, principalmente a
3. 3.
grandes articulaciones.
4. 4. El bazo puede estar aumentado de tamaño.

!
!
!

Gráfico de respuestas
Comentario

En cuanto a la clínica de la rubeola que es muy importante para el nacional, esta se caracteriza por
un período de incubación de 14 a 21 días; los pródromos que se presentan como un cuadro
catarral leve en el que lo más característico son las adenopatías dolorosas retroauriculares,
cervicales posteriores y postoccipitales (Respuesta 1 cierta) y el exantema que suele durar 3 días y
se resuelve mediante descamación (Respuesta 2 cierta). Otros hallazgos pueden ser la
esplenomegália (Respuesta 4 cierta), leucopenia, trombopenia,...

Entre las complicaciones se encuentra la artritis que afecta sobre todo a mujeres y niños pero en
articulaciones PEQUEÑAS (Respuesta 3 falsa), encefalitis y púrpura trombopénica.(R/3)

176. En el tratamiento de la Diabetes Mellitus tipo 1 en el niño se tendrá presente lo


siguiente, señale lo CORRECTO:

1. 1. La insulina de acción intermedia es de elección para la cetoacidosis.


2. 2. La glicemia debe mantenerse en valores menores de 140 mg/dL.
3. 3. La administración de bicarbonato es obligatoria para corregir la cetoacidosis.
4. 4. El horario de las comidas debe guardar relación con el esquema insulinico.
Gráfico de respuestas
Comentario

La respuesta correcta es la número 4. Ya que el horario de las comidas guardia estrecha relación
con la aplicación de insulina.

La insulina que se debe utilizar en caso de cetoacidosis es la rápida. Respuesta 1 incorrecta.

La glicemia preprandial debe mantenerse entre 70 y 130 mg/dL. Respuesta 2 incorrecta.

El bicarbonato se utiliza cuando el pH <6.9. Respuesta 3 incorrecta. (R/4)

177. A 40-year-old woman is admitted to the ER with a fever of 38 °C and right lumbar
pain. Her past medical history is unremarkable. Laboratory tests show: WBC count
25,000/mm3 with left shift, hemoglobin 13 g/dl, sodium 137 mg/dl, potassium 4.7 mg/dl,
AST 13 U/L, ALT 25 U/L, amylase 9 U/L and creatinine 1 mg/dl. An abdominal ultrasound
is performed, and it reveals a staghorn calculus. You run a urine culture that is reported
as positive after 48 hours. What is the most likely microorganism?

1. 1. Escherichia coli.
2. 2. Proteus mirabilis.
3. 3. Salmonella typhi.
4. 4. Candida albicans.
Gráfico de respuestas
Comentario
Proteus mirabilis. Staghorn calculi are branched stones that occupy a large portion of the collecting
system. Typically, they fill the renal pelvis and branch into several or all of the calices. They are
most frequently composed of mixtures of magnesium ammonium phosphate (struvite) and/or
calcium carbonate apatite. Struvite/calcium carbonate apatite stones also are referred to as
"infection stones" because of their strong association with urinary tract infection caused by specific

!
!
!

organisms that produce the enzyme urease, which promotes the generation of ammonia and
hydroxide from urea. The most frequent is Proteus mirabilis. (R/2)

178. La trombastenia de Glanzmann es un defecto de la función plaquetaria que


corresponde a:

1. 1. Adhesión de la plaqueta al vaso sanguíneo.


2. 2. Agregación plaquetaria.
3. 3. Actividad de la ciclooxigenasa.
4. 4. Almacenamiento de los gránulos plaquetarios.
Gráfico de respuestas
Comentario

Dentro de las alteraciones plaquetarias, uno de los temas poco preguntados es el de las
trombocitopatías: alteración de la función plaquetarias cuando el tiempo de hemorragia se
encuentra prolongado y el número de plaquetas es normal. Entre ellas podemos encontrar: la
trombastenia de Glanzman (AR), que consiste en un déficit de GpIIb/Ia (receptor del fibrinógeno )
necesario para la agregación plaquetaria y que cursa con plaquetas normales; la enfermedad de
Bernard- Soulier (AR), que consiste en un déficit de GpIb (receptor para el FVW sintetizado por el
endotelio) necesaria para la adhesión plaquetaria al endotelio y que presenta plaquetas gigantes;
la enfermedad de von Willebrand, que se debe a una alteración cuantitativa y/o cualitativa del FVW
y por tanto cursa también con alteración de la adhesión plaquetaria. Estos dos tipos de
alteraciones las podemos diferenciar mediante el test de ristocetina que valora la adhesión y está
alterada en Bernard-Soulier y en Von-Willebrand, la diferencia radica en que en la primera no se
corrige tras administrar plasma, mientras que en la segunda sí. Este test es normal en la
trombastenia de Glanzman donde encontramos alterado el test de agregación con ADP, adrenalina
o tromboxano.(R/2)

179. La melanina se sintetiza a partir de:

1. 1. Fenilalanina.
2. 2. Cisteína.
3. 3. Serina.
4. 4. Glicina.
Gráfico de respuestas
Comentario

Pregunta relativamente sencilla y muy directa, que debe saber contestar por conocimientos
generales de la carrera. Los melanocitos poseen unos orgánulos, los melanosomas, donde se
produce la melanina. Esto es gracias a una enzima, una tirosinasa, que oxida el aminoácido
tirosina y, tras otra serie de reacciones, acaba produciéndose la melanina. Lo importante en esta
pregunta es saber que el aminoácido precusor de la Tirosina es la Fenilalanina (respuesta 2
correcta). Fallos en esta enzima tirosinasa son los que originan el Albinismo, en el cual no hay
melanina y no existe pigmentación de la piel ni de otros tejidos melánicos.(R/1)

180. Which of the following clinical data does not fit the diagnostic criteria of medullary
sponge kidney?

1. 1. Hypertension is more common than overall population.

!
!
!

Infectious complications are common and invasive examinations should be


2. 2.
avoided.
3. 3. Hematuria, urinary infections and acute renal colic are frequent.
4. 4. Hypercalciuria is present in a half of stone producers patients.
Gráfico de respuestas
Comentario

Se trata de una pregunta directa sobre un dato fundamental de la enfermedad.

El riñón esponjoso medular consiste en la presencia de dilataciones de los conductos colectores de


Bellini. En la mayoría de las ocasiones, cursa de manera asintomática. No obstante, hay un
aumento de riesgo de formación de cálculos por la hipercalciuria y la ectasia, infecciones (por ello
se deben evitar las exploraciones instrumentales siempre que sea posible) y hematuria. No se ha
visto mayor incidencia de HTA y la insuficiencia renal es excepcional.(R/1)

181. A 79-year-old
man comes to the emergency department with complaints of fatigue and dyspnea. His
past medical history is significant for hypertension and type 2 diabetes. Current
medications include enalapril, spironolactone, atenolol and metformin. His functional
NYHA status has worsened for the past few days. He denies chest pain, syncopal
episodes or orthopnea. Vital signs are remarkable for slow pulse. Chest auscultation
reveals no abnormal findings in lung auscultation but a systolic murmur with a preserved
second sound on heart auscultation. He has no peripheral edema. The physician obtains
an ECG as shown in the picture. Which of the following options is correct?

1. 1. First degree AV block.


2. 2. Mobitz type 2 second degree AV block.
3. 3. Third degree AV block.
4. 4. Sinus bradycardia.
Gráfico de respuestas
Comentario

Vamos a interpretar este ECG: bradicardia rítmica a 42 lpm, onda P de morfología normal, PR
prolongado [pero si se da cuenta el PR no se va prolongado paulatinamente (como pasaría en el
Mobitz I)], ondas P bloqueadas (después de la onda T viene una onda P que no se sigue de un
complejo QRS), QRS estrecho y repolarización sin alteraciones. Conclusión: bloqueo
auriculoventricular de segundo grado Mobitz II. (R/3)

!
!
!

182. Regarding the previously described patient, which of the following is FALSE?

1. 1. Atenolol plays a important role in the symptoms of this patient.


2. 2. A permanent pacemaker should be placed as soon as possible.
Beta agonists such as isoproterenol should be considered in the initial
3. 3.
management.
4. 4. Cardiac monitoring is justified.
Gráfico de respuestas
Comentario

Dado que es un bloqueo nuevo (no diagnosticado previamente) habría que realizar un estudio e
iniciar tratamiento farmacológico antes de decidir colocar un marcapasos definitivo. Si nos damos
cuenta, el paciente está en tratamiento con betabloqueantes (atenolol); si el bloqueo es de causa
farmacológica habría que esperar a que se lavase para poder pensar en un tratamiento definitivo
.(R/2)

183. La hernia de Maydl consiste en:

1. 1. Hernia del divertículo de Meckel.


2. 2. Hernia que contiene una apendicitis aguda.
Hernia retrógrada con estrangulación del intestino en le saco herniario, con lo que
3. 3.
las asas del intestino toman la forma de una W.
Hernia que contiene una porción de la pared antimesentérica del intestino
4. 4.
delgado.
Gráfico de respuestas
Comentario

Es un pregunta muy complicada que no se puede deducir si no se conoce el concepto. No se


preocupe porque estas preguntas no son nada frecuentes. La hernia que contiene un divertículo de
Meckel se llama hernia de Littré (recuerde la marca de tabaco L&M). La hernia que contiene una
apendicitis aguda se llama hernia de Aymart. Recuerde que llamamos hernia de Richter a la que
contiene una porción de la pared antimesentérica del intestino delgado.(R/3)

184. Tenemos que plantear el tratamiento de un paciente con angina típica estable grave
(clase funcional III-IV), prueba de esfuerzo positiva precoz con tratamiento antianginoso
máximo y estenosis severa de la arteria coronaria derecha media. Todas las respuestas
son incorrectas, MENOS una. Señálela.

1. 1. Se debería indicar cirugía coronaria.


2. 2. Se debe indicar angioplastia y medidas de prevención secundaria.
3. 3. Se debe aconsejar al paciente reposo y continuar con tratamiento médico.
4. 4. Se debe realizar un estudio isotópico para comprobar isquemia miocárdica.
Gráfico de respuestas
Comentario

La respuesta correcta es la número 3; ya que en la enfermedad severa de un vaso, la angioplastia


es de elección sobre la cirugía coronaria, estando indicada en este paciente por encontrarse en
una situación basal vulnerable, con angina de mínimos esfuerzos y una prueba de detección de
isquemia positiva de forma precoz, lo que confiere mal pronóstico. Además el tratamiento
antianginoso está a dosis máximas por lo que se debe de optar por una actitud terapéutica activa.

!
!
!

Evidentemente, esto no exime del resto de medidas de prevención secundaria (control de TA,
lípidos, abandono del hábito tabáquico...) (R/2)

185. ¿Cuál de las siguientes enfermedades/manifestaciones no se asocia con el virus de


Epstein Barr?

1. 1. Linfoma de serosas.
2. 2. Linfoma cerebral primario.
3. 3. Linfoma de Burkitt.
4. 4. Cáncer de cavum.
Gráfico de respuestas
Comentario

Recuerde las patologías aquí presentes que se asocian al VEB, y tenga mucho cuidado con el
linfoma de serosas, relacionado con el VHH- 8, como el sarcoma de Kaposi y la enfermedad de
Castelman.(R/1)

186. Paciente de 60 años, operado de un cáncer de ciego, al que se practica una


hemicolectomía derecha sin incidentes perioperatorios. En los dos primeros días del
curso postoperatorio presenta fiebre elevada e hipoventilación en la base del hemitórax
izquierdo. ¿Cuál de las siguientes situaciones cree que es responsable de la
hipertermia?:

1. 1. Flebitis.
2. 2. Infección urinaria.
3. 3. Atelectasia pulmonar.
4. 4. Dehiscencia anastomótica.
Gráfico de respuestas
Comentario

Pregunta muy rentable para el ENARM. Nos encontramos ante una caso de fiebre en el 1º y 2º día
del postoperatorio de una cirugía digestiva que además nos describen "sin incidencias", y que se
acompaña de otros datos en la exploración que indican la existencia de una complicación nivel
pulmonar como es la presencia "de hipoventilación en el hemitórax izquierdo". Teniendo en cuenta
el tiempo postoperatorio, 1º y 2º día, y los signos pulmonares añadidos la causa más frecuente de
fiebre en este caso es la existencia de una atelectasia pulmonar (bien por mala ventilación durante
o después de la cirugía al alterarse la mecánica ventilatoria del paciente con una cirugía
abdominal).(R/3)

187. El estadío III laparoscópico de la enfermedad inflamatoria pélvica, supone el hallazgo


de:

1. 1. Presencia de eritema y edema de la serosa de las trompas.


2. 2. Secreción purulenta en la cavidad y fibrina en la serosa.
3. 3. Adherencias y/o abscesos tubo ovárico.
4. 4. Todos ellos.
Gráfico de respuestas
Comentario

La clasificación es la siguiente:

!
!
!

a) No complicada (limitada a trompas u ovarios)

1) Sin peritonitis pélvica

2) Con peritonitis pélvica

b) Complicada (masa inflamatoria o absceso que compromete trompa (s) u ovario (s)

1) Sin peritonitis pélvica

2) Con peritonitis pélvica

c) Diseminada a estructuras más allá de la pelvis y/o ruptura de absceso tubo-ovárico. (R/4)

188. Respecto a la finalización de la gestación en presentaciones podálicas, cuál de las


siguientes afirmaciones es INCORRECTA:

1. 1. Corresponde al 4% de las gestaciones con feto único.


Se obtienen peores resultados perinatales en los partos de nalgas vía vaginal que
2. 2.
en aquellos terminados por cesárea.
Está aceptada la cesárea electiva como terminación de elección de los partos en
3. 3.
pelviana.
La versión externa pretende disminuir el número de presentaciones podálicas y,
4. 4. por tanto, el número de cesáreas realizadas con esta indicación a partir de las 35
semanas.
Gráfico de respuestas
Comentario
La presentación podálica corresponde al 4% de las gestaciones con feto único, siendo la variedad
de nalgas puras es la más frecuente. Se obtienen peores resultados perinatales en los partos de
nalgas vía vaginal que en aquellos terminados por cesárea, por tanto está aceptada la cesárea
electiva como terminación de elección de los partos en pelviana. No obstante, la versión externa
pretende disminuir el número de presentaciones podálicas y, por tanto, el número de cesáreas
realizadas con esta indicación, pero su realización debe ser siempre en gestantes a término (>37s)
debido a que el proceso no está exento de riesgos. (R/4)

189. Mujer 36 años que ingresa por dolor en cuadrante superior derecho, hiperamilasemia
y aumento de fosfatasa alcalina y bilirrubina directa, en el ultrasonido se muestra
colelitiasis y dilatación del colédoco. Se inicia hidratación intravenosa y analgésicos, y a
las 48 horas los valores de amilasa y de fosfatasa alcalina se normalizan. El manejo más
apropiado para esta paciente es:

1. 1. Colecistectomía laparoscópica electiva en 8 semanas.


2. 2. CPRE antes del alta y colecistetomía laparoscópica en 8 semanas.
3. 3. Antibioticoterapia.
4. 4. Observación.
Gráfico de respuestas
Comentario

!
!
!

La respuesta correcta es la 2, ya que al hablarnos de un cuadro de pancreatitis aguda biliar, se


recomienda CPRE antes del alta y posteriormente la realización de colecistectomía laparoscópica.
(R/2)

190. Paciente de 17 años que presenta múltiples quistes sebáceos y lipomas en tejidos
blandos junto con múltiples osteomas craneales, consulta por presentar rectorragias de
repetición de escasa cuantía. Se le realiza una colonoscopia en la que se observan
innumerables pólipos que en la biopsia resultan ser pólipos adenomatosos colónicos.
¿Cuál de las siguientes afirmaciones es FALSA?

Ante el diagnóstico de poliposis adenomatosa familiar, será necesaria la


1. 1.
realización de una colectomía total.
Los sujetos con poliposis adenomatosa familiar están sujetos a un elevado riesgo
2. 2.
de presentar pólipos y/o cáncer en todo el tracto digestivo (de la boca al ano).
Estos pacientes con poliposis tienen especial riesgo de desarrollar adenomas o
3. 3.
carcinomas en el duodeno y en la ámpula de Vater.
Si se realiza una colectomía preservando el recto, el paciente deberá realizarse
4. 4. rectoscopias cada 3 ó 6 meses y deberán resecarse los adenomas que presente en
el muñón rectal y que midan más de 5 mm.
Gráfico de respuestas
Comentario

De los síndromes de poliposis, hay que tener claro qué tipo de pólipos producen y, en caso de
existir, qué signos extracolónicos cabe esperar, para así identificar la enfermedad en concreto.

El síndrome de Gardner, que es lo que nos están describiendo, comparte todos los hallazgos de la
poliposis adenomatosa familiar, añadiendo múltiples manifestaciones extraintestinales (osteomas y
otros tumores benignos).

En la poliposis colónica familiar, los tumores se desarrollan sobre los pólipos previos del colon, y no
en otras localizaciones del tracto digestivo, salvo a nivel duodenal o en el ámpula de Vater. El
tratamiento de elección es la colectomía total, con o sin reservorio. En caso de preservar el recto,
el paciente deberá realizarse rectoscopias cada 3 ó 6 meses, resecándose los adenomas que
presente en el muñón rectal y que midan más de 5 mm.(R/2)

191. Un neonato de 20 días de vida presenta vómitos desde hace una semana, alguno de
ellos en escopetazo. La exploración muestra una leve deshidratación. En los exámenes
sanguineos de urgencias destaca una alcalosis metabólica. ¿Qué actitud tomaría en
primer lugar frente a este paciente?

1.! ! 1.! Indicar!un!ultrasonido!abdominal.!


2.! ! 2.! Indicar!un!estudio!radiológico!con!papilla!de!bario.!
3.! ! 3.! Hidratación!oral!con!solución!hidratante!baja!en!sodio.!
4.! ! 4.! Observación!hospitalaria.!
Gráfico de respuestas
Comentario

Pregunta típica de ENARM. La presencia de vómitos y deshidratación también podría ser


compatible con una insuficiencia suprarrenal, pero no la alcalosis metabólica (todo lo contrario,
habría acidosis). Lo que quieren representarnos en este caso clínico es una estenosis hipertrófica
del píloro, que va haciéndose más grave a medida que pasan los primeros días de vida, hasta que

!
!
!

los vómitos son tan intensos que el niño se deshidrata y entra en alcalosis metabólica, como
consecuencia de la pérdida de ácido clorhídrico que se produce a nivel gástrico. La prueba que
habría que elegir, por lo tanto, sería el US abdominal, que mostrará lo que estamos buscando, la
hipertrofia pilórica. (R/1)

192. Masculino de 40 años, hipertenso, llega a urgencias por cuadro de cefalea intensa y
vómitos de pocas horas de evolución; en la exploración neurológica destaca rigidez
nucal y ptosis palpebral izquierda. El origen más probable de este cuadro clínico será:

1. 1. Hematoma cerebral hipertensivo.


2. 2. Hidrocefalia.
3. 3. Rotura de aneurisma cerebral.
4. 4. Meningitis.
Gráfico de respuestas
Comentario

Pregunta de dificultad moderada de un tema muy importante para el ENARM como es la


hemorragia subaracnoidea.

La causa más frecuente de hemorragia subaracnoidea es la rotura de aneurismas saculares


(respuesta 3 correcta) que se localizan preferentemente en la unión de la arteria comunicante
anterior con la cerebral anterior. Clínicamente, produce cefalea súbita de gran intensidad, rigidez
de nuca, naúseas y vómitos. Respecto al diagnóstico, ante la sospecha clínica, la TC craneal es la
técnica de imagen de elección. El tratamiento de la HSA es esencialmente médico (no es
susceptible de evacuación quirúrgica) y va dirigido a reducir la sintomatología dolorosa, soporte
vital y a evitar el resangrado. (R/3)

193. Qué prueba solicitaría para


confirmar el diagnóstico?:

1. 1. Difusión de monóxido de carbono.


2. 2. TAC de tórax.
3. 3. Biopsia pulmonar por videotoracoscopia.
4. 4. Broncoscopia.
Gráfico de respuestas
Comentario

!
!
!

La mejor forma de evaluar toda la vía aérea superior, incluída la tráquea, es la broncoscopia que
confirmará la sospecha diagnóstica de estenosis postintubación.

Imagen típica que te muestran de espirometría de un patrón obstructivo. (R/4)

194. Un paciente de 43 años, fumador de 24 paquetes/año, consulta por disnea de larga


evolución. Entre sus antecedentes figura un politraumatismo por accidente de tráfico a
los 24 años con estancia prolongada en la UCI y necesidad de ventilación mecánica por
un periodo de un mes, hipertensión arterial e hipercolesterolemia. Presenta tos con
expectoración de forma habitual desde hace más de 10 años. La exploración física
muestra a un paciente obeso, con ligero estridor inspiratorio más audible sobre la vía
aérea superior. La radiografía de tórax fue normal. La espirometría mostró los siguientes
valores: FEV1 980 ml (30%), FVC 2890 (74%), cociente FEV1/FVC 0.34. La curva flujo-
volumen puede verse en la imagen. ¿Cuál es el diagnóstico más probable?:

1. 1. EPOC tipo enfisema.


2. 2. EPOC tipo bronquitis crónica.
3. 3. Histiocitosis X.
4. 4. Estenosis subglótica postintubación.
Gráfico de respuestas
Comentario

Reafirmamos el concepto de la pregunta previa. La clave para contestar correctamente esta


pregunta es la morfología de la curva flujo- volumen, que es la típica de una obstrucción de vía
aérea superior. Esta se caracteriza por una forma “en meseta” tanto de los flujos inspiratorios como
espiratorios, cuya única causa de entre las que aparecen en las respuestas es la estenosis
postintubación, como puede deducirse del antecedente de intubación prolongada. La presencia de
estridor inspiratorio que se ausculta mejor sobre la vía aérea superior también apoya este
diagnóstico. Si sólo nos fijamos en los valores espirométricos, lo único que podemos deducir es
que se trata de un transtorno obstructivo, determinado por un cociente FEV1/FVC menor de 0.7. El
resto de las respuestas cursan con alteración obstructiva, pero sólo la 4 lo es de vía aérea
superior. (R/4)

195. La complicación más frecuente del Síndrome de Aspiración Meconial es:

1. 1. Persistencia del conducto arterioso.


2. 2. Hipertensión pulmonar persistente.
3. 3. Sepsis.
4. 4. Cianosis.
Gráfico de respuestas
Comentario

Debes dominar esta tabla para el ENARM, recuerda que el tapón de meconio da un efecto de
válvula y secundario a esto genera hipertensión pulmonar.

!
!
!

196. Una mujer de 30 años consulta por dolores generalizados de predominio diurno junto
con rigidez matutina. Refiere dificultad para conciliar el sueño habitual. A la exploración
presenta dolor a la palpación en cara anterior de espacios entre apofisis transversas de
C5-C7, encima de la espina escapular, cerca del borde medial de la escápula y la cara
posterior de la eminencia trocantérica. Los exámenes de laboratorio son normales. El
cuadro sólo mejora parcialmente con AINE. Señale la opción INCORRECTA:

1. 1. Se trata con toda probabilidad de una fibrositis o fibromalgia.


2. 2. Se ha involucrado una alteración de la fase 4 (no REM) del sueño en su etiología.
Se trata de un proceso inflamatorio banal del tejido conectivo, sin repercusión en
3. 3.
exámenes de laboraotrio.
4. 4. La combinación de alprazolam e ibuprofeno en el tratamiento es muy eficaz.
Gráfico de respuestas
Comentario

La fibromialgia NO es una enfermedad inflamatoria, prueba de ello es que los fármacos más
eficaces son los antidepresivos y ansiolíticos (psicofármacos), y por ello, ni los exámenes de
laboratorio ni otra prueba podrán demostrar organicidad, con excepción de la presión de puntos
gatillo, que no deja de tener cierto carácter subjetivo. Las zonas enunciadas (cervical inferior
C5- C7, supraespinoso, trapecio y trocantéreo) son puntos gatillo (hay un total de 18 bilaterales,
otros son occipucio, condrocostal, glúteo, rodilla, epicondíleo). La clínica es típica: dolor
GENERALIZADO y continuo ("Dr, me duele hasta punta del pelo"), con gran debilidad subjetiva (se
despiertan ya fatigados) e intolerancia al ejercicio, rigidez, parestesias e insomnio. Su etiología es
desconocida, pero al existir sueño no reparador se discute si la fase más implicada (fase 4) pueda
estar alterada. También se discuten factores psicológicos, del sistema nervioso? (R/3)

197. El riesgo de transmisión vertical madre – niño de infección VIH – SIDA es:

!
!
!

1. 1. Menor del 5%.


2. 2. 10% a 15%.
3. 3. 25% a 35%.
4. 4. 90% a 100%.
Gráfico de respuestas
Comentario

Pregunta complicada por ser memorística

Respuesta 3 correcta. 25-35%.

198. El tratamiento del fibroadenoma de mama usualmente es:

1. 1. Extirpación con anestesia local en paciente ambulatorio.


2. 2. Mastectomía simple.
3. 3. Cuadrantectomía.
4. 4. Extirpación quirúrgica seguida de radioterapia.
Gráfico de respuestas
Comentario

Es una patología benigna, que en el caso de que se decida operar, no suele requerir más que
extirpación ambulatoria. (R/1)

199. Un hombre de 68 años, que ha tenido recientemente un episodio sincopal, es


hospitalizado con un cuadro de insuficiencia cardíaca congestiva. Su presión arterial es
de 160/80 mmHg, la frecuencia del pulso, de 80 latidos por minuto y hay un soplo sistólico
rudo de grado III/VI. El ecocardiograma muestra un tabique interventricular
desproporcionadamente grueso y un movimiento anterior de la válvula mitral durante la
sístole. ¿Cuál de las siguientes manifestaciones es más probable que exista en este
enfermo?

1. 1. Irradiación del soplo hacia las carótidas.


2. 2. Disminución del soplo al apretar los puños.
3. 3. Retraso de la porción ascendente del carotidograma.
4. 4. Los signos de una estenosis mitral.
Gráfico de respuestas
Comentario

Pregunta de considerable dificultad, pero útil para repasar la Miocardiopatía hipertrófica (MCH).
Recuerde que esta enfermedad puede producir ICC por disfunción diastólica, y síncopes de
repetición o incluso muerte súbita, por arritmias ventriculares. Los datos más característicos de la
enfermedad (que pueden verse en el Ecocardiograma) son la hipertrofia ventricular de predominio
en el tabique y el movimiento sistólico anterior de la válvula mitral. También puede auscultarse un
soplo sistólico rudo localizado en ápex y borde esternal izquierdo. Por tanto, en este enfermo
sospecharíamos una MCH. En esta enfermedad el soplo aumenta o disminuye de intensidad con
algunas maniobras. Al apretar los puños aumenta la postcarga y este hecho produce una
disminución del soplo (opción 2 correcta). En la MCH la fracción de eyección del VI suele ser
normal o incluso está aumentada. Por último, la irradiación del soplo hacia las carótidas es típica
de la Estenosis aórtica, así como el retraso de la porción ascendente del carotidograma (pulso
parvus et tardus). (R/2)

!
!
!

200. ¿Qué podemos considerar FALSO respecto a la tuberculosis genital?:

1. 1. El síntoma más frecuente es la esterilidad.


2. 2. La vía de diseminación más frecuente es la hematógena.
Una imagen sugestiva de tuberculosis genital en la histerosalpingografía es la
3. 3. oclusión de la cavidad uterina como secuela de una endometritis tuberculosa o
síndrome de Netter.
4. 4. El tratamiento es principalmente quirúrgico.
Gráfico de respuestas
Comentario

El tratamiento de la tuberculosis genital es médico, exactamente con la misma pauta que se


recetan para la tuberculosis pulmonar.

Sólo se requerirá cirugia en caso de que la paciente presente adherencias u otras complicaciones
secundarias a las cicatrices residuales de la infección. (R/4)

201. La prueba de laboratorio para el diagnóstico de infección VIH – SIDA recomendada


en niños menores de 6 meses es:

1. 1. Test ELISA.
2. 2. Test WESTERN BLOT.
3. 3. p24.
4. 4. No es posible determinarlo.
Gráfico de respuestas
Comentario

Hay que recordar que no es adecuado realizar ELISA ni Western blot en menores de 6 meses, ya
que los anticuerpos de la madre podrían pasar a la circulación fetal, por lo que lo recomendado es
solicitar PCR del virus, p24 o cultivo de virus. En esta pregunta la correcta es la 3 ya que las otras
posibilidades no figuran entre las respuestas. (R/3)

202. Una mujer de 28 años acude a Urgencias por ictericia, astenia, anorexia y malestar
general. A la exploración presenta hepatomegalia de 4 cm y dolor a la palpación del
hipocondrio derecho. Los exámenes de laboratorio muestran: bilirrubina 10 mg/dl, GOT
150 mU/ml, GPT 200 mU/ml e hipergammaglobulinemia. Los estudios serológicos
posteriormente reflejan: HBsAg (-), anti-HBs (+), anti-HBc IgG (+), anti-VHC (-), ANA (+),
Ac antimitocondriales (-), Ac AML (+), anti-LKM1 (-). Se le practica biopsia hepática, que
es informada como hepatitis crónica activa. Respecto al tratamiento de esta paciente,
señale la afirmación INCORRECTA:

Dado que la paciente presenta una hepatitis crónica B, se obtendrá la mejor


1. 1.
respuesta con interferón alfa.
La base del tratamiento deben ser los glucocorticoides, recomendándose la
2. 2.
prednisona.
La asociación de glucocorticoides y azatioprina es más eficaz, pero tiene más
3. 3.
probabilidad de producir efectos secundarios severos.
Aunque en ocasiones el trasplante hepático es el único recurso terapéutico, la
4. 4.
enfermedad suele recurrir en el nuevo órgano.
Gráfico de respuestas
Comentario

!
!
!

Nos presentan una paciente con un cuadro de hepatopatía, serología viral negativa y que se asocia
a autoanticuerpos. Esto debe sugerirnos una posible hepatitis autoinmune. La hepatitis autoinmune
tipo 1 cursa con ANA y anti- músculo liso positivos. En la tipo 2, aparecen anticuerpos anti- LKM y
anti- citosol hepático. El tratamiento de las hepatitis autoinmunes sería, como en el resto de
procesos autoinmunitarios en general, con corticoides y en caso de falta de respuesta podríamos
añadir inmunosupresores, como la azatioprina. (R/1)

203. Señale qué afirmación, de las siguientes, relativa al meconio fetal normal, es
INCORRECTA:

1. 1. Retraso de eliminación sugiere Hirschsprung.


2. 2. Retraso de eliminación no sugiere ninguna enfermedad.
3. 3. Ausencia de meconio puede ocurrir en malformaciones anorrectales.
4. 4. Ninguna de las anteriores.
Gráfico de respuestas
Comentario

Pregunta sencilla sobre meconio. En caso que no cuente con muchos conocimientos sobre esta
patología, puede llegar a la respuesta. En la respuesta 1 le mencionan que el retardo en la
eliminación sugiere enfermedad de Hirschprung, pero en la opción 2 mencionan que no sugiere
ninguna enfermdad... por obvias razones al ver este tipo de respustas que se contraponen, debe
pensar inmediatamente que la respuesta se encuentra entre estas dos opciones. Por sentido
común, cree que el no eliminar el meconio es normal?, por lo que la respuesta correcta es la 2.

204. Gestante de 33 semanas con contracciones regulares (2 contracciones /10 minutos),


a la exploración cervical se detecta un cérvix permeable un dedo, borramiento 50%,
consistencia media y posición posterior. Mediante ecografía vaginal se obtiene una
cervicometría de 21 mm. Laboratorios dentro de límites normales. En esta situación,
usted indicaría:

Realizar test de fibronectina y sólo si es positivo realizar maduración pulmonar


1. 1.
fetal con corticoides e hidratación.
2. 2. Maduración pulmonar fetal con corticoides en ingreso hospitalario.
Maduración pulmonar fetal con corticoides y tocolisis intravenosa en ingreso
3. 3.
hospitalario.
4. 4. Tocolisis intravenosa, hidratación y reposo absoluto en ingreso hospitalario.
Gráfico de respuestas
Comentario
El caso clínico nos presenta una gestante con una amenaza de parto pretérmino con modificación
cervical y dinámica uterina regular. Dado que tenemos una longitud cervical menor a 30 mm la
actitud terapéutica correcta es el ingreso hospitalario para maduración pulmonar con corticoides y
la administración de tocolíticos para frenar la dinámica uterina.(R/3)

205. Un paciente de 50 años con artritis gotosa hace un mes presenta hiperuricemia y
excreción urinaria de ácido úrico de 200 mg al día. ¿Cuál es el tratamiento de elección?:

1. 1. AINEs.
2. 2. Alopurinol.
3. 3. Colchicina y uricosúricos.
4. 4. Colchicina y alopurinol.
Gráfico de respuestas
!
!
!

Comentario
Se trata de un paciente con gota intercrítica (paciente con hiperuricemia con episodio de artritis
aguda pasado y actualmente asintomático) en el que se demuestra una clara reducción de la
excreción renal de ácido úrico. La excreción normal debe ser de al menos 800 mg al día con una
dieta normal y de 600 mg en los pacientes que toman una dieta sin purinas. El tratamiento de
elección es la administración de un uricosúrico, añadiendo colchicina durante los primeros meses
para prevenir la aparición de nuevos episodios de artritis.(R/3)

206. Con relación a la etiología de la relajación del piso pélvico e incontinencia urinaria
de esfuerzo, ¿Cuál es el diagnóstico INCORRECTO?

1. 1. Atrofia genital por menopausia.


2. 2. Enfermedad pulmonar crónica.
3. 3. Infección crónica de las vías urinarias.
4. 4. Traumatismo obstétrico.
Gráfico de respuestas
Comentario

Pregunta obvia... si hay un problema en piso pélvico, y tres de las opciones le hablan de
alteraciones cercanas a dicha zona, inmediatamente debe de elegir la respusta 2 EPOC. Le
quieren hacer dudar por el hecho de toser con frecuencia. (R/2)

207. Hombre de 32 años que acude por alteración del tránsito intestinal junto con
episodios paroxísticos de dolor abdominal de tipo cólico. Refiere sentirse hinchado y con
"pesadez" después de las comidas. No ha perdido peso y no refiere ningún otro dato de
interés. Respecto al tratamiento de la enfermedad que sospecha indique la FALSA:

Si el paciente reconoce algún alimento que exacerbe los síntomas, es aconsejable


1. 1.
que lo retire de su dieta.
Es fundamental explicar claramente al paciente en qué consiste su enfermedad y
2. 2.
tranquilizarle respecto al pronóstico.
3. 3. El uso de enzimas pancreáticas puede estar indicado para evitar la maldigestión.
Algunos pacientes se benefician del uso de anticolinérgicos, sobre todo los que
4. 4.
presentan dolor abdominal.
Gráfico de respuestas
Comentario

Se trata de un caso de síndrome de intestino irritable. En este caso, la respuesta falsa es la 3, pues
el paciente no presenta maldigestión (no ha perdido peso), ni tenemos razones para pensar que
tenga disfunción pancreática. (R/3)

208. Todas las siguientes son actitudes correctas en el manejo de un paciente con
insuficiencia renal, EXCEPTO:

1. 1. Restricción de proteínas en la dieta.


2. 2. Limitar el fosfato de la dieta.
Quelar el fosfato con hidróxido de aluminio vía oral como tratamiento de
3. 3.
elección.
4. 4. Eritropoyetina recombinante humana.
Gráfico de respuestas

!
!
!

Comentario

Ha de conocer distintos aspectos del tratamiento de la IRC. El manejo de las alteraciones óseas es
importante. El hiperPTH secundario comienza por retención de P cuando sólo resta el 25% del
filtrado glomerular, así es interesante limitar el fosfato de la dieta y quelarlo con carbonato cálcico.
No se quela de elección con hidróxido de aluminio pues éste está implicado en la enfermedad ósea
adinámica (muy similar a la osteomalacia) y en la demencia dialítica. La restricción de proteínas en
la dieta si se inicia precozmente puede ralentizar la evolución de la IRC. La rHuEPO subcutánea se
utiliza en el tratamiento de la anemia en la IRC. (R/3)

209. Señale lo que NO sea cierto respecto al autismo o trastorno generalizado del
desarrollo (DSM- IV):

1. 1. El lenguaje está gravemente alterado y, cuando lo hay, no es comunicativo.


2. 2. La prevalencia está entre 2 y 4 casos por 10,000 habitantes.
3. 3. Se asocia siempre a retraso mental.
4. 4. Entre el 5 y el 15% alcanzan una vida sociolaboral casi normal al pasar los años.
Gráfico de respuestas
Comentario

En el autismo, según el DSM- IV uno de los items diagnósticos más importantes es la alteración
cualitativa de la comunicación por alteración en el lenguaje sin intentos compensatorios de
comunicación mediante otro tipo de expresión (opción 1).

Los datos de prevalencia que dan son correctos (respuesta 2). La conducta que presentan estos
niños es estereotipada, restringida, repetitiva y con manierismos u otros trastornos relacionados.
Es frecuente que se asocie a retraso mental, pero eso no significa que “siempre”. De hecho, el
riesgo de trastorno autista es mayor a menor CI. Por todo lo dicho, la opción falsa es la 3. La
opción 4 es correcta, porque aunque sólo un 1-2 % alcanza un estado normal e independiente,
entre un 5-20% logra un estado límite con la normalidad.(R/3)

210. Los medios de cultivo líquidos se emplean en las siguientes situaciones, EXCEPTO:

1. 1. Muestras con pocas bacterias.


2. 2. Muestras con bacterias anaerobias.
Seleccionar patógenos en muestras con flora habitual, mediante la adición de
3. 3.
antibióticos o sustancias inhibidoras.
4. 4. Identificación de diferentes bacterias presentes en la muestra.
Gráfico de respuestas
Comentario
Para identificar las diferentes especies bacterianas presentes en una muestra se utilizan medios
sólidos; se observa el tipo de colonia y, una vez aislado el agente, se completa la batería de
pruebas bioquímicas, inmunológicas, etc., necesarias para su identificación. (R/4)

211. Señale lo FALSO respecto al tumor filoides:

1. 1. Se presenta en la 4ª-5ª década de la vida.


2. 2. La clínica más frecuente es la mastodinia.
3. 3. Es de rápido crecimiento, siendo generalmente benigno.
4. 4. El tratamiento de elección es la mastectomía simple.

!
!
!

Gráfico de respuestas
Comentario

El tumor filoides (TF) de la mama es una patología poco frecuente y origen desconocido. A pesar
de su clasificación en benignos, borderline y malignos, su comportamiento tiende a ser incierto. Se
sitúa entre la 4º y 5º década de la vida, suelen presentar un rápido crecimiento y, aunque puede
hacer metástasis, son generalmente benignos. Se suele realizar mastectomía simple. Su forma de
presentación más frecuente es la de masa palpable. (R/2)

212. Niño de 7 años, acude a Urgencias por presentar un exantema ligeramente


pruriginoso en tronco y extremidades, sin afectación palmoplantar y con aspecto de
"encaje". Afebril. Refieren que el día previo al exantema tenía las mejillas muy "rojas".
Señale lo CORRECTO:

1.! ! 1.! El!virus!que!lo!produce!es!ARN.!


2.! ! 2.! Los!síntomas!generales!son!más!graves!en!niños!que!en!adultos.!
3.! ! 3.! Este!niño!ya!no!es!contagioso.!
4.! ! 4.! El!agente!responsable!no!se!ha!relacionado!con!hydrops!fetal.!
Gráfico de respuestas
Comentario

Pregunta de dificultad media sobre el eritema infeccioso o 5ª enfermedad, diagnóstico que


podemos hacer con los datos característicos que nos dan en el enunciado, como son exantema en
encaje, afectación de las mejillas (“bofetón”) y el estado afebril del niño, que además suele estar en
edad escolar (5-15 años).

La opción correcta es la 3, ya que el periodo de contagio es máximo antes de la aparición del


exantema, y a partir de este momento desaparece la contagiosidad.

El parvovirus es un virus de ADN, los síntomas generales son más frecuentes en adultos, aparece
típicamente en edad escolar.

Entre sus complicaciones, destacan los abortos y el hydrops fetal en embarazadas, las crisis
aplásicas en pacientes con anemias hemolíticas crónicas y, en inmunodeprimidos, la cronificación
de la enfermedad. (R/3)

213. Mujer de 35 años de edad, embarazo de 36 semanas de edad gestacional, sin


antecedentes personales de interés, que acude para realizar la prueba de monitorización
no estresante (NST). En el mismo no se aprecia ninguna aceleración de la FCF tras 20
minutos de prueba, y tras estimular manualmente al feto, tampoco hay aceleración alguna
tras 20 minutos, considerando la prueba no estresante no reactiva. Por este motivo se
decide realizar la prueba estresante o de Pose. Una prueba de POSE o de la oxitocina se
considera positiva cuando:

Aparecen desaceleraciones tardías o variables graves en menos del 30% de las


1. 1.
contracciones.
2. 2. Aparición de desaceleraciones precoces con todas las contracciones.
Aparición de desaceleraciones tardías en el 30% de las contracciones o
3. 3.
desaceleraciones variables graves con todas las contracciones.
4. 4. Aparición de alteraciones de la FCF como respuesta a la hipertonía uterina.
Gráfico de respuestas
!
!
!

Comentario

Pregunta sobre la monitorización fetal estresante que exige conocer los rangos de normalidad.
Recuerde que la prueba de Pose o de tolerancia a la oxitocina la realizamos ante un RAF negativo.
La prueba de Pose es positiva o patológica cuando tras conseguir 10 contracciones en el registro,
aparecen tres o más desaceleraciones tardías o dips II (en otras palabras, >30% de dips II). Por lo
tanto la opción correcta es la 3. Recordamos que será negativa si no aparecen dips II, o aparecen
menos de 2 en 10 contracciones (<20% de dips II en el registro). (R/3)

214. La tensión arterial:

1. 1. Mejora si se reduce el consumo de alcohol por debajo de 25-30 g diarios.


2. 2. Supone un factor de riesgo sólo si supera 140/90.
Si está alta es un factor de riesgo mayor para el desarrollo de insuficiencia
3. 3. cardíaca o ictus, aunque el tabaco supone mucho mayor riesgo para estas 2
patologías.
4. 4. Está regulada exclusivamente por los riñones.
Gráfico de respuestas
Comentario

Una pregunta sencilla y didáctica que nos permite revisar los efectos del alcohol sobre el corazón.
Como sabe, éstos dependen de la cantidad consumida diariamente. Se ha demostrado que el
consumo moderado (1-2 vasos de vino al día) es un factor protector cardiovascular. Sin embargo, a
partir de este punto el riesgo aumenta en proporción directa, por lo que la respuesta correcta es la
1.

Sobre el resto de las opciones, habría que decir:

•! R2: el riesgo cardiovascular es directamente proporcional a las cifras de la presión arterial,


por lo que no existe una cifra exacta a partir de la cual empieza a tener importancia. Por
ejemplo, unas cifras de 130/85 mmHg implican un riesgo mayor que 120/80 mmHg,
aunque en ninguno de los dos casos pueda hablarse de HTA. Por otra parte, es muy
arriesgado confiar en una opción como ésta, por el carácter restrictivo de su redacción.
•! R3: la hipertensión arterial es mucho más importante para el accidente cerebrovascular
que para la insuficiencia cardíaca. Por el contrario, el hábito tabáquico parece más
importante para la enfermedad coronaria que para los ACV.
•! R4: existen muchos factores que influyen en la presión arterial, no solamente los riñones,
sino también el sistema nervioso autónomo, diversas glándulas endocrinas, etc. (R/1)

215. Lactante de 6 meses que desde hace 2 días comienza con febrícula y algunos
vómitos ocasionales, y hoy comienza con evacuaciones blandas al principio y más tarde
se hacen liquidas, además de incrementarse los vómitos. Es invierno y el niño va a la
guardería donde ha habido otros niños con un cuadro similar ¿Cuál es la etiología más
probable en este caso?

1. 1. Salmonella enteritidis.
2. 2. Rotavirus.
3. 3. Campylobacter yeyuni.
4. 4. Adenovirus.

!
!
!

Gráfico de respuestas
Comentario

Detecte los datos claves en las preguntas: lactante con vómito y diarrea, en invierno y se encuentra
en una guarderia. La respuesta correcta es el rotavirus. Respuesta 2 correcta.

216. Paciente de 38 años con nódulo mamario indoloro, de bordes imprecisos. La


mastografía revela imagen nodular, con espículas en todos sus márgenes, y 10
microcalcificaciones finas, agrupadas en el interior. El diagnóstico más probable, entre
los que se citan, es:

1. 1. Fibroadenoma.
2. 2. Quiste.
3. 3. Carcinoma.
4. 4. Mamografía normal para la edad de la paciente.
Gráfico de respuestas
Comentario

Pregunta muy clínica que no se debe fallar. El diagnóstico más probable ante un nódulo con
bordes imprecisos, espículas en todos sus márgenes, y 10 microcalcificaciones finas agrupadas en
el interior es un carcinoma mamario. El resto de patologías no presentarían estas características.
(R/3)

217. La adrenalina es uno de los agentes más importantes que se utiliza en la reanimación
neonatal, con respecto a su uso NO es cierto lo siguiente:

1. 1. Se usa en asistolia.
Se usa si la frecuencia cardiaca se encuentra menor de 120 latidos por minuto
2. 2.
pero con una ventilación eficaz con oxigeno al 100%.
3. 3. Es útil por vía endotraqueal.
4. 4. Frecuencia cardiaca menor a 60 latidos por minuto.
Gráfico de respuestas
Comentario

En determinadas situaciones, en la reanimación del RN se recurre a la ventilación a presión


positiva intermitente y a la administración de adrenalina. El parámetro que determina la
administración de estas medidas es que la frecuencia cardíaca sea inferior a un límite definido
(respuesta 2). El uso de drogas como la adrenalina son útiles en situación de asistolia (respuesta
1) y pueden ser administradas por vía endotraqueal (respuesta 3). (R/2)

218. En un paciente de 80 años, gastrectomizado hace 20 años, estudiado por debilidad


crónica, se encuentra un componente M IgG lambda de 0.5gr/dl con el siguiente BH:
leucocitos 5,000/ul, plaquetas 150,000/ul, hemoglobina 6.5 gr/dl, VCM 125 fl e
hipersegmentación de neutrófilos en el frotis. El diagnóstico más probable es:

1. 1. Anemia megaloblástica por deficiencia de folato.


2. 2. Anemia megaloblástica por deficiencia de B12.
3. 3. Mieloma múltiple.
4. 4. Anemia mieloptísica.
Gráfico de respuestas

!
!
!

Comentario

Pregunta tipo caso clínico, que incluso con conceptos básico se puede sacar. La opción 4 es fácil
de descartar, no olvid que la anemia mieloptísica cursa con dacrocitos en el frotis y reacción
leucoeritroblastica (y nuestro paciente tiene pancitopenia). El diagnóstico de mieloma múltiple, es
poco probable, los niveles de componente M de este paciente son muy bajos para estos
diagnóstico. La duda puede surgir entre las opciones 1 y 2, pero si recuerda, el antecedente de la
gastrectomía de este paciente nos sirve para inclinarnos como diagnóstico más probable a la
anemia megaloblastica por déficit de vitamina B12 (opción 2 correcta).

219. Un recién nacido de 12 horas de vida presenta una ictericia marcada hasta raíz de
miembros inferiores. En la exploración física presenta un importante cefalohematoma,
sin ningún otro hallazgo de interés. Su madre es AB positivo. Solicita usted estudios de
laboratorio, los cuales reportan una cifra de bilirrubina total de 18 mg/dL con una
bilirrubina directa de 0.4 mg/dL. Señale la respuesta VERDADERA:

Se trata de una ictericia fisiológica ya que es monosintomática y ha aparecido en


1. 1.
las primeras 24 horas de vida.
El tratamiento consistirá en una exanguinotransfusión y administración de
2. 2.
fenobarbital intravenoso.
Podría tratarse de una enfermedad hemolítica del recién nacido por
3. 3. incompatibilidad Rh, por lo que es necesario solicitar un test de Coombs
indirecto.
Se trata de una ictericia patológica por hiperbilurrubinemia indirecta por aumento
4. 4.
en la producción.
Gráfico de respuestas
Comentario

Tema que se ha preguntado en reiteradas ocasiones en el ENARM así que lo repasaremos las
veces que sea necesario. La ictericia patológica es aquella que no cumple los criterios de la
ictericia fisiológica (aparición más allá de las 24 horas de vida, resolución antes de los 7 días de
vida, monosintomática, con hiperbilirrubinemia indirecta menor de 12 mg/dL, con incrementos
inferiores a 0.5 mg/dL al día).

Es frecuente encontrarla en niños que presenta un aumento de la producción de bilirrubina:


especialmente por cefalohematomas o fracturas producidas durante el parto.

La isoinmunizacións en este caso estarían descartadas al ser la madre AB positivo. (R/4)

220. Todos los datos de laboratorio pertenecen a las anomalías producidas por infección
VIH en el sistema inmune, EXCEPTO:

1. 1. Deplección de linfocitos T4.


2. 2. Activación policlonal de células B.
3. 3. Aumento de la producción de interferón gamma en respuesta a los antígenos.
4. 4. Aumento de beta-2-microglobulina.
Gráfico de respuestas

!
!
!

Comentario

La infección por VIH produce una inmunodepresión celular, por lo que no cabría esperar que
aumente la producción de IFN gamma ante antígeno alguno (respuesta 3 falsa). El resto de las
opciones sí ocurren como consecuencia de esta infección. Según ésta avanza, el número de
linfocitos CD4+ sufre un descenso progresivo. Como consecuencia de ello, dejan de ejercer su
función reguladora sobre el resto de las células de la inmunidad, por lo que se produce una
activación de las células B, que aumentan su producción de anticuerpos, lo que se expresa
mediante una hipergammaglobulinemia.

En cuanto a la beta-2- microglobulina, es una proteína que forma parte de la membrana de las
células, y que se libera normalmente como resultado de la muerte natural de las células
sanguíneas en general, especialmente los glóbulos blancos. De ahí su elevación en enfermedades
donde se produce un intenso recambio de estas células, como los linfomas o el mieloma múltiple.
El nivel normal es de unos 3 microgramos por litro. En pacientes infectados por el VIH, esta cifra
suele estar aumentada, como consecuencia de la paulatina destrucción que van sufriendo sus
linfocitos a medida que progresa la enfermedad. (R/3)

221. Señalar cuál de los siguientes procesos NO interviene en la patogenia del acné:

1. 1. Queratinización anormal del infundíbulo piloso.


2. 2. Aumento de la secreción sebácea.
3. 3. Respuesta aumentada de la glándula sebácea a los estrógenos.
4. 4. Liberación de mediadores de la inflamación.
Gráfico de respuestas
Comentario

Tema poco preguntado en el ENARM, pero vale la pena repasarlo y es bastante fácil.

El acné es una enfermedad crónica de etiología multifactorial. Entre los diversos factores
implicados, la alteración inicial se corresponde con una queratinización anómala del infundíbulo
piloso, lo que da lugar a la formación del comedón. Otro factor es el estímulo androgénico sobre la
glándula sebácea, esto lleva a una alteración cualitativa y cuantitativa en la producción del sebo.
Además se produce un aumento en la cantidad de Propionibacterium acnes, estas bacterias son
capaces de metabolizar los lípidos produciendo mediadores de la inflamación.(R/3)

222. De los factores de alto riesgo mencionados para el cáncer de mama, hay uno que
NO es cierto:

1. 1. Multiparidad.
2. 2. Embarazo en mayores de 30 años.
3. 3. Antecedentes familiares de cáncer de mama
4. 4. Cáncer previo en la mama contralateral.
Gráfico de respuestas
Comentario

Pregunta fácil sobre los factores de riesgo del cáncer de mama. Sabemos que el factor genético es
muy importante (opción 3). También es lógico pensar que el antecedente personal de cáncer de
mama es factor de riesgo (opción 4). Paridad y edad del primer embarazo son factores que
influyen en el riesgo de cáncer de mama.

!
!
!

Las mujeres nulíparas tienen mayor riesgo, por lo que la opción que debemos seleccionar es la 1,
ya que las multíparas tienen menor riesgo; cuanto mayor es la edad a la que se tiene el primer
embarazo, mayor es el riesgo a padecer cáncer de mama; aquí es donde se demuestra el carácter
hormonodependiente de este tipo de cáncer. (R/1)

223. ¿Cuál es el defecto campimétrico típico de los pacientes con adenoma hipofisario?

1. 1. Hemianopsia bitemporal.
2. 2. Disminución concéntrica del campo visual.
3. 3. Hemianopsia nasal.
4. 4. Escotoma central bilateral.
Gráfico de respuestas
Comentario

La hipófisis, al ser una estructura medial, podría afectar al quiasma óptico en sus fibras más
mediales. Dado que estas fibras recogen la información visual procedente de ambos campos
temporales, tendríamos una hemianopsia heterónima bitemporal (respuesta correcta 1).

224. La presencia de depósitos masivos mesangiales y subendoteliales en alguna


nefropatía lúpica, permiten establecer el diagnostico histológico de:

1. 1. Nefropatía lúpica mesangial.


2. 2. Nefropatía lúpica proliferativa difusa.
3. 3. Nefropatía lupica proliferativa focal.
Cualquier tipo histológico ya que lo típico del lupus es la producción de
4. 4. anticuerpos contra estructuras nucleares y por tanto la formación de
inmunocomplejos a cualquier nivel.
Gráfico de respuestas
Comentario

Esta pregunta es de dificultad intermedia y requiere conocer la inmunofluorescencia de los estadios


de la nefritis lúpica. En el estadio 1 o GN de cambios mínimos lúpica tiene depósitos mesangiales
de inmunoglobulinas y complemento. En el estadio 2 o GN mesangial lúpica hay depósito
mesangial de IgG, IgM, IgA y complemento. En el estadio 3 o GN focal y segmentaria lúpica se ven
depósitos subendoteliales en mesangio y asas capilares. En el estadio 4 o GN proliferativa difusa
lúpica se observan depósitos subendoteliales y mesangiales en casi todas las asas capilares. En la
GN lúpica membranosa o estadio 5 se ven depósitos subepiteliales de inmunoglobulinas y
complemento. Por último, en el estadio 6 o GN lupica esclerosante o terminal sólo hay
glomeruloesclerosis difusa con pocos depósitos y extensa afectación tubulointersticial. (R/2)

225. Mujer de 50 años con antecedentes de hepatitis crónica C que presenta cuadro de
astenia, anorexia, poliartralgias, púrpura palpable, Raynaud, acrocianosis y neuropatía
periférica. En los exámenes de laboratorio se observa VSG elevada, creatinina 2.5 mg/dl,
factor reumatoide positivo y disminución de las fracciones C3 y C4 del complemento. Los
ANA y ANCA son negativos. Usted sospecha que la paciente presenta:

1. 1. Panarteritis nodosa clásica.


2. 2. Policondritis recidivante.
3. 3. Granulomatosis de Wegener.

!
!
!

4. 4. Crioglobulinemia mixta esencial.


Gráfico de respuestas
Comentario

Pregunta muy característica de la crioglobulinemia mixta esencial. Hay que pensar en esta
enfermedad ante mujeres con antecedentes de infección por el VHC y clínica neurológica
(neuropatía periférica, mononeuritis múltiple), cutánea (Raynaud, púrpura palpable) y
reumatológica. Las crioglobulinas (IgM) activan el complemento, por lo que éste estará descendido
en sangre periférica. El factor reumatoide es característicamente positivo. (R/4)

226. Una de estas aseveraciones sobre los antidepresivos tricíclicos es INCORRECTA:

1. 1. Los índices de endogenicidad predicen una buena respuesta terapéutica.


Los antidepresivos tricíclicos son la indicación prioritaria en las depresiones
2. 2.
atípicas.
La clomipramina es eficaz en el tratamiento de la depresión y también en
3. 3.
trastornos obsesivo-compulsivos.
4. 4. Un síntoma de toxicidad temprana lo constituye el alargamiento del QRS.
Gráfico de respuestas
Comentario

El tratamiento más eficaz de la depresión atípica son los IMAOs. Después de estos, la eficacia
entre ISRSs y tricíclicos es similar, pero preferimos los ISRSs por sus menores efectos
secundarios. En la práctica clínica solemos empezar con un ISRSs, que tiene mejor tolerancia que
el IMAO. (R/2)

227. Un niño de 6 años de edad, presenta un desarrollo puberal precoz con cuadros de
hiperfagia y un gran aumento de peso, En la anamnesis destaca poliuria y polidipsia,
¿Qué sospecharía?

1. 1. Craneofaringioma.
2. 2. Glioma óptico.
3. 3. Histiocitosis X.
4. 4. Hemangiomas.
Gráfico de respuestas
Comentario

Cuadro compatible con craneofaringioma y alteración de hipotálamo, lo que explica la hiperfagia, el


aumento de peso y la diabetes insípida con poliuria y polidipsia, (R/1).

228. Hombre de 80 años, que consulta por un primer episodio de hematoquecia de 7 días
de evolución, sin otros síntomas asociados. En el momento de la exploración física

!
!
!

llevaba dos días asintomático. El tacto rectal no mostró alteraciones relevantes, y una
sigmoidocolonoscopia que llegó hasta el ángulo esplénico del colon fue normal. No
existían alteraciones en la BH, el Hcto y la VSG eran normales. El diagnóstico más
probable en este paciente sería:

1. 1. Enfermedad de Crohn.
2. 2. Angiodisplasia.
3. 3. Carcinoma de colon ascendente.
4. 4. Hemorragia diverticular.
Gráfico de respuestas
Comentario

La causa más frecuente de hemorragia digestiva baja en un anciano es el divertículo sangrante.


Esta pregunta encierra una trampa muy sutil. Aunque la localización más frecuente de los
divertículos son a nivel del colon descendente - sigma, la mayoría de los divertículos sangrantes se
localizan en el colon derecho. Dado que la colonoscopia no ha progresado más allá al ángulo
esplénico, cabe la posibilidad de no observarlos, por lo que la respuesta correcta sería la 4.

La angiodisplasia de colon es la causa más frecuente de hemorragia digestiva baja RECIDIVANTE


en el anciano, y aquí nos hablan de un solo episodio. (R/4)

229. Una paciente de 49 años, diagnosticada de enfermedad de Ménière hace 5 años,


período durante el cuál ha presentado diez episodios de vértigo e hipoacusia. En la
actualidad presenta permanentemente hipoacusia derecha de 60 dB y un tinnitus intenso.
El tratamiento más adecuado es:

1. 1. Tietilperacina a altas dosis.


2. 2. Dextrano de bajo peso molecular i.v.
3. 3. Drenaje del saco endolinfático.
4. 4. Laberintectomía.
Gráfico de respuestas
Comentario
El tratamiento del Meniere más efectivo va dirigido a controlar los síntomas mas que a solucionar la
etiología de la enfermedad. El síntomas más molesto que puede llegar a ser incapacitante, es el
vértigo. Cuando existe vértigo y no hay una audición útil, el tratamiento más efectivo es la
laberintectomía, que destruye todo el laberinto por lo que no se producirán más síntomas, aunque
se perderá lo poco que quede de audición. Si existe vértigo incapacitante con audición útil, se
prefiere la cirugía del saco endolinfático o la neurectomía vestibular, ambos conservan la audición.
Hoy en día también se realiza la instilación de gentamicina en la ventana redonda ya que es más
vestibulotóxica que cocleotóxica. (R/4)

230. ¿Cuál es el principal mecanismo de acción del DIU T de Cu 380°?

1. 1. Anovulación.
2. 2. Espesamiento del moco cervical.
3. 3. Altera la capacitancia de los espermatozoides.
4. 4. Alteración de la motilidad de las trompas.
Gráfico de respuestas
Comentario

!
!
!

El DIU tiene varios mecanismos, pero los libradores de cobre ejercen una acción espermicida,
dificultando la fertilizaciones. El cobre potencia la acción espermicida. Respuesta 3 correcta.

231. ¿Cuál de los siguientes antiepilépticos administrado en mujeres embarazadas puede


producir, en el recién nacido, una deficiencia de factores de la coagulación dependientes
de vitamina K?

1. 1. Carbamazepina.
2. 2. Fenitoína.
3. 3. Etosuximida.
4. 4. Clonazepam.
Gráfico de respuestas
Comentario

Una pregunta muy importante, dado que los efectos secundarios de los antiepilépticos se han
preguntado, sobre todo los del ácido valproico, la fenitoína y la carbamacepina.

La fenitoína es capaz de inducir el metabolismo de otras moléculas, entre ellas la vitamina K. Por
ello, en mujeres embarazadas, es posible que produzca un déficit de los factores de la coagulación
dependientes de esta vitamina (II, VII, IX, X). (R/2)

232. Masculino de 19 años, deportista habitual, acude a nuestra consulta porque hace
unos días, durante un partido de fútbol, sufre un choque frontal contra otro jugador.
Durante su estancia en el suelo, con gran dolor en la zona golpeada, sufre episodio de
mareo que se sigue de episodio sincopal, de pocos segundos de duración. En el
interrogatorio el paciente refiere varios episodios similares con las extracciones
sanguíneas y el dolor intenso. Niega clínica cardiológica. El paciente no tiene
antecedentes de muerte súbita familiar. Se realiza una ecocardiografía que resulta
normal. El ECG basal muestra un ritmo sinusal a 50 lpm y bloqueo incompleto de rama
derecha. En el Holter-ECG de 24h se aprecian varios episodios nocturnos de bloqueo
auriculoventricular de segundo grado tipo Wenckebach. ¿Cuál de las siguientes
opciones cree que es más CORRECTA?:

El!paciente!puede!ser!dado!de!alta!con!recomendaciones!de!tipo!higiénico<
1.! ! 1.!
dietéticas.!
Sería!recomendable!realizar!un!estudio!electrofisiológico!previo!a!la!colocación!
2.! ! 2.!
de!un!marcapasos!definitivo.!
Estaría!indicado!la!colocación!de!un!Holter!implantable!subcutáneo!para!poder!
3.! ! 3.! objetivar!eventos!arrítmicos!en!nuevos!episodios!clínicos!y!poder!así!justificar!el!
implante!de!un!marcapasos!definitivo.!
El!paciente!debería!ingresar!inmediatamente!en!el!hospital!con!monitorización!
4.! ! 4.! electrocardiográfica!ante!el!alto!riesgo!de!recidiva!del!cuadro!y!posible!muerte!
súbita!asociada.!
Gráfico de respuestas
Comentario
El cuadro que se expone aquí es el típico cuadro sincopal vasovagal con desencadenantes
habituales como son el dolor o la extracción de sangre. Realmente el diagnóstico es clínico y no
son necesarias más pruebas complementarias. La bradicardia sinusal y un bloqueo incompleto de
rama derecha son frecuentes en pacientes jóvenes con tono vagal aumentado sin tener que
asociarse a patología cardiológica. De la misma manera, un BAV tipo Wenckebach en gente joven,

!
!
!

con tono vagal aumentado y especialmente durante el sueño puede ser fisiológico y no se debe
realizar ninguna acción diagnóstica ni terapéutica. Por tanto, en este paciente, deberemos calmarle
y explicarle el excelente pronóstico de su enfermedad así como darle algunos consejos como evitar
desencadenantes, hidratarse abundantemente y realizar maniobras de contrapresión ante el inicio
de síntomas. (R/1)

233. En un paciente EPOC en tratamiento por reagudización infecciosa con eritromicina,


usted considera administrarle teofilina. ¿Cuál de las siguientes es cierta?:

1. 1. Pautaría la dosis habitual de teofilina.


2. 2. Mantendría niveles plasmáticos de 30 mg/ml.
3. 3. En ningún caso la administraría por vía intravenosa.
Si el paciente presenta náuseas y cefalea sospecharía efectos secundarios de esta
4. 4.
medicación.
Gráfico de respuestas
Comentario

Pregunta muy difícil que pese a estar en el bloque de neumología podría ser explicada mucho
mejor con unos conocimientos sencillos de farmacología. La teofilina es un fármaco principalmente
broncodilatador utilizado cada vez menos en el tratamiento crónico del EPOC, aunque tiene otros
efectos. El problema es que posee un margen terapéutico estrecho (10 - 20 mcg/ml) y que
situaciones que alteren sus niveles pueden provocar problemas serios (intoxicación que comienza
con náuseas y cefaleas) o reducir su eficacia. Recuerda el concepto de fármaco inhibidor
enzimático que es aquel que disminuye la eficacia de las enzimas que metabolizan otro fármaco o
él mismo, teniendo como consecuencia una disminución de su aclaramiento y una acumulación de
éste, provocando efectos adversos en fármacos con rango terapéutico estrecho como la teofilina.
Dentro de este grupo de inhibidores enzimáticos debe recordar: macrólidos, cimetidina, alopurinol,
esteroides,... . Otras situaciones que disminuyen el aclaramiento de la teofilina son: neonatos,
ancianos, hepatopatías, cardiopatías, cor pulmonales, fiebre, dietas ricas en hidratos de
carbonos,... Por el contrario aumentan dicho aclaramiento la edad inferior a 16 años. El tabaco, la
marihuana, dietas ricas en proteínas, fármacos inductores enzimáticos (isoprotenerol, fenobarbital,
rifampicina, hidracidas, fenitoína). (R/4)

234. Ante un recién nacido hijo de madre adicta a heroína que comienza a las pocas horas
de vida con irritabilidad, hipertonía, hiperreflexia, temblores y llanto continuo, sin
convulsiones, ¿qué tratamiento emplearía?:

1. 1. Fenitoína.
2. 2. Fenobarbital.
Unicamente sería necesario reposo en ambiente tranquilo, sin ruidos y con luz
3. 3.
tenue, al no presentar convulsiones.
4. 4. Naloxona.
Gráfico de respuestas
Comentario

La clínica del síndrome de abstinencia a opiáceos incluye hiperexcitabilidad (irritabilidad, llanto


agudo, taquicardia, taquipnea, diaforesis) y síntomas más vagos (febrícula, diarrea, rinorrea). En
cuanto a su tratamiento, no olvide dos pilares básicos: administración de fenobarbital a dosis
progresivamente decrecientes y mucha paciencia (conseguir la deprivación puede llevar semanas
o meses). (R/2)

!
!
!

235. Acerca de los antiarrímicos, ¿cuál de las siguientes respuestas es CORRECTA?:

Los antiarrítmicos de clase I aumentan la velocidad de ascenso del potencial de


1. 1.
acción al modificar la entrada de calcio en la célula.
2. 2. Los antiarrímicos de clase III acortan la duración del potencial de acción.
Los antiarrítmicos de clase II tienen un efecto betabloqueante y algunos de ellos
3. 3.
efecto estabilizador de membrana.
4. 4. La amiodarona y el sotalol son antiarrítmicos de clase IV.
Gráfico de respuestas
Comentario

Esta es una pregunta difícil acerca del mecanismo de acción de los antiarrítmicos TEMA MUY
POCO PREGUNTADO; es importante no obstante conocer lo distintos grupos de antiarrítmicos que
existen, cuáles son sus indicaciones y algún efecto secundario característico de algún fármaco,
como por ejemplo la amiodarona. Los antiarrítmicos de clase I actúan bloqueando los canales
rápidos del sodio, ralentizando así la fase 0 del potencial de acción y disminuyendo la velocidad de
conducción (respuesta 1 falsa). Los Ib acortan la duración del potencial de acción. Los fámacos
que la alargan son del grupo Ia, que prolongan el periodo refractario y el QT; ¡no olvide aprender
bien todo lo que prolonga el QT! Los antiarrítmicos de clase III, como son la amiodarona y sotalol
(respuesta 4 falsa) aumentan la duración del potencial de acción prolongando también el QT
(respuesta 2 falsa). Los de clase II incluyen a los betabloqueantes, digoxina y adenosina. Algunos
betabloqueantes tienen propiedades estabilizantes de membrana, efecto que en dosis altas
disminuye la despolarización (por tanto respuesta 3 correcta).

236. presenta dolor abdominal difuso que ocasionalmente se extiende desde el


epigastrio a la espalda, que mejora algo al flexionar el tronco hacia delante. El dolor se
acompaña de la presencia de ictericia, prurito, orinas colúricas y pérdida de peso. A la
exploración se palpa una vesícula biliar aumentada de tamaño y signos de tromboflebitis
en las piernas. En los exámenes de laboratorio está elevada la fosfatasa alcalina, la
gamma GT y la bilirrubina. ¿Cuál será el origen del cuadro de este paciente?

1. 1. Tumor de Klatskin.
2. 2. Cáncer de páncreas.
3. 3. Pancreatitis crónica.
4. 4. Colecistitis aguda.
Gráfico de respuestas
Comentario

Por todos los datos del enunciado de la pregunta, nos decantamos por la opción del cáncer de
páncreas, muy probablemente localizado en la cabeza, dada la clínica de colestasis por posible
compresión de vías biliares. Analicemos el caso clínico.

Estamos ente un hombre por encima de los 40 años, que es el grupo de población con mayor
incidencia de cáncer de páncreas. Presenta los síntomas y signos más frecuentemente
encontrados en los pacientes con cáncer de páncreas: dolor abdominal (en más del 80 %), ictericia
(en más del 80 % de los cánceres de cabeza de páncreas) y pérdida de peso (en más del 90% )
con vesícula palpable (alrededor del 30 % de los cánceres de cabeza de páncreas, signo de
Courvoisier-Terrier). Por otra parte, se acompaña de tromboflebitis migratoria (síndrome de
Trousseau), de naturaleza paraneoplásica, muy vinculada a este tipo de tumores. (R/2)

!
!
!

237. Todos los siguientes cuadros, EXCEPTO uno, pueden ser causa de metrorragia del
1º trimestre de gestación:

1. 1. Enfermedad trofoblástica.
2. 2. Desprendimiento prematuro de placenta.
3. 3. Embarazo ectópico.
4. 4. Hemorragia de implantación.
Gráfico de respuestas
Comentario

Dentro de las hemorragias del primer trimestre englobamos tres patologías: aborto, embarazo
ectópico y enfermedad trofoblástica, mientras que el desprendimiento prematuro de placenta lo
englobamos en las hemorragias del tercer trimestre junto a la placenta previa, la rotura uterina y la
rotura de vasos previos. La hemorragia de implantación sería una forma de amenaza de aborto y
por tanto, se incluye dentro de las hemorragias del primer trimestre. (R/2)

238. A 5-year-old child is brought to the pediatrician's office presenting with irritability
and sleeping difficulty during the last month. She often scratches her anal region, which
is erythematous at physical examination. She has not diarrhea or other symptoms. Scotch
tape test is positive. Which of the following is the best treatment option?

1. 1. Albendazole.
2. 2. Praziquantel.
3. 3. Trimethoprim-Sulfamethoxazole.
4. 4. Ivermectin.
Gráfico de respuestas
Comentario
Albendazole. Enterobius vermicularis is the most common parasite infection during childhood in
developed countries. The main symptom is perianal itching. Most patients have difficulty sleeping
because the female worms migrate out through the rectum onto the perianal skin to deposite eggs
during the night. Albendazole or mebendazole are first-line treatments. (R/1)

239. Durante el trabajo de parto de un niño se observa partículas de meconio en el liquido


amniótico ¿Que procedimiento debe realizarse luego de colocarlo bajo calor radiante?

1. 1. Secarlo y administrarle oxigeno.


2. 2. Secarlo y observar.
3. 3. Secarlo, evaluar calor y frecuencia cardiaca.
4. 4. Secarlo posicionarlo y succionar boca.
Gráfico de respuestas
Comentario

Debe leer muy detendamente esta pregunta y en caso de no encontrar la respuesta que busca
pues la que más se asemeje. La indicación de aspiración antes de llanto cada vez está más
discutida. Si el niño presenta buen patrón respiratorio y la sospecha de aspiración es baja, puede
observarse la evolución. (R/2)

240. A 7-month-old girl is being evaluated at a routinely check up for vaccination. She has
no medical history and lives with her mother because her parents are divorced. Physical

!
!
!

examination reveals a bluish pigmented area near the base of the spine. What should be
done in this case?

1. 1. Reassure the mother and follow routine visits as scheduled.


Betablockers have demonstrated to reduce the appearence of this lesion in some
2. 2.
cases.
3. 3. Laser therapy.
4. 4. Coagulation tests to rule out a coagulopathy.
Gráfico de respuestas
Comentario
Reassure the mother and follow routine visits as scheduled. The Mongolian spot (congenital dermal
melanocytosis) is a bluish spot located in the lumbar area, asymptomatic, presenting in up to 10%
of newborns. This stain is more common in Latin and Eastern countries (up 95%). It has no
pathological significance and usually disappears within three to five years of age. (R/1)

241. Paciente femenino de 25 años con amenorrea secundaria. Tras el tratamiento con
progesterona no menstrua. La administración de una secuencia de estrógenos y
progestágenos logra que la paciente menstrue. Los niveles de gonadotropinas son bajos.
El TAC de silla turca es normal. Lo más probable es que se trate de:

1. 1. Anovulación.
2. 2. Hipogonadismo hipogonadotropo.
3. 3. Insuficiencia hipofisaria.
4. 4. Prolactinoma.
Gráfico de respuestas
Comentario

Cuando existen niveles bajos de gonadrotropinas se procede inicialmente a la provocación con


progestágenos; si se produce hemorragia por deprivación, se confirma la existencia de estrógenos
ováricos y la integridad endometrial, y el diagnóstico es de anovulación.

En caso contrario, se procede a la prueba de provocación con estrógenos y progesterona. Si se


produce hemorragia, indica que el endometrio está intacto pero falta el estímulo proliferativo de los
estrógenos. Se tratará, pues, de una amenorrea hipoestrogénica hipogonadotropa, y el trastorno
residirá en problemas orgánicos o funcionales del sistema hipotálamo-hipofisario.

Si no hay respuesta, habrá que pensar en destrucción endometrial. (R/2)

242. A patient with grade C liver cirrhosis, presents to the emergency department with
fever (37.8 ° C) and continuous abdominal pain in both flanks, without nausea or vomiting.
Physical examination revealed the presence of mucocutaneous jaundice and a bruise on
the right thigh of the patient related to an accidental fall two days ago. The abdomen is
soft, nontender, without signs of peritoneal irritation, with dullness to percussion in both
flanks. Diagnostic paracentesis is performed, yielding the following results: WBC count
1334/mm3, with 45% neutrophils. Which of the following is CORRECT?

!
!
!

You must wait for the results of the ascitic fluid culture to initiate treatment
1. 1.
accordingly.
2. 2. This complication does not worsen the prognosis of the patient.
3. 3. The coexistence of renal failure increases mortality.
4. 4. The risk of recurrence after the first episode is minimal.
Gráfico de respuestas
Comentario

Una pregunta evidente, que se podría haber resuelto incluso sin saber nada del tema. Como se
sabe, la insuficiencia renal, por sí sola, puede ser suficiente como para comprometer la vida del
que la padece. Pues bien, si tiene un cuadro como el que nos plantean (peritonitis bacteriana
espontánea), ¿cuándo tendrá más probabilidades de morir? ¿Con insuficiencia renal o sin ella?

Aunque no supiéramos de qué enfermedad se trata, la insuficiencia renal es un agravante, con lo


que la respuesta 3 es correcta de forma forzosa.

243. ¿Cuál de los siguientes hallazgos radiológicos NO es propio de la afectación


articular de la artritis reumatoide?

1. 1. Patrón de afectación simétrico.


2. 2. Esclerosis subcondral.
3. 3. Erosiones óseas subcondrales.
4. 4. Deformación articular.
Gráfico de respuestas
Comentario

Pregunta asequible pero es importante que sepa diferenciar los hallazgos radiológicos de la
afectación articular de la artritis reumatoide (AR) de los propios de la artrosis. En la AR vamos a
encontrar un patrón simétrico, con pérdida de cartílago articular y erosiones subcondrales, con
osteopenia yuxtarticular (y no esclerosis subcondral que es propia de la artrosis). Así mismo, en
fase avanzada encontraremos deformidad articular. (R/2)

244. Una paciente caquéctica, consulta por deterioro progresivo. Tiene anemia, ascitis, y
nódulos metastáticos en áreas supraclaviculares y zona umbilical. ¿Cuál considera el
origen más probable del tumor?

1. 1. Estómago.
2. 2. Colon.
3. 3. Hígado.
4. 4. Páncreas.
Gráfico de respuestas
Comentario

Paciente con síndrome constitucional y anemia que presenta ascitis y nódulos metastáticos en
áreas supraclaviculares y zona umbilical. Es el típico caso clínico de adenocarcinoma gástrico que
ha metastatizado produciendo adenopatías metastásicas supraclaviculares (conocidas como
ganglio de Virchow) y diseminación por la superficie peritoneal, que ocasiona ascitis y nódulos
periumbilicales (conocidos como el nódulo de la hermana María José). (R/1)

245. Puede auscultarse un tercer tono fisiológico en todas, EXCEPTO una de las
siguientes circunstancias:

!
!
!

1. 1. En adultos jóvenes, fuertes, sanos.


2. 2. En mujeres gestantes.
3. 3. En pacientes mixedematosos.
4. 4. En pacientes con enfermedades febriles.
Gráfico de respuestas
Comentario

El tercer ruido se motiva por un llenado ventricular rápido y turbulento (sobrecarga de volumen).
Puede ser fisiológico en determinadas situaciones, como en niños, situaciones de alto gasto
cardíaco, durante el ejercicio,… Es lo que ocurre con las opciones 1, 2, y 4 (recuerde que la fiebre
produce taquicardia). Sin embargo, un paciente mixedematoso tendría tendencia a la bradicardia y
a una contractilidad menor, por lo que su situación hemodinámica no justificaría como normal un
tercer tono (respuesta 3).

246. Uno de los siguientes fármacos, cuando afecta al hígado, produce


fundamentalmente un cuadro de granulomatosis hepática. Señálelo:

1. 1. Isoniacida.
2. 2. Clorpromacina.
3. 3. Ketoconazol.
4. 4. Zidovudina.
Gráfico de respuestas
Comentario

Pregunta difícil sobre efectos adversos a nivel hepático de distintos fármacos. Es importante
recordar que el uso de clorpromacina se ha asociado a granulomatosis hepática. Respuesta 2
correcta.

247. Mujer de 25 años que consulta por hirsutismo, sin evidencia de claros signos de
virilización, y oligomenorrea. TA: 120/70 mmHg. Se detectan niveles altos de 17-
hidroxiprogesterona en plasma, que aumentan aún más tras el estímulo con ACTH. La
DHEA-s está elevada (al doble de lo normal). Testosterona ligeramente aumentada. LH y
FSH normales. Los 17-cetosteroides en orina están aumentados. ¿Cuál es el diagnóstico
de la paciente?

Hiperplasia suprarrenal congénita de inicio en el adulto por déficit de 21-


1. 1.
hidroxilasa.
2. 2. Tumor ovárico virilizante.
3. 3. Síndrome del ovario poliquístico.
4. 4. Hirsutismo idiopático.
Gráfico de respuestas
Comentario

Nos presentan una paciente joven con un cuadro de hirsutismo de aparición brusca y amenorrea
secundaria. Como datos analíticos de interés, destacan la elevación del cortisol (que no suprime
con dosis elevadas de DXT) cifras de ACTH suprimidas, y una concentración de DHEAs muy por
encima de los límites de la normalidad.

Nuestra primera sospecha diagnóstica, dada la rapidez de instauración del cuadro, debe ir dirigida
hacia una patología tumoral. El origen de dicho tumor parece estar en la glándula suprarrenal, ya
que presenta una producción de cortisol independiente de ACTH, y el andrógeno que se encuentra

!
!
!

elevado es la DHEAs, cuya producción depende mayoritariamente de esta glándula. Por todo esto,
el diagnóstico de sospecha debe ser un carcinoma suprarrenal, y la técnica de imagen de elección
es la TAC abdominal, óptima para valorar estructuras retroperitoneales (suprarrenales, aorta,
riñón). (R/1)

248. Un hombre de 37 años sin antecedentes de interés acude a Urgencias por haber
sufrido un síncope mientras caminaba. Había comenzado dos días antes con disnea de
esfuerzo. Exploración: peso 75 kg, TA 75/50 mmHg. Saturación de oxígeno 89%.
Auscultación pulmonar normal. Auscultación cardíaca: tonos rítmicos taquicárdicos a
130 lpm, abdomen normal. Extremidades: sin alteraciones. Biometría hemática: normal.
Troponina: 1.2 ng/mL (normal: <0.04). ECG: Taquicardia sinusal a 130 lpm. T invertida en
V1-V4. AngioTC torácica: defecto de repleción en ambas arterias pulmonares principales.
Una hora después de su llegada a Urgencias persiste TA 70/55 mmHg. ¿Cuál sería el
tratamiento inicial más adecuado?

Heparina no fraccionada, 10,000 U i.v. ante la sospecha clínica. Fibrinólisis con


1. 1. activador tisular del plasminógeno (tPA), 100 mg i.v., una vez confirmado el
diagnóstico.
2. 2. Enoxaparina: 80 mg s.c. cada 12 horas, comenzando ante la sospecha diagnóstica.
3. 3. Fondaparinux: 7.5 mg s.c. diarios.
4. 4. Tromboendarterectomía pulmonar de urgencia.
Gráfico de respuestas
Comentario

Se trata de un tromboembolismo pulmonar masivo, como nos resuelven al final del enunciado, y
nos piden el tratamiento inicial más adecuado. Dado que presenta hipotensión mantenida, este
paciente se considera de riesgo ALTO, y debe recibir fibrinolisis si no existen contraindicaciones.
Con esto ya podríamos resolver el caso (respuesta 1 correcta). Para dar más detalles,
puntualizariamos que la enoxaparina y el fondaparinux, se dan como tratamiento cuando hay
estabilidad hemodinámica, mientras que la heparina no fraccionada se reserva para el TEP con
compromiso hemodinámico, como es este caso. La tromboendarderectomía de urgencia es de uso
muy restringido, en caso de HTP crónica secundaria a TEP con clínica grave. Hay que recordar
que ante sospecha intermedia o alta de TEP, hay que comenzar el tratamiento anticoagulante aun
sin disponer de la confirmación diagnóstica. (R/1)

249. Respecto al reflujo vesicoureteral en la infancia, señale la opción INCORRECTA:

1. 1. Es preciso descartarlo ante un episodio de pielonefritis aguda.


En la mayoría de los casos se debe a una deficiencia de la musculatura del uréter
2. 2.
infravesical (reflujo primario).
3. 3. La prueba diagnóstica de elección es la urografía dinámica.
Según el grado de reflujo vesico ureteral se optará por profilaxis antibiótica o
4. 4.
solución quirúrgica.
Gráfico de respuestas
Comentario

El Reflujo Vesico Ureteral en la mayoría de los casos se debe a una deficiencia de la musculatura
del uréter infravesical (reflujo primario) (70% casos). Nunca olvide que la prueba gold standard
para su diagnóstico es la cistografía miccional seriada, en función de ella se gradúa el Reflujo
Vesico Ureteral y se optará por profilaxis antibiótica o cirugía para su tratamiento. (R/3)

!
!
!

250. Una de las siguientes características NO es propia de la colitis isquémica:

1. 1. Es la forma más frecuente de enfermedad isquémica del colon.


2. 2. En la mayor parte de los casos es no oclusiva.
3. 3. Asociación a coartación de aorta.
4. 4. Imagen de "huellas digitales" en la radiografía simple de abdomen.
Gráfico de respuestas
Comentario

La colitis isquémica se debe a una insuficiencia circulatoria a nivel del colon. Lo más frecuente es
que obedezca a causas no oclusivas (por ejemplo, hipoperfusión por bajo gasto en relación con
una insuficiencia cardíaca descompensada). La región más frecuentemente afectada es el ángulo
esplénico del colon (punto de Griffith), porque es una de las zonas peor irrigadas del colon. En
cambio, no se suele afectar el recto, puesto que su irrigación no sólo depende del sistema de las
mesentéricas (hemorroidal superior), sino que también recibe irrigación de otro origen (hemorroidal
media, rama de la hipogástrica, y hemorroidal inferior, rama de la pudenda).

La clínica característica es un dolor moderado tipo cólico, de inicio súbito, y que puede
acompañarse de rectorragia o diarrea sanguinolenta. Es poco frecuente el vómito. Puede
manifestarse de forma fulminante, pero lo más frecuente es que se trate de un cuadro subagudo.
Cuando progresa hacia la gangrena, el dolor se vuelve constante y aparecen síntomas de
abdomen agudo. En casos crónicos, pueden producirse estenosis segmentarias de colon, con el
consiguiente riesgo de obstrucción. En ocasiones, esta enfermedad se asocia a estenosis aórtica,
en cuyo caso se habla de síndrome de Heyde (no a coartación de aorta).

El diagnóstico definitivo se realiza por colonoscopía. Rara vez se recurre al enema opaco, el cual
revela el dato clásico de las “impresiones digitales” o estenosis en el segmento cólico afectado. Por
supuesto, el enema opaco está contraindicado en pacientes con sospecha de gangrena. La
arteriografía no es útil y no demuestra anomalías vasculares, lo cual sugiere que se trata de una
enfermedad de la microvascularización. (R/3)

251. ¿Qué tratamiento considera más adecuado para un anciano encamado y con
"atragantamientos" frecuentes por un ictus previo, que acude al Hospital por un cuadro
de fiebre y somnolencia, y en cuya placa de tórax se observa un infiltrado con
broncograma aéreo en el segmento superior del lóbulo inferior del pulmón derecho?

1. 1. Amoxicilina-ácido clavulánico.
2. 2. Penicilina G más gentamicina.
3. 3. Doxiciclina.
4. 4. Cefotaxima más aztreonam.
Gráfico de respuestas
Comentario

El paciente tiene varios factores de riesgo para padecer una neumonía aspirativa: edad avanzada,
encamamiento y dificultad para la deglución. Todo ello favorece el paso del contenido digestivo
hacia la vía aérea. Además, desde el punto de vista radiológico también la neumonía sugiere la
posibilidad de aspiración, ya que el infiltrado pulmonar se sitúa en zonas declives (como es el
segmento superior del lóbulo inferior, en un sujeto en decúbito supino).

!
!
!

Asumiendo la causa más probable (aspiración), habría que elegir un antibiótico que ofreciese una
buena cobertura frente a la flora mixta de la cavidad oral, incluyendo los anaerobios. Por ello,
deberíamos elegir amoxicilina – ácido clavulánico (R/1)

252. Señale la peculiaridad clínica que NO es característica del síndrome alcohólico fetal:

1. 1. Bajo peso al nacimiento.


2. 2. Microcefalia.
3. 3. Retraso mental.
4. 4. Hipoplasia de arco aórtico.
Gráfico de respuestas
Comentario

El síndrome alcohólico fetal es un síndrome dismórfico evitable que aparece tras el consumo de
alcohol durante la gestación. Cursa con retraso ponderoestatural, retraso mental, con microcefalia
asociada, facies peculiar (micrognatia, labio superior fino, filtrum nasal largo) y ocasionalmente
también asocia cardiopatía (hipertrofia septal asimétrica, no hipoplasia de arco aórtico). (R/4)

253. Un carcinoma de ovario que invade ambos ovarios, ascitis e implantes peritoneales
de diámetro superior a los 2cm., debe considerarse como un estadio:

1. 1. II C.
2. 2. III C.
3. 3. IV.
4. 4. I C.
Gráfico de respuestas
Comentario

El estadiaje de tumores de ovario no es muy útil de estudiar. Simplemente recuerde que el estadio
I es limitado al ovario, II extensión a pelvis, III extensión a peritoneo, IV metástasis y que la letra C
significa a grandes rasgos Cápsula rota Citología liquido peritoneal positiva. En este caso tenemos
extención peritoneal (estadio III) y asicitis tumoral (IIIC) Si los implantes peritoneales son mayor de
2 cm o hay adenopatias inguinales retroperitoneales también sería estadio IIIC (R/2)

254. En el tratamiento empírico inicial de la meningitis en el periodo neonatal se emplean


dos antibióticos asociados (generalmente las cefalosporinas de tercera generación
asociadas a la ampicilina) para cubrir los gérmenes más frecuentes en este período.
Señale, de ellos, el que suele ser resistente a las primeras y sensible a la segunda:

1. 1. Escherichia coli.
2. 2. Neumococo.
3. 3. Listeria monocytogenes.
4. 4. Pseudomonas aeruginosa.
Gráfico de respuestas
Comentario

La listeriosis durante el embarazo puede ser asintomática o causar una enfermedad febril, que
puede confundirse con una gripe o síndrome gripal. Habitualmente, el diagnóstico de certeza sólo
es posible mediante el hemocultivo, donde crecerá un bacilo aerobio, móvil y grampositivo. La
infección, ya sea oculta o manifiesta, puede estimular el trabajo de parto. La listeriemia materna
produce una infección fetal, que provoca característicamente lesiones granulomatosas

!
!
!

diseminadas con microabscesos. La eliminación de meconio es frecuente cuando existe infección


fetal.

El neonato es particularmente susceptible a la infección por Listeria (recuerde que los ancianos
también lo son, como a veces se pregunta en Neurología). La tasa de mortalidad de la listeriosis
neonatal es cercana al 50%. En estos casos, es usual la sepsis neonatal precoz, mientras que la
listeriosis de inicio tardío se manifiesta como una meningitis a las 3-4 semanas de edad.

Estas infecciones son muy parecidas a las causadas por estreptococo beta-hemolítico del grupo B.
Hay evidencia que el tratamiento antimicrobiano materno puede ser eficaz en caso de infección
fetal. Como tratamiento, se recomienda la combinación de ampicilina y gentamicina. (R/3)

255. La pancreatitis severa puede presentar las siguientes manifestaciones. EXCEPTO.

1. 1. Absceso pancreático.
2. 2. Seudoquiste pancreático.
3. 3. Ranson de 2 puntos.
4. 4. Pancreatitis con colecciones líquidas
Gráfico de respuestas
Comentario

La pancreatitis aguda severa se clasifica de distintas formas, pero la opción incorrecta es la 3, ya


que, para que se considere el Ranson en niveles de pancreatitis severes tiene que tener un punta
> o igual a 3. Otros patrones de severidad es un APACHE-II > o igual 8.(R/3)

256. Respecto a la enfermedad de Paget de la mama, señale la respuesta FALSA:

1. 1. A diferencia del eccema, presenta límites netos.


2. 2. Se asocia siempre a un adenocarcinoma ductal subyacente.
Se caracteriza histológicamente por las "células de Paget", que tiñen con PAS y
3. 3.
son diastasa resistentes.
4. 4. No es pruriginosa.
Gráfico de respuestas
Comentario

Pregunta de dificultad media sobre la enfermedad de Paget de la mama.

La enfermedad de Paget debe considerarse como la extensión de un tipo de cáncer de mama


ductal a la epidermis suprayacente, afectando pezón y la areola. Es poco frecuente, representando
el 1- 3% de todos los cánceres de mama. Clínicamente, consiste en una lesión eccematosa,
asociando también prurito (respuesta 4 falsa). Por ello, el eccema es su principal diagnóstico
diferencial. Una de las características por las que puede distinguirse del eccema es por ser
unilateral (el eccema suele afectar a ambas mamas). Otro rasgo diferencial son los límites, que
suelen ser difusos en el eccema y mejor definidos en el Paget mamario.

Recuerda que, histológicamente, son características las células de Paget. Son grandes,
redondeadas, PAS- positivas, con núcleos de gran tamaño y sin puentes intercelulares que las
unan. (R/4)

257. Señale de entre los siguientes antígenos HLA aquellos que estén relacionados
estrechamente con la diabetes mellitus tipo 1:

!
!
!

1. 1. B27, B29.
2. 2. DR8, DR9.
3. 3. D3, D4.
4. 4. B10, B12.
Gráfico de respuestas
Comentario

En la diabetes mellitus tipo 1 participan muchos genes y el mecanismo herediartio no está bien
conocido. El principal gen de predisposición a la DM-1se localiza en la región HLA del cromosoma
6. la mayoría de los diabéticos tiene el haplotipo HLA DR3, HLA DR4 o ambos. Sin embargo, la
mayoría de los individuos portadores de estos haplotipos no padecen diabetes y la mayoría de los
individuos afectos de esta enfermedad carece de pacientes de primer grado que la sufran. El DR2
puede tener papel protector. (R/3)

258. El principal factor etiológico del embarazo ectópico sería:

1. 1. La utilización de DIU.
2. 2. Endometriosis.
3. 3. Antecedentes de EIP.
4. 4. Edad materna.
Gráfico de respuestas
Comentario

Entre los factores de riesgo de embarazo ectópico, encontramos todas las opciones que nos
presentan, salvo la edad materna. De ellos, el más frecuente es la presencia o los antecedentes de
enfermedad inflamatoria pélvica. La inflamación de las trompas modifica la motilidad de las mismas
y produce fenómenos de cicatrización, por lo que predispone al embarazo ectópico. La cirugía
abdominal y la endometriosis producirían adherencias que pueden englobar a las trompas y así
modificar también su motilidad. El DIU disminuye el embarazo intrauterino, pero no es tan eficaz
con los extrauterinos, por lo que aparece un aumento de la frecuencia relativa de los mismos. (R/3)

259. A 9-year-old girl is brought to consultation for a routine visit. She is taller than
expected for her age. She has no previous medical history. Physical examination shows
mild scoliosis, disproportionately long arms and legs and abnormal joint flexibility. Her
father died suddenly and then the postmortem diagnosis of ruptured aortic aneurysm was
made. What is the most likely etiology of this condition?

1. 1. Mutations of the fibrillin 1 gene.


2. 2. Vitamin D deficiency.
3. 3. Defective collagen production.
4. 4. Mutations of the procolagen type 1 gene.
Gráfico de respuestas
Comentario
Mutations of the fibrillin 1 gene. Patients with Marfan syndrome have typical phenotype features,
such a high height and joint hyperlaxity. Family history is very useful and cases of sudden death
due to aneurysm rupture may be present, as in this case. The disease is caused by a mutation in
the fibrillin1 gene. Collagen mutation predispose to osteogenesis imperfecta. (R/1)

260. Un paciente de 27 años viene a la consulta con una historia de 1 año de duración de
dolor en ambos glúteos irradiado por cara posterior de muslo hasta su porción media. El

!
!
!

dolor es peor al amanececer; se acompaña de rigidez lumbar y mejora con el ejercicio.


¿Cuál de los siguientes procedimientos aportaría más información diagnóstica en este
paciente?

1. 1. Los niveles de proteína C reactiva y de antiestreptolisina O (ASLO).


2. 2. La resonancia magnética de columna lumbar.
3. 3. El tipaje HLA-B27.
4. 4. La radiografía de las articulaciones sacroilíacas.
Gráfico de respuestas
Comentario

Cuadro clínico compatible con espondilitis anquilosante: varón joven con afectación de esqueleto
axial (dolor en región glútea bilateral típicamente inflamatorio: peor en reposo y que mejora con el
movimiento). Dado que la presencia de sacroileítis radiológica es imprescidible para el diagnostico
de EA debemos solicitar en primer lugar una Rx de sacroilíacas (opción 4 correcta). La VSG y PCR
son inespecíficas, el tipaje HLA- B27 no es necesario para el diagnóstico y los ASLO no se utilizan
en la EA. (R/4)

261. ¿Qué afirmación sobre las atresias intestinales es FALSA?:

1. 1. Puede existir antecedente de polihidramnios.


2. 2. La asociación con síndrome de Down es frecuente.
La observación de calcificaciones intraperitoneales en la radiografía descarta este
3. 3.
diagnóstico.
4. 4. La atresia de colon puede asociarse a enfermedad de Hirschprung.
Gráfico de respuestas
Comentario

Las atresias intestinales son cuadros muchas veces de tediosa evolución, pues requieren múltiples
intervenciones quirúrgicas y ocasionalmente derivan hacia la presencia de secuelas tales como el
intestino corto. Son especialmente frecuentes en los niños con síndrome de Down, a nivel de
yeyuno-íleon. Más raramente afectan al colon, en el contexto de un megacolon agangliónico
extremo. Suele tener antecedente obstétrico de polihidramnios. Si generan peritonitis meconial
intraútero, en la radiografía de abdomen se aprecian calcificaciones peritoneales dispersas. (R/3)

262. Un paciente alcohólico de 55 años es llevado a urgencias por la policía. El paciente


se halla somnoliento y desorientado. Su aliento huele a alcohol. Las pupilas son
isocóricas normorreactivas, y los movimientos oculares, normales. En las maniobras de
Barré y Mingazzini se comprueba una paresia de las extremidades izquierdas, y al
explorar el reflejo cutáneo plantar, se halla un Babinski izquierdo. ¿Cuál de los siguientes
enunciados es correcto?

Nos hallamos únicamente ante un caso de intoxicación aguda enólica y no


1. 1.
debemos realizar ninguna exploración complementaria.
2. 2. El cuadro es específico de encefalopatía de Wernicke y se administrará B1.
3. 3. El paciente sufre una polineuritis aguda y se practicará una punción lumbar.
Debe practicarse una TC craneal o una angiografía cerebral para descartar un
4. 4.
accidente vascular cerebral.
Gráfico de respuestas
Comentario

!
!
!

La presencia de focalidad neurológica, con afectación de predominio crural y signo de Babinski


positivo (vía piramidal) orienta fuertemente hacia la respuesta 4. El resto de las opciones se
descartan con facilidad.

La encefalopatía de Wernicke obedece a un déficit de tiamina y se caracteriza por la tríada:


oftalmoparesia, ataxia y síndrome confusional.

Las polineuropatías son cuadros de instauración gradual, normalmente simétricos y de predominio


distal.

Por último, la intoxicación alcohólica aguda tampoco explica esta clínica (es posible que también la
tenga, porque huele a alcoho, pero no justifica la paresia ni el Babinski). (R/4)

263. Son signos clínicos de choque hipovolémico temprano, los siguientes EXCEPTO:

1. 1. Taquicardia.
2. 2. Cianosis.
3. 3. Estado mental alterado.
4. 4. Presión arterial normal.
Gráfico de respuestas
Comentario

Debes dominar la siguiente tabla, para fines del ENARM y la vida. La cianosis es una
manifestación clínica que aparece en etapas más avanzadas del proceso. (R/2)

264. Paciente de 42 años con carcinoma de cérvix, diagnosticado por biopsia, que invade
el labio anterior y 1 cm de la cúpula vaginal; los parametrios no están invadidos; la

!
!
!

rectoscopía, la urografía descendente y la cistoscopía son normales. En este caso, el


tratamiento más CORRECTO sería:

1. 1. Radioterapia externa más quimioterapia.


2. 2. Histerectomía total ampliada tipo Wertheim-Meigs más radioterapia externa.
3. 3. Radioterapia de contacto, seguida de histerectomía total simple.
4. 4. Ninguna de las anteriores.
Gráfico de respuestas
Comentario

Pregunta teórica acerca del tratamiento del carcinoma de cérvix, debiendo conocer datos básicos
del TNM. Recuerde que el estadio IIB implica afectación de parametrios. En este caso nos
presentan un carcinoma cervical estadio IIA por lo que el tratamiento de elección es la
histerectomía radical con linfadenectomía pélvica (cirugía de Wertheim-Meigs). La única opción
que te ofrece esta posibilidad es la respuesta número 2 (con RT externa adyuvante).

265. A 27-year-old woman that had an uncomplicated vaginal delivery three days ago
comes to the doctor's presenting with asthenia. Serologic test results are HBsAg Positive,
Anti HBsAg negative, IgM Anti HBcAg Positive, HBe Ag positive, Anti HBe Ag negative.
What is the best next step in the management of the newborn?

1. 1. Administration of hepatitis B immune globulin followed by vaccination.


2. 2. Hepatitis B vaccination.
3. 3. Initiate treatment with lamivudine.
4. 4. Periodic serologic control.
Gráfico de respuestas
Comentario
Administration of hepatitis B immune globulin followed by vaccination. Routine serological tests are
performed during pregnancy in order to treat connatal infection. In the case of VHB infection there
are measures of proved efficacy: at birth, administration of intravenous Ig and the first vaccination
dose.(R/1)

266. Una de las siguientes afirmaciones es típica de la artrosis primaria:

El recuento celular del líquido sinovial es con frecuencia superior a 3,000 células
1. 1.
por milímetro cúbico.
2. 2. Afecta principalmente a las articulaciones metacarpofalángicas.
3. 3. En la radiografías se aprecia osteoporosis.
4. 4. La viscosidad del líquido sinovial es elevada.
Gráfico de respuestas
Comentario

El líquido sinovial NO es inflamatorio, es un líquido de viscosidad normal (esto es, viscosidad no


disminuída), con recuento de células, glucosa y proteínas dentro de la normalidad. Tampoco los
exámenes de laboratorio ayudan al diagnóstico, pues los reactantes de fase aguda, BH y
bioquímica son completamente normales. Así, pues, la clínica es la base del diagnóstico, y los
cambios radiológicos lo más importante a conocer. Las articulaciones más afectadas son la cadera,
la rodilla, la columna y las articulaciones interfalángicas de manos y piés (recuerde que la
afectación de MCF es propia de la artritis reumatoide). En la radiografía habrá pinzamiento del

!
!
!

espacio articular, esclerosis subcondral, osteofitos, quistes yuxtaarticulares y deformidad


articular.(R/4)

267. Ante un paciente con una faringitis gonocócica, ¿cuál de los siguientes tratamientos
NO es eficaz?:

1. 1. Penicilina.
2. 2. Ciprofloxacina.
3. 3. Ceftriaxona.
4. 4. Espectinomicina.
Gráfico de respuestas
Comentario

Pregunta de dificultad intermedia sobre antibioterapia. El tratamiento de elección del gonococo son
las cefalosporinas de tercera generación, como la ceftriaxona (opción 3). El ciprofloxacino es una
quinolona y como tales antibióticos de amplio espectro y útiles frente a gramnegativos. La
espectinomicina es un antibiótico que aunque no es un aminoglucósido su espectro de acción es
parecido, y se extiende a varias bacterias grampositivas y gramnegativas, aunque en la práctica
queda limitado a la N. gonorrhoeae. La penicilina en principio es un antibiótico para grampositivos,
además muchas cepas del gonococo son productoras de betalactamasas (R/1).

268. Señale la respuesta FALSA en relación a la exploración del pulso venoso yugular:

Los pacientes con presión venosa muy elevada deben ser explorados en decúbito
1. 1.
supino para que se haga evidente la altura de la columna de presión yugular.
Los movimientos de aurícula y ventrículo derechos crean ciertos cambios en el
2. 2. flujo de retorno que se manifiestan por ondas de volumen en las venas principales
que abocan al corazón.
Las dos ondas positivas (a, v) son visibles y casi coincidentes con los ruidos
3. 3.
cardíacos.
La exploración se debe hacer de las venas yugulares internas y externas de ambos
4. 4.
lados del cuello.
Gráfico de respuestas
Comentario

Pregunta de baja dificultad de semiología cardíaca, sobretodo el pulso venoso yugular. Esta se
mide viendo el latido yugular con el paciente en decúbito y levantado el tórax unos 45º. Se mide la
altura con respecto al ángulo de Louis, que está a unos 5 cm de AD. La causa más frecuente de
elevación de la presión venosa es un aumento de la presió diastólica ventricular. El pulso venoso
yugular consta generalmente de dos ondas positivas ("a" y "v") y dos depresiones negativas "x" e
"y". La onda "a" es un onda prsistólica que se debe a la contracción auricular y se sitúa justo antes
de R1 mientras que la onda "v" tiene lugar durante la contracción ventricular y ocurre durante R2.
(R/1)

269. Señale qué afirmación, de las siguientes, relativa al meconio fetal normal, es
CORRECTA:

1. 1. Contiene escasos estafilococos.


2. 2. Excepcionalmente contiene estreptococos.

!
!
!

3. 3. Puede contener Proteus en casos de prematuridad.


4. 4. No contiene ningún germen.
Gráfico de respuestas
Comentario

Pregunta sencilla sobre el meconio. El meconio es estéril, por lo que la respuesta correcta es la
número 4.

270. En relación con el cuadro de sepsis neonatal de comienzo tardío, señale cuál de
estas afirmaciones NO es correcta:

1. 1. Suele presentarse a partir de la semana de vida.


2. 2. Los agentes etiológicos más frecuentemente implicados suele ser los anaerobios.
3. 3. La clínica es más insidiosa o solapada que la de la sepsis neonatal precoz.
El tratamiento de la meningitis nosocomial en un RN ingresado consiste en la
4. 4.
administración de vancomicina + cefotaxima.
Gráfico de respuestas
Comentario

La sepsis neonatal tardía no nosocomial, que aparece a partir del séptimo día de vida, tiene como
agentes más frecuentes determinados serotipos de SGB y E. coli. Estos serotipos peculiares tienen
como característica esencial la presencia de antígenos de adherencia tisular (por eso las
infecciones focales son más frecuentes). Su presentación no es tan abrupta como en las sepsis
precoces. Para tratar las meningitis nosocomiales ha de usar vancomicina (para cubrir los
estafilococos) y cefotaxima (que cubre bien a los gram negativos), pudiéndose asociar además
anfotericina B liposómica. (R/2)

271. ¿Cuál de las siguientes es CORRECTA a cerca de la anemia perniciosa?:

La médula generalmente muestra hipocelularidad con discreta elevación en el


1. 1.
número de megaloblastos.
2. 2. Es frecuente la presencia anticuerpos anti-celulas parietales.
3. 3. Es característico el aumento de absorción del hierro de los alimentos.
La administración de folato es tratamiento alternativo cuando no hay respuesta al
4. 4.
tratamieneto con vitamina B12.
Gráfico de respuestas
Comentario

La anemia perniciosa, es la causa fundamental de anemia megaloblástica por déficit de B12.


Consiste en una destrucción autoinmune (presencia de anticuerpos ), de las celulas parietales
gástricas productoras de Factor intrínseco, necesario para la absorción de la Vitamina B12, asi
como de acido clorhídrico, necesario para la absorción del hierro de los alimentos. El déficit de
B12, ocasiona un retardo en la división celular, con eritropoyesis ineficaz y aumento de la
celularidad de la medula ósea. La B12 logicamente, no se puede dar VO, si el transtorno consiste
en una alteración de su reabsorción. El folato es aconsejable darlo junto la B12, porque se produce
también un déficit de este, pero nunca como alternativa. (R/2)

272. Son complicaciones de la obesidad en niños:

1. 1. Alteraciones ortopédicas.
2. 2. Pubertad temprana.

!
!
!

3. 3. Desarrollo ulterior de diabetes mellitus tipo 2.


4. 4. Todas las anteriores.
Gráfico de respuestas
Comentario

Respuesta sencilla sobre complicaciones de obesidad. La respuesta correcta es la 4, ya que la


obesidad puede generar alteraciones ortopéidcas, pubertad temprana (por el exceso de grasa, que
incrementa los niveles de andrógenos) y el desarrollo ulterior de DM tipo 2.

273. Es una CONTRAINDICACIÓN absoluta para realizar un fórceps:

1. 1. Dilatación completa.
2. 2. Presentación cefálica.
3. 3. Cabeza en II plano.
4. 4. Sufrimiento fetal.
Gráfico de respuestas
Comentario

El fórceps es un método de extracción fetal que se emplea durante el período expulsivo,


conceptualmente semejante a unas tenazas. Exige, por tanto, dilatación completa y bolsa rota.
Requiere presentación cefálica (salvo el raro caso del fórceps de cabeza última, en el parto de
nalgas complicado) y cabeza normal (no puede usarse en hidrocefalias o anencefalias). Para
poderse emplear con seguridad, el punto guía debe alcanzar el tercer plano de Hodge, signo de
que el diámetro biparietal ha sobrepasado el estrecho superior.

El fórceps es un instrumento rápido, que resuelve malrotaciones fetales (tractor y rotador), lo que le
convierte en instrumento ideal en caso de sufrimiento fetal (siempre que se den las condiciones
que hagan posible su aplicación).(R/3)

274. Niño de 2 años de edad con fiebre de 39ºC de 48 horas de evolución asociado a
cuadro catarral leve. Acude por aparición de vesículas en pilares anteriores faríngeos así
como afectación palmo-plantar. Respecto a la patología que sospecha señale la
afirmación CORRECTA:

1. 1. Se caracteriza por presentar un exantema muy pruriginoso.


2. 2. Es causada por el virus Coxsackie B.
Se debe administrar gammaglobulina a neonatos expuestos en las primeras 72
3. 3.
horas.
4. 4. Puede ser causado por Enterovirus.
Comentario

El paciente padece la enfermedad mano-pie-boca que se caracteriza por aparición de vesículas en


mucosa oral, paladar y pilares anteriores faringeos y vesículas en manos y pies principalmente en
el dorso pero también en palmas y plantas.

Puede aparecer un exantema maculopapulosos en cara, brazos y nalgas.

Es causado por el Coxsackie A16 y enterovirus 71. Respuesta correcta 4.

!
!
!

Es importante no confundir esta enfermedad con varicela, en esta última se encontrarán lesiones
en distitnos estadíos y no habría afección palmo-plantar.

275. Respecto a las complicaciones que pueden producirse en esta patología señale la
respuesta INCORRECTA:

1. 1. Meningitis.
2. 2. Encefalitis.
3. 3. Convulsiones febriles.
4. 4. Neumonitis.
Gráfico de respuestas
Comentario

El enterovirus 71 puede causar meningitis y encefalitis, minetras que el Coxsackie A16 puede
causar miocarditis. La enfermedad exantemática que más se ha relacionado con las convulsiones
febriles es el exantema súbito.(R/3)

276. Señale cuáles son los elementos del triángulo de Hesselbach.

1. 1. Vasos epigástricos, ligamento de Cooper, borde lateral del recto abdominal.


Ligamento inguinal, vasos epigástricos y borde lateral del recto anterior
2. 2.
abdominal.
3. 3. Ligamento inguinal, vasos epigástricos, borde interno del recto abdominal..
4. 4. Vasos epigástricos, ligamento de Cooper, borde interno del recto abdominal..
Gráfico de respuestas
Comentario

Pregunta de cirugía en el internado, de anatomía de las hernias inguinales. El triángulo de


Hesselbach está compuesto por el ligamento inguinal, los vasoso epigástricos y el borde lateral del
recto abdominal. Respuesta 2 correcta.

!
!
!

277. Se recibe en consulta a un niño de 5 meses de edad, cuyo signo mayor es la pobre
ganancia ponderal. Peso al nacer: 3,700 gr. Peso actual: 4,500 gr. La madre refiere que
lacta de manera entrecortada con sudoración excesiva y se muestra irritable. Exámen
físico: taquipnea. ¿Cuàl es el diagnóstico más probable?

1. 1. Errores innatos del metabolismo.


2. 2. Mala técnica de lactancia.
3. 3. Infección intrauterina: TORCHS.
4. 4. Insuficiencia cardíaca congestiva.
Gráfico de respuestas
Comentario

La opción más probable, dado que sólo nos indican que existe una pobre ganancia ponderal
(recordamos que a los 5 meses el niño debería de haber duplicado su peso al nacer: 7,400 gr
aproximadamente, pesando en la actualidad 4,500 gr) y ningún otro signo clínico o síntoma que
nos haga sospechar la opción 1, 3 ó 4, sería la mala técnica de lactancia por la cual el niño se
alimenta de forma entrecortada y esto le genera una importante sudoración e irritabilidad. Sería
necesario instruir a la madre en una técnica adecuada para proporcionar el alimento a su hijo.(R/2)

278. En el tratamiento de la diabetes mellitus tipo 1 en niños, se tendrá presente lo


siguiente, señale lo CORRECTO:

1. 1. La insulina de acción intermedia es de elección para la cetoacidosis.


2. 2. La glicemia debe mantenerse en valores menores de 140mg/dl.
3. 3. La administración de bicarbonato es obligatoria para corregir la cetoacidosis.
4. 4. El horario de las comidas debe guardar relación con el esquema insulínico.
Gráfico de respuestas
Comentario

La insulina que se utiliza para la corrección de la cetoacidosis es la regular via iv. El bicarbonato
sólo se debe administrar cuando el pH es menor de 6.9, sinó, la acidosis se corregirá con la
insulina e hidratación administrada. El horario de las comidas se debe coordinar con las comida,
para hacer más fisiológica su curva y para que no haya hipoglicemias. (R/4)

279. Un hidradenoma está asociado con:

1. 1. Células nerviosas.
2. 2. La vejiga.
3. 3. Hernias.
4. 4. Glándulas sudoríparas.
Gráfico de respuestas
Comentario

!
!
!

Pregunta de dificultad elevada. El hidradenoma papilar es una neoplasia benigna. Se trata de un


nódulo bien delimitado localizado sobre todo en los labios mayores. Puede ulcerarse. Surge a partir
de las glándulas sudoríparas apocrinas de la vulva. Se han descrito asociaciones con hernias.
Respuesta 3 correcta.

280. Recién nacido de 20 horas de vida, edad gestacional 38 + 1 semana, peso al


nacimiento 2,800 gramos y Apgar 9/9. Madre de 35 años (G3 A2 P1), hipotiroidea en
tratamiento sustitutivo. El parto ha sido vaginal y eutócico, como antecedentes destaca
bolsa rota 5 horas antes del parto. Presenta ictericia conjuntival y cutánea hasta el
ombligo. No presenta cefalohematoma, ni otros signos de hemorragia. La palpación de
las clavículas es normal, y el cráneo no crepita. Ha recibido 3 tomas al pecho materno. El
grupo sanguíneo materno es O- y el del recién nacido es B+. Es FALSO que:

Un porcentaje de casos requerirá tratamiento con fototerapia y algunos con


1. 1.
exanguinotransfusión según las cifras de bilirrubina.
El diagnóstico diferencial habría que realizarlo principalmente entre cualquier
2. 2.
forma de isoinmunización e hipotiroidismo congénito.
3. 3. La isoinmunización anti-D no puede ser descartada en este caso.
La ruptura de membranas 5 horas preparto, no parece un factor de riesgo
4. 4. infeccioso relevante, a la hora de considerar sepsis como la causa de la ictericia
en este caso.
Gráfico de respuestas
Comentario

En los casos clínicos de ictericia neonatal es importante identificar posibles factores de riesgo:

- Grupo sanguíneo materno (ABO y Rh). Posible isoinmunización anti A/B si madre O y recién
nacido A, B o AB. Puede darse en el primer embarazo. Posible isoinmunización anti- Rh si madre
Rh – y recién nacido Rh + (90% son anti- D, 10% anti C o E).

- Considerar datos de hemorragia/hematomas/fracturas que al reabsorberse elevan la bilirrubina.

- Detectar factores de riesgo de infección bacteriana (todos los de sepsis neonatal: corioamnionitis,
fiebre materna, cultivo recto- vaginal positivo, IVU en el tercer trimestre, prematuridad… Para la
ruptura de membranas se considera >18 horas en RNT.

- Síndrome de Arias. Causado por lactancia materna. Es característico que al suspender la


lactancia materna, disminuya rápidamente el nivel de bilirrubina, pero habrá que recordar que NO
contraindica su administración.

- El hipotiroidismo congénito es causa de hiperbilirrubinemia tardía.

- La ictericia nunca será fisiológica en las primeras 24 horas de vida. (R/2)

281. En una sospecha de hematoma retroperitoneal la prueba diagnóstica de elección es:

1. 1. Ecografía.
2. 2. TAC.
3. 3. Arteriografía.
4. 4. Lavado peritoneal diagnóstico.
Gráfico de respuestas

!
!
!

Comentario

Es una pregunta muy directa. La causa más frecuente de hematoma retroperitoneal son las
fracturas pélvicas. Por ello todo politraumatizado debe someterse a una radiografía de pelvis. Debe
sospecharse en todo traumatismo con choque hipovolémico sin localización evidente de la
hemorragia. Con frecuencia no se acompaña de signos de irritación peritoneal. La prueba de
elección para el diagnóstico es el TC. En cuanto al tratamiento siempre debe realizarse exploración
quirúrgica en el caso de un trauma penetrante, siendo en el trauma cerrado variable según la
localización de las lesiones: si el hematoma es central- superior es siempre quirúrgico (suele
lesionarse la VCI), si es en flanco suelen lesionarse los riñones cuyo manejo suele ser
conservador, y el hematoma en cavidad pélvica suele presentar lesiones de arterias ilíacas
internas y no deben explorarse por riesgo de sangrado masivo. (R/2)

282. Le avisan de de un registro cardiotocográfico patológico, en una gestante de 42


semanas que está de parto. Al romper la bolsa, detecta un líquido amniótico intensamente
teñido de meconio. El RN nace con un Apgar al minuto y a los cinco minutos de 4 y 7,
precisando aspiración de secreciones y ventilación con mascarilla con presión positiva
intermitente. A las dos horas de vida el niño presenta dificultad respiratoria con quejido
espiratorio audible sin estetoscopio, disociación tóraco-abdominal ligera, retracción
xifoidea y tiraje intercostal marcados, sin aleteo nasal. En la gasometría muestra
hipoxemia con hipercapnia. ¿Cuál de las siguientes es CORRECTA?

1. 1. El paciente tiene un score de Silverman de 5.


En la radiografía de tórax es frecuente hallar derrame en las cisuras y
2. 2.
broncograma aéreo.
Otro hallazgo radiológico típico es la hipoventilación, con menos de 7 espacios
3. 3.
intercostales en la radiografía AP de tórax.
4. 4. Algunos pacientes se benefician del tratamiento con surfactante intratraqueal.
Gráfico de respuestas
Comentario
El cuadro descrito corresponde a un síndrome de aspiración meconial. Es característico de RN a
término o postérmino, con datos de sufrimiento fetal agudo, Apgar bajo al nacer y líquido amniótico
teñido de meconio (el RN hijo de madre diabética es propenso a padecer enfermedad de
membrana hialina). La clínica consiste en dificultad respiratoria severa (el Silverman del
paciente es de 7), con abombamiento torácico por atrapamiento aéreo (y, por tanto, con signos
radiológicos de hiperinsuflación), hipoxemia e hipercapnia. En la radiografía es frecuente
encontrar hiperinsuflación, infiltrados algodonosos, atelectasias y aire extraalveolar (el líquido en
cisuras es un dato sugestivo de taquipnea transitoria del RN y el broncograma aéreo de
enfermedad de membrana hialina). El tratamiento consiste en asistencia respiratoria,
antibioterapia y, en ocasiones, surfactante para mejorar la mecánica pulmonar. (R/4)

283. ¿Qué fármaco elegiría para tratar una neumonía en un niño de 5 años de edad, que
se acompaña de cefalea, malestar general, tos llamativa, radiografía con patrón
intersticial y que en los análisis de sangre presenta trombocitopenia y crioaglutininas?:

1. 1. Eritromicina.
2. 2. Penicilina G sódica.
3. 3. Amoxicilina.
4. 4. Trimetroprim-Sulfametoxazol.
Gráfico de respuestas
Comentario
!
!
!

Pregunta concreta que necesita de buen olfato clínico. Estamos ante una neumonía por
Mycoplasma pneumonae siendo el tratamiento de elección en este caso la eritromicina. (R/1)

284. ¿Cuál de los siguientes antiarrítmicos es más nocivo para la glándula tiroides?:

1. 1. Verapamil.
2. 2. Amiodarona.
3. 3. Propranolol.
4. 4. Lidocaína.
Gráfico de respuestas
Comentario

Esta pregunta puede resultarnos difícil si consideramos que es prácticamente imposible recordar
todos los efectos secundarios de cada fármaco, sin embargo hay que prestar especial atención y
conocer ciertos efectos adversos característicos de determinados fármacos como en el caso de los
antiarrítmicos. La amiodarona, es un antiarrítmico del grupo III muy eficaz en la mayoría de las
arritmias tanto supraventriculares como ventriculares malignas, siendo además el fármaco de
elección en el contexto de la reanimación cardiopulmonar y en la prevención de los episodios de
fibrilación auricular. Pero hay que tener en cuenta la gran cantidad de efectos secundarios, algunos
de ellos graves, que puede provocar, como son fibrosis pulmonar, alteraciones de la función
tiroidea (respuesta 3 correcta), depósitos corneales, hepatopatía, disminución del crecimiento,
alteraciones cutáneas, etc. Además la amiodarona también aumenta los niveles de digoxina
pudiendo desencadenar una intoxicación digitálica.(R/2)

285. Nos encontramos ante un hombre de 50 años que presenta febrícula, síndrome
constitucional y epistaxis de repetición con dolor en el tabique nasal. Se practica una
radiografía de tórax que muestra la presencia de infiltrados pulmonares cavitados
bilaterales. Los anticuerpos anti-citoplasma de neutrófilo resultan positivos. Señale cuál
de las siguientes afirmaciones en relación a la patología que sospecha NO es cierta:

La afectación renal aparece hasta en ¾ partes de los pacientes a lo largo de la


1. 1.
enfermedad.
Los títulos de c-ANCA se correlacionan positivamente con la actividad de la
2. 2.
enfermedad.
El tratamiento de elección se basa en ciclofosfamida, a la que se añaden
3. 3.
temporalmente corticoides a dosis altas.
La afectación más precoz y frecuente se produce a nivel de la vía respiratoria
4. 4.
superior.
Gráfico de respuestas
Comentario

Se trata de una pregunta asequible. Recuerde las características de la granulomatosis de


Wegener: vasculitis de mediano y pequeño calibre.

Tríada clínica: afectación de la vía respiratoria superior (lo más frecuente y precoz), infiltrados
pulmonares bilaterales, cavitados y no migratorios, glomerulonefritis (habitualemente no presente al
diagnóstico pero aparece en el 75% en la evolución de la enfermedad).

A pesar de ser altamente sensibles (88%) y específicos (95%), los niveles de c-ANCA no se
correlacionana con la actividad de la enfermedad (opción 2 falsa).

!
!
!

El tratamiento se basa en ciclofosfamida 2mg/kg/dia acompañada inicialmente de corticoides a


dosis altas (1mg/kg/dia) con descenso durante los primeros meses.(R/2)

286. Mujer de 50 años en cuya mastografía anual se visualizan en el cuadrante infero


ínterno de la mama izquierda un grupo de 7 microcalcificaciones de tamaño simétrico
agrupadas anárquicamente. En la exploración clínica no se detectan nódulos palpables.
¿Cuál sería su conducta a seguir?

1. 1. Repetir la mastografía en 6 meses para ver si el hallazgo sigue estable.


2. 2. Realizaría una biopsia con aguja gruesa en consulta.
3. 3. Realizaría una biopsia dirigida con arpón.
Tranquilizaría a la paciente, porque ese tipo de microcalcificaciones representan
4. 4.
un hallazgo muy común propio del envejecimiento del tejido mamario.
Gráfico de respuestas
Comentario
La aparición de microcalcificaciones agrupadas anárquicamente en número igual o superior a 5 de
tamaño simétrico constituye el signo sospechoso de malignidad que aparece más precozmente en
la mamografía. Si no se corresponden a una lesión palpable no podremos realizar BAG o PAAF en
consulta por lo que habrá que recurrir al marcaje de la lesión con un arpón y a su biopsia
diferida.(R/3)

287. En un paciente portador de una prótesis valvular aórtica que va a realizarse una
endodoncia y es alérgico a penicilina, estarían indicados los siguientes fármacos como
profilaxis de endocarditis, EXCEPTO:

1. 1. Vancomicina.
2. 2. Gentamicina.
3. 3. Teicoplanina.
4. 4. Azitromicina.
Gráfico de respuestas
Comentario

Es importante tener unas nociones básicas de la cobertura que tiene cada familia de antibióticos,
con ello se puede responder a unas cuantas preguntas, entre ellas ésta.

En los procedimientos dentales, la profilaxis buscará cubrir una posible bacteriemia por S. viridans,
microorganismo más frecuentemente implicado. De los antibióticos que hay entre las respuestas la
gentamicina es el único que sólo cubre gram negativos, por lo que sería la respuesta falsa.

Vancomicina y teicoplanina cubren grampositivos, incluso el S. epidermidis o el S. aureus oxacilin


resistente. La azitromicina es un macrólido que puede cubrir también la legionella, la Bordetella
pertussis (tos ferina) y neumonía atípica.(R/2)

288. ¿Cuál de las siguientes afirmaciones es FALSA?:

La "artritis colítica" periférica sigue una evolución paralela a los brotes de la


1. 1.
enfermedad intestinal.
Los pacientes con enfermedad intestinal inflamatoria crónica pueden presentar
2. 2.
artritis axial indistinguible de la espondiloartritis idiopática.

!
!
!

Un 40% de los casos de espondilitis asociada a enfermedad intestinal son


3. 3.
mujeres.
Los brotes de la espondilitis asociada a la enfermedad intestinal inflamatoria
4. 4.
crónica corren paralelos a los de dicha enfermedad.
Gráfico de respuestas
Comentario

La artritis de articulaciones periféricas asociada a la enfermedad inflamatoria intestinal tiene


característucamente un curso paralelo a los brotes intestinales. Es más probable que se produzaca
en los pacientes que tienen complicaciones como abcesos, poliposis pseudomembranosa, lesiones
perianales, eritema nodoso, estomatitis, uveitis o pioderma gangrenoso. No muestra predominio
sexual. La afectación del esqueleto axial tiene un curso independiente de los síntomas
gastrointestinales y predomina en hombres.(R/4)

289. La mayor frecuencia de muerte temprana en pacientes con lesión pancreática se


debe a:

1. 1. Absceso pancreático.
2. 2. Fistula pancreática.
3. 3. Fistula duodenal.
4. 4. Lesión vascular asociada.
Gráfico de respuestas
Comentario

Al estar el páncreas situado en el retroperitoneo, es muy frecuente que una lesión pancreática se
asocie a lesión de los grandes vasos (aorta o vena cava) que también se situan en este espacio
anatómico. Respuesta 4 correcta.

290. Mujer de 34 años, que acude al


servicio de urgencias con dolor en cuello de 3 días de evolución, febrícula y nerviosismo,
diagnosticada de amigdalitis aguda por médico de atención primaria y tratada con
ibuprofeno y amoxicilina-clavulánico. El dolor persiste y aparecen palpitaciones e
insomnio, motivo por el que acude a urgencias alarmada por la mala evolución del
cuadro. En los exámenes de laboratorios realizado se detecta una VSG de 85. La paciente

!
!
!

fue dada de alta con el mismo tratamiento y remitida a consultas de endocrinología, a la


que acude con el resultado de la gammagrafía tiroidea solicitada (ver imagen). ¿Cuál es
el diagnóstico más probable y el tratamiento que Ud. pautaría?

Ademona folicular hiperfuncionante - metimazol y betabloqueantes, previo a


1. 1.
radioyodo.
Tiroditis subaguda de De Quervain - betabloqueantes y corticoides dada la poca
2. 2.
respuesta a AINEs.
3. 3. Struma ovari - antitiroideos y cirugía.
4. 4. Bocio multinodular tóxico - antitiroideos y radioyodo.
Gráfico de respuestas
Comentario

La paciente padece una tiroiditis subaguda, vírica o de De Quervain. Posiblemente, el diagnóstico


de amigdalitis aguda que emitió el médico de atención primaria fuese correcto, pero de origen
vírico, afectando al tiroides secundariamente. Es lo que sucede habitualmente en esta entidad:
infección vírica unos días antes, normalmente de vías altas, afectándose después el tiroides.

Desde el punto de vista clínico, destacan los síntomas de hipertiroidismo junto con dolor en región
anterior del cuello. Encontraríamos una TSH suprimida y la VSG habitualmente elevada. En
cuando a la gammagrafía acompañante, como sucede en la práctica totalidad de las tiroiditis,
vemos abolida la captación de la glándula.(R/2)

291. When a patient has a parathyroid hyperplasia, pancreatic tumors and pituitary
tumors, which is the diagnosis?

1. 1. Cowden's syndrome.
2. 2. MEN type IIA.
3. 3. Von Recklinghousen disease.
4. 4. MEN type I.
Gráfico de respuestas
Comentario
MEN type I. Wermer's syndrome or MEN type I is a disorder defined by parathyroid hyperplasia,
pancreatic tumors and pituitary tumors. Hypercalcemia is not a usual finding in a Zollinger-Ellison
syndrome and when it appears it suggest a multiple endocrin disease and a worse prognosis.(R/4)

292. A 13-year-old child is brought to the pediatrician's office for a routine check-up. He
underwent an abdominal intervention one year ago. Findings in peripheral blood smear
are described as: "occasional red blood cells with bluish nuclear remnants" are often
seen in one of the following processes:

1. 1. Splenectomy.
2. 2. Hemoglobin precipitation.
3. 3. Pure red blood cells aplasia.
4. 4. Parvovirus infection.
Gráfico de respuestas
Comentario
Splenectomy. The red cells described in this blood smear have Howell–Jolly bodies inclusions.
These are histopathological findings of basophilic nuclear remnants (clusters of DNA) in circulating
erythrocytes and are seen with markedly decreased splenic function.(R/1)

!
!
!

293. Respecto a las siguientes afirmaciones sobre la infección en el paciente


neutropénico, señale la INCORRECTA:

El tratamiento empírico inicial con una única cefalosporina antipseudomonas


1. 1.
puede ser adecuado.
2. 2. El ectima gangrenoso está causado de forma exclusiva por Pseudomonas.
3. 3. El síndrome de Sweet puede ser un cuadro paraneoplásico.
4. 4. La tiflitis cursa clínicamente como una apendicitis.
Gráfico de respuestas
Comentario

Aunque el ectima gangrenoso es una infección típicamente asociada a P. aeruginosa en el


paciente neutropénico, también se ha descrito en el contexto de otros gramnegativos (respuesta 2
falsa).

294. Siendo la sospecha clínica la de apendicitis aguda, ¿cuál sería su actitud


terapéutica?:

1. 1. Sueroterapia y analgésicos.
2. 2. Intervención quirúrgica urgente: apendicectomía.
3. 3. Observación domiciliaria.
4. 4. Laparotomía exploradora.
Gráfico de respuestas
Comentario

Lo más probable es que se pregunte la apendicitis en forma de caso clínico, no olvide revisar los
criterios de Alvarado. El tratamiento de una apendicitis aguda siempre es quirúrgico
(apendicectomía) puesto que dejada a su evolución natural puede desembocar en una perforación
o una peritonitis difusa. El tratamiento se completa con antibioticoterapia profiláctica. Sólo en el
caso de que el diagnóstico se haga en la fase de plastrón apendicular se puede diferir la cirugía
tras un par de semanas de tratamiento antibiótico intravenoso.(R/2)

295. Una mujer joven acude a nuestra consulta refiriendo astenia, pérdida de peso e
hiperpigmentación progresiva. El estudio funcional de la paciente muestra un cortisol
plasmático en límites bajos y ausencia de respuesta del cortisol sérico al estímulo de
ACTH. ¿Cuál será probablemente la causa de su insuficiencia suprarrenal?:

1. 1. Tuberculosis.
2. 2. SIDA.
3. 3. Amiloidosis.
4. 4. Adrenalitis autoinmune.
Gráfico de respuestas
Comentario
Aunque esta pregunta se formule en forma de caso clínico, hace referencia a la etiología más
frecuente de una patología que puede deducirse por los datos aportados. Ante una paciente joven,
que presente astenia intensa, pérdida de peso, y en especial, hiperpigmentación lo primero que
hay que descartar es la existencia de una insuficiencia suprarrenal primaria (Enfermedad de
Addison). Dentro de las causas de esta enfermedad, la más frecuente es la adrenalitis autoinmune,

!
!
!

siendo la segunda en frecuencia la tuberculosis. Otras causas menos frecuentes serían la


existencia de infecciones oportunistas en pacientes con SIDA, las metástasis y patologías como la
amiloidosis.(R/4)

296. En gerontopsiquiatría, la melancolía involutiva se caracteriza por todos, MENOS uno


de los siguientes aspectos:

1. 1. Tendencia a la autodepreciación y autoacusación.


2. 2. Ideas de culpabilidad.
3. 3. Trastornos confusionales y oníricos.
4. 4. Disminución de la autoestima.
Gráfico de respuestas
Comentario
La "melancolía involutiva" ha desaparecido de las clasificaciones porque no es más que una
depresión endógena severa ("melancolía") de la ancianidad. Ahora bien, tiene sus peculiaridades
debido al momento del ciclo vital y a los factores orgánicos: predominan las preocupaciones
hipocondríacas (que a veces alcanzan un rango delirante de severidad, como en el Síndrome de
Cottard), y se asocia más que en la población general con degeneración cerebral vascular (no es
infrecuente que años después de una melancolía involutiva aparezca una demencia vascular). Lo
que claramente excluye el diagnóstico de demencia y de depresión, al menos provisionalmente
hasta que se resuelva, es la clínica confusional- oniroide (es decir, la disminución del nivel de
conciencia tipo delirium).(R/3)

297. Niño de 5 años de edad que, estando previamente bien, despierta por la noche con
fiebre de 39.8ºC, voz apagada, intenso babeo, estridor inspiratorio con escasa tos. A la
exploración destaca mal estado general, estridor inspiratorio e intenso tiraje
supraesternal y subcostal, así como tendencia a la hiperextensión del cuello. Se realiza
radiografía lateral de faringe, observando ocupación de espacio supraglótico. En la
biometría hemática destaca una leucocitosis con desviación izquierda. ¿Qué medida
terapéutica NO estaría aconsejada en este caso?:

1. 1. Aplicar oxígeno e intentar obtener una vía aérea artificial en quirófano.


2. 2. Aplicar adrenalina racémica y corticoides.
3. 3. Usar las medidas generales para descender la hipertermia que sufre el paciente.
4. 4. Mantener tranquilo al paciente.
Gráfico de respuestas
Comentario

Pregunta importatísima para el ENARM, es un cuadro de epiglotitis de libro.

Estamos ante un cuadro de epiglotitis aguda de tal forma que la 1, 3 y 4 serían opciones
terapeúticas a utilizar en este cuadro. Sin emabrgo, la número 2 se utiliza en el tratamiento de las
laringitis agudas (estridulosas) o laringotraqueitis viral.

Recuerde que por nada del mundo debes interar explorar la vía aerea porque sería peor, debe
asegurarla.(R/2)

298. ¿Cuál de las siguientes NO puede considerarse una reacción de hipersensibilidad


retardada?:

!
!
!

1. 1. Neumonitis en el cuidador de aves.


2. 2. Histoplasmosis.
3. 3. Beriliosis.
4. 4. Linfogranuloma venéreo.
Gráfico de respuestas
Comentario

Las reacciones de hipersensibilidad es un tema importante dentro de la inmunología. Las


reacciones de hipersensibilidad (RHS) se clasifican en 4 tipos, cada uno con sus características y
lo que es realmente difícil es clasificar las distintas patologías inmunológicas con el grupo al que
pertenece. La reacción tipo IV o retardada es mediada por células y necesita una sensibilización
previa. El ejemplo típico es el Mantoux (PPD), pero encontramos otros como el rechazo agudo de
trasplantes, los granulomas y la hipersensibilidad por contacto. La neumonitis del cuidador de aves
(neumonitis por hipersensibilidad) se produce por la mezcla de RHS tipo III y IV, lo mismo que
ocurre en la histoplasmosis y la beriliosis, por lo que muy frecuentemente encontramos granulomas
en la anatomía patológica. La Aspergilosis broncopulmonar alérgica es un trastorno complejo en el
que se han visto mezcladas varias RHS: tipo I y III fundamentalmente, aunque también tipo IV. El
linfogranuloma venéreo es una ETS provocada por Clamydia trachomatis serotipos L1, L2 y L3, por
ello no influye ninguna RHS.(R/4)

299. Todas las formas de presentación de la patología pancreática, son frecuentes,


EXCEPTO:

1. 1. Encefalopatía pancreática.
2. 2. Dolor de espalda.
3. 3. Síndrome tóxico-constitucional.
4. 4. Síntomas neuropsiquiátricos.
Gráfico de respuestas
Comentario

La encefalopatía pancreática en una entidad que se asocia a las pancreatitis agudas y que aparece
con muy baja frecuencia. Se puede apreciar clínica compatible con encefalopatía en el 4 a 20% de
todos los pacientes con pancreatitis aguda. Los síntomas pueden incluir desorientación, confusión,
delirio, y alucinaciones.

Debido a que el alcohol y el síndrome de deprivación también producen estos síntomas a veces es
imposible discernir cual es la causa subyacente. El edema cerebral, las hemorragias y la necrosis
focal, presumiblemente secundarias a alteraciones del flujo sanguíneo microvascular han sido
relacionadas con episodios de pancreatitis aguda.(R/1)

300. La oxigenoterapia crónica domiciliaria está indicada en el tratamiento de la EPOC


cuando:

El paciente cumple el tratamiento broncodilatador completo y la PaO2 es menor


1. 1.
de 65 mmHg.
El paciente cumple el tratamiento broncodilatador completo y la PaO2 es inferior
2. 2.
a 55 mmHg en las agudizaciones.
3. 3. La disnea es de mínimos esfuerzos.
El paciente cumple el tratamiento broncodilatador completo y la PaO2 es inferior
4. 4.
a 55 mmHg, estando el paciente en situación basal.
Gráfico de respuestas
Comentario
!
!
!

Pregunta sencilla que no debe fallar en ningún momento. Es importante saber que las únicas dos
medidas que mejoran la supervivencia en el paciente EPOC son el dejar de fumar y la
oxigenoterapia domiciliaria si está indicada. Esta indicación es en pacientes que no fumen y que en
tratamiento broncodilatador completo su pO2 basal es menor que 55mmHg y si está entre 55- 60
en caso de presentar alguna complicación como hipertensión pulmonar, cor pulmonale, ICC,
arritmias, policitemia o reducción del intelecto. Recuerde que el uso es de16h/día al menos y no se
realiza descanso nocturno.(R/4)

301. Un paciente de 27 años es incapaz de impedir el desplazamiento anterior del fémur


sobre la tibia cuando la rodilla está flexionada. ¿Cuál de los siguientes ligamentos está
dañado?:

1. 1. Cruzado anterior.
2. 2. Colateral peroneo.
3. 3. Rotuliano.
4. 4. Cruzado posterior.
Gráfico de respuestas
Comentario
La maniobra de exploración que nos describen es la “prueba del cajón posterior”. El
desplazamiento hacia anterior del fémur sobre la tibia (o al revés), con la rodilla flexionada a unos
90º, nos indica la posible rotura del ligamento cruzado posterior. Este ligamento suele lesionarse
en traumatismos directos sobre la tibia con la rodilla en flexión. Además, en casos crónicos puede
observarse una deformidad en recurvatum – la rodilla se hiperextiende al no existir el tope del
ligamento cruzado posterior. La lesión aislada del ligamento cruzado posterior puede ser
subsidiaria de tratamiento conservador (potenciar cuádriceps), pero si se combina con la lesión de
una o más estructuras laterales (inestabilidad posterolateral) es necesaria la reconstrucción
quirúrgica.(R/4)

302. Respecto a la prueba de la tuberculina, todas las afirmaciones siguientes son


correctas, EXCEPTO:

La lectura de la reacción se debe hacer midiendo el diámetro de eritema medido a


1. 1.
las 48-72 horas.
La reacción positiva del PPD sólo indica que probablemente hay alguna infección
2. 2.
micobacteriana.
La repetición de las cutirreacciones puede hacer que aumenten de tamaño (efecto
3. 3. booster), aunque no pueden producir una reacción positiva en un sujeto no
infectado.
15% de los sujetos con tuberculosis pulmonar activa reciente y 50% de los
4. 4.
enfermos con tuberculosis miliar presentan un Mantoux negativo.
Gráfico de respuestas
Comentario

Sobre la técnica de realización del Mantoux,

- Lo que se mide es la INDURACIÓN, y no el eritema.

- El inóculo es a nivel INTRADÉRMICO, no subcutáneo.

!
!
!

Y, efectivamente, lo que significa un Mantoux positivo es que existe INFECCIÓN, y no


ENFERMEDAD tuberculosa.(R/1)

303. María es una mujer de 29 años de edad, embarazada de 28 semanas, que debuta con
un cuadro clínico de dolores musculares generalizados, con escalofríos y sensación
distérmica, sin foco aparente. Tras 10 días de evolución, se añade al cuadro clínico un
dolor a nivel de la faringe, con cuadro febril de hasta 38.8ºC, cefalea y adenopatías
cervicales. Se diagnostica de un cuadro mononucleósico, con germen identificado como
citomegalovirus. Respecto a la infección congénita por CMV, son todas correctas,
EXCEPTO:

1. 1. La mayoría de los infectados intraútero están asintomáticos al nacimiento.


La microcefalia, calcificaciones periventriculares y la coriorretinitis son los
2. 2.
hallazgos más característicos de la forma sintomática.
La IgG positiva anti-CMV en el suero del recién nacido es diagnóstica del
3. 3.
proceso.
4. 4. Es causa frecuente de sordera y retraso mental.
Gráfico de respuestas
Comentario

De todas las infecciones connatales, la debida al citomegalovirus es la más frecuente. Se da


especialmente cuando la madre padece la primoinfección o cuando está coinfectada por CMV y
por VIH.

- Si la madre adquiere la infección durante la primera mitad del embarazo, su hijo puede presentar
la tríada sintomática (situación poco frecuente): microcefalia, calcificaciones periventriculares y
corioamnionitis.

- Si la adquiere en la segunda mitad (más frecuente), estará asintomático al nacer y después


desarrollará hipoacusia neurosensorial.

Para diagnosticarlo, puede tomar un cultivo de orina en la primera semana de vida o determinar la
IgM antiCMV en el recién nacido. Lógicamente, la IgG carece de valor, puesto que esta
inmunoglobulina atraviesa la placenta y, por ello, es posible encontrarla en sangre del neonato sin
que esté infectado.(R/3)

304. Mujer de 46 años que en una rectosigmoidoscopia, prescrita por estreñimiento y


dolor abdominal de reciente aparición, se detecta una masa ulcerada en cara anterior de
sigma. Dos años antes había sido intervenida de un carcinoma de endometrio y en su
familia existían antecedentes de varios familiares fallecidos por cáncer intestinal. A la
exploración física se aprecian lesiones cutáneas en brazos y tronco sugestivas de
tumores sebáceos. El diagnóstico más probable en este caso sería el de síndrome de:

1. 1. Peutz-Jeghers.
2. 2. Cowden.
3. 3. Muir-Torre.
4. 4. Turcot.
Gráfico de respuestas
Comentario

!
!
!

Una pregunta muy peligrosa. La historia familiar de cáncer de colon, teniendo también en cuenta el
cáncer de endometrio, es muy sugestiva de síndrome de Lynch tipo II… Pero, si nos dan algún
dato suele ser por algo. Si añade a esto los tumores sebáceos, debe responder inmediatamente el
síndrome de Muir-Torre. De hecho, este síndrome está muy relacionado con el Lynch II, incluso
desde el punto de vista genético, por lo que los hallazgos descritos también podrían encontrarse en
él. Sin embargo, la presencia de tumores sebáceos sólo pueden encontrarse en la respuesta 3.

305. ¿El limite recomendado de 15 mmHg para el neumoperitoneo, tiene relación con que
parámetro?

1. 1. Distensibilidad de la pared abdominal.


2. 2. Compresibilidad de la pared arterial.
3. 3. Presión arterial media.
4. 4. Presión venosa.
Gráfico de respuestas
Comentario

El límite de 15mmHg de presión de aire en u pneumoperitoneo es el limite a partir del cual se


impediria el retorno venoso. A partir de 15 mmHg se colapsarian las venas abdominales y no
podria retornar la sangre al corazón.(R/4)

306. La enfermedad de Köhler es una osteocondrosis que afecta, ¿qué hueso?

1. 1. Tuberosidad anterior de la tibia.


2. 2. Polo inferior de la rótula.
3. 3. Tuberosidad posterior del calcáneo.
4. 4. Escafoides tarsiano.
Gráfico de respuestas
Comentario

Las osteocondrosis son enfermedades poco importantes en el ENARM. Sin embargo, si tiene
tiempo, no estaría de más repasar los nombres y la localización típica de cada una (sería una
pregunta difícil y valiosa, porque no la acertaría más de un 10-15%). Las distintas osteocondrosis
se fueron describiendo con el nacimiento de la radiología, por ello cada una lleva un epónimo
distinto. La osteocondrosis de Köhler se localiza en el escafoides tarsiano. Tenga cuidado, porque
esta de Köhler (I) se puede confundir con la de Köhler II o enfermedad de Freiberg, que se localiza
en la cabeza de los metatarsianos.(R/4)

307. ¿Cuál es la conducta a adoptar en un recién nacido cuya madre se le diagnosticó


varicela durante el parto?

1. 1. Aplicar la vacuna de la varicela dentro de las 24 horas del nacimiento.


Aplicar la gammaglobulina específica hiperinmune dentro de las 72 horas del
2. 2.
nacimiento.
3. 3. Indicar aciclovir a partir de los 7 días del nacimiento.
4. 4. Conducta expectante, medidas de aislamiento de contactos hasta el alta.
Gráfico de respuestas
Comentario

La profilaxis postexposición en RN con madres con varicela es la siguiente:

!
!
!

Inmunoglobulina inmune contra varicela-zoster, la cual previene la transmisión de varicela en los


neonatos expuestos, disminuye la sintomatología.

La CDC recomienda la administración de la Ig a los RN con exposición reciente a VVZ en casos


de: neonatos cuyas madres presentan signos y síntomas de varicela al rededor del parto (5 días
antes y 2 días después). Respuesta 2 correcta.

308. Respecto a la hipertrofia benigna de próstata es FALSO que:

1. 1. Es la principal causa de obstrucción del tracto urinario inferior en el hombre.


2. 2. Puede condicionar insuficiencia renal crónica.
3. 3. Puede originar litiasis a nivel vesical.
La intensidad de los síntomas es directamente proporcional al tamaño de la
4. 4.
glándula prostática.
Gráfico de respuestas
Comentario

La hiperplasia benigna de próstata es uno de los temas más preguntados en esta asignatura. Esta
pregunta, en concreto, es tan sencilla que la falsedad de la opción 4 debería reconocerse casi al
instante.

En la hiperplasia benigna de próstata no existe una correlación entre el tamaño de la glándula y la


presencia de síntomas (respuesta 4 falsa). Existen próstatas pequeñas pero muy obstructivas
donde la hiperplasia afecta de forma preferente a la región periuretral, lo que produce una clínica
desproporcionada si se compara con el volumen de la glándula.

Por el contrario, existen pacientes con próstatas mucho más voluminosas, pero con escasa
afectación periuretral, y por tanto, con menos síntomas obstructivos de los que cabría esperar si
sólo tuviéramos en cuenta el tamaño prostático.

El resto de las opciones son claramente correctas: efectivamente, es la causa más frecuente de
obstrucción urinaria en el varón (respuesta 1 co-rrecta). En cuanto a las respuestas 2 y 3, son
claramente correctas. Aparte, en el ENARM, cuando existen dudas, resulta imprudente desafiar al
poder del “puede”…(R/4)

309. Masculino de 76 años con antecedentes de HTA en tratamiento crónico con tiacidas.
Acude a urgencias porque la familia dice que le encuentra "raro". En el control de
laboratorio obtenido se detecta Na de 123 mmol/l, K 4.5 mmol/l, Cr 1 mg/dl, Urea 40 mg/dl.
En la exploración física presenta signos de depleción de volumen. ¿Cuál es la causa más
probable de la hiponatremia que presenta?:

1. 1. Hiponatremia por diuréticos.


2. 2. SIADH.
3. 3. Cirrosis.
4. 4. Síndrome pierde sal.
Gráfico de respuestas
Comentario

Pregunta típica de diagnóstico diferencial de hiponatremia. Para este tipo de preguntas es


importante buscar los antecedentes del paciente y la situación hídrica. En este caso, el paciente
está deshidratado y ha estado tomando tiacidas, y además es mayor, hecho que asocia

!
!
!

fuertemente a efectos secundarios de tiacidas. El otro cuadro compatible con la descripción


descrita es el sídrome pierde sal cerebral, que se caracteriza por elevada natriuresis y
deshidratación en el contexto habitualmente de un EVC, factor que no presenta en este caso.(R/1)

310. ¿Cuál de los siguientes antiarrítmicos está más indicado en el tratamiento de una
fibrilación auricular secundaria a un síndrome de Wolff-Parkinson-White?

1. 1. Amiodarona.
2. 2. Flecainida.
3. 3. Propranolol.
4. 4. Verapamil.
Gráfico de respuestas
Comentario

En esta pregunta la RESPUESTA CORRECTA es la número 2 (flecainida).

La fibrilación auricular, en el contexto del Wolff-Parkinson-White, es una urgencia cardiológica.

En una persona sana, los impulsos originados en la fibrilación auricular (unas 600 ondas por
minuto), afortunadamente, son filtrados por el nodo AV. Éste sólo deja pasar algunos, por lo que la
respuesta ventricular es característicamente irregular, en general entre 100 y 150 lpm (esto es muy
variable, dependiendo de la capacidad de conducción del nodo AV en cada individuo).

En un paciente con vía accesoria, la situación es muy distinta. Es posible que ésta conduzca de
forma anterógrada (de aurícula a ventrículo), con períodos refractarios cortos, es decir, con buena
capacidad de conducción. La vía accesoria no se comporta como un “filtro” de impulsos, como
haría el nodo AV… Por ello, por esta vía podrían pasar todos o casi todos los impulsos auriculares
al ventrículo. En consecuencia, si la aurícula va a 600, el ventrículo recibiría 600 impulsos
eléctricos por minuto… Y eso puede inducir a fibrilación ventricular.

Cuando se produce esta situación, se habla de fibrilación auricular preexcitada. En estos casos, se
precisa del empleo de fármacos que aumenten la refractariedad de las células de la vía accesoria,
siendo de elección la procainamida, flecainida o propafenona. Nunca deben emplearse fármacos
que bloqueen el nodo AV, pues esto puede facilitar la conducción por la vía accesoria de forma
indirecta, por lo que están contraindicados digoxina, verapamil o diltiacem. Los betabloqueantes
frenan el nodo AV y podrían también frenar la vía, pero el efecto es muy variable y, en general, no
se recomiendan.

La amiodarona intravenosa, aparte de su efecto antiarrítmico, tiene un efecto calcioantagonista


marcado. Como consecuencia de este efecto, induciría una vasodilatación periférica, cuya
respuesta compensatoria sería una estimulación simpática refleja. Las catecolaminas serían
contraproducentes en esta circunstancia, porque acortan el período refractario de las vías de
conducción (entre ellas, la vía accesoria), con lo que llegarían más impulsos al ventrículo, pudiendo
desencadenarse una fibrilación ventricular.(R/2)

311. Un paciente de 62 años, ex fumador reciente, consulta por disnea de varios meses
de evolución y tos no productiva. En ambas manos existen acropaquias muy evidentes.
En la auscultación se detectan crepitantes teleinspiratorios y en la Rx tórax se aprecia un
patrón reticular. La pletismografía objetiva una pérdida de volumen y la TC confirma los
hallazgos radiográficos. Si se realiza un lavado broncoalveolar, sería esperable:

!
!
!

1. 1. Predominio de los linfocitos CD8+ sobre los CD4+.


2. 2. Aumento de los neutrófilos.
3. 3. Aumento de los eosinófilos por encima del 50%.
4. 4. Incremento de las células de Langerhans.
Gráfico de respuestas
Comentario

Caso clínico típico. Los datos que te deben llevar a pensar que se trata de un paciente con fibrosis
pulmonar idiopática son: 60 años, disnea y tos seca, crepitantes teleinspiratorios, acropaquias y
patrón reticular. En el LBA típico hay aumento de neutrófilos.(R/2)

312. ¿Cuál de las siguientes característica aparece con más frecuencia en la EPOC tipo
bronquitis que en la tipo enfisema?:

1. 1. Disnea.
2. 2. Tos y expectoración abundante.
3. 3. Disminución del murmullo vesicular en la auscultación pulmonar.
4. 4. Alteración de la capacidad de difusión.
Gráfico de respuestas
Comentario

Pregunta sencilla para la cual debe tener claras aquellas cosas que en un caso clínico nos
aproximan más a enfisema o más a bronquitis crónica, ya que este tema es importante. Debe
saber identificar conceptos que nos indiquen bronquitis crónica como tos y expectoración
abundantes, infecciones y agudizaciones frecuentes, hábito pícnico, cianosis, alteraciones
gasométricas graves, poliglobulia, HTP, cor pulmonale y escasa alteración en la DLCO. Por el
contrario si lo que quieren es que enfoquemos el caso a un enfisema encontraremos expresiones
como la disnea intensa, hábito asténico, poca alteración gasométrica, alteraciones de la DLCO,
retracción elástica alterada, intenso esfuerzo respiratorio, disminución del murmullo vesicular y RX
característica con hiperinsuflación y silueta cardiaca alargada. En nuestro caso nos preguntan algo
muy sencillo, ya que es la tos y expectoración abundante lo que indica bronquitis crónica, y
además es lo que la define clínicamente.(R/2)

313. A 60-year-old female is diagnosed with multifocal invasive ductal breast carcinoma
and is scheduled to undergo mastectomy plus axillary dissection and postoperative
chemotherapy. What is the best timing for breast reconstruction surgery?

1. 1. It depends on the number of positive nodes.


2. 2. Postoperative chemotherapy is a contraindication to immediate reconstruction.
Immediately, since breast reconstruction does not interfere with the natural
3. 3.
history of the disease.
4. 4. In patients aged 60 or over, it is better to use external prosthesis.
Gráfico de respuestas
Comentario

Pregunta sencilla sobre el cáncer de mama. La reconstitución mamaria se puede realizar


inmediatamente, ya que no está contraindicada su realización a pesar de los tratamientos. Es muy
habitual colocar los expansores mamarios inmediatamente posterior a la cirugía. Respuesta
correcta 3.

!
!
!

314. Aunque la pseudoartrosis es una complicación que puede aparecer en cualquier


fractura, su presencia es más frecuente en determinadas localizaciones ¿Cuál de los
siguientes huesos NO suele cursar con pseudoartrosis cuando se produce una fractura
del mismo?

1. 1. Escafoides.
2. 2. Clavícula.
3. 3. Astrágalo.
4. 4. Tibia.
Gráfico de respuestas
Comentario

La pseudoartrosis o ausencia de consolidación, en este caso avascular, es una complicación


relativamente frecuente en todas las localizaciones reflejadas en las opciones excepto la clavícula.
La fractura de clavícula, sobre todo las de 1/3 medio, tienen una tasa de excelentes resultados
mediante tratamiento conservador, con muy bajo índice de trastornos de consolidación, Ya que se
trata de un hueso bien vascularizado. Se ha visto que las fracturas tratadas inicialmente mediante
reducción y osteosíntesis presentan una tasa elevada de pseudoartrosis, de alrededor del 5%.(R/2)

315. Paciente que acude a urgencias con piel seca, mucosas secas, lagrimas
disminuidas, llenado capilar de 2 segundos, oliguria, ruidos cardiacos de buena
intensidad. ¿Qué tipo de deshidratación tiene?

1. 1. Moderada.
2. 2. Severa.
3. 3. Grave.
4. 4. Leve.
Gráfico de respuestas
Comentario

Hay que tener presente que los grados de deshidratación son distintos en lactantes, niños y
adultos, pues se considera más grave en lactantes y niños.

El grado de deshidratación se corresponde con el % de peso perdido pero a veces no se dispone


de ese dato y se tiene que recurrir a signos clínicos que de manera indirecta permiten estimar el
grado de deshidratación.

En pediatria, clasificamos la deshidratación en:

- Leve o de primer grado: Pérdida de < 5% (lactantes), < 3% (niños).

- Moderada o de segundo grado: Pérdida 5-10% (lactantes), 3-7% (niños)

- Grave o de tercer grado: > 10% (niños), > 7 % (lactantes)

Si tenemos en cuenta los signos clínicos, la clasificación sería:

1. Deshidratación leve: sin signos ni síntomas.

!
!
!

2. Deshidratación moderada: sed, sequedad de piel y mucosas, comportamiento inquieto o irritable,


ojos hundidos.

3 .Deshidratación grave: los síntomas se agravan; choque, con pérdida parcial del conocimiento,
falta de diuresis, extremidades frías y húmedas, pulso rápido y débil, tensión arterial baja o no
detectable, y palidez.

En cuanto a los adultos:

- Deshidratación leve: La sed es el único síntoma, pero es necesario saber que la sed no es un
signo específico del déficit de agua, ya que aparece en otros síndromes (diabetes descompensada,
hemorragias o estados de ansiedad)

- Deshidratación moderada: piel y mucosas secas, paciente debilitado, con taquicardia, hipertermia
leve, la disminución del peso corporal en un 5%. Además disminución de la diuresis y aumento del
residuo en orina.

- Deshidratación grave: Al cuadro anterior se le agregan graves trastornos de la conciencia


(obnubilación, delirio, estupor, que pueden finalizar en un estado de coma). Se profundiza la
hipotensión arterial y la hipertermia y la muerte sobreviene cuando la pérdida de agua alcanza
entre 6 y 10 litros en una persona de talla adulta.

En esta pregunta, independientemente de que el paciente sea un niño o un adulto se trata de un


caso de deshidratación moderada. Ten en cuenta que la respuesta 2 y 3 podrian ser el mismo
grado de deshidrtación, pues severa y grave son sinónimos.(R/1)

316. Usted sabe que el síndrome de ovario poliquístico (SOP) se caracteriza por una
alteración hormonal compleja. Respecto a la fisiopatología, es FALSO que:

1. 1. Afecta al 4-10% de las mujeres en edad reproductiva.


Existe una concentración sérica media de LH elevada, con FSH normal o en el
2. 2.
límite inferior de la normalidad.
En un 20-30% de casos aparece hiperprolactinemia moderada, que alcanza
3. 3.
niveles anovulatorios.
El DHEA-S se encuentra elevado en el 20% de las pacientes con ritmos
4. 4.
circadianos de ACTH y cortisol normales.
Gráfico de respuestas
Comentario

El síndrome de ovario poliquístico es muy frecuente, afectando al 1-5% de las mujeres,


aumentando hasta el 10% en edad reproductiva, incluso más en algunas series. La fisiopatología
es compleja y en algunos aspectos no conocida del todo. Aparece un aumento de los pulsos de la
GnRH por parte del hipotálamo, el cual estimula un aumento constante de la LH, la cual duplica a
la FSH. La prolactina puede elevarse en un 20-30% de las pacientes, pero NO llega a alcanzar
niveles anovulatorios. Los andrógenos suprarrenales pueden elevarse en torno al 20% de las
pacientes.(R/3)

317. Un niño de 8 años presenta un cuadro de cefalea, fiebre, exantema maculopapular


generalizado y dolor centrotorácico punzante. En las exploraciones complementarias se
aprecia en el LCR una pleocitosis linfocitaria con glucorraquia y proteinorraquia
normales, y en el electrocardiograma se observa una elevación generalizada del

!
!
!

segmento ST con concavidad hacia arriba. ¿Qué diagnóstico es el más probable ante
dicha constelación de hallazgos?

1. 1. Infección por Mycoplasma pneumoniae.


2. 2. Infección por virus herpes simple 2.
3. 3. Infección por Coxsackie B.
4. 4. Infección por el virus de la parotiditis.
Gráfico de respuestas
Comentario

Una elevación generalizada del ST hacia arriba es sugestivo de una pericarditis, sobre todo si
además es cóncavo. Por otra parte, sabe que la causa más frecuente de pericarditis aguda es
vírica (Coxsackie B), por lo que debe elegir la respuesta 3.

En cuanto al resto del cuadro clínico, no era una pregunta tan difícil. Prácticamente todos los virus
pueden justificar exantemas cutáneos, y los enterovirus (entre ellos el coxsackie) son también una
causa frecuente de meningitis vírica. El mismo virus, por tanto, puede justificar todo lo que nos
cuentan de este niño…

318. Los betabloqueantes son útiles en el tratamiento de la angina de pecho porque, entre
otras acciones:

1. 1. Dilatan las arterias coronarias.


2. 2. Disminuyen la frecuencia cardiaca.
3. 3. Aumentan la tensión arterial.
4. 4. Aumentan el poder contráctil del corazón.
Gráfico de respuestas
Comentario

Es una pregunta muy fácil y todos deben conocer el verdadero motivo por el cual los
betabloqueantes ejercen un efecto tan favorable en los pacientes con angina de pecho. Esto es
debido a que al disminuir la frecuencia cardiaca, están disminuyendo el trabajo cardiaco, lo que a
su vez provoca una disminución de los requerimientos de oxígeno. Este efecto es muy beneficioso
en estos pacientes. Los efectos secundarios de los beta- bloqueantes pueden ser bradicardia,
bloqueos cardíacos, disminución de la contractilidad cardiaca con insuficiencia cardiaca,
vasoconstricción periférica, broncoespasmo, fatiga muscular, depresión, pesadillas, alucinaciones,
hiperglucemia, disminución del flujo renal....(R/2)

319. En la glomerulonefritis aguda postestreptocócica, una de las siguientes


afirmaciones es FALSA:

El título de antiestreptolisina O (ASLO) se eleva en la mayoría de los casos de


1. 1.
origen faringoamigdalar.
Un título normal de ASLO no descarta el origen postestreptocócico de una
2. 2.
glomerulonefritis aguda.
Tras una infección estreptocócica, la elevación del título de ASLO aumenta el
3. 3.
riesgo de desarrollar una glomerulonefritis aguda.
El tratamiento precoz por antibióticos de la infección estreptocócica inicial puede
4. 4.
decapitar la esperada elevación del título de ASLO.
Gráfico de respuestas
Comentario

!
!
!

Pregunta de elevada dificultad acerca de un aspecto concreto de la GN aguda post-estreptocócica:


el ASLO o antiestreptolisina O. En referencia al diagnóstico de la GNA, llama la atención que los
cultivos de faringe o de piel sólo resultan positivos para el estreptococo betahemolítico en
alrededor de una cuarta parte de los casos no tratados previamente con antibióticos. Por el
contrario, las pruebas serológicas de una infección estreptocócica reciente suelen ser positivas en
más del 75% de las GNA. En la mayoría de los casos de origen faringoamigdalar se observa una
elevación significativa del título de antiestreptolisina O (ASLO) al cabo de 3-5 semanas del
comienzo de la infección. Dicha elevación no guarda relación con el riesgo de desarrollar
glomerulonefritis ni con su gravedad o pronóstico (LA OPCION FALSA ES LA 3). En los casos de
infección cutánea, el título de ASLO se eleva poco, siendo mucho más frecuente el hallazgo de
elevaciones significativas de los títulos de antihialuronidasa, de antiestreptocinasa y de anti-
DNAasa B.(R/3)

320. Mujer de 20 años que acude a su consulta refiriendo menstruaciones irregulares que
incluyen baches amenorreicos. No relaciones sexuales. Antecedentes personales:
enfermedades propias de la infancia; amigdalectomía; no alergias medicamentosas
conocidas. Exploración: en el tacto rectal parece palparse un aparato genital normal;
resto de exploración física sin datos de interés salvo talla de 162 cm y peso de 75 kg. Se
realiza ultrasonido que muestra un útero regular de 60 mm y unos anexos ligeramente
aumentados de tamaño con múltiples imágenes econegativas compatibles con folículos,
la mayor de los cuáles tiene 9 mm de diámetro. Señale la respuesta INCORRECTA:

Lo más probable es que encontremos en los exámenes de laboratorio un cociente


1. 1.
LH/FSH mayor de 2.
El tratamiento de elección de esta paciente se basa en la administración de
2. 2.
antiandrógenos tipo acetato de ciproterona.
3. 3. Esta paciente mejoraría su cuadro clínico con una simple reducción del IMC.
4. 4. Algunas pacientes pueden presentar Acantosis nigricans.
Gráfico de respuestas
Comentario

Sospeche un SOP cuando, como en este caso, se encuentre con una mujer con menstruaciones
irregulares y baches amenorreicos, sobrepeso, y hallazgos ecográficos de quistes en ovarios. Los
antiandrógenos no son el tratamiento de elección en estas pacientes. Debe iniciarse el tratamiento
con reducción de peso y metformina, y anticonceptivos orales si no desea gestación. El acetato de
ciproterona, un progestágeno con efecto antiandrógeno, está indicado para el tratamiento del
hirsutismo y el acné en estas pacientes.(R/2)

321. Señale cuál de las siguientes respuestas acerca de la problemática social del niño
NO es correcta:

Cuando se adopta a un niño debe procurarse que sea en la edad más cercana al
1. 1.
nacimiento para favorecer el vínculo entre padres e hijo.
Se debe informar al niño de su situación en cuanto pueda comprenderlo, como
2. 2.
muy tarde a los 4 años.
Los niños pequeños no deben asistir a las ceremonias que rodean a la muerte de
3. 3.
un progenitor porque crean estado de confusión al no comprenderlo.
La reacción más frecuente de los adolescentes frente a la separación o divorcio de
4. 4.
los padres, es el pensamiento mágico de que sus padres van a volver a unirse.
Gráfico de respuestas
Comentario

!
!
!

Esta pregunta acerca de la problemática social del niño carece de relevancia. Ante la separación o
divorcio de los padres el determinante mas importante de las respuesta iniciales es la edad del
niño. Algunos niños presentan depresión, otros lo niegan y evitan el tema como principal
mecanismo de defensa. La mayoría se aferra a la fantasía de que la separación no es real y de
que volverán a unirse. En relación a la muerte de un ser querido simplemente recuerde que la edad
a la que los niños comprenden que la muerte es permanente y comienzan a entenderla como un
proceso biológico es entre los 5 y 10 años. El fallecimiento de un familiar es la pérdida más dura
para un niño y la reacción ante ella está también esta en función de la edad. Está demostrado que
los niños necesitan hablar de la pérdida con adultos, pues les alivia las penas. El que los niños
más pequeños no comprendan todas las dimensiones de la muerte no es razón para que se les
oculte y no asistan a las ceremonias que la rodean (R/3)

322. Mujer de 78 años de edad, con antecedentes de HTA en tratamiento con lisinopril,
que ingresa en unidad coronaria por IAM inferior evolucionado no revascularizado. La
paciente rechaza hacerse coronariografía, puesto que se encuentra asintomática y le da
miedo la prueba. A las 48 horas de ingreso, la paciente comienza con disnea brusca,
intolerancia al decúbito, saturación O2 85% a pesar de O2 en mascarilla y le cae la TA a
85/60 mmHg. En la exploración física, la paciente está sudorosa, mal perfundida y con
crepitantes hasta campos medios. El puso es débil, rítmico a 95 lpm, y se le ausculta un
soplo pansistólico rudo intenso en borde esternal izquierdo irradiado en barra. Ante la
sospecha de complicación mecánica, ¿qué NO esperaría encontrar en esta paciente?:

1. 1. Salto oximétrico en VD.


2. 2. Ondas V grandes en el registro de PCP con el catéter de Swan-Ganz.
3. 3. Acinesia inferior con hipercontractilidad del resto de segmentos no infartados.
4. 4. Índice cardíaco < 2.2 l/min/kg.
Gráfico de respuestas
Comentario

Generalmente el soplo de IM aguda del caso anterior son soplos que se oyen mal. Sin embargo, el
soplo de la rotura del SIV (séptum interventricular) es un soplo como el descrito en el enunciado.
Por tanto, la sospecha es de rotura del SIV, precisamente la complicación que es “más rara”. Lo
que no veremos son las ondas V grandes en el registro de PCP porque esto es lo que se ve en el
caso de que hubiera sido una IM aguda severa (la sangre retrocede del VI a la AI y de aquí a
través de las venas pulmonares al capilar pulmonar, produciendo unas ondas V gigantescas). Todo
lo demás se puede ver en la rotura del SIV.(R/2)

323. Un paciente es intervenido quirúrgicamente de forma urgente por sospecha de


apendicitis aguda. Durante la cirugía se aprecia apendicitis aguda con peritonitis fecal y
una formación purulenta localizada. ¿Qué actitud debe adoptarse con los antibióticos
utilizados profilácticamente?:

1. 1. Suspender tras la cirugía.


2. 2. Mantener 24-48 horas.
3. 3. Poner otra dosis postoperatoria.
4. 4. Mantenerlos como tratamiento durante 7-10 días.
Gráfico de respuestas
Comentario

Pregunta fácil y deducible que no debe plantarle mucha duda. En el tratamiento de la apendicitis
aguda, la cirugía se complementa con la administración de antibioterapia PROFILACTICA, frente a

!
!
!

gramnegativos y anaerobios (flora saprófita de colon). Cuando la apendicitis progresa a peritonitis


el tratamiento antibiótico ya no es profiláctico sino TERÉEUTICO y debe estar dirigido contra los
mismos microorganismos, siendo una combinación útil cefalosporina + metronidazol, por
ejemplo.(R/4)

324. Among the following cyanotic congenital heart diseases, only one does NOT typically
present with decreased pulmonary flow:

1. 1. Tricuspid atresia.
2. 2. Single ventricle without pulmonary stenosis.
3. 3. Pulmonary atresia with intact ventricular septum.
4. 4. Transposition of the great arteries with pulmonary stenosis.
Gráfico de respuestas
Comentario

Para contestar preguntas de este tipo es muy importante que tengan clara una clasificación de las
cardiopatías congénitas que las divida en cianóticas - acianóticas y, a su vez, con flujo pulmonar
aumentado o normal. Ésta es una pregunta difícil pero que puede deducir fácilmente teniendo en
cuenta en qué consiste la malformación en cada una de las opciones. Tengan en cuenta que, en
general, el flujo pulmonar estará disminuido en aquellas cardiopatías en las que exista una atresia
o estenosis de válvulas derechas (tricúspide o pulmonar), como ocurre en las opciones 1, 3 y 4. En
el ventrículo único sin estenosis pulmonar, en la que existe una única cavidad ventricular que
recibe sangre de las dos aurículas, ocurre lo contrario, es decir, existirá plétora pulmonar.(R/2)

325. Respecto al cáncer de cérvix, cuál de las siguientes afirmaciones es INCORRECTA:

1. 1. El síntoma más frecuente es la leucorrea maloliente.


2. 2. La lesión precursora es la displasia o carcinoma in situ.
Entre pacientes con cáncer cervical in situ que no han sido tratadas, del 30% al
3. 3.
70% desarrollarán carcinoma invasor en un período de 10 a 12 años.
El cáncer cervical progresa de manera ordenada, pero ocasionalmente se puede
4. 4.
presentar un tumor pequeño con metástasis a distancia.
Gráfico de respuestas
Comentario

El cáncer de cérvix en la mayoría de los casos permanece asintomático. En etapas tardías, el


síntoma más precoz y característico es la metrorragia. También es posible que aparezca leucorrea,
que en la enfermedad avanzada, es fétido, llamado "en agua de lavar carne".(R/1)

326. Mujer en el 8º mes de gestación que de forma brusca e indolora padece una
metrorragia de sangre líquida y roja, de moderada cuantía. El cese del sangrado es
espontáneo y cuando usted la explora, ni la madre ni el feto se encuentra afectados.
Señale la respuesta FALSA:

1.! ! 1.! El!tratamiento!consiste!en!reposo!en!cama!y!beta<adrenérgicos.!


2.! ! 2.! En!caso!de!sangrado!repetido!y!grave!debemos!realizar!una!cesárea.!
El!problema!puede!estar!provocado!por!cicatrices!uterinas!por!cirugías!previas!
3.! ! 3.!
sobre!el!útero.!
Es!conveniente!repetir!los!tactos!vaginales!para!cercionarnos!del!descenso!de!la!
4.! ! 4.!
presentación.!
Gráfico de respuestas

!
!
!

Comentario

La hemorragia del tercer trimestre ante la que nos encontramos es la placenta previa. Recuerde
que el sangrado suele ser un sangrado intermitente y cada vez con mayor frecuencia y que no
existen más datos asociados (ni dolor ni alteraciones del tono uterino). La conducta es expectante
si la hemorragia cede como es el caso de la pregunta, realizar una ecografía para definir el tipo de
placenta previa y decidir la vía del parto y ante todo evitar las exploraciones vaginales porque
pueden precipitar un nuevo episodio de hemorragia.(R/4)

327. Acude al Servicio de Urgencias paciente masculino de 35 años aquejado de náuseas


y vómitos de 48h de evolución, sin diarrea y con un cuadro de debilidad intensa y
febrícula. Presenta hiperpigmentación cutánea, si bien reconoce haber estado
bronceándose en la playa. En los exámenes de laboratorio básicos destaca la presencia
de una glucemia de 57 mg/dL, con una cifra de sodio en plasma de 132 meq/L y un potasio
sérico de 5.6 meq/L. El paciente presenta una hipotensión muy marcada y con signos de
deshidratación evidentes.¿Cuál de las siguientes afirmaciones es cierta en cuanto a la
crisis suprarrenal?:

Puede desencadenarse en casos de hipotiroidismo hipofisario tratado inicialmente


1. 1.
con hormonas tiroideas.
Debe tratarse de forma urgente con soluciones hipertónicas para compensar el
2. 2.
déficit de líquidos y de Na+.
Lo importante es el tratamiento de soporte y se debe esperar a conocer la etiología
3. 3.
antes de administrar los glucocorticoides.
La causa más frecuente es la exacerbación de una insuficiencia suprarrenal
4. 4.
crónica no tratada por algún episodio de estrés.
Gráfico de respuestas
Comentario

Esta pregunta hace referencia a la crisis suprarrenal. Respecto a ella, merece la pena recordar:

La causa más frecuente de crisis suprarrenal es la suspensión brusca de un tratamiento


prolongado con glucocorticoides.

El tratamiento con suero salino isotónico es de vital importancia, así como la administración precoz
de glucocorticoides intravenosos, incluso sin conocer la etiología del cuadro.

En caso de coexistir un déficit de hormonas tiroideas, siempre se administrarán primero los


glucocorticoides y posteriormente las hormonas tiroideas. Si se invierte el orden, se corre el riesgo
de desencadenar una crisis suprarrenal.

Por último, señalar que en estos pacientes, si el tratamiento sustitutivo se pauta de forma
adecuada, no tiene por qué existir un marcado descenso de la supervivencia.(R/1)

328. ¿Cómo se podría prevenir el desprendimiento de retina?:

1. 1. Fotocoagulando las degeneraciones periféricas.


2. 2. Prohibiendo el deporte a los miopes.
3. 3. Prohibiendo el deporte a los hipermétropes.
4. 4. Usando lentes de contacto.
Gráfico de respuestas

!
!
!

Comentario

Es una pregunta que no debería fallar, puede sacarla incluso por exclusión, ya que las opciones 2
a 4 son como mínimo exóticas. En sujetos expuestos a padecer desprendimiento por presentar
áreas de degeneración periféricas es preciso hacer profilaxis mediante fotocoagulación con láser,
por lo que la opción correcta es la 1.

329. En relación a las anemias megaloblásticas, señale la FALSA:

1. 1. La causa más frecuente de anemia megaloblástica es el déficit de B12.


Los pacientes con déficit de ácido fólico tienen más tendencia a padecer un estado
2. 2.
de malnutrición subyacente que los que tienen déficit de B12.
Las manifestaciones gastrointestinales por déficit de ácido fólico suelen ser más
3. 3.
difusas y más graves que las de la anemia perniciosa.
El déficit de folato durante el embarazo puede causar defectos del tubo neural en
4. 4.
el feto.
Gráfico de respuestas
Comentario

Pregunta de elevada dificultad. Encontrar la opción verdadera no es muy difícil, pero no olvide
fijarse en las otras opciones que contienen importante información sobre las anemias
megaloblasticas. Los depósitos de acido fólico son mucho más pequeños que los de vit. B12 y por
lo tanto es de esperar que la causa más frecuente de anemia megaloblastica sea por folicopenia
(opción 1 falsa). Le recuerdo además que estudios recientes han demostrado que la administración
de acido fólico durante el embarazo reduce la aparición de las alteraciones del tubo neural.(R/1)

330. Acude a su consulta una mujer de 45 años con un bulto no palpable en la mama
izquierda. La mastografía indica un BI-RADS 4 (sospechosa de malignidad). El ginecólogo
decide operar sin hacer más pruebas y enviar la muestra intraoperatoriamente para que
el patólogo le confirme la sospecha de carcinoma. ¿Cómo tiene que enviar la muestra?:

1. 1. En formol.
2. 2. En fresco.
3. 3. En alcohol.
4. 4. Da igual en qué medio fijador, lo importante es que se mande inmediatamente.
Gráfico de respuestas
Comentario

Pregunta dudosa y confusa pues rige más la costumbre de cada hospital que la escasa evidencia
disponible. La necesidad de urgencia en un estudio intraoperatorio viene definida principalmente
por el tiempo de quirófano que se pierde (y el dinero que supone) esperando el diagnóstico.

Por ello lo importante es que se reciba la muestra en laboratorio lo más rápido posible y que el
diagnóstico se emita igualmente con la menor demora. Se dio como correcta la respuesta 2,
mandar la muestra en fresco porque es la mejor técnica para el envío de muestras intraoperatorias.

331. Acude a urgencias una niña de 12 años por presentar fuertes dolores en hipogastrio
que comenzaron hace dos meses, pero que han ido en aumento. Aún no ha tenido la
menarquía. En una el USG abdominal se observa una masa en pelvis de 8x5 cm ecomixta
bien delimitada. Señale lo CIERTO sobre la patología que sospecha:

!
!
!

1. 1. Habría que determinar las gonadotropinas, que probablemente sean bajas.


2. 2. Habría que someterle a una intervención quirúrgica.
3. 3. Habría que realizar una ecografía renal.
4. 4. Habría que extirpar las gónadas por el riesgo de desarrollar tumor.
Gráfico de respuestas
Comentario

Cuadro clínico típico de libro, no puede fallarlo. El cuadro clínico es el típico del himen imperforado,
que cursa con dolor abdominal cíclico, progresivo, por el acúmulo menstrual retenido, que es la
masa que se visualiza en el USG o hematocolpos. A la exploración se vería el himen imperforado
con una masa azulada por detrás, hematocolpos. Se cura con la incisión y evacuación de dicho
contenido vaginal.(R/2)

332. En la telangiectasia hemorrágica hereditaria (Rendu-Osler):

1. 1. Está alargado el tiempo de coagulación.


2. 2. Las plaquetas están disminuidas.
3. 3. El tiempo de hemorragia está alargado.
4. 4. Todos los estudios de coagulación son normales.
Gráfico de respuestas
Comentario

Se estudia en el grupo de las trombocitopatías, aunque realmente no es una alteración de la


función plaquetaria, sino una púrpura vascular por alteraciones a nivel de los vasos, pero que
también cursa con tiempo de hemorragia alargado (opción 4 correcta). Tan sólo recuerda que es
una malformación vascular AD caracterizada por la aparición de neoformaciones telangiectásicas
en forma de manchas puntiformes de color rojo violáceo, que al romperse, dan lugar a
manifestaciones clínicas de la enfermedad, fundamentalmente epistaxis y hemorragias digestivas.
La localización preferencial de estos angiomas suele ser la cara, pulpejos de los dedos, axilas y
mucosas, siendo sumamente características la presencia de telangiectasias en labios y lengua. El
tratamiento es puramente sintomático. Recuerde que ante un tiempo de hemorragia alargado hay
que determinar el número de plaquetas, porque la causa más frecuente es la trombopenia; sin
embargo si el número de plaquetas es normal habrá que sospechar una Enfermedad de von
Willebrand. Un TTPa alargado es típico del tratamiento con heparina, y si esta no se está utilizando
debemos investigar la posibilidad de que se trate de una hemofilia mediante la cuantificación de los
factores de coagulación. No olvide que en el caso del tratamiento con ACO el que se alarga es el
TP. Un cuadro que cursa con trombopenia pero con todos los estudios de coagulación normales
debe recordarnos la PTT y su péntada: trombopenia, anemia hemolítica microangiopática con
esquistocitos, fiebre afección neurológica transitoria y fluctuante y disfución renal y cuyo
tratamiento de elección es la plasmaféresis.(R/3)

333. Si al hacer una ecografía cerebral a un prematuro nos encontramos con imágenes
anómalas periventriculares e hiperecogenicidad en más del 50% del espacio ventricular
con dilatación ventricular, el diagnóstico es:

1. 1. Hemorragia intraventricular tipo II y áreas de leucomalacia.


2. 2. Hemorragia intraventricular tipo III y áreas de leucomalacia.
3. 3. Hemorragia intraventricular tipo IV y áreas de leucomalacia.
4. 4. Hemorragia intraventricular tipo V y áreas de leucomalacia.
Gráfico de respuestas
Comentario

!
!
!

Los estudios Ultrasonográficos permiten clasificar las HIV en cuatro grupos fundamentales:

Grado I: Cuandola hemorragia está localizada en la matriz germinal, sin o mínima hemorragia
intraventricular.

Grado II: El sangrado es intraventricular y ocupa entre el 10 y el 50 % del ventrículo.

Grado III: El sangrado intraventricular es mayor del 50 % y el ventrículo lateral está distendido.

Grado IV: a lo anterior se asocia sangramiento intraparenquimatoso.(R/2)

334. ¿Cuál de los siguientes perfiles corresponde a un pseudohipoparatiroidismo tipo II?


(respuesta de AMPc nefrógeno a infusión de PTH: AMPc-PTH; respuesta del fósforo
urinario a la infusión de PTH: Po-PTH):

1. 1. PTH baja, AMPc-PTH aumenta, Po-PTH aumenta.


2. 2. PTH alta, AMPc-PTH aumenta, Po-PTH no aumenta.
3. 3. PTH normal, AMPc-PTH aumenta, Po-PTH aumenta.
4. 4. PTH baja, AMPc-PTH no aumenta, Po-PTH no aumenta.
Gráfico de respuestas
Comentario

Pregunta complicada sobre pseudohipoparatiroidismo, pero que puede contestarse conociendo


algunos detalles de esta patología. Para empezar, descartamos las respuestas 1 y , pues si la PTH
estuviese baja estaríamos ante un hipoparatiroidismo, no ante un PSEUDOhipoparatiroidismo. El
tipo II se caracteriza por un calcio bajo y un fósforo elevado, pero con PTH elevada. Son capaces
de inducir cAMP nefrogénico, pero no hay respuesta fosfatúrica tras la infusión de PTH (el defecto
en la acción de la PTH está situado más allá de la síntesis del cAMP).(R/2)

335. Un paciente de 53 años acude por síndrome cistítico de meses de evolución. Su


médico le ha tratado varias veces como infección de orina a pesar de haber encontrado
repetidos urocultivos negativos. Refiere hematuria ocasional. No es fumador. En los
exámenes de laboratorio se observa piuria franca y una orina con un pH reiteradamente
elevado. En este caso:

Es poco probable la resolución del proceso, ya que al desconocerse su patogenia


1. 1.
los tratamientos existentes son escasos y con malos resultados.
2. 2. Lo más útil será la realización de una PCR en orina.
3. 3. Se deberá iniciar de inmediato tratamiento con BCG.
4. 4. Se debe proponer de inicio una cistectomía radical.
Gráfico de respuestas
Comentario

La tuberculosis genitourinaria se produce principalmente por diseminación hematógena de una


primoinfección pulmonar. La prueba más útil para el diagnóstico es la PCR de orina y, en ausencia
de ésta o de un cultivo de Löwenstein positivo, será necesario realizar diagnóstico diferencial con
otras entidades como el ca. in situ de vejiga.(R/2)

!
!
!

336. A patient is found to have high grade intraepitelial dysplasia on follow-up cervical
cytology. Colposcopy reveals no atypical lesions in the transition zone. Which of the
following is the most appropriate next step in management?

1. 1. New cervical cytology and colposcopy.


2. 2. Cryotherapy.
3. 3. Endocervical curettage.
4. 4. Total hysterectomy.
Gráfico de respuestas
Comentario

No puede fallar esta pregunta. Después de tener una citología alteralda hay que realizar una
colposcopía y si está es normal, se deberá repetir de nuevo la citología y si procede la
colposcopía.(R/1)

337. Un recién nacido pretérmino, de 3 días de vida, tiene episodios de apnea de 5-10
segundos, presentando después taquipnea durante 10-15 segundos. No se observan
cambios de color de la piel ni de la frecuencia cardíaca. La conducta más apropiada en
este caso es:

1. 1. Administrar cafeína.
2. 2. Administrar teofilina.
3. 3. Aplicar presión positiva continua en la vía aérea.
4. 4. Tranquilizar a los padres.
Gráfico de respuestas
Comentario

Este RNPT presenta lo que se conoce como respiraciones periódicas que son apneas de 5-10
segundos seguidas de respiraciones rápidas de 10-15 segundos y se diferencian de la apnea
idiopática en que la primera no se acompaña de cianosis ni bradicaria y que altera las fases 3 y 4
del sueño mientras que la apena idiopática provoca alteraciones en la fase REM y si aparecen las
alteraciones ya mencionadas. La actitud en este caso es la observación y tranquilizar a los
padres.(R/4)

338. Las siguientes alteraciones se han relacionado con el empleo de anticonceptivos


orales, EXCEPTO:

1. 1. Poliarteritis nudosa.
2. 2. Embolia pulmonar.
3. 3. Síndrome de Budd-Chiari.
4. 4. Colitis isquémica transitoria.
Gráfico de respuestas
Comentario

Es importante que conozca el mecanismo de acción los efectos secundarios y las


contraindicaciones de los anticonceptivos orales. Aunque esta pregunta es bastante fácil si tiene en
cuenta los efectos tromboembólicos de los anticonceptivos. La anticoncepción hormonal es un
factor de riesgo para la enfermedad tromboembólica y para el infarto agudo de miocardio. Induce
una elevación de los factores de coagulación I- II- VII- IX- X y plasminógeno. Aumentan la actividad
fibrinolítica. La edad superior a 35 años quintuplica los riesgos.(R/1)

!
!
!

339. El tratamiento de la intoxicación por órganofosforados comprende:

1. 1. Atropinización del paciente, adrenalina y oxigenoterapia.


2. 2. Atropinización y monitorización de funciones vitales.
3. 3. Atropinización y antibióticos profilácticos.
4. 4. Atropinización y lavado gástrico.
Gráfico de respuestas
Comentario

Medidas generales

a) ABC

b) Prevención de la absorción

En ingestión

- Lavado gástrico

- Carbón activado.

- En intoxicaciones dérmicas

Sacar toda la ropa, bañar con agua y jabón al paciente, (proporcionar protección al personal
médico que realice esta maniobra). Estos pacientes deben ser observados por lo menos 6 a 8
horas, antes de ser dados de alta.

c) Tratamiento

- Neumonitis química

- Insuficiencia respiratoria

- Convulsiones

- Coma.

d) Aumento de la eliminación: Hemodiálisis o hemoperfusión no están generalmente indicados.

Antídotos

El tratamiento específico incluye un agente antimuscarínico; como es la atropina y un reactivador


enzimático; como lo es la obidoxima o la pralidoxima.(R/2)

340. A 57-year-old woman comes to the clinic for a follow-up appointment because she
has been having a rash over her face that has not improved with first-line treatments.
Further questioning reveals that she has a 6-week history of weakness and asthenia.
Physical examination shows obvious proximal weakness, with muscle strength 3/5. Scaly

!
!
!

erythematous eruptions overlying her knuckles and elbows are noted. These clinical
findings are consistent with which of the following options?

1. 1. Eaton-Lambert.
2. 2. Rheumatoid arthritis.
3. 3. Dermatomyositis.
4. 4. Systemic sclerosis (scleroderma).
Gráfico de respuestas
Comentario

Dermatomyositis. You have to recognize some combination of words and rapidly link them to a
diagnosis. Skin rash + proximal muscle weakness in a middle-aged woman (commonly from 40 to
60 years old) = DERMATOMYOSITIS. Further useful points about this disease are:

1) Gottron papules over the knuckles and periorbital heliotropic rash (the most specific skin signs).

2) The cause seems to be an autoimmune reaction to muscle tissue in genetically susceptible


people. Familial clustering occurs, and HLA subtypes -DR3, -DR52, and -DR6 seem to be the
genetic predisposition.

3) Dermatomyositis may be typically associated to cancer. 4) Treatment is with corticosteroids,


usually combined with immunosuppressants or IV immune globulin.(R/3)

341. ¿Cuál de las siguientes no es una manifestación de resolución de íleo paralítico post
operatorio?

1. 1. Expulsión de gases.
2. 2. Desaparición del cuadro febril.
3. 3. Presencia de ruidos hidroaéreos.
4. 4. Disminución de distensión abdominal.
Gráfico de respuestas
Comentario

La respuesta correcta es la 2, ya que no es común que los pacientes con íleo paralítico
postquirúrgico se presenten con fiebre, por lo que su desaparición no se relaciona con la mejoría
del cuadro.

342. Señale la opción FALSA:

El primer signo de aparición de pubertad en la mujer es la aparición de botón


1. 1.
mamario o telarquia.
El primer signo de aparición de pubertad en el varón es la aparición de vello en la
2. 2.
zona púbica o pubarquia.
El estirón puberal es algo más precoz en las niñas frente al varón, coincidiendo
3. 3.
con el estadio de Tanner II.
4. 4. Un volumen testicular inferior a 3 cc indica estadio Tanner I.
Gráfico de respuestas
Comentario

En el ENARM, alguna vez se ha hecho alusión al del desarrollo puberal, definido por los estadios
de Tanner (I- V), siendo

!
!
!

el Tanner I el estadio prepuberal o infantil. Recuerde que el signo físico inicial que marca el
comienzo de la pubertad en el varón es el incremento del tamaño testicular por encima de 4 cc, y
en la mujer la aparición de telarquia o botón mamario (opción 1 verdadera). La aparición de vello
axilar y pubiano es fruto de un fenómeno independiente denominado adrenarquia. Durante la
pubertad acaece el estirón puberal, siendo más precoz en la mujer (Tanner II) frente el varón
(estadios III-IV), generalmente siendo máximo previo a la aparición de la menarquia (edad
aproximada 12 años). Por último un dato que muestra la impregnación estrogénica de la pubertad
es el incremento de los ovarios y cuerpo uterino en la mujer, invirtiéndose la relación cuerpo-cuello
existente en estadios infantiles.(R/2)

343. Con respecto al cáncer colorrectal hereditario no asociado a poliposis es cierto que:

Es más frecuente encontrar tumores metacrónicos que en el cáncer colorrectal


1. 1.
esporádico.
2. 2. Es más frecuente en el colon derecho.
3. 3. Esta asociado a mutaciones en genes (MSH2, MLH1, MSH6 y PMS2).
4. 4. Todas son ciertas.
Gráfico de respuestas
Comentario

El síndrome de cancer colorrectal no asociado a poliposis (CCHNP), tradicionalmente llamado


Síndrome de Lynch se caracteriza por la presencia de tumores colónicos en pacientes menores de
40 años y sobre todo en el colon derecho. Asimismo se asocia a la presencia tanto de tumores
metacrónicos como sincrónicos en otros lugares de la anatomía como endometriales, ováricos,
tumores uroteliales, gástricos, etc. A nivel génico se caracteriza por mutaciones en genes (MSH2,
MLH1, MSH6 y PMS2).(R/4)

344. Los fármacos anticolinérgicos pueden desencadenar:

1. 1. Depresión.
2. 2. Toxicomanía.
3. 3. Estados confuso oníricos.
4. 4. Ataques de pánico.
Gráfico de respuestas
Comentario

Ya solo el nombre del grupo farmacológico asusta, pues nos hace dudar en las miles de reglas
para distinguir los efectos simpático, parasimpáticos, y sus antagónicos... sin embargo es
importante que los conozca, pues son de mucha utilidad como correctores de efectos
extrapiramidales de los antipsicóticos. La pregunta recalca cual es el efecto secundario principal de
los anticolinérgicos, la tendencia a la confusión, en especial en pacientes ancianos o con previa
disminución del nivel de conciencia (no dar en delirium ni en demecia!). De hecho, podría
relacionar este dato con el grupo de fármacos que se utilizan para el tratamiento de las demencias,
los anticolinesterásicos, justo los antagónicos.(R/3)

!
!
!

345. A 70 year-old man comes to the physician


because of severe low back pain that interferes with sleep. He had a colon
adenocarcinoma that was treated with surgery and chemotherapy two months ago.
Physical examination reveals decreased mobility of the spine and 37.4ºC. X-ray is shown.
Which of the following test should be done in this case?

1. 1. Bone scintigraphy.
2. 2. Positron emission tomography.
3. 3. Bone-density scan.
4. 4. Take a biopsy.
Gráfico de respuestas
Comentario
En un paciente oncológico, sometido a quimioterapia reciente, y que se presenta con fiebre y
lumbalgia, debemos sospechar la existencia de una discitis infecciosa. En la imagen a la que nos
refiere la pregunta (una radiografía simple en proyección lateral de la columna lumbar), deberíamos
observar: 1) la morfología de las vértebras lumbares, que puede alterarse en presencia de
metástasis (no es el caso de la imagen), y 2) la morfología de los discos (espacios) intervertebrales
(que sí está alterado). Está claro que el espacio entre la tercera y la cuarta vértebra lumbares ha
desaparecido. Este patrón de afectación discal es típico de la discitis infecciosa (rápida destrucción
del espacio discal). Aunque es posible que nos planteemos realizar al paciente varias pruebas
complementarias para descartar la afectación neoplásica (gammagrafía, resonancia, revisión de
marcadores tumorales, e incluso PET), la biopsia (posiblemente mediante punción guiada por TAC)
permitirá tomar muestra para estudio microbiológico (sobre todo) y anatomopatológico
(secundariamente).(R/4)

!
!
!

346. ¿En cuál de las siguientes vasculitis es EXCEPCIONAL tener que recurrir a la biopsia
para su diagnóstico?:

1.! ! 1.! Arteritis!de!Takayasu.!


2.! ! 2.! Arteritis!temporal.!
3.! ! 3.! PAN.!
4.! ! 4.! Churg<Strauss.!
Gráfico de respuestas
Comentario
Dado el calibre de los vasos a los que afecta la arteritis de Takayasu, no se recurre a la biopsia
`para su diagnóstico; en general, ante un cuadro clínico compatible, el diagnóstico se realiza
mediante arteriografía. En el resto de vasculitis que se mencionan, siempre que sea posible, debe
realizarse un diagnóstico anatomopatológico. EL lugar de elección para la realización de la biopsia
es un tema de difícil resolución; en general, se realizará del órgano afectado que sea más
accesible ( piel, nervio, riñón...etc). Hay que tener en cuenta, en cuanto a la enfermedad de
Wegener, que el pulmón es el órgano que mas rentabilidad diagnóstica tiene en las biopsias ( ya
que puede observarse la vasculitis y la formación de granulomas ). En cuanto a la biopsia en la
vasculitis de células gigantes, a pesar de que le paciente haya iniciado tratamiento y la posibilidad
de negativización de la biopsia, ésta debe realizarse siempre que sea posible.(R/1)

347. A 67-year-old woman is referred to the gynecologist by her primary care physician.
She is concerned about irregular and intermittent vaginal bleedings for the past 2 months.
Her last menstrual period was 12 years ago. Her obstetric history is gravida 3 para 3
abortus 0. Physical examination reveals normal external genitalia, atrophic cervix and no
abnormalities in the palpation of uterus and ovaries. Ultrasound study shows a
hyperechogenic endometrium with 7-mm thickness. Which of the following options is the
most advisable and sensitive test to confirm a diagnosis?

1. 1. Cervical conization and biopsy.


2. 2. Pap smear test.
3. 3. Endometrial cytology.
4. 4. Hysteroscopy and endometrial biopsy.
Gráfico de respuestas
Comentario

Pregunta sencilla sobre hiperplasia endometrial. La respuesta correcta es la 4, se tiene que realizar
una histeroscopia y biopsia endometrial.

348. ¿Cuál de las siguientes es la causa más frecuente de coagulación intravascular


diseminada durante el embarazo?

1. 1. Diabetes gestacional.
2. 2. Placenta previa.
3. 3. Hipertensión arterial crónica asociada al embarazo.
4. 4. Desprendimiento precoz de placenta.
Gráfico de respuestas
Comentario
!
!
!

La coagulación intravascular diseminada (CID) es una alteración fisiopatológica sistémica,


trombohemorrágica, que se manifiesta en algunas situaciones clínicas bien definidas y que se
acompaña de alteraciones de laboratorio que indican activación de procoagulantes, activación
fibrinolítica, consumo de inhibidores de la coagulación y evidencias bioquímicas de daño o fallo
orgánico.

Los accidentes obstétricos son eventos que tienen asociación habitual con CID, siendo el abruptio
o desprendimiento prematuro de placenta la causa más frecuentemente involucrada (respuesta 4
correcta). La embolia de líquido amniótico con CID es el más catastrófico de los accidentes
obstétricos, y se caracteriza por la aparición aguda de insuficiencia respiratoria, colapso circulatorio
y CID. En el desprendimiento prematuro de placenta normoinserta con CID, sustancias
placentarias o tisulares, incluyendo tromboplastina, penetran a la circulación uterina y
posteriormente a la circulación sistémica, activando el sistema de la coagulación. En el feto muerto
in utero, la incidencia de CID se aproxima a un 50% si éste está retenido más de cinco semanas.

Los signos más destacados son púrpura y petequias; menos frecuentemente, se observan bullas
hemorrágicas, cianosis distal y a veces gangrena. Los pacientes con CID crónica, generalmente
desarrollan hemorragias de pequeña cuantía u ocultas y trombosis difusa en vez de hemorragias
fulminantes.

Los datos de laboratorio utilizados para el diagnóstico de CID son la medición de los productos de
degradación del fibrinógeno/fibrina (PDF), el tiempo de protrombina, la concentración de
fibrinógeno y el recuento plaquetario.

El principal esfuerzo terapéutico ha de estar enfocado a tratar la enfermedad de base que origina la
CID. En pacientes obstétricas, la eliminación del factor desencadenante mejora en forma dramática
el cuadro hemostático. Cuando existe evidencia clínica de trombosis, la terapia con heparina puede
ser útil. Si existe depleción significativa de factores de coagulación e inhibidores, la terapia de
reemplazo con plasma fresco congelado puede reducir el riesgo de hemorragia.(R/4)

!
!
!

349. RNT 40 semanas, masculino, obtenido por cesárea por sufrimiento fetal, con peso al
nacer de 3,200 g, apgar 4-8, y dificultad respiratoria. Ingresa a UCIN para su manejo; ¿Cual
serían sus indicaciones hídricas?

Solución intravenosas a 60 ml/kg/día ; Gluc 6 mg/kg/min; Na 3 mEq/kg/día; K 2


1. 1.
mEq/kg/día, Ca 200 mg/kg/día.
2. 2. Solución intravenosas a 60 ml/kg/día; Gluc 5.5 mg/kg/min; sin electrolitos.
Solución intravenosas a 80 ml/k/día ; Gluc 5.5 mg/kg/min; Na 3 mEq/kg/día; K 2
3. 3.
mEq/k/día, Ca 200 mg/kg/día.
Solución intravenosas a 100 ml/kg/día ; Gluc 6 mg/kg/min; Na 3 mEq/kg/día; K 2
4. 4.
mEq/k/día, Ca 200 mg/kg/día.
Gráfico de respuestas
Comentario

Pregunta complicada que es muy fácil fallar, ya que es memorística y es muy improbable que la
pregunten en el ENARM.

La respuesta correcta es la 2.

350. Respecto a la artritis psoriásica, es cierto que:

1. 1. La artritis siempre precede a la psoriasis.


2. 2. La psoriasis siempre precede a la artritis.
3. 3. La psoriasis precede a la artritis en la mayoría de los casos.
4. 4. La afectación cutánea y articular suelen comenzar de forma simultánea.
Gráfico de respuestas
Comentario

La artropatía es una complicación que aparece en el 5-8% de las personas que presentan psoriasis
cutáneo. Habitualmente los síntomas articulares comienzan después de años de presentar la
afectación cutánea, sin embargo ambos procesos pueden comenzar de forma simultánea o incluso
los síntomas articulares pueden preceder a la enfermedad cutánea. Haciendo muy dificil en ese
caso el diagnóstico de la enfermedad. El objetivo principal de la enfermedad es el control de las
manifestaciones articulares que como la artritis reuamtoide es una artritis erosiva y destructiva. De
hecho el manejo de ambos procesos es prácticamente similar. Las complicaciones extrarticulares
no son graves ni son evitables.(R/3)

351. Masculino que se presenta en la sala de urgencias con clara verborrea caracterizada
por volubilidad, prolijidad, asociaciones de ideas rápidas y superficiales, asonancias y
juegos de palabras. La aceleración del pensamiento hace que cambie rápidamente de un
tema a otro sin terminar la exposición, con gran distraibilidad. ¿De qué trastorno son
típicas estas alteraciones del discurso?:

1. 1. Manía.
2. 2. Depresión.
3. 3. Trastorno obsesivo.
4. 4. Demencia.
Gráfico de respuestas
Comentario
Expansividad, presión/fuga de pensamiento con taquilalia/saltos del discurso, distraibilidad y juegos
del lenguaje (son muy típicas las rimas por asonancia y los juegos de palabras) caracterizan el

!
!
!

síndrome maníaco. Si no hay delirios, aluncinaciones ni deterioro autista, no hay


esquizofrenia.(R/1)

352. Un paciente consulta por fiebre, tos y dolor torácico, después de volver de unas
vacaciones en un lujoso hotel, donde veraneaba en la playa. También refiere diarrea. Se
detecta una Tª de 38.8 ºC. Desde el punto de vista de laboratorio, destaca una leucocitosis
con desviación a la izquierda, así como unas cifras de Na+ de 131. Rx tórax: infiltrados
difusos bibasales, de carácter alveolar. En el sedimento de orina, se detectan 8-10
hematíes por campo. Se confirma el diagnóstico mediante un test de detección de
antígeno en orina. ¿Cuál de los siguientes antibióticos es de elección en la actualidad
para el tratamiento de esta infección?

1. 1. Eritromicina.
2. 2. Levofloxacino.
3. 3. Rifampicina.
4. 4. Ceftriaxona.
Gráfico de respuestas
Comentario

En esta pregunta se resumen los aspectos más importantes de la legionelosis:

- Clínica: neumonía atípica en verano+ diarrea/obnubilación+ hiponatremia.

- Diagnóstico: detección de antígeno en orina

- Tratamiento: levofloxacino(R/2)

353. En un paciente originario del Caribe


que presenta adenopatías, hipercalcemia y serología positiva para HTLV-1, ¿qué
patología sospecharía?

!
!
!

1. 1. Leucemia / linfoma de células T del adulto.


2. 2. Linfoma de Hodgkin.
3. 3. Sarcoidosis.
4. 4. Granulomatosis linfomatoidea.
Gráfico de respuestas
Comentario

La hipercalcemia es un dato característico del linfoma T del adulto, que también se asocia a
eosinofilia sanguínea y está relacionado con el retrovirus HTLV- I. Suele ser muy agresivo. Es
típico de personas originarias del Caribe y de Japón.

354. En la semana 42 de gestación, en una paciente sin embarazos previos, con un test
de Bishop de 3 y controles normales hasta ese momento, la finalización del mismo
debería ser:

1. 1. Cesárea.
2. 2. Inducción con oxitocina.
3. 3. Inducción con prostaglandinas y amniorrexis.
Inducción con prostaglandinas y cuando el cérvix sea favorable, oxitocina y
4. 4.
amniorrexis.
Gráfico de respuestas
Comentario
Para evaluar la manera de finalizar una gestación cronológicamente prolongada, es básico el test
de Bishop, que se basa en los datos de la exploración cervical de la paciente. Si estamos ante un
Bishop menor de 5, la inducción comenzará con prostaglandinas intravaginales, y posteriormetne
oxitocina con amniorrexis. Si por el contrario, el test de Bishop es mayor de 5 podemos comenzar
directamente con oxitocina y rotura de bolsa amniótica.(R/4)

355. Con respecto a la micosis fungoide, señale lo CORRECTO:

1. 1. Es una proliferación neoplásica de linfocitos T con inmunofenotipo CD8+.


2. 2. Es más frecuente en la primera década de la vida.
3. 3. Habitualmente debuta bruscamente con el estadío eritrodérmico.
La parapsoriasis en grandes placas es una dermatosis a partir de la cual puede
4. 4.
desarrollarse la micosis fungoide.
Gráfico de respuestas
Comentario

Los linfomas cutáneos no se preguntan demasiado en el ENARM. De estudiar uno, debe centrarse
en la micosis fungoide, el más preguntado y por tanto el más importante. La parapsoriasis en
grandes placas es una dermatosis crónica, bastante infrecuente. Se caracteriza por presentar
placas ovaladas de tonalidad rosada, descamativas y muy persistentes. Hoy en día, muchos
autores aceptan su relación con la micosis fungoide. Las células T de la micosis fungoide son
CD4+ (respuesta 1 falsa), es una enfermedad de ancianos (respuesta 2 falsa), el prurito es un
síntoma muy típico y la evolución es muy crónica, durante muchos años.(R/4)

356. Respecto a las alteraciones hormonales que aparecen en el síndrome del ovario
poliquístico, señale cuál de las siguientes afirmaciones es FALSA:

!
!
!

1. 1. LH/FSH > 2.5.


2. 2. Aumento de la androstendiona y testosterona.
3. 3. Aumento de la progesterona.
4. 4. Disminución del estradiol.
Gráfico de respuestas
Comentario

Esta pregunta es muy importante porque debe conocer perfectamente el síndrome de ovario
poliquístico y sus alteraciones hormonales.

Es una afección muy frecuente en que está aumentada la LH con niveles de FSH bajos o inferiores
a los normal, por lo que aumenta la relación LH/FSH.

Hay un aumento leve de andrógenos, aumento de la estrona (los andrógenos circulantes son
convertidos a estrona en la grasa periférica) y descenso del estradiol.

La LH aumentada estimula la teca, produciendo más andrógenos (también hay una


sobreproducción suprarrenal) que va a provocar obesidad, hirsutismo y anovulación.

La esterilidad es el síntoma más frecuente (73%) y se debe precisamente a la falta de ovulación,


que sería lo que provoque el aumento de progesterona, por ello es falso que esté aumentada.(R/3)

357. La causa más frecuente de amenorrea secundaria es:

1. 1. Anovulación.
2. 2. Anorexia nerviosa.
3. 3. Realización de deporte continuado.
4. 4. Gestación.
Gráfico de respuestas
Comentario

Pregunta básica que no puede fallar. La causa más frecuente de amenorrea secundaria es el
embarazo.!!!!(R/4)

358. En la asfixia neonatal se produce un aumento del cortocircuito derecha-izquierda


para mantener transitoriamente el flujo cerebral, a costa de la perfusión de otros órganos,
entre los que NO se encuentra:

1. 1. El pulmón.
2. 2. Las glándulas suprarrenales.
3. 3. Los riñones.
4. 4. El intestino delgado.
Gráfico de respuestas
Comentario

Existen órganos que pueden tolerar una isquemia relativa durante períodos de tiempo más o
menos largos, como sucede con el riñón, el hígado, el pulmón o el intestino delgado. Sin embargo,
la isquemia suprarrenal supone una amenaza inmediata para la vida, igual que la cardíaca o la
cerebral. Por ello, en situaciones como la descrita, el flujo sanguíneo tiende a redistribuirse hacia

!
!
!

cerebro, corazón y suprarrenales, descendiendo temporalmente la perfusión de otros órganos


(respuesta 4 correcta).

359. A 30-year-old woman from Cleveland city (Ohio) presents with malaise, asthenia and
skin lesions on her left forearm and hand. She denies any chills, nausea or vomiting. Vital
signs are within normal limits, except for her body temperature, which is 38.2ºC. Her skin
lesions are mostly papulopustular, purplish and well demarcated. Some microabscesses
are seen surrounding the purple rash. Which of the following is the most appropriate
treatment?

1. 1. Intravenous penicillin.
2. 2. Oral isoniazid and rifampin.
3. 3. Itraconazole.
4. 4. Amphotericin B.
Gráfico de respuestas
Comentario

Itraconazole. This patient is presenting with signs and symptoms suggestive of Blastomycosis
(remember that disease is endemic in the south-central and north-central US and in the north of
Mexico). The yeast form of the causative agent (Blastomyces spp) can be seen in wet preparations
of purulent material obtained from the skin lesions. TREATMENT: Itraconazole, fluconazole, or
amphotericin B. Although, Amphotericin B is also included in the possible answers, this drug may
be reserved for very severe infections and life-threatening situations.(R/3)

360. Hablando de la epidemiología del VHB, señalar la FALSA:

1. 1. El hombre es el único reservorio del VHB.


2. 2. La hepatitis B se considera una enfermedad de transmisión sexual.
3. 3. La transmisión vertical es de madre a su hijo.
4. 4. En zonas de baja endemia es más frecuente la transmisión vertical.
Gráfico de respuestas
Comentario

El mecanismo de transmisión vertical (madre-hijo) es un mecanismo muy frecuente en países de


baja endemia respecto a otros mecanismos. Asimismo existe un mecanismo de transmisión sexual
y el sanguíneo, siendo el hombre el único reservorio que existe.(R/4)

361. El método utilizado para detectar el cáncer de la mama en etapa preclínica es:

1. 1. Mastografía.
2. 2. Termografía de mama.
3. 3. Ultrasonido mamario.
4. 4. Tomografía de mama.
Gráfico de respuestas
Comentario

Pregunta de un tema muy importante para el ENARM. El método usado para el screening del
cáncer de mama es la mastografía.

En mujeres jóvenes, de menos de 40 años, es más eficaz el ultrasonido. De todos modos, los
programas de screening

!
!
!

suelen ir dirigidos a mujeres de más de 40 años; generalmente el screening consiste en


mamografías bianuales a mujeres entre 40 y 70 años.(R/1)

362. ¿Cuál de los siguientes NO es cierto con relación a la fibrosis quística?:

1. 1. Su herencia es autosómica recesiva.


2. 2. El gen responsable se encuntra localizado en el brazo largo del cromosoma 7.
La afectación respiratoria durante el primer año de vida es signo de mal
3. 3.
pronóstico.
4. 4. Nunca se manifiesta por hemoptisis.
Gráfico de respuestas
Comentario

Esta pregunta de fibrosis quistica (FQ) es importante saber reconocer el caso.

Sera típico que comenten a un RN sin expulsión de meconio en las primeras 48 hrs de vida.

La alteración se encuentra en el gen que codifica el regulador de la conductancia de


transmenbrana, y está localizado en el brazo largo del cromosoma 7. Se hereda como rasgo
autosómico recesivo y la mutación mas prevalente es la F508.

El principal factor pronostico es la afectación pulmonar por lo que cuanto antes se inicie peor
evolución. La hemoptisis leve suele ser bastante frecuente en el contexto de exacerbaciones
respiratorias y sobre todo a partir delos 10 años cuando se desarrollan bronquiectasias. La
hemoptisis masiva aparece en el 0.2 % de los niños y en el 1.7 de los mayores de 18 siendo
necesario radiografía y estudio de coagulación antes de iniciar tratamiento. La fostatasas,
peptidasas y otras enzimas intestinales en liquido amniótico están disminuidas.

La respuesta de la pregunta es sencilla. "NUNCA...." y nos piden la incorrecta. Este tipo de


aseveraciones son muy restrictivas y a menudo erróneas. En el examen y en la medicina real.(R/4)

363. La densitometría para la detección precoz de la osteoporosis está indicada en


mujeres:

1. 1. Nulíparas.
2. 2. De más de 40 años.
3. 3. Ooforectomizadas antes de los 35 años.
4. 4. Hipertensas.
Gráfico de respuestas
Comentario

Los estrógenos son hormonas que se encargan de manetener una correcta mineralización ósea.
En el caso de las mujeres ooforectomizadas, el déficit de estrógenos puede originar una
osteoporosis que debe de ser controlada estrictamente. Por este motivo se recomiendan
densitometrias en estas pacientes.(R/3)

364. Con relación al disgerminoma ovárico, señale la afirmación INCORRECTA:

1. 1. Es bilateral en más del 20% de los casos.


2. 2. Secreta hCG.

!
!
!

3. 3. Es un tumor de los cordones sexuales.


4. 4. Predomina en mujeres menores de 30 años.
Gráfico de respuestas
Comentario

Pregunta fácil sobre los disgerminomas ováricos.

La respuesta incorrecta es la 3, ya que este tumor no pertenece a los tumores de los cordones
sexuales sino a los tumores germinales de ovario.

Es el tumor maligno más frecuente de los germinales y el maligno más habitual en pacientes
menores de 30 años. Es radiosensible, aunque el tratamiento es por cirugía. El resto de opciones
son correctas.(R/3)

365. An otherwise healthy 3-year-old boy is brought to the emergency department with a
sudden onset of pallor, asthenia, fever and malaise. Laboratory studies reveal
leukocytosis with lymphocytosis and neutropenia; normocytic and normochromic
anemia; and thrombocytopenia. Which of the following is the most likely diagnosis?

1. 1. Acute lymphoblastic leukemia.


2. 2. Congenital bone marrow aplasia.
3. 3. Acquired bone marrow aplasia.
4. 4. Acute myeloid leukemia.
Gráfico de respuestas
Comentario

Las leucemias agudas producen una clínica relacionada con el déficit de las tres series: palidez y
astenia (anemia), trombocitopenia y mayor susceptibilidad a la infección (fiebre).

La rapidez en la aparición del cuadro también habla a favor de este diagnóstico. Dado que en la
BH encontramos neutropenia y linfocitosis, lo más probable es que la leucemia aguda sea
linfoblástica (respuesta 1 correcta), no mieloblástica.

366. A 37-year-old woman 4 pregnancy and 4 births, comes to the ER due to severe
metrorrhagia with expulsion of grapelike cluster into the vagina. A pregnancy test is
performed and the result is positive. On examination, the cervix is closed and blood
traces are found. Uterine size is large for gestational age. Pelvic ultrasound reveals a
"snowstorm" appearance with no gestational sac or fetus present. Suction curettage is
performed. Which of the following is INCORRECT:

!
!
!

1. 1. B-hCG determination, pelvic examination and chest x-ray.


In 90% of cases, the process is resolved successfully, the beta-HCG is negative
2. 2.
after 6-10 weeks.
3. 3. The risk of obstetric complications in a future pregnancy is not increased.
If a new trophoblastic disease occurs in a subsequent pregnancy in 33% of cases
4. 4.
will be the same histological type.
Gráfico de respuestas
Comentario

Tema importante en el ENARM. Se presenta un caso clínico de enfermedad trofoblástica, en la que


el tratamiento inicial es el legrado por aspiración. Para el seguimiento, utilizamos la beta- HCG y
también la ecografía ginecológica. Debe descartar enfermedad extragenital y para ello se realiza
placa de tórax. En el 90% de los casos, el proceso se resuelve satisfactoriamente, negativizándose
la beta-HCG a las 6-10 semanas. Tras este evento, las nuevas gestaciones no tienen
incrementado el riesgo de complicaciones obstétricas, pero siempre habrá que hacer estudio
histológico tras el parto, para excluir enfermedad recurrente. En caso de ocurrir una nueva mola,
en el 68-80% de los casos será del mismo tipo histológico.(R/4)

367. En la evaluación del niño que tiene estatura corta, la información diagnostica más
importante es obtenida de:

1. 1. Edad ósea.
2. 2. Historia ampliada del crecimiento en la familia.
3. 3. Curva de crecimiento.
4. 4. Dosaje de T4 y TSH séricas.
Gráfico de respuestas
Comentario

El indicador más sensible para detectar precozmente las alteraciones del crecimiento es la
velocidad de crecimiento (respuesta 3 correcta).

368. En relación al dioptrio ocular pueden hacerse las siguientes afirmaciones, EXCEPTO
que:

1.! ! 1.! De!las!lentes!que!lo!integran!la!córnea!es!la!de!mayor!potencia.!


En!el!ojo!emétrope!los!rayos!luminosos!procedentes!de!un!punto!en!el!infinito!
2.! ! 2.!
forman!su!imagen!en!la!retina.!
3.! ! 3.! La!imagen!retiniana!es!invertida!respecto!al!objeto!real!y!de!tamaño!normal.!
4.! ! 4.! La!imagen!percibida!es!derecha!respecto!al!objeto!y!de!tamaño!normal.!
Gráfico de respuestas
Comentario

Pregunta muy difícil. Sin embargo debemos tener claro que las opciones 2 y 3 son correctas: la
córnea tiene una potencia de 43 dioptrías (el cristalino sólo aporta 17) y lo ideal (ojo emétrope) es
que la imagen procedente del infinito se forme en la retina. Un sistema óptico se denomina
centrado cuando todas sus superficies ópticas tienen simetría de revolución respecto a un eje
común que se denomina eje óptico, por lo cual todo rayo que pasa por él no sufre desviación
alguna. La imagen retiniana sí es invertida pero de tamaño menor al objeto (en la retina no caben
imágenes de casas). La imagen percibida es la ya procesada por lo que es derecha y de tamaño
similar al objeto.(R/3)

!
!
!

369. Al hacer un estudio mediante una microtoma de sangre fetal se obtiene un valor de
pH de 7.23. Con estos resultados, la actitud más CORRECTA sería:

Se trata de un pH claramente patológico, y es necesaria una extracción inmediata


1. 1.
del feto.
Se trata de un valor de pH prepatológico, y es necesario repetir la prueba en unos
2. 2.
15-20 min.
Este valor de pH indica claramente un sufrimiento fetal, y es necesario hacer una
3. 3.
amnioscopia en ese momento.
Se trata de un pH prepatológico, y debería repetirse la prueba en unos 40-60
4. 4.
minutos.
Gráfico de respuestas
Comentario

Un valor en el pH fetal de 7.23 (7.21-7.25) estaría en el rango prepatológico (respuesta 2 correcta).

En caso de que el pH fuese de 7.20 o menor, habría que poner fin inmediato a la gestación. Ante
un valor en este rango, lo que debemos hacer a continuación es dejar a la paciente en decúbito
lateral izquierdo durante 15-30 minutos, repitiendo después la microtoma, para ver si mejora o no.
Si el registro no mejora, estaría indicado finalizar la gestación.(R/2)

370. Paciente de 56 años, en tratamiento por una infección del tracto urinario con
nitrofurantoína. Comienza con tos seca, fiebre, escalofríos y disnea. En la radiología de
tórax aparecen infiltrados parcheados y derrame pleural. Si realizamos una
toracocentesis, con mucha probabilidad nos encontraremos con:

1. 1. Líquido con predominio de polimorfonucleares.


2. 2. Predominio linfocítico.
3. 3. Un trasudado.
4. 4. Un derrame eosinófilo.
Gráfico de respuestas
Comentario

La nitrofurantoína produce una reacción aguda pulmonar caracterizada por eosinifilia, fiebre
escalofríos, tos y disnea; presenta patrón alveolar en bases y derrame pleural eosinofílico en 1/3
de los casos. En las fases crónicas puede producir daño intersticial difuso. La presencia de
derrame pleural eosinofílico debe orientar a enfermedad por fármacos o parásitos, o a la presencia
de aire o sangre en le espacio pleural. La presencia de eosinofilia pleural prácticamente descarta la
TB pleural y disminuye la probabilidad de neoplasia.(R/4)

371. Mujer de 45 años hipotiroidea, bien controlada con levotiroxina, que refiere diarrea
acuosa sin productos patológicos, de inicio súbito hace 3 meses, con pérdida inicial de
peso, que posteriormente se ha estabilizado, presentando ahora diarrea líquida, con
frecuencia nocturna. Realizamos exámenes de laboratorio, un tránsito intestinal, un
enema opaco y una colonoscopía que son normales. En la biopsia de colon que ha
tomado durante la endoscopía, esperas encontrar:

1. 1. Mucosa de colon morfológicamente normal.


Infiltrado inflamatorio mucoso con microabscesos, invasión de lámina propia,
2. 2.
submucosa y muscular con formación de granulomas no caseificantes.
3. 3. Agregados de histiocitos con cuerpos de inclusión de Michaelis-Gutmann.

!
!
!

Infiltrado inflamatorio linfoplasmocitario en la lámina propia, sin ulceraciones y


4. 4.
engrosamiento de la capa subepitelial de colágeno.
Gráfico de respuestas
Comentario

La colitis microscópica o linfocítica y la colitis colágena son dos enfermedades de etiología


desconocida con inflamación de la mucosa colónica, pero sin ulceración. Algunos autores
consideran que los dos trastornos pertenecen al mismo espectro de enfermedad y se han descrito
algunos casos de evolución de una microscópica a colágena.

El principal síntoma de ambas es la diarrea acuosa crónica. Es frecuente la pérdida de peso


inicialmente, pero tras unas semanas o meses luego se estabiliza. La diarrea con frecuencia es de
intensidad fluctuante. No suele haber sangrado. No hay esteatorrea. La microscópica se da igual
en ambos sexos, pero la colágena es más frecuente en mujeres. Se asocian a muchas otras
enfermedades como: artritis, hipotiroidismo, hipertiroidismo, diabetes mellitus, hepatitis crónica
activa, celíaca y otras. Al hacer la colonoscopía, la mucosa suele tener un aspecto macroscópico
normal. El diagnóstico es histológico. En la colitis microscópica hay un infiltrado inflamatorio mixto,
agudo y crónico con neutrófilos y células plasmáticas, pero sin ulceraciones ni microabscesos. En
la colágena además hay un engrosamiento de la capa subepitelial de colágeno.

No hay estudios que permitan concluir cuál es el mejor tratamiento para estos pacientes. Se
recomienda iniciarlo con antidiarreicos. Si no hay mejoría, se pasa a antiinflamatorios, comenzando
con aminosalicilatos, y cambiando a esteroides si no hay mejoría con aquellos. Aunque el
pronóstico es muy variable, en general es bueno y no hay evidencia de que predispongan a
cáncer.(R/4)

372. La afectación más frecuente en la polineuritis diabética es:

1. 1. Proximal, bilateral, simétrica, dolorosa y motora.


2. 2. Distal, bilateral, simétrica y motora.
3. 3. Distal, bilateral, simétrica y sensitiva.
4. 4. Proximal, bilateral, simétrica y sensitiva.
Gráfico de respuestas
Comentario

En la diabetes mellitus puede aparecer una gran variedad de alteraciones del sistema nervioso
periférico, que globalmente se clasifican en polineuropatías simétricas y asimétricas. No obstante,
esta pregunta hace referencia a la polineuritis diabética, que es la forma más frecuente, así que
vamos a centrarnos en ella.

La polineuropatía sensitiva distal suele ser, como dice la respuesta 3, distal, bilateral, sensitiva y
simétrica. Afecta preferentemente a fibras gruesas, con parestesias e hipoestesia en guante y
calcetín. También puede producir pérdida de la sensibilidad vibratoria y arreflexia distal. En algunos
pacientes, se sobreañade la afectación de fibras de pequeño calibre, en cuyo caso aparece
sensación urente en los pies, con un característico empeoramiento nocturno. Existen otras
variantes de este cuadro, como la forma pseudotabética y la pseudosiringomiélica, pero son más
infrecuentes.(R/3)

373. Paciente de 76 años de edad, paridad 3-0-1-3, con menopausia a los 52 años. Relata
que desde hace al menos 4-5 años presenta prurito vulvar de intensidad variable que ha
sido tratado algunas veces mediante automedicación y otras por indicación de su médico

!
!
!

de familia con preparados tópicos (cremas y lavados). El prurito ha evolucionado así con
periodos intermitentes pero desde hace 3-4 meses nota además una pequeña tumoración
en el labio mayor izquierdo de la vulva y de la que se producen pérdidas serohemáticas
al roce desde hace unos días; por ello consulta al ginecólogo. Al interrogatorio refiere
disuria ocasional y el estado general es bueno. ¿Cuál es el diagnóstico más probable en
esta paciente?

1. 1. Herpes genital.
2. 2. Enfermedad de Paget de la vulva.
3. 3. Granuloma crónico de la vulva.
4. 4. Carcinoma vulvar escamoso.
Gráfico de respuestas
Comentario

El cáncer escamoso vulvar es una neoplasia relativamente rara que suele aparecer entre los 65 y
75 años, siendo excepcional antes de los 30 años. El síntoma más frecuente es el prurito vulvar
que suele ser de larga evolución, como es el caso que nos presentan. Se asocia a tumoraciones
en genitales externos y ulceraciones junto a otros síntomas tardíos como la secreción
serohemática o dolor por sobreinfección o extensión a tejidos vecinos.

Los síntomas urinarios, como la disuria que refiere la paciente del caso clínico, aparecen cuando
hay extensión al meato uretral. La lesión se localiza de forma más frecuente en los labios mayores,
seguida de los labios menores.(R/4)

374. An 87-year-old male is admitted to hospital for acute cholecystitis. His past medical
history is significant for chronic bronchitis and heart failure. On the fourth day of
admission, the patient continues with fever, abdominal pain and persistent leukocytosis.
He is currently in nil per os (NPO) status and his treatment includes intravenous fluid
therapy and piperacillin/tazobactam. Which of the following is the best next step in
management?

1. 1. Emergent cholecystectomy.
2. 2. Biliary drainage by percutaneous cholecystostomy.
3. 3. Change his antibiotic therapy to metronidazole / cefotaxime.
4. 4. Add gentamicin to his current treatment.
Gráfico de respuestas
Comentario

El tratamiento inicial de la colecistitis aguda consiste en dieta absoluta, aporte de líquidos i.v.,
analgésicos y antibióticos ajustados al antibiograma. El tratamiento definitivo es quirúrgico. Existe
controversia sobre el momento en el que proceder a la cirugía. Una de las actitudes más
aceptadas es el tratamiento conservador (que suele ser efectivo en el 75% de los casos), enfriando
la colecistitis y haciendo cirugía diferida a las 4-6 semanas.

Inicialmente, si tras 48 horas de tratamiento conservador el paciente presenta una evolución


desfavorable, se procede a la cirugía precoz.

Sin embargo, en pacientes con alto riesgo quirúrgico (como en este caso), este procedimiento
implica una morbimortalidad elevada. La colecistostomía percutánea es un procedimiento
mínimamente invasivo que no requiere anestesia general y que mediante la descompresión

!
!
!

vesicular y drenaje de la bilis permite la resolución del cuadro de colecistitis en pacientes no


candidatos a cirugía (respuesta 2 correcta).

375. En una paciente sometida a tratamiento de inducción de la ovulación con citrato de


clomifeno, ¿en caso de que no funcione dicho fármaco, cuál se podría utilizar?:

1. 1. Metformina.
2. 2. Danazol.
3. 3. Progestágenos.
4. 4. Gonadotropinas.
Gráfico de respuestas
Comentario

Respuesta 4 correcta. En caso que el clomifeno no funcione, está indicado el uso de


gonadotropinas.

Citrato de clomifeno: fue el primer fármaco utilizad o para inducir la ovulación. Actúa por unión a los
receptores estrogénicos del hipotálamo bloqueando la retroalimentación negativa del estradiol y
activa el mecanismo neuroendocrino para la secreción de GnRH . Se emplea para la inducción de
la ovulación en pacientes con oligoanovulación y niveles de estrógenos normales (ovario
poliquístico).

Gonadotropinas: obtenidas por purificación a partir de la orina de mujeres postmenopáusicas (hM


G ) o por recombinación genética (rFSH, rLH), se utilizan para la estimulación ovárica en técnicas
de reproducción asistida ya que recluían folículos y estimulan su crecimiento al mismo tiempo que
facilitan su maduración.(R/4)

376. ¿Cuál de las siguientes dermatosis paraneoplásicas constituye a menudo un signo


precoz de carcinoma pancreático?:

1. 1. Hipertricosis lanuginosa adquirida.


2. 2. Acantosis nigricans.
3. 3. Dermatomiositis.
4. 4. Tromboflebitis migratoria.
Gráfico de respuestas
Comentario

Las dermatosis paraneoplásicas son un tema absolutamente extraordinario en el ENARM, y su


estudio no es nada fácil. Son un tema poco rentable, así que céntrsee en lo más característico de
cada una y aprenda solamente a qué tumor se asocian.

El síndrome de Trousseau es una tromboflebitis superficial que se asocia sobre todo al cáncer de
páncreas. Aparece como una o varias flebitis superficiales en extremidades, que cursan por brotes,
cediendo en semanas y apareciendo luego en otras localizaciones. También se ha descrito
asociado a otros tumores, a enfermedad de Hodgkin y a infecciones por rickettsias. En este último
caso, probablemente sea debido a la lesión que producen este tipo de bacterias en el endotelio de
pequeños vasos.(R/4)

377. Señale la respuesta CORRECTA:

!
!
!

1. 1. La dermatitis seborreica se inicia a los 3 meses de edad.


2. 2. La dermatitis atópica se inicia a los 3 meses de edad.
3. 3. La dermatitis del pañal es exclusiva del lactante.
4. 4. La dermatitis atópica del lactante afecta flexuras.
Gráfico de respuestas
Comentario

La respuesta correcta es la 2, ya que la dermatitis atópica se inicia a los 3 meses de edad. La


dermatitis seborreica se inicia posteriormente, mientras que la dermatitis del pañal no es exclusiva
al lactante.

378. Todo lo que sigue es normal en un recién nacido, EXCEPTO:

1. 1. El cierre del conducto arterioso.


2. 2. El cierre de la arteria umbilical.
3. 3. Mayor proporción de agua extracelular que en edades posteriores.
4. 4. Mayor capacidad para concentrar la orina que a edades posteriores.
Gráfico de respuestas
Comentario

El riñón del recién nacido, como bien pueden suponer, tiene una cierta inmadurez funcional; de
este modo, se muestra más torpe que el riñón de otras edades para desembarazarse de un exceso
de solutos, pues el túbulo no es capaz de concentrar la orina debidamente. El resto de las
opciones enumeradas en esta pregunta (1, 2, y 3) son correctas.(R/4)

379. A 74-year-old female comes to the emergency department with several episodes of
metrorrhagia in the past two weeks. Her past medical history is significant for infiltrating
ductal breast cancer diagnosed 5 years ago. She underwent mastectomy and
lymphadenectomy and received 6 cycles of chemotherapy; Tamoxifen therapy was then
prescribed, which she is still currently taking. She has had regular annual follow-up visits,
except for the last year. No abnormalities were found until now. Mark the wrong answer
regarding this case:

Since episodes of metrorrhagia are self-limited, watchful waiting approach


1. 1.
without any further test is advisable.
2. 2. Her current medical problem may be a side effect of her medication.
3. 3. The ultrasound can show thickened endometrium with multicystic appearance.
4. 4. The diagnosis can be obtained by performing hysteroscopy and multiple biopsies.
Gráfico de respuestas
Comentario

Pregunta importante sobre el manejo de complicaciones del tratamiento coadyuvante del cáncer de
mama. Tenemos el caso clínico de una paciente con cáncer de mama en tratamiento con
tamoxifeno y presenta metrorragias. Recordemos que uno de los efectos secundarios más
importantes del tamoxifeno es la hiperplasia de endometrio, que puede debutar con metrorragia y
aumenta el riesgo para desarrollar cáncer de endometrio (opción 2).

Lo primero que debemos hacer es explorar a la paciente y realizar un ultrasonido, que nos dará
muchas veces un patrón endometrial multi- microquístico.

!
!
!

Pero la certeza diagnóstica, y la prueba más importante es la histeroscopia, con la que podemos
hacer una biopsia dirigida

Así pues, una cosa debemos tener claro, y es que ante una metrorragia en una paciente
postmenopáusica (más aun, si está en tratamiento con THS o tamoxifeno, fármacos que estimulan
el endometrio), no podemos quedar indiferentes y no hacer nada, aunque sean episodios aislados,
con lo que la respuesta incorrecta es la 1.

380. El aumento patológico de alfafetoproteína en el suero de la gestante puede asociarse


a:

1.! ! 1.! Hipertiroidismo!materno.!


2.! ! 2.! Hepatopatía!materna!grave.!
3.! ! 3.! Malformaciones!fetales!del!SNC.!
4.! ! 4.! Incompatibilidad!Rh.!
Gráfico de respuestas
Comentario

Pregunta importante sobre el screening bioquímico del segundo trimestre. La alfafetoproteína está
elevada característicamente en madres con fetos portadores de defectos del tubo neural (opción 3
correcta), como el mielomeningocele. Conviene aclarar que, en el embarazo gemelar, el nivel de
alfafetoproteína es mayor que en la gestación única (lógicamente), pero no estará en rango
patológico. Recuerde que la alfafetoproteína está muy descendida en la gestante con feto con
síndrome de Down, y que la podemos encontrar elevada en mujeres con un tipo de tumor de ovario
germinal como el del seno endodérmico (tumor de muy mal pronóstico).(R/3)

381. Señale la respuesta FALSA en relación a los fármacos antituberculosos:

La isoniacida se elimina por el riñón, por lo que la dosis debe reducirse en


1. 1. pacientes con insuficiencia renal, en tanto que la rifampicina, al eliminarse por la
bilis, no requiere dicha reducción de dosis.
El efecto adverso más importante es la hepatotoxicidad, que es más frecuente en
2. 2.
pacientes ancianos, alcohólicos o hepatópatas.
Durante el tratamiento con isoniacida y rifampicina siempre se deben realizar
3. 3. controles muy periódicos de transaminasas, lo cual es obligado en todos los
pacientes, tengan o no síntomas de hepatitis tóxica asociada.
La isoniacida puede ocasionar una neuropatía periférica que se previene
4. 4.
administrando piridoxina.
Gráfico de respuestas
Comentario
Los efectos secundarios de los fármacos antituberculosos son múltiples en algunos casos
coincidiendo entre ellos. Este es el caso de la hepatotoxicidad de la isoniacida, de la rifampicina y
de la pirazinamida. La rifampicina lo que hace es favorecer el efecto hepatotóxico de la isoniacida.
La American Thoracic Society recomienda determinar niveles séricos de transaminasas al inicio del
tratamiento y a intervalos mensuales en mayores de 35 años. Sin embargo esto cada vez está más
discutido y sí que se considera imprescindible realizar esta monitorización en pacientes
sintomáticos. Se indica retirada del tratamiento en caso de que la elevación persista por encima de
cinco veces el valor normal. El resto de opciones son ciertas. Descartamos la acción inductora del
rifampicina porque interfiere con uno de los citocromos P450.(R/3)

382. Señale la respuesta FALSA en relación a los fármacos antituberculosos:

!
!
!

La isoniacida se elimina por el riñón, por lo que la dosis debe reducirse en


1. 1. pacientes con insuficiencia renal, en tanto que la rifampicina, al eliminarse por la
bilis, no requiere dicha reducción de dosis.
El efecto adverso más importante es la hepatotoxicidad, que es más frecuente en
2. 2.
pacientes ancianos, alcohólicos o hepatópatas.
Durante el tratamiento con isoniacida y rifampicina siempre se deben realizar
3. 3. controles muy periódicos de transaminasas, lo cual es obligado en todos los
pacientes, tengan o no síntomas de hepatitis tóxica asociada.
La isoniacida puede ocasionar una neuropatía periférica que se previene
4. 4.
administrando piridoxina.
Gráfico de respuestas
Comentario
Los efectos secundarios de los fármacos antituberculosos son múltiples en algunos casos
coincidiendo entre ellos. Este es el caso de la hepatotoxicidad de la isoniacida, de la rifampicina y
de la pirazinamida. La rifampicina lo que hace es favorecer el efecto hepatotóxico de la isoniacida.
La American Thoracic Society recomienda determinar niveles séricos de transaminasas al inicio del
tratamiento y a intervalos mensuales en mayores de 35 años. Sin embargo esto cada vez está más
discutido y sí que se considera imprescindible realizar esta monitorización en pacientes
sintomáticos. Se indica retirada del tratamiento en caso de que la elevación persista por encima de
cinco veces el valor normal. El resto de opciones son ciertas. Descartamos la acción inductora del
rifampicina porque interfiere con uno de los citocromos P450.(R/4)

383. A 20-week-old infant is brought to the pediatrician's office for a routine visit. His
mother says that the child is excessively sleepy, with poor muscle tone and he often
makes a low or hoarse sound when he cries. Physical examination shows a well-
nourished infant, but macroglossia is remarkable and an umbilical hernia is present. The
child is often constipated. What is the most likely diagnosis?

1. 1. Congenital myasthenia.
2. 2. Congenital hypothyroidism.
3. 3. Werdnig-Hoffman syndrome.
4. 4. Duchenne muscular dystrophy.
Gráfico de respuestas
Comentario
Congenital hypothyroidism. Hypothyroidism does not give specific symptoms at birth but a prompt
diagnosis and treatment are important to prevent irreversible sequelae such as mental retardation.
Umbilical hernia and hypoactivity are the most common signs.(R/2)

384. Dentro de la sintomatología causada por una oclusión completa de la arteria


cerebelosa posteroinferior izquierda, NO se encuentra:

1. 1. Hipoestesia facial izquierda.


2. 2. Paresia de miembros derechos.
3. 3. Ataxia de miembros izquierdos.
4. 4. Hipoestesia de miembros derechos.
Gráfico de respuestas
Comentario

!
!
!

Pregunta de dificultad moderada de un tema importante para el ENARM como es el EVC. La


oclusión de la arteria cerebelosa posteroinferior (PICA) produce un síndrome bulbar lateral o
síndrome de Wallenberg. Clínicamente es caracteriza por:

1) síndrome vertiginoso con naúseas y vómitos por afectación de los núcleos vestibulares,

2) hipoestesia facial ipsilateral por afectación del núcleo del trigémino,

3) hipoestesia corporal contralateral por afectación del tracto espinotalámico,

4) ataxia cerebelosa ipsilateral secundaria a la afectación del pedúnculo cerebeloso inferior y


cerebelo,

5) síndrome de Horner ipsilateral,

6) disartria y disfagia,

7) diplopia (respuesta 2 incorrecta).

385. Paciente de 29 años, fumador importante y asintomático desde el punto de vista


respiratorio. En una espirometría de rutina realizada en su empresa se aprecia: CVF 4 l
(112%), VEF1 322 l (98%), VEF1/CVF 82%, FEF 25-75% 1.7 l (40%). ¿Qué le recomendaría?:

Dejar de fumar, pues la obstrucción de las vías pequeñas puede desaparecer por
1. 1.
completo.
2. 2. Da igual dejar de fumar, pues ningún cambio ya establecido se puede recuperar.
3. 3. Iniciar tratamiento con beta-2-agonistas.
4. 4. Realizarse una gasometría.
Gráfico de respuestas
Comentario

Pregunta de dificultada moderada que debe saber manejar bien, ya que es importante manejar con
soltura los datos obtenidos de una espirometría. Aunque falle la respuesta no debe dudar en
reconocer el caso que se nos presenta.

En principio la espirometría es normal, pero nos debe llamar la atención dos datos: disminución del
FEF25-75 o flujo mesoespiratorio y un leve aumento de la CVF. El FEF25-75 es la medida más
sensible de la obstrucción precoz de las vías respiratorias, sobre todo las de pequeño tamaño, por
lo que suele ser la primera alteración detectada en fumadores. Por ello vemos que el tabaco ya
está haciendo daño en el árbol bronquial del paciente y en él, es vital que deje de fumar, ya que
estos cambios pueden revertir en estadios precoces. El aumento de la CVF, aunque está en los
límites de la normalidad, puede ser explicada por un comienzo de atrapamiento aéreo.(R/1)

!
!
!

386. Niño de 18 meses


derivado por su pediatra por estancamiento ponderal desde los 12 meses junto con dolor
abdominal y diarrea crónica. Lactancia materna hasta el año de vida. Diversificación
alimentaria sin incidencias con introducción del gluten al 9º mes y leche de vaca al año
de vida. En los exámenes de laboratorio destaca una Hb de 10 gr/dl con un VCM 75fl y
datos de ferropenia con seropositividad a títulos altos de los anticuerpos antigliadina,
antiendomisio y antitransglutaminasa. Ante su sospecha indica la realización de una
endoscopia digestiva (imagen nº ##) con toma de biopsias. Señale la respuesta FALSA:

Todavía no se conoce el mecanismo molecular preciso por el cual se produce la


1. 1.
enfermedad, aunque hay diversas teorías.
El diagnóstico de sospecha se establece sobre la base de datos clínicos y de
2. 2.
laboratorio, pudiendo ser necesario realizar una biopsia intestinal.
Los anticuerpos antiendomisio y antitransglutaminasa se relacionan más
3. 3.
estrechamente con el daño de la mucosa que los antigliadina.
4. 4. El HLA más frecuentemente implicado es el DQ5.
Gráfico de respuestas
Comentario

El cuadro clínico presentado sugiere la presencia de una celiaquía como causa de diarrea crónica
y restraso ponderal. Los títulos altos de anticuerpos (más sensibles y específicos los anticuerpos
antitransglutaminasa) junto con el inicio de dieta con gluten apoyan el mismo, así como la anemia
ferropénica crónica por descamación intestinal epitelial. Puede ser necesaria una biosia
diagnóstica que mostrará un infiltrado linfocitario en la lámina propia, posteriormente hiperplasia de
criptas y atrofia vellositaria en el estadio final. La predisposición individual la marca la presencia de
HLA DQ2 y/o DQ8.

387. Tras la exclusión dietética del gluten de la dieta, el paciente experimenta una rápida
mejoría en su sintomatología con recuperación de los percentiles de peso y talla. Con
respecto al seguimiento ulterior de este paciente señale la respuesta FALSA:

La lesión histológica puede ser parcheada, con zonas preservadas, por lo que se
1. 1.
aconseja la toma de múltiples biopsias.
Tras exclusión alimentaria los anticuerpos implicados suelen descender a cifras
2. 2.
normales lo cual apoya el diagnóstico.
3. 3. Es imprescindible no excluir el gluten hasta la realización de la primera biopsia.
Existen medicaciones hoy en día que han demostrado claramente evitar la
4. 4. inflamación tras la ingesta de cantidades importantes de gluten, como por ejemplo
el larazótido.

!
!
!

Gráfico de respuestas
Comentario

El único tratamiento hasta la fecha efectivo es la dieta exenta de gluten de por vida.

Recuerde que el consumo de gluten mantiene activa la enfermedad, y la consecuente elevación de


anticuerpos secundaria. En los casos dudosos , puede estar indicada la realización de una
segunda biopsia tras dieta sin gluten para confirmar la normalización histológica. Un aspecto
importante es el hecho de que frecuentemente se presenta de forma parcheada por lo que la
biopsia debe ser en múltiples territorio de la unión duodeno-yeyunal.(R/4)

388. Con respecto a la neoadyuvancia en el cáncer de recto, indique la respuesta


INCORRECTA:

1. 1. Está indicada cuando el tumor afecta a las vesículas seminales.


2. 2. Está indicada cuando existen adenopatías positivas.
Si se da quimioterapia preoperatoriamente, ya no hay que darla en el
3. 3.
postoperatorio.
En algunos casos permite hacer resecciones anteriores en pacientes en los que, de
4. 4.
entrada, estaba indicada una amputación abdominoperineal.
Gráfico de respuestas
Comentario
Parece existir evidencia suficiente para recomendar la neoadyuvancia (es decir el tratamiento
médico con radio o quimioradioterapia) en el cáncer de recto en estadio II (T3: afectación fascia
propia del recto y T4: afectación de estructuras vecinas) y III (adenopatías positivas con cualquier
T). El efecto fundamental de la neoadyuvancia lo realiza la radioterapia, disminuyendo las recidivas
locales. En este tumor la quimioterapia preoperatoria, que no siempre se da, tiene un efecto
“sensibilizador” para la radioterapia, es decir la hace más efectiva. Por eso, el haber recibido
quimioterapia en el preoperatorio, en el recto, a diferencia de otros cánceres como el de mama, no
evita su administración de forma adyuvante si está indicada (B2 con factores de riesgo o C y D de
la clasificación de Dukes). La neoadyuvancia permite, en caso de respuestas, realizar Cirugías
radicales (R0: sin restos de enfermedad) y aumenta la tasa de conservación de esfínteres, siendo
en la actualidad poco frecuente tener que realizar una amputación abdomino perineal (sólo se hace
si no hay respuesta, si hay afectación esfinteriana franca, incontinencia previa del paciente o
tumores muy bajos que no permiten margen libre distal).(R/3)

389. Secundípara en la 39ª semana de gestación que comienza con dinámica de parto.
Entre sus antecedentes obstétricos destaca una cesárea anterior por una desproporción
pelvicocefálica. El embarazo actual ha transcurrido con normalidad. Se hace un registro
cardiotocográfico que demuestra una frecuencia cardíaca de 110 lpm. La amnioscopía
nos muestra un líquido teñido de meconio. Se realiza una microtoma de sangre fetal, con
un resultado de un pH de 7.24. ¿Cuál de las siguientes actuaciones es la MENOS útil?

1. 1. Colocar a la madre en decúbito lateral.


2. 2. Repetir la microtoma en unos 15 minutos.
3. 3. Suspender gotero oxitócico, si se estaba utilizando.
4. 4. Administración de glucosa a la madre.

!
!
!

Gráfico de respuestas
Comentario

Nos presentan un pH fetal prepatológico (7.20 a 7.25). Esto podría deberse al comienzo de un
sufrimiento fetal. Colocar a la madre en decúbito lateral, impidiendo así la compresión de la cava,
mejorará la hemodinámica uterina. En caso de estar utilizando un oxitócico, efectivamente habría
que retirarlo, porque incrementan la tensión parietal en el músculo uterino, dificultando el
intercambio útero placentario al comprimir los vasos mediante las contracciones. También es lógico
repetir la microtoma en 15 minutos mientras se toman estas medidas, para comprobar la evolución
del pH fetal.

Lo que no tiene sentido es administrar glucosa a la madre. No es que se trate de una actitud
manifiestamente perjudicual, sino únicamente fuera de indicación, no habría motivos para
hacerlo.(R/4)

390. De las siguientes edades medias, ¿cuál es FALSA?:

1. 1. Cáncer germinal de ovario a los 20 años.


2. 2. Menopausia a los 51 años.
3. 3. Cáncer de cérvix a los 40 años.
4. 4. Menarquia tardía a los 18 años.
Gráfico de respuestas
Comentario

Pregunta no muy importante que puede responder más o menos fácilmente. Debe conocer que la
amenorrea primaria se define como al ausencia de la menstruación cuando la mujer ha cumplido
los 16 años (no los 18). La causa más frecuente de amenorrea primaria es la disgenesia gonadal
(de ellas la más frecuente es el síndrome de Turner), le siguen las alteraciones müllerianas e
himenales y en tercer lugar el síndrome de Morris (feminización testicular).(R/4)

391. ¿Cuál de los siguientes hallazgos ecocardiográficos es un criterio mayor en el


diagnóstico de la endocarditis infecciosa?

1. 1. Derrame pericárdico.
2. 2. Insuficiencia mitral moderada.
3. 3. Absceso periaórtico.
4. 4. Insuficiencia aórtica severa.
Gráfico de respuestas
Comentario

Una pregunta sobre un tema esencial para el ENARM: las endocarditis infecciosas. En este caso,
se contesta directamente si conocemos los criterios de Durack.

CRITERIOS MAYORES

•! Hemocultivos positivos:
•! - Microorganismos típicos en, al menos, dos hemocultivos separados: S. viridans, S. bovis,
HACEK, S.aureus, Enterococcus.
•! - Hemocultivos persistentemente positivos.

!
!
!

•! Hallazgos ecocardiográficos:
•! - Ecografía con vegetación o jet valvular, absceso o nueva dehiscencia en prótesis
valvular.
•! - Nueva regurgitación valvular.

CRITERIOS MENORES

•! ADVP o cardiopatía predisponente.


•! Fiebre mayor de 38ºC.
•! Fenómenos vasculares: embolias en arterias mayores, infartos sépticos pulmonares,
aneurismas micóticos, aneurismas micóticos, hemorragias intracraneales o conjuntivales,
manchas de Janeway.
•! Fenómenos inmunológicos: glomerulonefritis, nódulos de Osler, manchas de Roth, factor
reumatoide.
•! Ecocardiografía sugestiva, sin ser criterio mayor.
•! Hallazgos microbiológicos (cultivo o serología) que no llegan a ser criterio mayor.(R/3)

392. A 32-year-old female comes to your office with an intense pain with defecation
followed by small amounts of bright red blood in her stools. She has suffered from
constipation for many years, but it has currently worsened notably. Her vital signs are
within normality and laboratory workup reveals no anemia. Physical examination gives
the diagnosis. Which of the following is the most likely diagnosis?

1. 1. Thrombosed external hemorrhoid


2. 2. Bleeding internal hemorrhoid
3. 3. Anal fissure
4. 4. Perianal abscess
Gráfico de respuestas

Comentario

Anal fissure. Intense constipation can cause an anal fissure. Anal fissures cause intense pain and
occasional blood with defecation or manipulation. Pain causes sphincter contraction which in return
increases pain.(R/3)

393. Al auscultar a un paciente que se queja de fatiga progresiva y ortopnea, escucha un


soplo holositólico, seguido de un desdoblamiento amplio del segundo tono y un tercer
tono bajo tras el cierre de la válvula aórtica. El soplo lo encuentra irradiado a la axila. Lo
más probable es que se trate de:

1. 1. Estenosis mitral.
2. 2. Estenosis pulmonar.
3. 3. Insuficiencia mitral.
4. 4. Insuficiencia aórtica.
Gráfico de respuestas

Comentario

La estenosis mitral y la insuficiencia aórtica producen un soplo diastólico. La estenosis aórtica, la


estenosis pulmonar, y la insuficiencia mitral producen un soplo sistólico. Los dos hechos que deben
hacer inclinarnos hacia la insuficiencia mitral serían la irradiación hacia la axila y el carácter

!
!
!

holosistólico del soplo. El desdoblamiento amplio del segundo ruido aparece frecuentemente por
la hipertensión pulmonar, y el tercer ruido puede indicar que la insuficiencia mitral es severa.(R/3)

394. Señale la respuesta CORRECTA con respecto al neumotórax en Pediatría:

1. 1. El latido cardíaco está desplazado hacia el lado enfermo.


2. 2. La simetría torácica descarta siempre la existencia de un neumotórax.
3. 3. La irritabilidad o la aparición de apnea pueden ser signos tempranos.
4. 4. Es más frecuente en prematuros.
Gráfico de respuestas

Comentario

En el neumotórax, el latido se desplaza hacia el lado sano.

Los neumotórax de pequeño tamaño no generan asimetría torácica. Si bien muchas veces es
asintomático, desde el punto de vista clínico, lo hemos de sospechar ante la presencia de
irritabilidad o pausas de apnea en recién nacidos con causas desencadenantes (SAM, reanimación
con presión positiva a dosis altas). Es más frecuente en postérminos.(R/3)

395. A 48-year-old man goes to the ER because of a two-month history of persistent cough
and hemoptysis. He reports no fever, chest pain or rhinorrhea, but he has lost his appetite
and he has lost 6 kg. He reports no relevant issues regarding his personal or family
history. A CT-scan is performed. It reveals a mass in the upper left lobe. He is admitted
into hospital, where the diagnosis of lung cancer is confirmed with fibrobronchoscopy,
and he undergoes surgery without any major complications. However, his physician is
concerned because the patient spends uncountable hours on his laptop searching for the
latest information regarding his illness. What psychological defense mechanism is he
using?

1. 1. Denial.
2. 2. Repression.
3. 3. Intellectualization.
4. 4. Delusional projection.
Gráfico de respuestas

Comentario

Intellectualization. Physchological defense mechanisms are our unconscious means of responding


and adapting to different situations. They usually allow us to decrease anxiety, and can be
classified in three groups: (1) immature, more commonly used by children or teenagers (eg, fantasy,
projection, passive aggression, acting out), (2) neurotic, common in adults, they provide short term
relief, but often result in long-term complications (eg, intellectualization, reaction formation,
dissociation, displacement, repression); and (3) mature, most often used by adults, and allow for a
healthy and more constructive approach to reality (eg, humor, sublimation, suppression, altruism,
anticipation).(R/3)

!
!
!

396. Mujer de 58 años que acude a su


consulta refiriendo metrorragia escasa autolimitada de 4 días de evolución. Exploración
ginecológica: genitales externos y vagina normales, cérvix multípara bien epitelizado sin
evidencia de sangrado en el momento actual. Útero y anexos no tumorales. Realiza un
USG transvaginal que se muestra en la imagen. Señale la respuesta INCORRECTA:

Conducta expectante dado que no sangra en el momento actual, recomendando


1. 1. volver a consulta si se repite el episodio de sangrado para realizar legrado
fraccionado.
Realizaremos una histeroscopía diagnóstica con toma de muestras para estudio
2. 2.
anatomopatológico.
3. 3. Se puede realizar legrado fraccionado.
Si se diagnostica de hiperplasia simple de endometrio realizaremos tratamiento
4. 4.
con acetato de medroxiprogesterona.
Gráfico de respuestas
Comentario

Ante una mujer de casi 60 años con metrorragia, caben varias posibilidades, pero por supuesto
debemos descartar la peor de ellas: el cáncer de endometrio. También puede deberse a una
hiperplasia endometrial, pero no es imposible que se trate de una lesión maligna, por lo que una
respuesta como la 1 no puede ser correcta.

En cuanto a la imagen, es posible que no sepa interpretarla con gran precisión, pero eso no
significa que no pudieras acertar la pregunta, por la razón explicada. Lo que muestra es un
endometrio cuyo grosor es distinto en unos puntos que en otros.(R/1)

397. Con respecto a la patología que posiblemente padece la paciente del caso clínico
anterior indique cuál es el método diagnóstico de elección:

1. 1. Legrado fraccionado.
2. 2. USG vaginal.
3. 3. Citología cervical en triple toma.
4. 4. Biopsia dirigida por histeroscopía.
Gráfico de respuestas

!
!
!

Comentario

Dado que existe la posibilidad de que estemos ante un cáncer de endometrio, es imperativa la
realización de una histeroscopía, con toma de biopsias sobre las áreas sospechosas de
malignidad, para posterior estudio histológico (respuesta correcta 4).

398. A 28-year-old female from Quintana Roo who was diagnosed with HIV infection 2
years ago, is admitted with Pneumocystis jirovecii pneumonia. Her current CD4 count is
80/microL, and viral load is 300,000 copies/mL. The patient improves with treatment and
she is encouraged to receive zidovudine, lamivudine, nelfinavir, and trimethoprim-
sulfamethoxazole. What is the most appropriate drug to be added to her current regimen?

1. 1. Pentamidine.
2. 2. Ganciclovir.
3. 3. Azithromycin.
4. 4. Itraconazole.
Gráfico de respuestas
Comentario

Itraconazole. This question describes an HIV-positive patient that is at high risk of developing
disseminated histoplasmosis. Its causative agent, Histoplasma capsulatum is a dimorphic fungus
that is endemic to the mexican southeast. A very typical and suggestive HINT is a history of
exploring caves or exposure to bats, cleaning bird cages or coops. HIV-positive patients with CD4
below 100/microL and who live in endemic areas should receive prophylaxis against this agent. The
recommended drug for this is itraconazole.(R/4)

399. ¿Cuál es el tratamiento de elección de un paciente con enfermedad de Churg-Strauss


con hemorragia pulmonar e insuficiencia cardiaca severa?:

1. 1. Ciclofosfamida.
2. 2. Metotrexate.
3. 3. Ciclofosfamida y prednisona.
4. 4. Azatioprina.
Gráfico de respuestas
Comentario

Recuerde que el tratamiento de las vasculitis, en general, incluyen corticoides a dosis altas y, en
función de la gravedad, inmunosupresores. En la enfermedad de Wegener, de entrada, debe darse
ciclofosfamida y corticoides; en la PAN clásica y la microscópica, en función de la severidad, pero,
dado que suelen ser enfermedades muy graves en general, se precisa añadir también
inmunosupresores; en cuanto al Churg-Strauss, es cierto que puede responder sólo al tratamiento
con corticoides a dosis altas, pero en casos graves, como éste, que presenta hemorragia pulmonar
(proceso con alto riesgo de enfermedad) e insuficiencia cardiaca, se precisa también tratamiento
intensivo. En cuanto a la elección del inmunosupresor, es muy variable, pero, en general, el más
utilizado es la ciclofosfamida, ya que es el que mejores resultados ha demostrado en vasculitis
graves como el Wegener y la PAN clásica. (R/3)

400. Indique cuál de las siguientes definiciones sobre el control neural de la mirada es
CORRECTA:

!
!
!

Movimientos sacádicos: desplazan rápidamente la fóvea hacia un objeto visual


1. 1.
situado en la periferia.
Movimientos de persecución: mantienen las imágenes durante la rotación
2. 2.
sostenida de la cabeza, y son iniciados por estímulos visuales.
Movimientos vestibulooculares: desplazan los ojos en direcciones opuestas de
3. 3.
modo que la imagen quede ubicada en ambas fóveas.
Movimientos optocinéticos: mantienen sobre la fóvea la imagen de un objeto en
4. 4.
movimiento
Gráfico de respuestas
Comentario

Las definiciones correctas son las siguientes:

1- Movimientos sacádicos: desplazan rápidamente la fóvea hacia un objeto visual situado en la


periferia.

2- Movimientos de persecución: mantienen sobre la fóvea la imagen de un objeto en movimiento

3- Movimientos vestibulooculares: mantienen fijas en la retina las imágenes durante los


movimientos breves de cabeza

4- Movimientos optocinéticos: mantienen las imágenes durante la rotación sostenida de la cabeza,


y son iniciados por estímulos visuales.(R/1)

401. La luxación de Lisfranc se produce a nivel de la articulación:

1. 1. Tobillo.
2. 2. Calcáneoastragalina.
3. 3. Tarsometatarsiana.
4. 4. Calcáneo-cuboidea.
Gráfico de respuestas
Comentario

La fractura luxación de Lisfranc asocia una fractura en la base del 2º metatarsiano, la rotura del
ligamento de Lisfranc en la misma localización y una luxación de las bases de los metatarsianos
hacia lateral, perdiendo su relación de congruencia con el resto del tarso (las cuñas y el cuboides).
Existen distintas variantes, pero se caracterizan por su inestabilidad y por la necesidad de
estabilización quirúrgica con tornillos y agujas. Son lesiones asociadas a una inmovilización larga y
a secuelas por dolor en muchos casos.(R/3)

402. ¿Cuál de las siguientes sustancias NO tiene utilidad en el diagnóstico de un tumor


de ovario?:

1. 1. HCG.
2. 2. Alfafetoproteína.
3. 3. CA-125.
4. 4. Progesterona.
Gráfico de respuestas
Comentario

!
!
!

Esta pregunta de marcadores tumorales es sencilla. El conocimiento de los marcadores que se


elevan en los tumores germinales es importante:

Ca 125 esta elevado en el 70% de los canceres de ovario pero recuerde que también puede
estarlo en la endometriosis y la gestación.

El CEA y CA 19.9 es más especifico de cancer mucinoso.

Teratomas, disgerminomas, y sobre todo el tumor seno endodermico o saco vitelino, elevan la
alfetoproteina.

El coriocarcinoma secreta HCG, y el carcinoma embrionario ambas. (Recuerde que el seminoma


testicular NO eleva marcadores).

Los tumores de los cordones sexuales funcionantes producen hormonas: los de la granulosa y teca
producen estrógenos, los de las células de Sertoli y Leydig testosterona y los ginandroblastomas
secretan estrógenos y andrógenos.

La progesterona no tiene utilidad como marcador tumoral en el cáncer de ovario.(R/4)

403. Los componentes de las sales de Rehidratación Oral, fórmula OMS/UNICEF, son:

1. 1. NaCl: 3.5 g; ClK 1.5 g; Citrato trisódico dihidratado: 2.9 g; Glucosa: 20 g.


2. 2. NaCl: 2.5 g; ClK 3.5 g; Citrato trisódico dihidratado: 1.0 g; Glucosa: 10 g.
3. 3. NaCl: 2.5 g; ClK 1.0 g; Citrato trisódico dihidratado: 3.0 g; Glucosa: 30 g.
4. 4. Ninguna de las anteriores.
Gráfico de respuestas
Comentario

Pregunta fácil y que ya ha sido preguntado en el ENARM sobre la composición del VSO.

La respuesta correcta es la 1.

!
!
!

404. Mujer de 58 años que acude a su


consulta refiriendo metrorragia escasa autolimitada de 4 días de evolución. Exploración
ginecológica: genitales externos y vagina normales, cérvix de multípara bien epitelizado
sin evidencia de sangrado en el momento actual. Útero y anexos no tumorales. Realiza
un ultrasonido transvaginal que se muestra en la imagen. Señale la respuesta
INCORRECTA:

Conducta expectante dado que no sangra en el momento actual, recomendando


1. 1. volver a consulta si se repite el episodio de sangrado para realizar legrado
fraccionado.
Realizaremos una histeroscopia diagnóstica con toma de muestras para estudio
2. 2.
anatomopatológico.
3. 3. Se puede realizar legrado fraccionado.
Si se diagnostica de hiperplasia simple de endometrio realizaremos tratamiento
4. 4.
con acetato de medroxiprogesterona.
Gráfico de respuestas
Comentario

Ante una mujer de casi 60 años con metrorragia, caben varias posibilidades, pero por supuesto
debemos descartar la peor de ellas: el cáncer de endometrio. También puede deberse a una
hiperplasia endometrial, pero no es imposible que se trate de una lesión maligna, por lo que una
respuesta como la 1 no puede ser correcta.

En cuanto a la imagen, es posible que no sepa interpretarla con gran precisión, pero eso no
significa que no pudiera acertar la pregunta, por la razón explicada. Lo que muestra es un
endometrio cuyo grosor es distinto en unos puntos que en otros.(R/1)

405. Con respecto a la patología que posiblemente padece la paciente del caso clínico
anterior indique cuál es el método diagnóstico de elección:

1. 1. Legrado fraccionado.
2. 2. Laparoscopia.
3. 3. Citología cervical en triple toma.
4. 4. Biopsia dirigida por histeroscopia.

!
!
!

Gráfico de respuestas
Comentario

Dado que existe la posibilidad de que estemos ante un cáncer de endometrio, es imperativa la
realización de una histeroscopia, con toma de biopsias sobre las áreas sospechosas de
malignidad, para posterior estudio histológico (respuesta correcta 4).

406. Un RN pretérmino de 30 semanas de edad gestacional, con un peso de 1,100 gramos,


realiza apneas de 30 segundos de duración con bradicardia y cianosis de forma
frecuente. Señale entre las siguientes situaciones asociadas, cuál es MENOS frecuente:

1. 1. Hemorragia intraventricular.
2. 2. Anemia.
3. 3. Enterocolitis necrotizante.
4. 4. Hiperglucemia.
Gráfico de respuestas
Comentario

Los neonatos expresan inespecíficamente su patología desde el punto de vista respiratorio, a


través de pausas de apnea, distrés o polipnea. Cualquier enfermedad neonatal tiene una expresión
común, por tanto. Cardiopatías, procesos intracraneales, anemia, infecciones, alteraciones
digestivas, alteraciones metabólicas (hipoglucemia, no hiperglucemia - que suele ser iatrogénica y
autolimitada- ) pueden ser causa de cese de la respiración.(R/4)

407. Un RN se ha intubado en la sala de partos por presentar apnea desde el nacimiento.


La madre había recibido un derivado opiáceo. ¿Cuál de los siguientes fármacos y qué vía
de administración utilizaría de 1ª elección?:

1. 1. Naloxona por vía intratraqueal.


2. 2. Adrenalina por vía intratraqueal.
3. 3. Bicarbonato sódico por vía intravenosa.
4. 4. Teofilina por sonda nasogástrica.
Gráfico de respuestas
Comentario

Una causa de depresión respiratoria neonatal es la administración de opiáceos como anestésico a


la madre. La depresión respiratoria provocada se reversibilizará a través de la administración al
recién nacido de naloxona. La naloxona, antagonista opiáceo, se puede administrar por vía
intratraqueal aprovechando la intubación del niño (respuesta correcta 1) es e tratamiento de
elección no solo en RN también en depresiones respiratorias en adultos esto es muy preguntado
en el ENARM.(R/1)

408. La mayoría de las bronquiolitis van a ser tratadas de forma ambulatoria, sin
necesidad de tratamiento farmacológico, pero sí se deben aplicar determinadas medidas
generales de tipo sintomático. Señale cuál de las siguientes opciones acerca del
tratamiento ambulatorio de la bronquiolitis es FALSA:

El niño debe ser observado y en caso de que aparezca dificultad respiratoria, mal
1. 1. aspecto, pausas de apnea o rechazo del alimento, debe ser llevado con la mayor
brevedad posible a un servicio de urgencias.

!
!
!

Nos debemos asegurar que el niño tome algo de alimento por boca, si se fatiga
2. 2.
con las tomas, se pueden fraccionar.
3. 3. Se debe evitar el humo del tabaco en todo el domicilio.
4. 4. La postura del lactante en la cuna debe ser el decúbito lateral izquierdo.
Gráfico de respuestas
Comentario

Esta pregunta de bronquiolitis no es demasiado importante. Las recomendaciones para le


tratamiento de las bronquiolitis en el domicilio son: asegurar la toma de líquidos suficientes
(fraccionar las tomas e ingesta de líquido); favorecer la posición en decúbito supino con inclinación
de la cabeza 30º y ligera hiperextensión (descanso semiincorporado); lavados nasales con suero;
evitar tabaco, humos e irritantes bronquiales; vigilar signos de empeoramiento y acudir a urgencias
si mal aspecto, agitación o somnolencia no habitual, rechazo del alimento o vómitos, apnea,
cianosis o empeoramiento de la dificultad respiratoria; tomar temperatura; seguimiento por su
pediatra.(R/4)

409. A 7-year-old child is brought to the pediatrician's office presenting with a lump on
the upper right side of her neck that is swollen and erythematous. She suffered a
pharyngeal infection a few days ago. She is febrile and complaints of pain in the affected
region but general appearance shows no signs of serious illness. There is no fluctuance
on palpation. Which is the most likely causative agent in this case?

1. 1. Staphylococcus aureus.
2. 2. Streptococcus pneumonia.
3. 3. Streptococcus viridians.
4. 4. Toxoplasma gondii.
Gráfico de respuestas
Comentario
Staphylococcus aureus. Cervical lymphadenopathy is a common finding after pharyngeal infections.
Lymph nodes related to bacterial infections are usually enlarged unilaterally while viral infections
tend to produce bilateral lymphadenopathies. The most frequent germ isolated is Staphylococcus
aureus followed by group A streptococcus.(R/1)

410. Niño de 3 años que consulta por historia de estreñimiento desde el período neonatal
(con retraso en la evacuación de meconio), retraso ponderal y dolor abdominal frecuente.
Si usted sospecha una enfermedad de Hirschprung, ¿Qué respuesta sería la
INCORRECTA?:

El niño realiza evacuaciones abundantes con esfuerzo importante y la madre


1. 1.
refiere que “mancha la ropa interior”
2. 2. En la exploración física usted encuentra el abdomen distendido.
3. 3. Al hacer tacto rectal, presenta tono del esfínter aumentado y ampolla rectal vacía.
4. 4. En la biopsia: aumento de acetilcolinesterasa.
Gráfico de respuestas
Comentario

La Enfemedad de Hirschprung (EH) se da por un trastorno en la migración de los neuroblastos


desde la cresta neural. Fruto de esta anomalía, hay una ausencia de células ganglionares en los
plexos mientéricos. El tramo agangliónico se muestra estenótico. El territorio proximal a este se
dilata secundariamente. El sexo masculino se afecta más que el femenino. Desde un punto de
vista clínico las manifestaciones se dan desde el nacimiento (es un cuadro congénito). Hay que
sospechar una EH ante

!
!
!

casos de estreñimiento crónico que debuten antes de los dos años de edad. Desde un punto de
vista manométrico, en la EH destaca una asuencia en la relajación del esfinter anal interno. El
tratamiento es quirúrgico y consiste en resecar el segmento agangliónico.(R/1)

411. La colitis ulcerosa (CU) y la enfermedad de Crohn (EC) son dos trastornos
inflamatorios crónicos y diferenciados de causa desconocida. Para establecer el
diagnóstico diferencial entre ambas hay que tener en cuenta las siguientes
consideraciones excepto una:

1. 1. La CU afecta el colon y la EC puede afectar todo el tracto digestivo.


En la EC existen tres patrones de presentación clínica: inflamatorio, fistulizante y
2. 2.
obstructivo.
La colonoscopía y la biopsia definen la extensión, siendo posible realizar el
3. 3.
diagnóstico diferencial entre ambas.
4. 4. El diagnóstico diferencial debe realizarse con el linfoma.
Gráfico de respuestas
Comentario

Una pregunta discutible hasta cierto punto.

Las respuestas 1, 2 y 4 son claramente ciertas. Es muy importante saber que una de las
principales diferencias entre la CU y la EC es el territorio de afectación. Mientras la EC puede
afectar a todo el tracto digestivo, la CU se limita al colon. Otro dato que es conveniente recordar es
que el recto sólo puede estar afectado en CU En la EC existen los patrones de presentación que
relata la pregunta y siempre se debe hacer el diagnóstico diferencial con el linfoma por sus
posibles similitudes.

La respuesta 3 aunque aparentemente sea correcta, se podría considerar falsa porque nos dicen
que la colonoscopía y la biopsia definen la extensión… Y en el caso de esta última es falso, ya que
la biopsia no permite definir la extensión de la enfermedad. No obstante, es importante recordar
que con ambas técnicas se hace el diagnóstico y el estudio de extensión que permite diferenciar
CU de EC.(R/3)

412. Un hombre de 50 años acude a consulta refiriendo dolor abdominal epigástrico


ocasional y diarrea de dos años de evolución. Se le practica un test de cuantificación de
grasa en heces de 24 h, determinándose la presencia de esteatorrea. La siguiente prueba
que le realizamos es la prueba de la D-xilosa, que es normal. ¿Cuál es el diagnóstico más
probable en este paciente?

1. 1. Enfermedad de Crohn.
2. 2. Amiloidosis.
3. 3. Pancreatitis crónica.
4. 4. Linfoma intestinal.
Gráfico de respuestas
Comentario

La D-xilosa se absorbe en el yeyuno por difusión pasiva, sin necesidad de que intervengan las
enzimas pancreáticas o las sales biliares. Por ello, está alterada en procesos que afectan a la
pared del tubo digestivo, sin verse influida por alteraciones pancreáticas.

!
!
!

El test de D-xilosa consiste en administrar este azúcar en ayunas, y luego se mide su presencia en
orina, cuando pasan unas horas. Podría estar más o menos alterada en la enfermedad celíaca, en
la infiltración intestinal por amiloide, en un linfoma intestinal o en el contexto de la enfermedad de
Whipple. En cambio, en la pancreatitis crónica no habría ningún tipo de problema para absorber D-
xilosa, porque su absorción es independiente de las enzimas de esta glándula.(R/3)

413. ¿Cuál es el mecanismo de acción de la atorvastatina?:

1. 1. Aumento de la liberación de esteroles.


2. 2. Inhibición de la HMG-CoA reductasa.
3. 3. Aumento de la eliminación de sodio.
4. 4. Bloqueo de receptores a2.
Gráfico de respuestas
Comentario

La lovastatina pertenece al grupo de las estatinas que actúan inhibiendo la HMG-CoA reductasa,
(enzima limitante en la síntesis de colesterol). Por este mecanismo reducen el contenido hepático
de colesterol aumentando los receptores de LDL, acelerando el aclaramiento de LDLy reduciendo
entre un 25-60% el LDLc (colesterol transportado por las LDL). También pueden reducir los niveles
de triglicéridos, al disminuir la producción hepática de VLDL, teniendo mayor efecto la atorvastatina
y simvastatina. Los niveles de HDLc se elevan moderadamente. Además de efecto sobre los
lípidos, actualmente se ha visto que las estatinas tienen acción "pleiotrópica" pues mejora la
función endotelial y vasomotora, inhiben la agregabilidad plaquetaria y modulan la respuesta
inflamatoria. Hay distintos tipos de estatinas.(R/2)

414. Paciente de 33 años que presenta parálisis facial periférica izquierda. El test de
lacrimación (Schirmer), el reflejo estapedial y la electrogustometría son normales. Ante
esta situación, el origen más probable de la lesión estará en:

1. 1. El conducto auditivo interno.


2. 2. La primera porción del facial intrapetroso.
3. 3. La tercera porción del facial intrapetroso.
4. 4. En la glándula parótida.
Gráfico de respuestas
Comentario

La exploración del paciente nos revela que sus tres ramas intrapetrosas no están afectadas:

- Nervio petroso superficial mayor: Lacrimación normal.

- Nervio del estribo: Reflejo estapedial normal.

- Nervio cuerda del tímpano: Electrogustometría normal.

De las opciones que nos ofrecen, la única opción posible es la 4, ya que la glándula parótida es
distal a la salida de todas estas ramas. Si la lesión estuviese en el conducto auditivo interno,
ocurriría lo contrario: las tres estarían afectadas, ya que es proximal a todas ellas.

415. ¿Cuál de los siguientes trastornos electrolíticos NO se produce en la enfermedad de


Addison?:

!
!
!

1. 1. Hiponatremia.
2. 2. Hipercalcemia.
3. 3. Alcalosis.
4. 4. Hiperazoemia.
Gráfico de respuestas
Comentario
Para comprender las distintas alteraciones iónicas que aparecen en la patología suprarrenal es
básica la fisiología de esta glándula. En la insuficiencia suprarrenal primaria (Enf. Addison) existe
una ausencia de mineral y glucocorticoides. Las acciones de la aldosterona en el túbulo distal renal
conllevan un aumento en la reabsorción de sodio y una pérdida de potasio e hidrogeniones. Por
este motivo, en ausencia de aldosterona existirá hiponatremia, hiperpotasemia y acidosis
metabólica. La ausencia de cortisol produce una menor eliminación de agua libre, lo que también
contribuye a la existencia de hiponatremia. La hipercalcemia es un hallazgo relativamente
frecuente en estos pacientes, aunque el mecanismo de producción sea desconocido.(R/3)

416. What should be the first step in the management of a trauma patient who is not
breathing?

1. 1. Ventilation with Ambu bag.


2. 2. Cricothyroidotomy.
3. 3. Removal of secretions, foreign bodies and blood of the upper respiratory tract.
4. 4. Endotracheal intubation.
Gráfico de respuestas
Comentario

Esta pregunta es muy importante ya que el manejo del politraumatizado es un tema muy rentable
para el ENARM. El primer paso a realizar ante todo politraumatizado, y que se recuerda con el
acrónimo ABCDE, es comprobar la vía aérea (Airway) y realizar la extracción de cuerpos extraños
y elevar la barbilla con tracción anterior de la mandíbula, antes de realizar cualquier maniobra
invasiva como puede ser una traqueotomía o cricotiroidotomía. La ventilación e intubación serían
dos pasos que seguirían al anterior en caso de que no fuera suficiente.(R/3)

417. Con respecto a la prueba de la tuberculina, ¿cuál de las siguientes afirmaciones es


cierta?

1. 1. Se administra por vía i.m.


2. 2. Una prueba de la tuberculina no puede sensibilizar a una persona no infectada.
3. 3. La reacción se considera positiva cuando el área de eritema es > 10 mm.
4. 4. La reacción debe leerse a las 24 horas.
Gráfico de respuestas
Comentario

Teng cuidado con esta pregunta, porque es más difícil de lo que parece.

La 2 es correcta. Efectivamente, nadie se sensibiliza por haberse sometido a la


intradermorreacción de Mantoux. Existen casos en los que, con un primer Mantoux, el resultado es
negativo, siendo el segundo positivo, pero se trata de sujetos previamente sensibilizados en los
que se ha producido un fenómeno de “olvido” inmunológico. Al realizarse la prueba por segunda
vez, ya positiviza, debido a que el primer Mantoux sirvió como “recordatorio” a la inmunidad (efecto
Booster).(R/2)

!
!
!

418. ¿En cuál de las siguientes manifestaciones clínicas se requiere el acetato de


ciproterona como tratamiento de elección?:

1. 1. Defectos luteínicos.
2. 2. Amenaza de aborto.
3. 3. Hirsutismo manifiesto.
4. 4. Endometriosis.
Gráfico de respuestas
Comentario

Es una pregunta importante en cuanto que debe conocer un síndrome, el del ovario poliquístico y
el hirsutismo es una de sus manifestaciones de la que debe conocer el tratamiento. Se realiza con
anticonceptivos orales, que disminuyen la producción de esteroides suprarrenales y ováricos,
reduciendo el hirsutismo en 2/3 de los pacientes. A veces se añaden antiandrógenos
(espironolactona, acetato de ciproterona, flutamida, cimetidina, finasteride...).(R/3)

419. ¿En cuál de las siguientes patologías NO hay participación del cérvix?:

1. 1. Vaginosis.
2. 2. Sífilis.
3. 3. Gonococo.
4. 4. Molluscum contagiosum.
Gráfico de respuestas
Comentario

Las 3 primeras opciones pueden producir cervicitis tanto de manera especifica como inespecífica.
En cambio la infección por Molluscum contagiosum es de genitales externos.(R/4)

420. En un lactante de dos meses con ictericia apreciable desde la 2ª semana de vida,
evacuaciones de color blanquecino, hepatomegalia de 6 cm de consistencia dura,
esplenomegalia de 3 cm, en el que se realizó una gammagrafía hepática con HIDA
observándose una captación normal en el hígado con ausencia de paso a intestino, ¿cuál
es la primera actitud a tomar?

1. 1. Intervención quirúrgica de Kasai.


2. 2. Trasplante hepático.
3. 3. Laparotomía exploradora urgente con biopsia hepática y colangiografía.
Dieta hipercalórica con triglicéridos de cadena media, vitaminas A, D, E y K,
4. 4.
fenobarbital y colestiramina.
Gráfico de respuestas
Comentario

La presencia de ictericia, acolia y coluria caracteriza un síndrome colestásico. Éste puede deberse
a causas intra o extrahepáticas, y para distinguirlo se ha realizado una gammagrafía con HIDA
(ácido hidroxi- indol- acético). Dado que existe un retraso en la evacuación del contraste, el
problema muy probablemente sea extrahepático, que es lo que impide que llegue al intestino.
Sabiendo esto, el siguiente paso sería conocer cuál es la causa exacta, por lo que elegiremos la
respuesta 3. Las opciones 1 y 2 no son válidas porque, hasta este momento, sólo sabemos que
tiene una colestasis de origen extrahepático, pero no el diagnóstico exacto, por lo que aún no
podemos establecer el tratamiento adecuado.(R/3)

!
!
!

421. Atiende en el lugar del accidente a un politraumatizado que se encuentra


inconsciente, sin ventilación espontánea, presenta TA 80/50, varias heridas sangrantes
en extremidades y fractura abierta de tibia derecha. La actitud inmediata es:

1. 1. Despejar la vía aérea e intubar.


2. 2. Canalizar vía central y administrar expansores plasmáticos.
3. 3. Controlar la hemorragia con torniquetes.
Colocar collarín cervical y descartar lesión medular con exploración neurológica
4. 4.
completa.
Gráfico de respuestas
Comentario

Esta pregunta es muy importante ya que el manejo del politraumatizado es un tema muy
preguntado en el ENARM. Casi siempre se pregunta del mismo modo, le piden la actitud prioritaria
ante un accidentado; se responde conociendo el acrónimo ABCDE, y sabiendo q lo primero que
hay que hacer ante un politraumatizado es controlar la vía aérea (Airway), con la extracción de
cuerpos extraños y elevando la barbilla, y si es necesario intubar. Conociendo que esta es la
primera medida se responden bastantes de las preguntas de este tema.(R/1)

422. La frecuencia de la convulsión febril es:

1. 1. 20%.
2. 2. 8%.
3. 3. 20%.
4. 4. 5%.
Gráfico de respuestas
Comentario

Las crisis febriles aparecen en un 5% de los niños, y son un proceso típico de la edad infantil (entre
los 3 meses y los 5 años), que se relaciona más frecuentemente con el aumento de la temperatura,
lo que da lugar a una crisis el primer dia del proceso febril, independientemente del origen del
mismo. NO SE RELACIONAN CON UN MAYOR RIESGO DE EPILEPSIA.(R/4)

423. ¿Cuál de los siguientes cuadros clínicos NO mejoraría su agudeza visual con gafas?:

1. 1. Miopía.
2. 2. Hipermetropía.
3. 3. Presbicia.
4. 4. Astigmatismo irregular.
Gráfico de respuestas
Comentario
Pregunta sencilla aplicando conocimientos básicos de defectos de refracción. Las gafas se usan
para corregir los defectos de refracción. Usamos lentes esféricas principalmente para la corrección
de la miopía, hipermetropía y la presbicia. Las lentes cilíndricas se usan, de forma general, para
corregir el astigmatismo asociado o no a otras ametropías. Sin embargo en el astigmatismo
irregular, en el que no existen focos claros definidos, no es posible la corrección con lentes
convencionales. La mejor corrección posible se consigue con la utilización de lentes de contacto
rígidas.(R/4)

424. Las metástasis inguinales en el cáncer epidermoide de ano:

!
!
!

1. 1. Son siempre síncronas con el tumor primario.


2. 2. Son susceptibles de disección profiláctica.
3. 3. Son susceptibles de disección terapéutica.
4. 4. No existe diseminación inguinal en el cáncer de ano.
Gráfico de respuestas
Comentario

Los tumores anales son relativamente raros. Más frecuentes en hombres jóvenes.

La mayoría son asintomáticos, siendo el dolor y la hemorragia las principales manifestaciones. El


más frecuente es el carcinoma de células escamosas. Se suele presentar como ulceración del
borde anal, de hecho toda ulceración crónica en la región anal que no cicatriza debe considerarse
un carcinoma hasta que no se demuestre lo contrario con biopsia. Tiene tendencia a la
diseminación hacia ganglios inguinales.

El tratamiento es la escisión local amplia o la radioterapia, con similares resultados. Se lleva a cabo
una linfadenectomía inguinal en caso de que haya ganglios positivos. Es raro que se realice
amputación abdominoperitoneal.(R/3)

425. Los signos de hipoxia fetal son indicación de administración de O2 a


concentraciones elevadas a la madre, y de inducción del parto. Entre estos hallazgos se
encuentran los siguientes, EXCEPTO:

1. 1. Deceleración variable o tardía en la frecuencia cardíaca fetal.


Acidosis de tipo metabólico y respiratorio, en el análisis de sangre del cuero
2. 2.
cabelludo, con pH menor de 7.20.
3. 3. Bradicardia fetal con disminución de la variabilidad entre cada latido.
4. 4. Disminución de la resistencia vascular fetal detectada con USG doppler.
Gráfico de respuestas
Comentario

La hipoxia fetal se asocia además a acidosis. Ambas inducen en el feto bradicardización,


disminución de la variabilidad, CIR y elevación de las resistencias vasculares fetales (no
dismunción de las mismas, como aparecen en la opción 4). Este hecho se puede detectar a través
de USG con sonda de doppler.(R/4)

426. ¿Cuál de las siguientes alternativas es una medida eficaz en el tratamiento de la


enfermedad diarreica aguda en niños?

1. 1. Difenoxilato y atropina.
2. 2. Loperamida.
3. 3. Rehidratación oral.
4. 4. Racecadotrilo.
Gráfico de respuestas
Comentario

Por nada del mundo puede fallar el abordaje de diarrea en pediatría.

La rehidratación oral es una de las medidas más eficaces en el tratamiento del síndrome diarreico
agudo. Recordemos que el suero oral hizo merecedores del Nobel a sus inventores. Respuesta 3
correcta. No es necesario administrar loperamida.

!
!
!

Composición del vida suero oral es de: glucosa 111mEq/lt, sodio 90mEq/lt, cloruro 80 mEq/lt,
citrato 30 mEq/lt, potasio 20 20 mEq/lt, HCO3 30 mEq/lt.

Existen 3 planes de rehidratación dependiendo el grado de deshidratación.

PLAN A

Para prevenir la deshidratación y desnutrición: Se aplica en paciente con diarrea aguda NO


deshidratados.

- Alimentación Continúa A

- Bebidas abundantes B

- Consulta educativa C

Dar “Vida Suero Oral” VSO de la siguiente de la siguiente manera:

Dar 1 taza (150 ml) después de cada evacuación en niños mayores de 1 año y media taza (75 ml)
después de cada evacuación en niños menores de 1 año. Se ofrece a cucharaditas o sorbos. Se
realiza en domicilio.

PLAN B

El suero oral se administra a 100ml/Kg de peso en cuatro horas.La dosis se fracciona en tomas
cada 30 minutos y se ofrece lentamente con taza y cucharita para no sobrepasar la capacidad
gástrica y así disminuir la probabilidad del vómito. La rehidratación con este plan puede durar de 2
a 8 hrs. en el hospital.

PLAN C

Es el plan de rehidratación intravenosa que se emplea en la deshidratación severa. Se solución


Hartmann y si esta no esta disponible se utiliza solución salina al 0.9% en un esquema para tres
horas.

Primera hora : 50 ml/kg/hora

Segunda hora : 25 ml/Kg/hora

Tercera hora : 25 ml/Kg/hora.

(R/3)

427. Femenino de 44 años que acude a la consulta de ginecología por menorragias, sin
otra sintomatología acompañante. La exploración ginecológica muestra vagina y cérvix
normales, mamas normales y tacto vagino-abdominal con útero aumentado de volumen,
de consistencia dura y superficie irregular. El USG confirma el diagnóstico de tres
miomas de entre 3-4 cm de diámetro, de localización submucosa. En los exámenes de

!
!
!

laboratorio se detecte una anemia ferropénica moderada. ¿Qué tipo de tratamiento


propondría a esta paciente?

1. 1. Miomectomía abdominal.
2. 2. Análogos de GnRH.
3. 3. Tratamiento médico de la anemia ferropénica y observación de los miomas.
4. 4. Tratamiento de la anemia ferropénica e histerectomía abdominal.
Gráfico de respuestas
Comentario

Nos presentan el caso de una paciente con miomas, en la que éstos tienen repercusión clínica:
menorragias y anemia ferropénica. Si tratamos la anemia exclusivamente con hierro, lo más
probable es que reaparezca pasado un tiempo, porque la causa persistiría. Por ello, hay que
ofrecerle una solución para el problema de base, que son los miomas.

Los análogos de la GnRH no son el tratamiento definitivo de los miomas. Se utilizan previamente a
la cirugía, porque disminuyen su tamaño, pero no de forma aislada. Por otra parte, el tipo de
cirugía preferible en esta paciente es la histerectomía, puesto que ya no se encuentra en edad
reproductiva. La miomectomía se reserva para pacientes jóvenes que todavía no han cumplido sus
deseos genésicos.(R/4)

428. Los requerimientos diarios de insulina en los pacientes con DM tipo 1 disminuyen:

Entre uno y seis meses después del diagnóstico, bajo tratamiento con
1. 1.
insulinoterapia intensificada.
2. 2. Ante una apendicectomía.
3. 3. Durante la adolescencia.
4. 4. Por la presentación asociada de hipertiroidismo.
Gráfico de respuestas
Comentario
Tras el debut de una diabetes mellitus tipo 1 puede existir un periodo de unos meses de duración,
conocido como "luna de miel" en el que todavía se mantiene cierta reserva insulínica propia y se
puede reducir o incluso suspender el aporte de insulina. En la adolescencia, por el efecto de la GH
como hormona contrarreguladora, puede ser necesario tener que aumentar las dosis de insulina
(fenómeno del alba). En el resto de las respuestas, también va ser necesario tener que aumentar la
insulina porque corresponden a situaciones de estrés.(R/1)

429. La hemorragia uterina disfuncional puede ser controlada administrando la hormona


faltante que generalmente es:

1. 1. Estrógenos.
2. 2. Testosterona.
3. 3. Progesterona.
4. 4. Tiroideas.
Gráfico de respuestas
Comentario

En el tratamiento de la hemorragia uterina disfuncional se controla con la administración de


progesterona únicamente o combinada, por lo que la respuesta correcta es la 3.

!
!
!

430. ¿Cuál de los siguientes tumores produce virilización?:

1. 1. Tecoma.
2. 2. Gonadoblastoma.
3. 3. Tumor de Brenner.
4. 4. Tumor de seno endodérmico.
Gráfico de respuestas
Comentario

Esta pregunta de tumores de ovario es muy sencilla y se responde con conocimientos básicos
como que los andrógenos son los responsables de la virilización y en el ovario se producen en la
teca, por lo que un tumor de esta capa producirá virilización.

Por el contrario, un tumor de la granulosa produce estrógenos y producirá feminización.

El gonadoblastoma, el disgerminoma y el tumor del seno endodérmico, procedentes de la célula


germinal y el tumor de Brenner, con epitelio transicional de vejiga no secretan hormonas
esteroideas. Debe recordar que la causa de virilización de origen ovárica más frecuente es el
androblastoma.(R/1)

431. Respecto a las diferencias entre la leche materna y la leche de vaca, señale la
afirmación CORRECTA:

1. 1. La leche de vaca contiene una cantidad de calcio superior a la leche materna.


2. 2. La leche materna contiene mayor cantidad de vitamina K que la leche de vaca.
3. 3. La leche de vaca es mas rica en hidratos de carbono que la leche materna.
4. 4. La leche materna tiene una relación caseína/seroproteinas de 70/30.
Gráfico de respuestas
Comentario

Aunque es una pregunta que relativamente carece de importancia, es un dato curioso saber que la
leche de vaca contiene más calcio que la leche materna.(R/1)

432. Paciente de 22 años que acude por


primera vez a la consulta para realizar una revisión ginecológica. Menarquia a los 14 años,

!
!
!

reglas normales, no dolorosas, nuligesta. Se realiza USG vaginal, objetivando en región


anexial izquierda lo que se muestra en la imagen. Señale lo INCORRECTO:

1. 1. Es el tumor germinal ovárico más frecuente.


2. 2. Pueden producir alfafetoproteína.
3. 3. Son benignos.
4. 4. Habitualmente cursan con dismenorrea.
Gráfico de respuestas
Comentario

Teniendo en cuenta que se trata de una revisión rutinaria, en una mujer joven, sin factores de
riesgo para tumoración maligna y, por lo demás, asintomática, aparte de las características de la
imagen, debemos concluir que seguramente se trata de un quiste ovárico totalmente benigno.
Observe que se trata de una tumoración completamente regular, tanto en su contorno (redondeado
y liso) como en su contenido. Estos quistes suelen ser, simplemente, hallazgos incidentales y casi
siempre son asintomáticos, por lo que la respuesta incorrecta es la 4.

433. Usted decide intervenir a la paciente del caso clínico anterior con fines diagnóstico-
terapéuticos. Señale la opción CORRECTA:

Los análogos de GnRH tras la intervención pueden resultar eficaces para el


1. 1.
control de la enfermedad.
La anexectomía laparoscópica es el tratamiento más idóneo, debido al contenido
2. 2.
de la tumoración.
La quistectomía laparoscópica sin biopsia intraoperatoria es el tratamiento más
3. 3.
oportuno.
4. 4. En los embarazos suelen disminuir de tamaño.
Gráfico de respuestas
Comentario

Difícilmente vamos a poder tratar un quiste (tumoración benigna, pero tumoración) con tratamiento
médico. Por ello, lo que debemos hacer es extirparlo, lo cual hoy día es posible mediante cirugía
laparoscópica. Dada la juventud de la paciente y las características ecográficas del quiste, no
precisamos biopsia intraoperatoria, ya que se trata de una neoplasia benigna casi con total
seguridad.
Recordemos que una tumoración que sí puede reducir su tamaño con tratamiento médico son los
miomas, utilizando agonistas de la GnRH. No obstante, el mioma no tendría un contenido líquido
(anecoico, como en este caso), sino sólido. Y, en cualquier caso, lo que se suele conseguir es que
reduzcan su tamaño, pero para que desaparezcan por completo sería necesaria la extirpación
quirúrgica.(R/3)

434. Durante el embarazo, la artritis reumatoide suele:

1. 1. Afectar las rodillas.


2. 2. Mejorar.
3. 3. Producir manifestaciones renales.
4. 4. Producir nódulos.
Gráfico de respuestas
Comentario

!
!
!

Aproximademente el 70-80 % de las pacientes con AR experimenta una remisión clínica o una
mejoría sustancial durante el embarazo. A pesar de ello la paciente requiere alguna intervención
terapeútica la medida de elección son los corticoides que no sea de larga vida media, ya que no
atraviesan las barrera placentaria. Los AINEs se podrían utilizar si el beneficio superase al riesgo
pero habitualmente se pueden sustituir por corticoides a dosis bajas. Los AINEs pueden provocar
el cierre precoz del conduto arterioso, prolongar la gestación y el parto e inducir hemorragias
intracraneales en prematuros. Dentro de los fármacos modificadores de la enfermedad se deben
evitar como normal general. El más seguro sería no obstante la sulfasalazina.(R/2)

435. En relación a los divertículos colónicos, señala el enunciado FALSO:

1. 1. Su incidencia aumenta con la edad.


2. 2. Son más frecuentes a nivel de sigma.
3. 3. Son protusiones en fondo de saco de toda la pared intestinal.
4. 4. La hemorragia ocurre sólo en un pequeño porcentaje de pacientes.
Gráfico de respuestas
Comentario
Los divertículos de colon son herniaciones de la mucosa colónica a través de las capas musculares
de la pared del colon. La enfermedad diverticular, o diverticulosis, es más frecuente en personas
ancianas. Se localizan fundamentalmente en colon descendente. Suelen ser asintomáticos. Pero
cuando dan clínica, lo más frecuente es la hemorragia (que generalmente es de divertículos de
colon ascendente).(R/3)

436. Mujer de 35 años que acude a su revisión ginecológica rutinaria, con ciclos
regulares, siendo su última regla hace 17 días. En el ultrasonido transvaginal se observa
un útero en retroflexión, regular, con endometrio lineal de segunda fase y en uno de los
ovarios se observa una imagen econegativa de 31 mm, sin otros hallazgos de interés.
Ante esta situación usted decide:

1. 1. Completar el estudio con marcadores tumorales.


Completar el estudio con pruebas radiológicas más específicas como resonancia
2. 2.
magnética o TAC pélvico.
3. 3. Nuevo ultrasonido de control en 4-6 semanas.
4. 4. Cirugía citorreductora.
Gráfico de respuestas
Comentario

La imagen que nos describen en el caso clínico hace referencia al diagnóstico casual de un quiste
de contenido líquido, puesto que es econegativo, en el contexto de una mujer en edad fértil. Lo
más probable que dicha imagen corresponda con un folículo funcional, por el periodo del ciclo en
que se encuentra la paciente, por lo que lo más correcto sería repetir la exploración en 1 ó 2 meses
para evidenciar como dicha imagen ha desaparecido.(R/3)

437. El tumor uterino que tiene mayor riesgo de hemorragia es el:

1. 1. Subseroso.
2. 2. Intramural.

!
!
!

3. 3. Intraligamentario.
4. 4. Submucoso.
Gráfico de respuestas
Comentario

Los miomas submucosos se asocian con un aumento del sangrado menstrual y son más
sintomáticos que los demás. Respuesta 4 correcta.

Miomas subserosos (40%): situados bajo el peritoneo visceral uterino . Pueden alcanzar gran
tamaño, siendo por su localización poco sintomáticos.

Miomas intramurales: son los más frecuentes (55%) y proliferan enla porción central del miometrio.

Miomas submucosos (5-10%): hacen protrusión en la cavidad uterrina, por lo que son los más
sintomáticos. Pueden ser pediculados y prolapsarse a través del orificio cervical (se habla entonces
de mioma parido)(R/4)

438. Los gemelos unidos se forman cuando:

Entre el 4º y 8º día postfecundación antes de la diferenciación de las células del


1. 1.
amnios.
2. 2. La división ocurre después de la formación del disco embrionario.
Si la división ocurre cuando el amnios ya está establecido, 8 días después de la
3. 3.
fertilización.
4. 4. Los gemelos son dicigóticos, pero existe una anomalía en el desarrollo amniótico.
Gráfico de respuestas
Comentario

Existen dos tipos de embarazo gemelar: dicigótico (70%) y monocigótico (30%). Los gemelos
monocigóticos son genéticamente idénticos. Entre los monocigóticos, algunos nacen unidos en
grados muy diversos, desde una pequeña conexión superficial hasta compartiendo órganos vitales.

La división de los embriones de los gemelos independientes se produce durante los diez días
siguientes a la fecundación. En cambio, en los gemelos unidos se produce más tardíamente,
cuando el disco embrionario ya está formado. Por eso, la división será incompleta, y al nacimiento
se manifiesta como una conexión anatómica entre los dos gemelos.(R/2)

439. La lubricación de la vagina durante el coito, se debe a:

1. 1. Trasudado vaginal.
2. 2. Secreción de glándulas de Bartholino.
3. 3. Flujo cervical.
4. 4. Fluido endometrial.
Gráfico de respuestas
Comentario

La lubricación vaginal se produce por la segregación del flujo vaginal, un líquido viscoso segregado
por el cuello de la matriz y las paredes de la vagina con diversos fines. La lubricación de los labios,
en cambio, se debe a las glándulas de Bartolino. Respuesta 1 correcta.

!
!
!

440. Señale el cuadro clínico que NO esperaría encontrar en un adolescente con fibrosis
quística:

1. 1. Diabetes mellitus.
2. 2. Azoospermia obstructiva.
3. 3. Amenorrea primaria.
4. 4. Pólipos nasales.
Gráfico de respuestas
Comentario

Es importante conocer los aspectos clínicos mas relevantes de la fibrosis quística. La alteración de
la proteína reguladora de la conductancia transmebrana origina excreción defectuosa de cloro y
excesiva reabsorción de sodio, lo que causa secreciones muy espesas en diferentes órganos y la
subsiguiente obstrucción de sus conductos (páncreas, epidídimo, hígado, vía biliar, intestino, tracto
respiratorio...). El movimiento de los cilios alterado por la alta viscosidad de las secreciones facilita
la colonización de gérmenes patógenos en el epitelio respiratorio, lo que ocasiona una respuesta
inflamatoria crónica que puede dar lugar a pólipos nasales. A menudo existe amenorrea
secundaria a consecuencia de la enfermedad crónica y la reducción marcada del peso corporal,
pero no produce amenorrea primaria.(R/3)

441. Un hombre de 36 años de edad es valorado porque tiene una pierna roja, caliente y
dolorosa, desde hace cuatro días, con dolor e impotencia funcional. Sin antecedentes
familiares ni personales de interés, ni tampoco traumáticos ni quirúrgicos sobre dicha
pierna. La eco-Doppler de dicha pierna nos muestra un trombo en la vena poplítea. Se le
trata con heparina de bajo peso molecular y, posteriormente, comienza con
anticoagulantes orales. Un estudio posterior revela que este paciente es portador de la
mutación asociada al factor V de Leyden. ¿Cuál de las siguientes pautas de duración del
tratamiento anticoagulante es la más adecuada para este paciente?:

1. 1. 2 meses.
2. 2. 1 mes.
3. 3. Toda la vida.
4. 4. 2 años.
Gráfico de respuestas
Comentario

Los pacientes con una mutación en el factor V de Leyden que sufren un episodio tromboembólico
tienen mucho riesgo de recurrencia, por lo que la anticoagulación debe mantenerse de por vida, al
no tratarse de una condición reversible.(R/3)

442. ¿Cuál de las siguientes NO se considera un indicador de alto riesgo en el TEP?:

1. 1. Hipotensión mantenida.
2. 2. Choque cardiogénico.
3. 3. Insuficiencia respiratoria.
4. 4. Elevación de troponina.
Gráfico de respuestas
Comentario

La valoración del riesgo en pacientes con TEP es un elemento básico para decidir el tratamiento.
El tratamiento primario (fibrinólisis) se reserva para pacientes de alto riesgo (hipotensión o choque

!
!
!

cardiogénico) y en pacientes con riesgo intermedio (disfunción de VD o elevación de marcadores


de daño miocárdico) se debe valorar de forma individualizada la relación riesgo/beneficio de su
empleo. Los pacientes con bajo riesgo se benefician de tratamiento anticoagulante. La insuficiencia
respiratoria no se contempla como indicador de alto riesgo.(R/3)

443. Dentro del tratamiento no farmacológico de la HTA lábil se incluyen las siguientes
medidas dietéticas, EXCEPTO:

1. 1. Dieta sin sal.


2. 2. Reducción de peso.
3. 3. Ejercicio físico isométrico.
4. 4. Suspender la toma de anticonceptivos orales.
Gráfico de respuestas
Comentario

Son medidas no farmacológicas que disminuyen la PA:

- Restricción de la ingesta de sal

- Disminución de peso

- Ejercicio físico aeróbico

- Evitar el estrés

La suspensión de fármacos causantes de HTA, como los anticonceptivos orales, también ayuda a
controlar la PA. La supresión del tabaco no reduce los niveles de PA, pero reduce el impacto de la
HTA sobre los órganos diana. El ejercicio isométrico (levantar pesas) no se recomienda como
método para tratar la HTA, pues mal realizado puede incluso incrementarla (al realizar
Valsalva).(R/3)

444. Paciente femenino de 30 años consulta por un nódulo mamario que se diagnostica
de carcinoma ductal infiltrante. Tiene antecedentes familiares, ya que su madre y su
abuela han padecido esta misma enfermedad. El estudio genético muestra la presencia
de BRCA1, c-erb B2 y alteraciones de la p53. Receptores estrogénicos positivos. Sobre
las características citadas de esta paciente, señale cuál puede considerarse un factor de
buen pronóstico:

1. 1. Presencia de receptores estrogénicos positivos.


2. 2. Alteraciones en p53.
3. 3. Edad inferior a 35 años.
4. 4. Presencia de BRCA1.
Gráfico de respuestas
Comentario

Tanto las alteraciones en el p53, antecedentes familiares, la positividad para c-erb B2, la edad
menor de 35 años y la presencia de BRCA1 son factores demostrados de mal pronóstico. Por el

!
!
!

contrario, es la AUSENCIA de receptores hormonales la que confiere un mal pronóstico y no la


presencia de los mismos, como indica la opción 1.(R/1)

445. Paciente femenino de 29 años, con historia de alteraciones del ciclo frecuentes,
consistentes en oligomenorrea, hirsutismo en la escala de Ferriman-Galley de 11 puntos,
IMC 31 y lesiones verrugosas aterciopeladas y pigmentadas en nuca, axila y pliegue
submamario. Respecto a la patología que sospecha, sólo con la clínica, es FALSO que:

1. 1. Muy probablemente padezca resistencia insulínica.


2. 2. El cociente LH/FSH es superior a 2.
3. 3. Tiene riesgo incrementado de padecer patología endometrial.
4. 4. La primera medida a tomar será la pérdida de peso.
Gráfico de respuestas
Comentario

Se presenta un caso clínico típico de una paciente con síndrome de ovario poliquístico (SOP), la
cual presenta alteraciones menstruales por defecto e hirsutismo. Se considera hirsutismo en la
escala de Ferriman- Galley una cifra superior a 8. Además presenta en piel lesiones de acantosis
nigricans, las cuales implican resistencia insulínica en el 90% de las veces. Además hay un
hiperandrogenismo muy importante. En estas pacientes obesas con resistencia insulínica, el
cociente LH/FSH suele estar próximo a 1 o incluso ser superior la FSH. Cuando son pacientes
delgadas sin resistencia insulínica, suele ser superior a 2. Estas pacientes, al no ovular y no
compensar los estrógenos con progesterona, tienen más riesgo de hiperplasia de endometrio.
Obviamente, la primera medida de salud será la pérdida de peso.(R/2)

446. ¿Cuándo se aconseja intervenir quirúrgicamente la hernia umbilical infantil?:

1. 1. Se debe intervenir cuanto antes para evitar el riesgo de estrangulación.


A partir de los 3 ó 4 años de vida, al ser frecuente su cierre espontáneo antes de
2. 2.
esa edad.
Con el fin de utilizar prótesis en la cirugía, se recomienda esperar a que el
3. 3.
paciente sea mayor.
4. 4. Si el niño presentara vómitos durante los accesos febriles.
Gráfico de respuestas
Comentario

Dentro de las masas umbilicales en un recién nacido, la hernia umbilical se clasifica como una
masa grande que se puede detectar a nivel del cordón (respetándolo) ya en el periodo neonatal.

Está cubierta de piel, es blanda y reductible, aumentando de tamaño en valsalva (cuando el niño
llora). Suele desaparecer a los tres años y no se trata antes de esta edad a no ser que esté
incarcerada o estrangulada.

El onfalocele también es una masa grande pero está cubierta sólo por peritoneo y, a diferencia de
la hernia, su tratamiento sí que es quirúrgico.(R/2)

447. Todas las siguientes son posibles manifestaciones de inicio de la fibrosis quística
y, por tanto, indicaciones para la realización del test del sudor, EXCEPTO:

1.! ! 1.! Tos!crónica.!


2.! ! 2.! Sabor!salado!al!besarle.!

!
!
!

3.! ! 3.! Esteatorrea.!


4.! ! 4.! Invaginación!intestinal.!
Gráfico de respuestas
Comentario

La fibrosis quística (FQ) es un tema fundamental para el examen de la que debe conocer
prácticamente todo. No obstante, esta es una pregunta muy difícil.

La FQ es la enfermedad genética más letal en raza caucásica. Se debe a mutación del gen CFTR
(proteína reguladora de la conductancia transmembrana), que está en el brazo largo cromosoma 7.
La mutación más prevalente en nuestro medio es la delta- F508. Es una enfermedad con herencia
AR, por lo que es necesario tener mutados los dos alelos para expresar la enfermedad.

La alteración de la regulación de los canales iónicos de las membranas celulares produce


secreciones deshidratadas y espesas, lo que origina la clínica. La afectación pulmonar determina la
mortalidad de esta enfermedad.

Para el diagnóstico es necesario una sospecha y una confirmación. La sospecha es por la clínica,
antecedentes familiares o cribado neonatal positivo (elevación de tripsinógeno inmunoreactivo). La
confirmación es mediante el test del sudor, alteración en la diferencia de potencial nasal y estudio
genético (este último indicado en menores de 3 meses).

En el manual viene una lista con las distintas manifestaciones clínicas que son indicación para
realizar el test del sudor. De las 4 respuestas, todas son correctas menos la invaginación intestinal.
No la confunda con el íleo meconial, que sí sería sugestivo.(R/4)

448. Mujer de 23 años de edad con diagnóstico de lupus eritematoso sistémico que
presenta cifras de hemoglobina de 9.8 g/dl, microcitosis, hierro sérico disminuido, niveles
de transferrina reducidos y ferritina sérica elevada. ¿Qué tipo de anemia le parece más
probable que presente esta paciente?:

1. 1. Anemia aplásica.
2. 2. Anemia megaloblástica.
3. 3. Anemia de inflamación crónica.
4. 4. Anemia ferropénica.
Gráfico de respuestas
Comentario

Esta es una pregunta que no debe de fallar, solo con conceptos básicos se puede contestar.

Las opciones 2 es una anemia macrocítica, por lo que descartamos con facilidad esta opción. La
duda puede surgir entre las opciones 3 y 4, porque ambas pueden cursar con microcitosis; para
diferenciarlas siempre debe de fijarse en los valores de ferritina sérica, que expresan como se
encuentran los depósitos de hierro en el organismo; en este paciente están elevados, y además
nos hablan de una enfermedad crónica como el LES, por lo que la opción 3 es la más correcta.

449. Una mujer de 55 años, posmenopáusica, consulta por astenia y disnea de esfuerzo.
En la anamnesis refería ligera epigastralgia y pirosis ocasional. No metrorragias.
Laboratorio: Hb 6 g/dl, VCM 69 fl, sideremia 13 microgramos/dl, ferritina 4 ng/ml.

!
!
!

Endoscopia digestiva alta: pequeña hernia hiatal por deslizamiento sin signos de
esofagitis. ¿Cuál es la actitud más CORRECTA con esta enferma?

1. 1. Administrar hierro oral y ver evolución de la anemia.


2. 2. Tratar con inhibidores de la bomba de protones y evaluar a los tres meses.
3. 3. Recomendar una colonoscopia completa.
4. 4. Solicitar un evaluación ginecológica.
Gráfico de respuestas
Comentario

Se nos presenta paciente de 55 años con anemia ferropénica. Se realiza estudio endoscópico alto
en el que se visualiza hernia de hiato sin otros hallazgos lo que no justifica la anemia de la
paciente, motivo por el que está indicado un estudio con colonoscopia completa.

Con la edad de la paciente y la gravedad de la anemia no está indicado administrar hierro, y


esperar, ya que el cuadro es sugestivo de patología orgánica.

Por prevalencia es más habitual encontrar lesiones con potencial hemorrágico en colon que en ID
por lo que se debe realizar la colonoscopia antes que la cápsula endoscópica.(R/3)

!
!
!

!
!
!

!
!
!

!
!
!

!
!
!

!
!
!

!
!
!

!
!
!

!
!
!

!
!
!

!
!

Vous aimerez peut-être aussi